Ths Nguyễn Thanh Tú
eBook Collection
CÁC
Một số bài toán số học trong các kỳ thi học sinh giỏi, Olympic trong nước và nước
WORD VERSION | 2023 EDITION ORDER NOW / CHUYỂN
CÁC KÍ HIỆU VÀ CỤM CHỮ CÁI VIẾT TẮT
Kí hiệu Tên tiếng Anh Tên tiếng Việt
IMO International Mathematical Olypiad Kì thi Olimpic Toán quốc tế
VMO Viet Nam Mathematical Olympiad Kì thi học sinh giỏi quốc gia môn toán Việt Nam
IMO SL International Mathematical Olypiad Shortlist Tuyển tập các bài toán trong danh sách rút gọn đề nghị cho kì thi Olimpic Toán quốc tế
X TST X Team Selection Tests Kì thi chọn đội tuyển X
NMO National Mathematical Olympiad Kì thi học sinh giỏi toán quốc gia
TT & TT
Tạp chí Toán học và tuổi trẻ
HSG Học sinh giỏi
HSG QG Học sinh giỏi quốc gia
Phần 1. ĐẶT VẤN ĐỀ
1. Lý do chọn đề tài
Số học là một phân nhánh toán học lâu đời nhất. Nó được hầu hết mọi người thường xuyên sử dụng trong những công việc thường nhật cho đến các tính toán khoa học.
Trong chương trình toán phổ thông, số học vốn là một phân môn khó. Các bài toán số học thường xuyên xuất hiện trong các kì thi học sinh giỏi các cấp và nó luôn là một thách thức đối với học sinh. Trong những bài toán số học ấy, chúng ta có thể nhận thấy rằng: các tính chất các ước nguyên tố thường xuyên được sử dụng một cách tinh tế và đẹp mắt. Tuy nhiên, hiện nay, các tài liệu viết riêng và chuyên sâu về nó không nhiều. Điều đó gây không ít khó khăn trong việc tiếp cận hướng giải quyết các bài toán số học đó.
Chính vì vậy, để giải quyết phần nào các khó khăn đó, chúng tôi chọn đề tài “Số nguyên tố và các bài toán liên quan” để trao đổi cùng các thầy cô và các em học sinh chuyên toán, góp phần nâng cao chất lượng giảng dạy môn toán trong trường THPT chuyên.
Phần nội dung chuyên đề bao gồm ba chương:
- Chương 1. Các bài toán vềước nguyên tố. Nội dung của chương này xoay quanh các vấn đề về phân tích chính tắc và các định lý cổ điển liên quan đến ước nguyên tố của một số nguyên.
- Chương 2. Thặng dư bậc hai.
- Chương 3. Các bài toán về hệ số nhị thức và một số định lý khác. Nội dung này xoay quanh các tính chất ước nguyên tố trong hệ số nhị thức trong khai triển Newton và định lý Polignac.
Trong các chương, hệ số ví dụ minh họa được đưa ra từ dễ đến khó theo quan điểm của chúng tôi. Trong mỗi ví dụ thường có định hướng, dẫn dắt tại sao có lời giải như vậy. Điều đó giúp cho việc học trở nên thuận lợi hơn.
2. Mục đích nghiên cứu
Đề tài “Số nguyên tố và các bài toán liên quan” được chúng tôi lựa chọn trao đổi cùng các đồng nghiệp về các tính chất, định lý về các số nguyên tố, ước nguyên tố của một số tự nhiên, sự phân tích chính tắc,... Vận dụng các định lý, tính chất đó giải quyết các bài toán số học trong các kì thi học sinh giỏi. Thông qua đề tài này, chúng tôi muốn nhấn mạnh, làm rõ tầm quan trọng của số nguyên tố trong các bài toán số học trong các kì thi học sinh giỏi Quốc gia và Quốc tế
3. Nhiệm vụ nghiên cứu
Nghiên cứu các phương pháp giải bài toán số học trong các kì thi học sinh giỏi toán quốc tế, quốc gia Việt Nam và các nước trên thế giới. Cung cấp một tài liệu nhỏ về phương pháp dạy học phân môn số học cho các đồng nghiệp dạy chuyên toán, và phương pháp tự nghiên cứu cho học sinh chuyên.
4. Đối tượng và khách thể nghiên cứu
Đối tượng nghiên cứu của đề tài là học sinh chuyên Toán, đội tuyển học sinh giỏi quốc gia môn toán.
5. Phạm vi nghiên cứu
- Kiến thức trong phạm vi chương trình thi học sinh giỏi quốc gia của Bộ Giáo dục và Đào tạo.
- Nghiên cứu các đề thi học sinh giỏi toán quốc tế và học sinh giỏi toán quốc gia các nước.
- Nghiên cứu các tài liệu số học có liên quan.
6. Phương pháp nghiên cứu
- Nghiên cứu lý luận: nghiên cứu các tài liệu, chương trình của Bộ Giáo dục và Đào tạo, các tài liệu số học hiện hành, các tạp chí toán học trong và ngoài nước, các tài liệu từ internet,…
- Trao đổi, tọa đàm với các giáo viên chuyên và học sinh chuyên trong nước.
- Tổng hợp, tổng kết kinh nghiệm.
Phần 2. PHẦN NỘI DUNG
Chương 1. Các bài toán về ước số nguyên tố
1.1 Định nghĩa
Một số nguyên dương p được gọi là số nguyên tố, nếu nó chỉ có hai ước số dương là 1 và chính nó.
Nếu p không phải số nguyên tố thì p được gọi là hợp số.
Nhận xét: 2 là số nguyên tố chẵn duy nhất.
1.2 Một số định lý, bổ đề cần dùng
1.2.1 Định lý 1 ( Định lý cơ bản của số học)
Mọi số tự nhiên lớn hơn 1 đều có thể phân tích một cách duy nhất thành tích các
thừa số nguyên tố
1.2.2 Định lý 2 (Tính vô hạn của tập số nguyên tố)
Tập hợp các số nguyên tố là vô hạn.
1.2.3 Định lý 3
Cho p là một số nguyên tố. Nếu p | ab thì p | a hoặc p | b.
1.2.4 Định lý 4 (Định lý Fermat nhỏ)
Cho p là một số nguyên tố và a là một số nguyên sao cho (a, p) = 1. Khi đó
1 1(mod) p ap ≡ .
Chứng minh
Ta xét hệ thặng dư thu gọn mod p là A = {1, 2, …, p - 1}
Do (a, p) = 1 nên B = {a, 2a, …, (p - 1)a} cũng là hệ thặng dư thu gọn mod p.
Do đó ta có
Mà p nguyên tố nên (1.2…(p - 1), p) = 1.
Từ đó ta có điều phải chứng minh.
1.2.5 Định lý 5 (Định lý Euler)
Cho a là một số nguyên, n là số nguyên dương, (a, n) = 1. Khi đó
() 1(mod) a an ϕ ≡ .
Chứng minh
Ta xét hệ thặng dư thu gọn mod n là A = { 12() ,,..., n aaaϕ }.
Do (a, n) = 1 nên B = { 12().,.,...,. n aaaaaaϕ } cũng là hệ thặng dư thu gọn mod n Do đó ta có () 12()12()12() ....(.).(.)...()....(mod) a nnn aaaaaaaaaaaaan ϕ ϕϕϕ ≡≡
Kết hợp với ( 12() n aaaϕ ,n) = 1 ta dễ dàng suy ra đpcm.
1.2.6 Định lý 6 (Định lý Wilson)
Cho p là một số nguyên tố lẻ. Khi đó
(1)!1(mod) pp −≡− .
Chứng minh
(Xin nhường cho bạn đọc).
1.2.7 Bổ đề LTE
a) Cho p là số nguyên tố lẻ, x và y không chia hết cho p, x – y ⋮ p,n là một số nguyên dương. Khi đó
()()(). nn ppp vxyvxyvn −=−+
b) Cho p là số nguyên tố lẻ, x và y không chia hết cho p, x + y ⋮ p, n là số nguyên dương lẻ. Khi đó
()()(). nn ppp vxyvxyvn +=++
c) Cho x, y lẻ, n là số nguyên dương chẵn. Khi đó
22
222 ()()()1 nn vxyvxyvn −=−+− .
Chứng minh (Tham khảo “Phạm Văn Quốc – bổ đề LTE”)
1.2.8 Cấp phần tử
1.2.8.1 Các định nghĩa Định nghĩa 1
1.2...(1).2...(1)(mod). paapap −≡−
Cho n > 1 và a là một số nguyên dương, (a, n) = 1. Số nguyên dương k nhỏ nhất
thỏa mãn ak ≡ 1 (mod n) được gọi là cấp của a modulo n
Kí hiệu k = ordn(a).
Định nghĩa 2
Cho n > 1 và a là một số nguyên dương, (a, n) = 1. Nếu ϕ(n) = ordn(a) thì a được
gọi là một căn nguyên thủy modulo n.
Nhận xét: Từ định nghĩa trên ta dễ dàng suy ra
+) Nếu a là căn nguyên thủy ( mod n) thì mọi số cùng lớp với a theo (mod n) đều là căn nguyên thủy (modn).
1.2.8.2 Các định lý
Định lý 1
Cho a, n thỏa mãn n > 1, (a, n) = 1. Khi đó ax ≡ 1 (mod n) ⇔ x ⋮ ordn (a).
Chứng minh
Giả sử ax ≡ 1 (mod n).
Đặt k = ordn (a).
Theo thuật toán Euclid ta có x =kq + r, 0 ≤ r < k.
Khi đó 1 ≡ ax ≡ (ak)qar ≡ ar (mod n).
Suy ra ar ≡ 1 (mod n) r = 0 (theo định nghĩa).
Vậy x ⋮ k.
Chiều ngược lại hiển nhiên.
Hệ quả
Cho a, n thỏa mãn n > 1, (a, n) = 1. Khi đó ϕ(n) ⋮ ordn (a).
Định lí 2
Nếu a là căn nguyên thủy (mod n) thì tập A = {1, a, a2 ,…,ah-1} là hệ thặng dư thu
gọn (mod n) (lúc này h = φ(n))
Định lí 3
Nếu p là một số nguyên tố thì có đúng φ(p - 1) căn nguyên thủy (mod p)
Định lít 4
5
Nếu p là một số nguyên tố lẻ và a là một căn nguyên thủy (mod p2) thì a cũng là
căn nguyên thủy (mod pn) với n ≥ 3.
Định lý 5 (Định lý về sự tồn tại căn nguyên thủy)
Cho m là một số nguyên, m > 1 khi đó m có căn nguyên thủy khi và chỉ khi m có một trong 4 dạng sau: 2, 4, pα, 2p α (trong đó p là 1 số nguyên tố lẻ)
(Phần chứng minh các định lí trên, xin nhường cho bạn đọc).
1.3 Các ví dụ điển hình
Trong mục này, để thuận lợi cho công tác giảng dạy, dựa vào độ phức tạp các bài toán, tôi sẽ chia thành hai phần. Phần một là các ví dụ cơ bản, giúp học sinh làm quen dần với các dạng toán. Phần hai là các bài toán nâng cao, đòi hỏi kĩ năng xử lí tinh tế và được cập nhật trong các kì thi học sinh giỏi gần nhất.
1.3.1 Các ví dụ cơ bản
Trong phần này, tôi sẽ đưa ra các ví dụ điển hình về tính chất ước nguyên tố. Vận dụng các tính chất đó vào giải quyết các bài toán liên quan. Tuy nhiên, các ví dụ được chọn không quá khó, nhưng thường xuyên xuất hiện. Cố gắng giúp học sinh hình thành, phát triển tư duy trong phân môn số học.
Ví dụ 1 (Đề tham khảo 30/4/2021) Giả sử a, b, c là các số nguyên dương thỏa mãn hệ phương trình
=
Chứng minh rằng a =b =c.
=
Lời giải
2143()
=
2143()
2143()
ϕ ϕ ϕ
ab bc ca
Nhận xét: đây làmột dạng của hệ hoán vị vòng quanh nên việc giải quyết hệ này thông thường ta xét a ≤ b ≤ c sau đóchứng minha = b = c.
Mặt khác, bài toán liênquan đếnphi hàmEuler nên thông thường tasử dụng phân tích chính tắc để đánhgiá là mộthướng đi chung. Mời cácbạn theodõi lời giải sau đây.
Ta có (21, 43) = 1 a, b, c cùng chia hết cho 43. Do đó, a, b, c ≥ 43. Xét phân tích chính tắc của a, b, c như sau
nmk abc iii apbpcp ααα ===
===
=−−
n aa i app α ϕ = ∏ ⋮⋮
Chứng minh tương tự ta được ϕ(a), ϕ(b), ϕ(c) chẵn.
Do (21, 2) = 1 a ⋮ 2, b ⋮ 2. c ⋮ 2.
Từ đó ta có
m i b abbbb p ϕ = ∏
==−≤−−=
1
a ≤ b.
Chứng minh tương tự ta được b ≤ c; c ≤ a.
Từ đó suy ra a = b = c
Ví dụ 2 (Chọn ĐT 10 KHTN 2020) Tìm tất cả các số nguyên tố p sao cho
3p + 5p – 1
là số nguyên tố.
Nhận xét: Bài toán này khá đơn giản. Trước tiên ta thử nghiệm, dự đoán p = 3, sau đó
xét p khác 3 và chứng minh 3p + 5p – 1 không là số nguyên tố. Lời giải cụ thể như sau:
Lời giải
TH1: p = 3k + 1, (k chẵn) ta có
3p + 5p – 1 = 3.27k + 5.125k – 1 ≡ 3(-1)k + 5.(-1)k – 1 ≡ 0 (mod 7).
Nên 3p + 5p – 1 không là số nguyên tố.
TH2: p = 3k + 2, (k lẻ) ta có
3p + 5p – 1 = 9.27k + 25.125k – 1 ≡ 2(-1)k + 4(-1)k – 1 ≡ 0 (mod 7).
Vậy p = 3k p = 3.
Thử lại thấy p = 3 thỏa mãn.
Ví dụ 3 (Chọn ĐT 10 KHTN 2020) Tìm x, y nguyên dương sao cho 2x + 5y + 2 là số chính phương.
Nhận xét: Bài này cũng giống như bài trên, Tính chất chủ đạo sử dụng ở đây là mọi ước của pn đều có dạng pk, 0 ≤ k ≤ n. Cụ thể ta có lời giải sau:
Lời giải
Xét x ≥ 2 ta có 2x + 5y + 2 ≡ 3 (mod 4) 2x + 5y + 2 không là số chính phương.
Do đó x = 1.
Khi đó ta đặt 5y + 4 = n2 ⇔ (n -2)(n + 2) = 5y .
a ba b
n ab n −= −= == +=
Từ đó tìm được (x, y) = (1, 1).
Ví dụ 4 (Olimpic KHTN 2014) Tìm tất cả bộ ba số (x, n, p) với x, n nguyên dương, p nguyên tố thỏa mãn
3 22(1) n xxp+=−
Lời giải Ta có 33 2 23(1)233 (1)(3)3.
n n n
xxpxxp xxxp +=−⇔++= ⇔+−+=
Do d = (x + 1, x2 – x + 3) = (x + 1, (x+1)(x-2) + 5) = (x +1, 5).
Nếu d = 1, do x + 1 > 1 và x2 –x + 3 > 1 nên 2 2
13 3 1 33.
+=
−+=
Từ đó tìm được (x, n, p) =(2, 2, 2) và (x, n, p) = (1, 1, 2).
Nếu d = 5 thì x + 1 = 5m
Thay vào phương trình ban đầu ta được 2 22 5(25155)35 (531)3.5.
Nếu m ≥ 2 thì 5m2 – 3m + 1 > 3.
Do d = (m,5m2 – 3m + 1) = 1 m = 3 và 5m2 – 3m + 1 = 5n – 2 vô nghiệm.
Vậy ta tìm được ba bộ (x, n, p) như trên.
Ví dụ 5 (Olimpic KHTN 2015) Tìm tất cả các số nguyên tố p sao cho 3p + 4p là số chính phương.
Lời giải Ta có 3p + 4p = x2 (x – 2p) (x + 2p) = 3p
8
12 23 23(31)0 23
ppn pnpn pn
x n x +− += =− = −=
Do đó 2p + 1 = 3p – 1 p = 2 (thỏa mãn)
Ví dụ 6 (TH & TT_ T10/507) Tìm số nguyên tố p và hai số nguyên dương a, b sao cho pa + pb là số chính phương.
Lời giải
TH1: a = b suy ra 2.pa = x2 p = 2, a lẻ (p, a, b) = (2, 2n + 1, 2n + 1), n ∈ ℕ
TH2: a ≠ b.
Không mất tính tổng quát ta giả sử a > b pb(pa – b + 1) = x2 .
Nhận xét: đây là bài toán cơ bản nhất mà sau khi dạy lý thuyết về cấp phần tử và có
thể lấy làm ví dụ minh họa mở đầu.
Lời giải
Ta có 2 21(mod)21(mod). hh pp ≡− ≡
Đặt (2)|2 p kordkh = nhưng k không là ước của h
Nếu k ≤ h thì 2k – 1 < 2h + 1 = p (loại) k > h
Nếu k lẻ mà k | 2h k | h (vô lý).
Vậy k chẵn k = 2t | 2h t | h t ≤ h. (1)
b ab pu pv = +=
Mà (pb, pa – b + 1) = 1 nên 2 2 1.
Do đó b chẵn.
Ta có pa – b = (v – 1)(v + 1).
Nếu v = 2 thì pa – b = 3 a – b = 1 và p = 3 (p, a, b) = (3, 2k + 1, 2k).
Nếu v > 2 thì p | v – 1 và p | p + 1 p | 2 p = 2 2a – b = (v - 1)(v + 1).
Suy ra a – b = 3.
Ta có 2k – 1 = 22t – 1 = (2t - 1)(2t + 1).
Do t ≤ h 2t – 1 < 2h + 1 = p (p, 2t - 1) = 1.
2t + 1 ⋮ p 2t + 1 ≥ p = 2h + 1 t ≥ h. (2)
Từ (1) và (2) suy ra t = h k = 2h
Ví dụ 9 (HSG Tỉnh THPT Phú Yên 2021) Cho p là số nguyên tố lẻ. a và b là hai số nguyên dương lẻ sao cho a + b chia hết cho p và a – b chia hết cho p – 1. Chứng minh rằng
2 ba abp + ⋮
12 2221,23 12
k lk v klv v −= −= == = += .
Từ đó suy ra (p, a, b) = (2, 2k + 3, 2k).
Kết luận…
Ví dụ 7 (TH&TT_ T10/512) Chứng minh rằng với mọi n nguyên dương thì ta luôn có n và 2 21 n + là hai số nguyên số nguyên tố cùng nhau.
Lời giải
Gọi p là số nguyên tố và p | 2 21 n + thì theo một tính chất quen thuộc về bậc của một số nguyên ta có p ≡ 1 (mod 2n + 1).
p = 2n + 1k + 1, k ∈ ℕ * p > 2n + 1 .
Mặt khác, ta dễ chứng minh bằng quy nạp được 2n + 1 > n, với mọi n ∈ ℕ *
Từ đó ta có đpcm.
Ví dụ 8 (TH&TT_T11/513) Cho h là số nguyên dương sao cho p = 2h + 1 là một số nguyên tố. Tìm bậc của 2 modulo p.
9
Nhận xét: Bài này có hình thức khá đẹp mắt mà tôi rất thích. Nó thực chất đã xuất hiện khá lâu, trên TH&TT và chuyên đề cấp phần tử (2014) của thầy Nguyễn Duy Liên.
Lời giải
Ta có a, b lẻ nên baab + chẵn nên 2 baab + ⋮
Không mất tính tổng quát ta giả sử a ≥ b
Ta có (1) (1)(1)0(mod)(mod)abbabbkpab aaaaaapaap −=−=−≡ ≡
Do b lẻ nên 0(mod) aaaa ababpabp ++ +≡ ⋮⋮
Từ trên ta có 0(mod) baaa ababp +≡+≡
Từ đó ta có đpcm.
Ví dụ 10 (TH&TT_ T11/472) Cho n, k là các số nguyên dương và p là số nguyên tố lẻ
Chứng minh rằng tồn tại số nguyên dương a sao cho pk là ước của ϕ(a), ϕ(a + 1), …, ϕ(a + n).
Nhận xét: Những bài chứng minh sự tồn tại hoặc tồn tại vô hạn các số nguyên dương thỏa mãn điều kiện nào đó, ta thường nghĩ đến định lý Dirichle hoặc xây dựng hệ phương trình đồng dư sau đó áp dụng định lý thặng dư Trung Hoa.
Lời giải
Gọi q0, q1, …, qn là các số nguyên tố sao cho qi ≡ 1 (mod pk), điều này tồn tại theo bài toán trên.
Theo định lí Thặng dư Tru ng Hoa tồn tạo số nguyên dương a sao cho
Do đó trong hai số u và a phải có một số bằng 1, trong hai số v và b phải có một số bằng 1.
Lại có u và v không đồng thời bằng 1 nên chỉ có thể xảy ra các khả năng sau:
+) a = b = 1 2u – 1.3v – 1 = 2 v = 1, u = 2 n = 6 (thỏa mãn).
+) b = u = 1 3v – 1 – 2a – 1 = 1.
Phương trình này quen thuộc và dễ dàng giải được a ∈ {2, 4} n ∈ {12, 48}.
+) a = v = 1 2u – 1 – 3b – 1 = 1 b ∈ {1, 2} n ∈ {6, 18}.
Hay a + i ≡ 0 (mod qi).
≡ ≡− ≡−
Nếu p | n thì p – 1 | ϕ(n)
0(mod) 1(mod) ... (mod) n
aq aq anq
0 1
Từ đó ta tìm được n ∈ {2, 3, 6, 12, 18, 48 }.
Ví dụ 12 (TH&TT - 249) Giả sử a1, a2, …, an là các số nguyên dương phân biệt thỏa mãn 12
111 ...1 n aaa +++=
=− ∏
Mặt khác ta lại có 1 1 1 1 (...)(1). mi m mii i pppp αα α ϕ =
Từ đó suy ra pk | qi – 1 | ϕ(a + i), i = 0, 1, 2, …, n.
Đó chính là đpcm.
Ví dụ 11 (Pi 233) Cặp số nguyên dương a, b được gọi là cân bằng nếu hai số nguyên dương a, b có cùng tập ước nguyên tố. Tìm tất cả các số nguyên dương n sao cho n và
n + 6 là cặp cân bằng.
Lời giải
Giả sử n là số nguyên dương sao cho (n, n + 6) là cặp cân bằng.
Dễ thấy n > 1.
Gọi p là ước nguyên tố bất kì của n p | (n, n + 6) p ∈ {2, 3}.
Do đó
và số lớn nhất trong các số a có dạng 2p với p là một số nguyên tố. Hãy xác định các số a1, a2, …, an.
Lời giải Không mất tính tổng quát ta giả sử a1 = max {a1, a2, …, an} = 2p với p nguyên tố.
Ta có 231 23 23
n n = −= +=
23 23236.(1) 623
ab uvab uv
TH1: a = 0 thì u = 0 3v – 3b = 6 ⇔ 3b(3v – b - 1) = 6.
Từ đó dễ dàng tìm được b = 1, v = 2 n = 3 (thỏa mãn).
TH2: Nếu b = 0 v = 0 2u – 2a = 6 a = 1, u = 3 n = 2 (thỏa mãn).
TH3: a,b, u, v > 0 thì 2u – 1.3v – 1 – 2a – 1.3b – 1 = 1.
+++=−=−= ⇔=⇔=−
1111121 ...11 22 21 2(21).... ...2
n n n
p aaaapp mp pmpaaa aaap
Do đó p| 23 n aaa (vì (p. 2p - 1) = 1).
Suy ra p là ước của một trong các số ai, i = 2, 3, …, n.
Không mất tính tổng quát, giả sử p | a2.
Mà a1 = 2p > a2 a2 = p
Từ đó suy ra 3412 34 34
111111123 ...11 22 23 2(23)..... ...2
+++=−−=−−= ⇔=⇔=−
n n n
p aaaaappp MppMpaaa aaap
Suy ra p | 34 (23).... npaaa .
Nếu có số ai chia hết cho p (với i = 3, 4, …, n) thì ai = p = a2 (vô lý) nên p | 2p –
p = 3.
Vậy số lớn nhất a1 = 6, a2 = 3. Các số còn lại nhỏ hơn 6 nên thuộc tập {1, 2, 4, 5}.
Từ đó dễ dàng chứng minh được tồn tại duy nhất bộ {2, 3, 6} thỏa mãn.
1.3.1 Các bài toán nâng cao
Trong phần này, chúng tôi cố gắng đưa ra các ví dụ minh họa mang tính thời sự và tinh tế hơn. Các hướng giải quyết phức tạp hơn (theo quan điểm cá nhân)
Ví dụ 13 (IMO SL 2018) Tìm tất cả các cặp số nguyên dương (n, k) phân biệt sao cho tồn tại số nguyên dương s sao cho số ước nguyên dương của sn và sk là bằng nhau.
Lời giải (Dựa theo lời giải của v_Enhance trên Aops)
TH1: Nếu n | k hoặc k | n thì hiển nhiên không tồn tại s.
TH2: n và k không có số nào là bội của số kia.
Ta sẽ chứng minh luôn tồn tại số nguyên dương s sao cho số ước của sn và sk là bằng nhau.
Nhận xét rằng: với a > b ≥ 0 và vọi M nguyên, M ≥ b + 1 luôn tồn tại số nguyên
không âm c sao cho
11 1 acM bcM +++ = ++
Thật vậy, ta chỉ việc chọn ()1(1)(1)10cMabbbbbb =−−+≥++−−+> , luôn thỏa mãn đẳng thức trên.
Bây giờ ta xét
β αβγαγ
∏
' '' 1
mMmMmMmM mMmMmMmM
1 121 .... 111 m ss s ss
Vậy các cặp (s,k) mà không có số nào là bội của số còn lại luôn thỏa mãn bài toán.
Ví dụ 14 (Turkey EGMO TST 2020) Kí hiệu ( )pm là số các ước nguyên tố phân
của m . Tìm tất cả các số nguyên dương n thỏa mãn đẳng thức sau:
( ) 222(5)24fnfnn +++=−
Lời giải Vì mọi ước nguyên tố đều lớn hơn hoặc bằng 2 nên:
( ) 22 242(5)224 nfnfnn −=+++≥+ ≥ .
Với 4 n = : (18)(21)4ff+> (loại).
Với 5 n = : (27)(30)3362.54ff+=+==− (thỏa mãn).
Với 6 n = : (38)(41)8ff+> (loại).
Với 7 n = : (51)(54)3210ff+=+< (loại).
Ta chứng minh bổ đề: nếu n là một hợp số thì () fnn ≤
= =
111 '''' 111
111 111
........... im im
ijm ijm
Xét hai trường hợp:
β αβγαγ
nppqqrr kppqqrr
Với '' ,(1,2,...,),,(1,2,...,),,1,0.llsslismimjααγγ>=<=≥≥
Ta chọn '' 11 max{,...,,,...,} im M ααγγ > . Khi đó theo nhận xét trên ta luôn tìm được các số 1,..., i λλ sao cho
Nếu np = với ,2tt∈≥ ℕ . Khi đó ()1pn = và () t fnppn =<= .
Nếu ()2pn ≥ mà () fnn > , vì ()1 () 2 pn pn + < nên ước nguyên tố thứ ()pn của
1
iMiMiMiM iMiMiMiM
++ ++++ == ++++−
1 121 ..... 111 i ss s ss
n cũng lớn hơn n , vô lý vì n không thể có hai ước nguyên tố phân biệt cùng lớn hơn n
Bổ đề trên được chứng minh.
Quay lại bài toán.
Và các số '' 1 ,..., m λλ sao cho 14
chỉ xét 8 n ≥ và sử dụng bổ đề ta có ( ) ( ) 222222 fnnfnn +≤+ +≤
và ( ) ( ) 222555 fnnfnn +≤+ +≤
Nếu 2 (2) fnn += thì 2 |2|22 nnnn + = (vô lý).
Tương tự nếu 2 (5) fnn += thì 2 |5|55 nnnn + = (vô lý).
Nếu 2 (2)1fnn+=− thì 2 1|21|34nnnn −+ ≤ (vô lý).
Tương tự nếu 2 (5)1fnn+=− thì 2 1|51|67nnnn −+ ≤ (vô lý).
Suy ra ( ) 2 52fnn+≤− (1).
Nếu 2 (2)2fnn+=− thì 2 2|22|46nnnn −+ ≤ (vô lý).
Do đó ( ) 2 23fnn+≤− (2).
Từ (1) và (2) suy ra ( ) 222(5)2524fnfnnn +++≤−<− (loại).
Vậy 5 n = là giá trị duy nhất thỏa mãn đề bài.
Ví dụ 15 (IMO Shortlist 2020) Cho trước số nguyên dương k . Chứng minh rằng tồn tại số nguyên tố p sao cho có thể chọn các số nguyên dương phân biệt { } 123 ,,...,1;2;...;1 k aaap + ∈− thỏa mãn 123iiii aaaai +++ chia hết cho p với mọi 1,2,..., ik = .
Lời giải Trước tiên ta xây dựng dãy số hữu tỉ { } 3 1 k n n q + = như sau:
==== + =∀∈
ℕ
r qrsrsnk s =∈=∀=+ ℕ
n nnnnn n
>>
>−
Max, Max Max nn ijji ijk
prps prsrs ≤≠≤+
1 1;;; 1 nn
qqMqMq M n qqn n +
2 1234 3 * 4
Trong đó M là một số hữu tỉ tùy ý thỏa mãn điều kiện sau: 1111 111...1 1233 M k
>++++
Tiếp theo, với mỗi 1;2;...;3nk=+
ta chọn { }1;2;...;1 n ap∈− sao cho 1(mod), nnn arsp ≡ trong đó 1 n s là phần tử nghịch đảo của n s trong { }1;2;...;1 p Điều này là khả thi do , s nn rpp ⋮⋮ . Ta đi chứng minh { }n a thỏa mãn bài toán.
+ ij∀≠ thì ijaa ≠ .
Thật vậy giả sử 11(mod)(mod)ijiijjijji aarsrsprsrsp = ≡ ≡ (vô lý).
Do đó ta có
1111
(mod) . iiiiiiiiiiii iiii
≡≡
aaaarrrrssssip aaaaip +++++++++ +++ ⋮
Đó chính là điều phải chứng minh.
Ví dụ 16 (Centroamarican 2020 – Pr 6) Ta gọi một số nguyên dương N là số
Inteoceanic nếu nó có phân tích chính tắc
12 12.... kx xx kNppp =
thỏa mãn
1212......kk pppxxx +++=+++ .
Hãy tìm tất cả các số Inteoceanic nhỏ hơn 2000.
Lời giải
Trước hết ta chứng minh k ≤ 3.
Thật vậy, nếu k ≥ 4 thì
không thỏa mãn.
Vậy k ≤ 3.
Với k = 3 ta có
Ví dụ 17 (BMO 2019) Kí hiệu P là tập hợp gồm tất cả các số nguyên tố.
Tìm tất cả các hàm : fPP → thỏa mãn đẳng thức sau ()()()()fqpfpq fpqfqp +=+
với mọi , pqP ∈
Lời giải
Từ đẳng thức đã cho ta có ()()()()fqfpqp fpfqpq −=−
Với , pq là các số nguyên tố lẻ bất kì thì qppq là số chẵn nên dễ thấy (),()fpfq phải cùng tính chẵn lẻ
Giả sử (),()fpfq đều chẵn thì do : fPP → nên ()()2fpfq==
Thay vào ta được pqqp = , vô lý.
Do đó với mọi số nguyên tố lẻ p thì ()fp lẻ.
12 17
ppp N +++
+++≥+++= >≥≥>
...235717 22...2222000 kk
k xxxx xx
12 12
1212331233 2235 11 123.22.42222000xxxxxxxxxxNppp +++++ =>=≥≥> (loại).
Với k = 2 ta có p1 + p2 ≤ 10 nên chỉ có các cặp (p1, p2) sau
(2, 3), (2, 5), (2, 7), (3, 5), (3, 7).
TH1: x1 + x2 = 2 + 3 = 5 ta có các nghiệm sau
N ∈ {24.3, 23.32, 22.33, 21.34}.
TH2: x1 + x2 = 2 + 5 = 7, ta có 54.23 > 2020 nên ta tìm được
N ∈ {53.24, 52.25, 5.26}.
TH3: x1 + x2 = 2 + 7 ta dễ dàng tìm được nghiệm duy nhất N = 28.7
TH4: x1 + x2 = 3 + 5 = 8.
Ta có 37.5 > 2000 nên dễ dàng chứng minh được không tồn tại N.
TH5: x1 + x2 = 3 + 7 = 10 cũng dễ chứng minh được không tồn tại N.
Với k = 1 dễ dàng tìm được N ∈ {22, 33}.
17
Thế 2 q = và p là một số nguyên tố lẻ bất kì vào đẳng thức đã cho ta được (2)()2 ()(2)2 ffppfpfp−=−
Vì vế phải là một số lẻ nên ()fp và (2) f khác tính chẵn lẻ. Lại có ()fp lẻ nên (2)2 f = .
Bây giờ xét p là một số nguyên tố lẻ bất kì: 2()2 ()22 fppfpp−=− .
Xét hàm số 2 ()2x gxx=− . Ta chứng minh (1)() gxgx +< với mọi 3. x ≥
Thật vậy: ( ) ( ) 2 12 (1)()1222120 xxx gxgxxxx + +−=+−−−=+−< với mọi 3. x ≥
Do đó từ đẳng thức 2()2 ()22 fppfpp−=− .
ta thu được () fpp = với mọi số nguyên tố lẻ p
Kết luận: hàm số cần tìm là hàm () fpp = (thử lại thấy thỏa mãn).
Ví dụ 18 (Canada TST 2021) Cho n là một số nguyên dương. Gọi p1, p2, …, pn là các số nguyên tố phân biệt lớn hơn 3. Chứng minh rằng 12 21 nppp + có ít nhất 4n ước số
Lời giải
Bổ đề 1. Cho a, b là hai số nguyên dương lẻ. Khi đó (2a + 1, 2b + 1) = 2(a, b) + 1.
18
Đây là bổ đề khá quen thuộc, xin không chứng minh lại ở đây.
Trở lại bài toán
Nếu (p, 3) = 1 thì theo bổ đề trên ta có (2p + 1, 23 + 1) = 2(p,3) + 1 = 3
2p + 1 không chia hết chi 9.
Ta gọi một số nguyên dương là “tốt” nếu nó là số Squarefree lẻ, không chia hết cho 3
và lớn hơn hoặc bằng 5.
Đặt T(n) là số ước nguyên tố phân biệt của n, d(n) là số ước của n
Bài toán trở thành: Chứng minh rằng với mọi số tốt a ta có () (21)4aTa d +≥ (*)
Quy nạp theo T(a).
Nếu T(a) = 1 hay a = p ≥ 5, p nguyên tố.
Khi đó 1 21(21)(2...1) aa +=+++ .
Mà 1 2...13 a ++> , lẻ và không chia hết cho 3 nên nó có ít nhất một ước nguyên tố
lớn hơn hoặc bằng 5
Do đó (*) đúng khi T(a) = 1.
() (21)44. aTa d +≥=
Giả sử (*) đúng khi T(a) = k, ta xét số nguyên tố (,)1,5, papp =≥ là số nguyên tố
Khi đó
Ví dụ 19 (Hướng tới kì thi VMO và Olimpic Toán) Cho p là số nguyên tố và a, b, c là 3 số nguyên bất kì. Chứng minh rằng tồn tại các số nguyên x, y, z không đồng thời chia hết cho p sao cho
222 axbycz ++
chia hết cho p.
Lời giải (Tham khảo từ một bạn trên Group Hướngtới kì thi VMO và Olimpic Toán)
TH1: Nếu một trong ba số a,b, c chia hết cho p thì bài toán hiển nhiên đúng.
Bởi vì, chẳng hạn c ⋮ p, ta chỉ việc chọn (x, y, z) = (p, p, 1) thỏa mãn.
TH2: Cả ba số a, b, c cùng không chia hết cho p, chọn z = 1. Bài toán trở thành 22 (mod)axbycp +≡−
có nghiệm.
Do có 1 2 p + thặng dư bình phương (mod p) nên 22 , axcby đều có 1 2 p + số dư khi
chia cho p khi x và y chạy khắp ℤ .
Nếu các số dư của ax2 luôn khác số dư của –c – by2 khi chia cho p thì vô lý vì khi đó có p + 1 số dư phân biệt.
Do đó phải tồn tại x, y ∈ Z sao cho 22 (mod)axcbyp ≡−− .
Đến đây ta có điều phải chứng minh.
Nên
++
pap paap paa L
+=++
++
p
2
4
224222 2121(21)(21)[21,21] papaapap ++ +>+>++>++
Suy ra (21)(21) [21,21] 3 ap ap L ++ >++=
Suy ra ()1 (21)(21) ([21,21])2([21,21])2.4 2 ap apap Ta dd dLd + ++ ++≤++=≥
Từ đó (*) đúng với T(a) = k + 1.
Nhận xét: Thực chất, bài toán này chỉ là một hệ quả trực tiếp của định lý Cauchy –
Davenport!
Ví dụ 20
a) Cho x0, k ∈ ℤ, k ≥ 1 và p là số nguyên tố với 0 1(mod)xp ≡ / Đa thức 111 2(1). ()1.... kkk ppppPxxxx =++++
Giả sử q là một ước nguyên tố của P(x0), chứng minh rằng q ≡ 1 (mod pk).
b) Chứng minh rằng tồn tại vô số số nguyên tố dạng pkt + 1.
Lời giải
a) Ta có 00 ()|1. k p Pxx
Gọi 0 () q hordx = h | pk h = pn với n ≤ k
Nếu n ≤ k – 1 thì 1 0 1(mod) k p xq ≡ P(x0) ≡ p (mod q)
Mà P(x0) ≡ 0 (mod q) p = q
20
Mà theo định lý Fermat nhỏ 1 0000(mod)(mod) k pp xxpxxp ≡ ≡
Do đó x0 ≡ 1 (mod p) trái với giả thiết
Do đó n = k tức là h = pk
Từ đó suy ra q – 1 ⋮ pk q ≡ 1 (mod pk).
b) Giả sử có hữu hạn số nguyên tố dạng pkt + 1 là q1, q2, …, qm
Xét x0 = q1 q2 qm + 1
Ta có x0 ≡ 1 + 1 ≡ 2 (mod p) 0 1(mod)xp ≡ /
Gọi q là ước nguyên tố của P(x0), q luôn tồn tại vì P(x0) > 1, theo phần a) thì ta có 1(mod) k qp ≡ .
Do đó, tồn tại qi, i = 1, 2, …, m sao cho p = q .
Suy ra x0 ≡ 1 (mod q) P(x0) ≡ p (mod q) p = q vô lý do 1(mod) k qp ≡
Vậy điều giả sử là sai suy ra có vô số số nguyên tố dạng pkt + 1.
Nhận xét: Đây là một trường hợp đặcbiệt của định lý Dirichlet sau:
“Cho a, blà hai số nguyên dương nguyên tố cùng nhau. Khi đó có vô hạn số nguyên tố có dạng an + b”. Tuy nhiên, hiện nay chưa có chứng minh sơ cấp nào cho địnhlý trên. Áp dụng bài toán này ta có thể dễ dàng giải quyết bài toán sau trên tạp chí Toán học và tuổi trẻ
Ví dụ 21 (Bình Dương 2020) Cho p là một số nguyên tố lẻ. Giả sử
222 122 (1)1.... p p p xaxaxax +=++++
a) Chứng minh rằng 1 (1)(1) k k ak + +−+ chia hết cho p với mọi k = 1, 2, …, p – 2.
b) Khi chia a1, a2, …, ap -2 cho p ta được bao nhiêu số dư phân biệt? Vì sao?
Lời giải
a) Ta có
11
b) Theo phần a) ta có
(1)(1)(mod) 1(mod),21. k k kpkn akp pkpkn += ≡−+≡ −−=+
1(mod),2
Do đó với mọi m, n ∈ {1, 2, …, 3 2 p } thì 22 (mod). nm aap ≡
Và với mọi m, n ∈ {0, 1, 2, …, 3 2 p }thì m, n ∈ {1, 2, …, 3 2 p }
Do mọi số dư của ak chia p đều là số lẻ nên chọn được đúng 1 2 p số dư
Ví dụ 22. Cho hai số nguyên a, b và n là số nguyên dương. Chứng minh rằng n! là ước của số 1 ()(2)...((1)). n Abaababanb =+++−
Lời giải Nếu n = 1, bài toán hiển nhiên đúng.
Nếu n ≥ 2.
Gọi p là một số nguyên tố bất kì, p ≤ n ta phải chứng minh vp(n!) ≤ vp(A).
TH1: p | b vp(bn - 1) = (n - 1)vp(b) ≥ n – 1
Mà 111 (!) 22 p kkk kkk ∞∞∞ ===
vp(n!) ≤ n – 1 ≤ vp(bn - 1) ≤ vp(A)
TH2: (p, b) = 1.
Gọi k = vp(n!) ta chứng minh B = a(a+b)(a+2b) …[a + (n - 1)b] ⋮ pk .
Thật vậy, do (p, b) = 1 nên tồn tại c nguyên sao cho bc ≡ 1 (mod pk)
kkk kp k k k k
(1)(1)(1)(1)
2 1 1 1
Ta có Bcn = ac(ac+bc) … [ac + (n-1)bc] ≡ ac(ac+1)(ac + 2) … (ac + (n - 1)) Do ac(ac+1)(ac + 2) … (ac + (n - 1)) ⋮ n! nên ac(ac+1)(ac + 2) … (ac + (n - 1)) ⋮ pk
akCk pppk k k k k k k kp k
+−+=+−+ =+−+ ≡+−+ −+ ≡+−+≡
Suy ra Bcn ≡ 0 (mod pk) B ≡ 0 (mod pk)
Từ đó ta có điều phải chứng minh.
++ + + + Từ đó có đpcm. 22
Ví dụ 23 (NICE MO 2021). Với mỗi số nguyên tố p , kí hiệu { }1;2;...;1 p Sp=−
Tìm tất cả các số nguyên tố p sao cho tồn tại hàm : ppfSS →
thỏa mãn .().(())1 nfnffn là bội của p với mọi p nS ∈
Lời giải
Ta chứng minh f là một đơn ánh. Thật vậy:
Giả sử tồn tại , p abS ∈ sao cho ()() fafb = . Khi đó ( ) ( ) ( ) ( ) ffaffb = .
Theo bài ra ta có ( ) ( ) ( ) ( ) .().1.().10(mod). afaffabfbffbp −≡−≡
( ) ( ) ( ) ( ) .()..().(mod). afaffabfbffbp ≡
Lại có ()() fafb = và ( ) ( ) ( ) ( ) ffaffb = .
Mặt khác ()fa và ( ) ( ) ffa đều không chia hết cho p (do tập giá trị của hàm f là p S ).
Do đó (mod)abp ≡
Nhưng , p abS ∈ nên ab = . Do đó f là một đơn ánh.
Nhưng tập nguồn và tập đích của hàm f đều là p S có hữu hạn phần tử nên f phải là
một song ánh. Từ đây ta có { }(1);(2);...;(1) p fffpS −≡
và
( ) ( ) ( ) { } (1);(2);...;(1) p ffffffpS −≡ .
Do đó () () ()
Thử lại thấy 2 p = là giá trị thỏa mãn.
Ví dụ 24(IMO 2018) Cho 12,,...aa là dãy vô hạn các số nguyên dương. Giả sử tồn tại số nguyên dương N đủ lớn sao cho
aaaa aaaa ++++
121 231 ... nn n
là số nguyên với mọi nN ≥ . Chứng minh rằng ( ) n a là dãy dừng.
Lời giải Đặt
121 231 ... nn n n
Saaaa aaaa =++++
Ta có: 1 , nn SS + ∈ℕ nên 1 1 11
nnn nn n
SSaaa aa + + +
−=+∈ ℤ với mọi nN ≥ ( ) ()()
+−
11111 11111
aaaaaaa aaaaaa +++ +++
| |. nnnnn nnnn
Ta chứng minh bổ đề sau: nếu |()() xyyxyz thì ( ) ( ) { } ( ) ( ) ( ) { } Min,Max, ppppp xzyxz υυυυυ ≤≤
với mọi số nguyên tố p .
Thật vậy
+) Nếu ( ) ( ) ( ) { } Min, pppyxzυυυ < thì:
( )( ) ( ) ( ) ( ) ( ) ( ) 2 ppppp yxyzyxyxy υυυυυ −−=<+= (vô lý).
+) Nếu ( ) ( ) ( ) { } >Max, pppyxzυυυ thì:
( )( ) ( ) ( ) ( ) ( ) ( ) ( ) pppppp yxyzxzxyxy υυυυυυ −−=+<+= (vô lý).
Bổ đề hoàn toàn được chứng minh.
Trở lại bài toán
1 3
≡=− ∏
1 1.().1!(mod). p i ififfipp =
Theo định lý Wilson ta có () [] () 3 1!11!1(mod). pppp
−≡− −≡−
Vậy 11(mod) p ≡− suy ra 2 p = .
Ta có ( )( ) 11111 | nnnn aaaaaa +++
Nên theo bổ đề trên suy ra
Do đó
111 Min,Max,ppnpnppn aaaaa υυυυυ + ≤≤
( ) ( ) { } ( ) ( ) ( ) { }
( ) ( ) 111gcd;|gcd;nn aaaa + ( ) ( ) 111 lcm;|lcm;nn aaaa + ( ) ( ) 111 gcd;||lcm;nn aaaaa
Từ đây ta thu được ( ) { } 1 gcd; n aa là dãy số nguyên dương không giảm và bị chặn trên
bởi 1a ; ( ) { } 1 lcm; n aa là dãy số nguyên dương không tăng và bị chặn dưới bởi 1a . Do
đó ( ) { } 1 gcd; n aa và ( ) { } 1 lcm; n aa là các dãy dừng.
Tức là tới một lúc nào đó, ta luôn có ( ) ( ) 111 gcd;.lcm;. nnn aaaaaa = là một hằng số, suy ra ( ) n a là dãy dừng.
Đó chính là điều phải chứng minh.
Ví dụ 25 (CGMO 2019) Cho p là một số nguyên tố thỏa mãn 2019 |21 p . Xét dãy
==
aa p aaan +−
=+∀≥
Chứng minh rằng: với mọi n thì ( )1 n ap + ⋮
Lời giải
Ta chứng minh bằng quy nạp mệnh đề sau: 1 (mod), 2 n n a ap + ≡ với mọi n ∈ ℕ .
+)Với 0 n = : 0 1 1(mod) 2 a ap =≡ (đúng).
+) Giả sử mệnh đề trên đúng tới 0 nk=≥ , tức là 1 (mod). 2 k k a ap + ≡
Ta cần chứng minh 1 2 (mod). 2 k k a ap + + ≡
Thật vậy ( ) 2 2 1 1 21 21 1 424 kk k kkk apa pa aaa + + ++
+− =+=+ . Vì
Suy ra 1 2 (mod). 2 k k a ap + + ≡ Mệnh đề được chứng minh.
Bây giờ sử dụng liên tiếp mệnh đề trên ta thu được: 11 1 1 2422...(mod). nn n nn aaa ap + ≡≡≡≡=
Ta đi chứng minh với mọi n thì ( )1 n ap + ⋮ .
Thật vậy, giả sử 1 11 121 110(mod). 22 n n nn ap + +≡+≡≡
Khi đó 122 21(mod)21(mod). nn pp ≡− ≡
Đặt ord2 p h = thì |22hn và |2019 h
Vì 2019 là số lẻ nên |1hn
Nhưng khi đó thì 1 21(mod), n p ≡ và do đó 11(mod)2 pp ≡−⇔= (vô lý).
Vậy ( )1 n ap + ⋮ với mọi n.
Ví dụ 26. Cho m, n nguyên dương, n > 1. Chứng minh rằng {1n, 2n, …, mn} là hệ thặng dư đầy đủ mod m khi và chỉ khi m là số square-free và (n, j(m)) = 1.
Lời giải
Phần thuận: Giả sử {1n, 2n , …, mn} là hệ thặng dư đầy đủ mod m.
Xét
a a impp =
= ∏ (với p, p1, …, pt là các số nguyên tố, a nguyên dương).
1
( ) ( ) 22
1 1 212. kkkkk apapaaap + + +−=+− ⋮
xyp xypp = = = ≡ ≡ ≡ ≡ ∏ ∏ ∏
i xyp xypp
2 1 1 2 2 1 (mod) (mod)(1) (mod) (mod)
26
Do n ≥ 2 Þ (mod) 2 nn a naxym ≥ ≡
Ta có
Do đó
≡=−≡−
==≥ ∏
pp m pp pppp
(do a ≥ 2)
Nếu k ≠ p – 1 thì k = (p - 1)q + r, 1 ≤ r ≤ p – 2, q Î ℕ . (mod)(,)(,)(mod). kr iipSpkSprp ≡ ≡
∏∏
==
ii
2 1 22 11
a a a i a i aatt aa ii ii
Nên từ (1) suy ra tồn tại x ≠ y thuộc (1, 2, …, m) thoả mãn xn º yn (mod m) (vô lý).
Vậy m là số square – free.
Ta có phương trình xr º 1 (mod p) có không quá r nghiệm.
Mà r Î {1, 2, …, p - 2} nên tồn tại a Î {1, 2, …, p - 1} sao cho 1(mod). r ap ≡
Vì (a, p) = 1 nên 1 1 {}p i ia = là hệ thặng dư thu gọn (mod p).
Suy ra
1 i i mp =
= ∏ +) Nếu (n, j(m)) = d > 1 Þ tồn tại số nguyên tố q | d Þ q | pi – 1, với i nào đó.
Bổ đề: Phương trình x3 º 1 (mod pi) có ít nhất q nghiệm.
Thật vậy phương trình 1 1 0(mod) 1 p q
x p x ≡ là phương trình đa thức bậc pi – q – 1 nên có
tối đa (pi – q - 1) nghiệm.
Mà 1 10(mod) p ixp −≡ có đúng pi – 1 nghiệm trên p ℤ nên phương trình xq – 1 º 0 (mod pi) có ít nhất q nghiệm. (Bổ đề được chứng minh)
Gọi a, b là 2 trong số các nghiệm đó.
Theo định lý thặng dư thặng dư Trung Hoa, tồn tại các số tự nhiên n1, n2 sao cho n1 º b (mod pj), j ≠ i; n1 º a (mod pi); n2 º b (mod pi).
Dễ thấy có thể chọn n1, n2 Î {1, 2, …, m} và n1 ≠ n2
Hiển nhiên ta có 1212(mod)(mod)qqnn nnmnnm ≡ ≡ (vô lý)
Vậy phần thuận được chứng minh.
Ví dụ 28 (GGTH) Đặt S(n, k) = 1 1
với n = p là số nguyên tố lẻ thì
n k i i = với n, k nguyên dương, n lẻ. Chứng minh rằng
a) S(p,k) º -1 mod p Û k chia hết cho p – 1.
b) S(p,k) º 0 mod p Û k không chia hết cho p – 1.
Lời giải
Nếu k ⋮ p - 1 thì
1 1 ()1(mod),1,1. k kp p ippip =≡∀=−
Do đó 111 111 (,)()(mod) (1)(,)0(mod).
=≡≡ −≡
Spriiaaip aSprp ===
Do (ar – 1, p) = 1 Þ S(p, r) º 0 (mod p).
Nhận xét. Ta có thể gọi a là một căn nguyên thuỷ mod p thì không cần sử dụng phương trình đồng dư, bạn đọc có thể xem ở lời giải bài sau (Ví dụ 29).
BàitoántrênthựcchấtlàmộtcáchphátbiểukháccủabaitoánHunggary–Israel năm 2009, bạn đọc có thể tìm thấy một lời giải khác trên mathlinks.ro!
Ví dụ 29 (Mạc Đăng Nghị). Đặt S(n, k) = 1 1
minh rằng với n = pm, m nguyên dương thì
n ki = với n, k nguyên dương, n lẻ. Chứng
a) S(pm, k) º 0 mod pm nếu k không chia hết cho p – 1;
b) S(pm, k) º -pm – 1 mod pm nếu k chia hết cho p – 1.
Lời giải (Bài này là trường hợp tổng quát của bài trên)
a) Xét k ⋮ p – 1.
Với p lẻ bất kì. Gọi g là một căn nguyên thuỷ của pm. Khi đó {g, 2g, …(pm - 1)g} và {1,2,…, pm - 1} là hai hệ thặng dư giống nhau (mod pm).
Khi đó ta có
gk + (2g)k + … + ((pm - 1)g)k º 1k + 2k + … + (pm - 1)k Þ (gk - 1)S(pm, k) º 0 (mod pm).
Do j(pm) = pm – 1(p - 1) nên j(pm) ⋮ p – 1.
Lại do g là căn nguyên thuỷ (mod p) nên
gk – 1 º 0 (mod pm) Û k ⋮ j(pm).
Do đo nếu k ⋮ p – 1 thì k ⋮ j(pm) nên
gk – 1 ⋮ pm và (gk – 1, p) = 1
Vậy khi đó S(pm , k) º 0 (mod pm).
b) Xét k ⋮ p – 1.
Ta chứng minh bằng quy nạp.
Nếu m = 1, Theo định lý Fermat nhỏ, ta có
S(p, k) = 1k + 2k + … +(p - 1)k º 1 + 1 + … + 1 º - 1 (mod p).
Giả sử S(pn, k) º - pn – 1 (mod pn).
Ta có
nkknk
Vì A là tích của 4 số nguyên dương liên tiếp nên A luôn có ước số là 2; 3.
Vậy ta xét trường hợp A có đúng 3 ước số là 2; 3; p (p ≥ 5).
+) n lẻ (n ≥ 3).
Khi đó n và n + 2 là 2 số lẻ liên tiếp, nguyên tố cùng nhau nên suy ra n, n + 2 là hai luỹ thừa 3a và pb .
Vì n + 1 chẵn và nguyên tố cùng nhau với n và n + 2 nên n + 1 = 2k , k ≥ 2.
Từ đó suy ra
A = (2k - 1)2k(2k +1)(2k + 2) = 2k+1(2k - 1)(2k +1)(2k-1 +1) (1).
Vì k ≥ 2 nên 3 thừa số sau ở (1) đều lẻ
Suy ra
11 (,)12...(1)
++=+++−
Mặt khác
Suy ra
Spkp ppppp ppppppp
kknknknknknnk nknknnk
12...(1)()[(p+1)(2)...(1)] ...[((1)1)((1)2)...((1)1).
=+++−+++++++−+ +−++−+++−+−
Do p – 1 | k Þ k ≥ 2 Þ kn ≥ n + 1
( (2k +1 , 2k - 1 ) = (2k +1, 2k +1 -2) = (2k +1, -2) = 1
và (2k +1, 2k-1 +1) = (2(2k-1+1) -1, 2k-1 +1) = (-1, 2k-1 +1) = 1.
(,)(1)(2)...(1)
+ = −−−+
1111
(12...(1))(12...(1))(mod)
+
Spkipipipp pkippp pSpkkppSpkp pp pSpkkpSpk
−=
+)
1 1111 (,)(,)(mod)(.).(mod). nnnnnnnnn SpkpSpkpppmppmppp + +−++≡≡−+≡−+≡−
Từ đó có điều phải chứng minh.
Ví dụ 30 (Hà Tĩnh TST 2014) Tìm tất cả các số nguyên dương n sao cho tích n(n+1)(n+2)(n+3)
có đúng 3 ước số nguyên tố. Lời giải
30
Điều này không xẩy ra vì a, c ≥ 1 mà 3 không chia hết cho p
Vậy với n lẻ chỉ có n = 3 thoả mãn.
+) n chẵn ( n ≥ 2). Khi đó (n + 1), (n + 3) là hai số lẻ liên tiếp nguyên tố cùng nhau suy ra (n + 1), (n + 3)
là hai luỹ thừa 3a và pb .
Vì n + 2 chẵn, nguyên tố với n +1 và n + 3 nên n + 2 = 2k với k ≥ 2.
Khi đó
A = (2k - 2)(2k -1)2k(2k + 1) = 2k+1( 2k-1-1)(2k -1)(2k +1) (2)
+) Nếu k = 2 thì 2k-1 - 1= 2 - 1 = 1 và n = 2k - 2 = 22 - 2 = 2, thì A = 2.3.4.5 = 23.3.5 có
đúng 3 ước số nguyên tố
+) Nếu k ≥ 3 thì 3 thừa số sau ở (2) đều là số lẻ, mặt khác ( 2k -1, 2k +1) = 1, (
2k -1, 2k-1 -1 ) = (2(2k-1-1) +1, 2k-1- 1) = 1.
Khi đó chỉ có thể xảy ra một trong hai trường hợp sau:
+) Hoặc
Suy ra
Lời giải Từ tính chất duy nhất trong phân tích số nguyên dương thành thừa số nguyên tố ta có:
nếu vk chia hết cho u, thì (glc(u,v))k chia hết cho u. (1)
Dễ thấy các bộ (1, m, n ) và (a, 1, n) với mọi số nguyên dương a, m, n là các nghiệm
của bài toán.
Bổ đề: Nếu a > 1 và (a +1)n chia hết cho am +1 thì m lẻ
Chứng minh bổ đề
Nếu m chẵn thì am + 1 ≡ 2 (mod (a+1)), nên (am + 1, a +1) bằng 1 hoặc 2.
Từ (1) suy ra 2n ⋮ am + 1 Þ am +1 = 2s
Ta có s ≥ 2 vì a > 1. Khi đó am = 2s - 1 ≡ - 1(mod 4), điều này không xẩy ra vì am là một số chính phương.
Như vậy a > 1 và m là số nguyên dương lẻ lớn hơn 1 và hiển nhiên n > 1.
+) Hoặc
1 1
213 21 212
ka kb kc p −= += +=
với a, b, c nguyên dương.
3c = 2k + 1 = 2.3a + 3 Û 3c – 1 = 2.3a – 1 + 1
Û c – 1 = 2; a – 1 = 0 Û c = 2; a = 1
Gọi p là số nguyên tố lẻ sao cho m = pr, b = ar
Vì r lẻ, b + 1 chia hết cho a + 1, nên (b+1)n chia hết cho bp +1 = am + 1.
Từ đó suy ra (b +1)n -1 ⋮ B = (bp + 1)/(b + 1).
Theo (1) thì (B, b+1)n-1 chia hết cho b.
Vì p lẻ và từ công thức nhị thức suy ra
Khi đó n = 6
p −= += += với a, b, c nguyên dương.
1 1
ka kb kc
21 213 212
Suy ra pc = 2k + 1 = pa + 3.
Điều này không xẩy ra vì a, c nguyên dương, p > 3.
và p nguyên tố, p > 3.
Vậy với n chẵn thì chỉ có n = 2, n = 6 thoả mãn.
Vậy n = 2, n = 3, n = 6.
Ví dụ 31 (Đề KT đội tuyển Hà Tĩnh 2015). Tìm tất cả các bộ ba các số nguyên dương (a, m, n) sao cho
1(11)1 (mod(1)). 11 ppbb pb bb ++−+ =≡+ ++
Vậy số nguyên tố p chia hết cho (B, b +1) suy ra (B, b+1) = p.
Từ đó pn-1 chia hết cho B, suy ra B là luỹ thừa của p.
Ta có bp +1 chia hết cho p do đó (b+ 1)n chia hết cho p, cho nên b + 1 ⋮ p Þ b = kp -1.
Áp dụng công thức nhị thức ta có: bp +1 = (kp - 1)p +1 = ((kp)p -…-
2 11 (mod) 1 ppbb Bpkp bkp ++ ==≡ +
222223 ()1)(mod) p Ckpkpkpkp +−≡
Điều này chứng tỏ b chia hết cho p nhưng không chia hết cho p2
Vì B là một luỹ thừa của p, suy ra B = p
Trường hợp nếu p ≥ 5 thì
121222 1 ...1(1)2. 1 p pppppp b bbbbbbbp b + =−+−+>−=−≥> +
Từ đó ta có:
p = 3, B = p Þ b2 - b +1 = 3 Þ ar = b = 2 Þ a = 2 và m = p = 3.
Vậy (2, 3, n) là nghiệm của bài toán với n ≥ 2
Vậy nghiệm của bài toán là: (1, m, n); (a, 1, n) với a, m, n * N ∈ ; (2, 3, n)
với n ≥ 2, n Î ℕ .
Chương 2. Thặng dư bậc hai
Phần này, bản thân tôi cũng đã báo cáo trong chương trình bồi dưỡng giáo viên chuyên Toán, năm đầu tiên do Viện NCCC về Toán tổ chức. Qua đó, cũng nhận được khá nhiều góp ý của các bạn đồng nghiệp và chuyên đề cũng đã được chỉnh sửa nhiều cho phù hợp với thực tiễn giảng dạy học sinh chuyên toán hiện nay.
2.1.
Lí
thuyết
2.1.1 Các định nghĩa Định nghĩa 1
Cho số nguyên tố p. Số nguyên a được gọi là số chính phương (mod p) nếu tồn tại số nguyên x sao cho x2 ≡ a ( mod p).
Nhận xét:
+) Mọi số chính phương đều là số chính phương (mod p)
+) a ≡ 0 (mod p) thì a2 ≡ a (mod p) nên mọi a ≡ 0 (mod p) đều là số chính phương (mod p). Do đó, từ đây về sau, ta chỉ xét số nguyên a sao cho (a, p) = 1.
+) Mọi số nguyên lẻ đều là số chính phương (mod 2)
Định nghĩa 2 ( Kí hiệu Legendre)
Cho p là số nguyên tố lẻ. 0 a p = , nếu a ≡ 0 (mod p). 1 a p = nếu a là số chính phương mod p
a p
= -1 nếu a không là số chính phương mod p.
Kí hiệu trên gọi là kí hiệu Legendre.
2.1.2 Các định lí Định lý 1
Cho p là một số nguyên tố lẻ. Khi đó
1 2 11(mod) p a ap p =⇔≡ (1)
1 2 11(mod) p a ap p =−⇔≡− (2)
Chứng minh
Giả sử a là số chính phương mod p, khi đó, tồn tại số tự nhiên x sao cho
x2 ≡ a (mod p)
Do (a, p) = 1 nên (x, p) = 1.
Theo định lí Fermat ta có
1 1 2 1(mod) p p xap≡≡
Ngược lại, nếu có (1) thì với mỗi k ∈ {1, 2, …, p - 1} có duy nhất một số '{1,2,...,-1}kp ∈ sao cho k.k’ ≡ a (mod p)
Nếu tồn tại k = k’ thì k.k’ = k2 ≡ a (mod p) a là số chính phương (mod p).
Trái lại, tập {1, 2, …, p - 1} được chia thành 1 2 k tập con {k, k’} rời nhau sao cho k.k’
≡ a (mod p). (p - 1)! ≡
1 2 1(mod) p ap ≡
Mặt khác, theo định lí Wilson, ta có (p - 1)! ≡ −1 (mod p)
Từ đó suy ra
1 ≡ −1 (mod p) = 2 (vô lí vì p lẻ)
Vậy (1) được chứng minh xong.
Theo định lí Fermat, ta có ap – 1 ≡ 1 (mod p)
Do đó, (2) được chứng minh.
Định lí 2 (Bổ đề Gauss)
(n chính là số bội số của a trong khoảng ( ; 2 p p ))
Chứng minh
Ta có ka ≡ rk (mod p)
Cho k chạy từ 1 đến (p - 1)/2 rồi nhân các đẳng thức đó lại ta được: 1 1 2 2 1
1 2 1 2
p p ≡ ≡−
1(mod) 1(mod)
ap ap
Cho p là số nguyên tố lẻ, a là số nguyên, (a, p) = 1. Xét tập {ka| k = 1, 2, …, 1 2 p }.
Gọi rk ≡ ka (mod p), 1 ≤ rk ≤ p
Gọi A = | 2 kk p rr > ; B = | 2 kk p rr <
p p k k
p arp = ∏
≡
p n kkkkk krArBrArB rrrprrp =∈∈∈∈ ∏∏∏∏∏
Ta có với mọi rk ∈ A thì p – rk ≤ 1 2 p
Mặt khác Nếu ri ∈ A, rj ∈ B thì p – ri ≠ rj vì nếu p – ri = rj thì ri + rj = p
a(i + j) ≡ 0 (mod p) i + j ≡ 0 (mod p).
Điều này không thể vì i và j thuộc 1 (1;) 2 p nên 1 < i + j < p .
Từ đó suy ra 1 1 2 2 1
đpcm.
11 !(1)! 22
p p n k k
pp ar =
≡≡− ∏ (*)
Mà 1 (!,)1 2 p p = nên từ (*) suy ra 1 2 (1)(mod) p n ap ≡−
Từ đó ta có một số hệ quả quan trọng sau.
Hệ quả 1. Cho p là số nguyên tố lẻ. Gọi n là số các số chẵn thuộc ( ; 2 p p ) thì ta có 1 2 2(1)(mod) p n p ≡− .
Chứng minh
Từ định lí 2, cho a = 2 ta có điều phải chứng minh.
36
Hệ quả 2. Cho p là số nguyên tố lẻ. Khi đó
1 2 1 ()(1) p p =−
Chứng minh
(Hiển nhiên)
Hệ quả 3. Cho p là số nguyên tố lẻ. Khi đó 2 ()1 p = ⇔ p ≡ ± 1 (mod 8)
và 2 1 8 2 (1) p p =−
Chứng minh
Giả sử p = 4k + 1, tập các số chẵn trong khoảng (0; p) là {2i | 1 ≤ i ≤ 2k}
Khi đó, số các số chẵn trong khoảng ( ; 2 p p ) là n = k
Tương tự như vậy, nếu p = 4k – 1 thì ta cũng tính được số các số chẵn trong khoảng ( ; 2 p p ) là n = k.
Do đó, theo hệ quả 1, ta có
1
2 2(1)(mod) p n p ≡− ≡ (-1)k (mod p)
2 là số chính phương mod p ⇔
1 2 12(1)(mod) p k p ≡≡−
⇔ k chẵn ⇔ p ≡ ± 1 (mod p).
2 không là số chính phương mod p ⇔ k lẻ ⇔ p ≡ ± 3 (mod p) đpcm.
Hệ quả 4. Cho p là số nguyên tố lẻ. Gọi n là số các bội của 3 trong khoảng ( ; 2 p p
). Khi đó
1
2 3(1)(mod) p n p ≡−
Chứng minh
Được suy ra hiển nhiên từ định lí, trong trường hợp a = 3.
Hệ quả 5. Cho p là số nguyên tố có dạng 6k ± 1. Khi đó, 3 là số chính phương mod p khi và chỉ khi p ≡ ± 1 (mod 12).
Chứng minh
Giả sử p = 6k + 1. Tập hợp các số là bội của 3 trong khoảng (0; p) là {3i | 1 ≤ i ≤ 2k}
Mà
3i > 1 1 266 ppikik ⇔>=+⇔≥+
Do đó, số các số là bội của 3 trong khoảng ( ; 2 p p ) là n = k
Tương tự khi p = 6k – 1, ta cũng chứng minh được n = k.
Vậy trong cả hai trường hợp ta đều chứng minh được số các số là bội của 3 trong khoảng ( ; 2 p p ) là n = k.
Vì vậy, theo hệ quả 4, ta có
1 2 3(1)(mod) p n p ≡− ≡ (-1)k (mod p)
Định lí 3
3 ()1 p = ⇔ (-1)k ≡ 1 (mod p) ⇔ k chẵn ⇔ p ≡ ± 1 (mod 12). đpcm.
Cho p là số nguyên tố. a ≡b (mod p), (a, p) = (b, p) = 1. Khi đó
()() ab pp =
Chứng minh
Do a ≡ b (mod p) 11 22 (mod) pp abp ≡ ()() ab pp =
đpcm.
Định lí 4
Khi đó
Cho p là một số nguyên tố. a1, a2, …, an là các số nguyên không chia hết cho p.
1212 ()()()...() nn aaaa aa pppp =
Chứng minh
(Phần chứng minh khá đơn giản, xin nhường cho bạn đọc)
Định lí 5
Cho p là số nguyên tố lẻ, a là số nguyên không chia hết cho p. Khi đó
1 2 (1)(mod) p s ap ≡−
Chứng minh
p k
(tính chất này đã được chứng minh trong tài liệu “Một số bài toán về phần nguyên ” của cùng tác giả).
Do p hoăc q có dạng 4k + 1 nên 11 ()() 22 pq là số chẵn.
Nên s1 và s2 có cùng tính chẵn lẻ.
Theo định lí 5, ta có
ka p =
Gọi n là số các bội của a trong khoảng ( ; 2 p p ), theo định lí 2, ta có
1 2 (1)(mod) p n ap ≡−
Vì vậy, ta chỉ cần chứng minh s – n là số chẵn.
(Phần này khá đơn giản, xin nhường lại cho bạn đọc).
Định lí 6 (Luật tương hỗ Gauss)
Cho p, q là hai số nguyên tố lẻ phân biệt.
Khi đó:
a) Nếu có ít nhất một trong hai số có dạng 4k + 1 thì
()() pq qp = .
b) Nếu cả hai số có dạng 4k + 3 thì
()(). pq qp =−
Chứng minh
a) Đặt s1 =
Khi đó
1 2 (1)(mod) p s qp ≡−
1
2 2 (1)(mod) q s pq ≡−
Từ đó dễ dàng suy ra điều phải chứng minh.
b) Chứng minh tương tự.
2.2. Các ví dụ
2.2.1 Các ví dụ cơ bản
Phần này, chúng tôi cố gắng lựa chọn các ví dụ đơn giản nhất về thặng dư bậc hai. Giúp học sinh có hình dung cơ bản về định hướng sử dụng thặng dư bậc hai trong các bài toán số học.
Ví dụ 1. Chứng minh rằng mọi ước nguyên tố của số m = 2n – 1, (n lẻ) đều có dạng p = 8k ± 1.
Lời giải
Ta có p | 2n – 1 2n ≡ 1 (mod p) ⇔ (2m)2.2 ≡ 1 (mod p).
⇔ (2m + 1)2 ≡ 2 (mod p) ⇔ 2 1 p = ⇔ p = 8k ± 1.
Từ đó có điều phải chứng minh.
Ví dụ 2 (JBMO 2007). Cho p là số nguyên tố. Chứng minh rằng số 7p + 3p – 4
không là số chính phương.
iqip s pq==
11 22 2 11 ; pq ii
Lời giải
TH1: p = 2 ta có 7p + 3p – 4 = 19 không là số chính phương.
TH2: p lẻ. Giả sử 7p + 3p – 4 = x2 với x nguyên dương.
Ta có 3p ≡ 3 (mod p) 7p + 3p – 4 ≡ -1 (mod p).
40
Suy ra 1 1 p =
p ≡ 1 (mod 4).
Xét (mod 4), ta có 7p + 3p – 4 ≡ 7 + (-1)p ≡ 2 (mod 4) x2 ≡ 2 (mod 4), vô lí vì một
số chính phương chia 4 không thể dư 2.
Từ đó ta có điều phải chứng minh.
Ví dụ 3. Cho p là số nguyên tố lẻ. Khi nào p – 2 là số chính phương (mod p)?
Lời giải
Ta có
===−−=−
−−+
Ví dụ 4. Cho p = 8n – 1 và q = 4n – 1 là các số nguyên tố. Chứng minh rằng -2 là một căn nguyên thủy (mod p).
Lời giải
Ta có
p −=−≡
nên
2 p
Nếu – 2 không là căn nguyên thủy (mod p) thì tồn tại r sao cho 0 < r < p – 1 và (-2)r ≡ 1 (mod p) r | 2q r = 2.
Mà 1 ≡ (-2)2 ≡ 4 (mod p) p = 3 (không thỏa mãn).
Vậy có điều phải chứng minh.
Ví dụ 5. Cho p là số nguyên tố có dạng 8k + 5 hoặc 8k + 7. x, y là hai số nguyên thỏa
mãn x2 + 2y2 chia hết cho p.
Chứng minh rằng x ⋮ p và y ⋮ p.
Lời giải
2y2 ⋮ p y ⋮ p
22 2 ()() xy pp =
1 = 2 2212 ()()()()() y ppppp == . (1)
Nếu p = 8k + 5 2
p p
p p
1 2 1 8
1 ()(1)1 2 ()(1)1.
=−= =−=−
Thay vào (1) được 1 = -1 (vô lí).
Nếu p = 8k + 1 2
Từ đó suy ra đpcm.
p p
Vậy điều giả sử là sai
p p
1 2 1 8
1 ()(1)1 2 ()(1)1
=−=− =−=
cũng không thoả mãn (1)
Ví dụ 6. Cho p là số nguyên tố lẻ. Chứng minh rằng nếu tồn tại các số nguyên x, y sao cho p = x2 + 2y2
thì p ≡ 1 (mod 8) hoặc p ≡ 3 (mod 8).
Lời giải
Dễ thấy cả x và y đều không chia hết cho p
Ta có x2 ≡ -2y2 (mod p)
===
xy ppp
Ví dụ 7. Cho p là số nguyên tố lẻ và p ≡ 1 hoặc 3 (mod 8). Chứng minh rằng luôn tồn tại các số nguyên x, y sao cho p = x2 + 2y2 .
Lời giải
Đặt f(u, v) = u + rv và kp =
Khi đó 1 kpk <<+ do p là số nguyên tố
Xét tất cả các cặp (u, v) với 0 ≤ u ≤ k và 0 ≤ v ≤ k
Có tất cả (k + 1)2 cặp phân biệt.
Mà (k + 1)2 > p nên theo nguyên lý Dirchlet tồn tại (u1, v1) ≠ (u2, v2) sao cho
f(u1, v1) ≡ f(u2, v2) (mod p).
⇔ u1 + rv1 ≡ u2 + rv2 (mod p) ⇔ (u1 – u2) ≡ - r(v1 – v2) (mod p).
Hay tồn tại a, b nguyên không đồng thời bằng 0 sao cho
a ≡ -rb (mod p) a2 ≡ r2b2 (mod p) a2 ≡ -2b2 (mod p).
⇔ a2 + 2b2 ≡ 0 (mod p).
Mà 0 ≤ |u1 - u2| ≤ k và 0 ≤ |v1- v2 | ≤ k nên a2 + 2b2 ≤ 3k2 < 3p.
Từ đó suy ra a2 + 2b2 = p hoặc a2 + 2b2 = 2p.
Nếu a2 + 2b2 = 2p thì a phải là số chẵn hay a = 2c suy ra b2 + 2c2 = p.
Từ đó suy ra điều phải chứng minh.
Ví dụ 8. Chứng minh rằng mọi ước nguyên tố lẻ, lớn hơn 3 của số n = x2 + 3y2 , x, y
nguyên dương ( x ⋮ p, y ⋮ p) đều có dạng p = 6k + 1.
Lời giải
Ta có x2 ≡ - 3y2 (mod p) 22313 1()()() xy pppp
Nếu p = 12k + 5 thì 13 1,1 pp
Nếu p = 12 k + 11 thì 13 1,1 pp
Nếu p = 12k + 1 thì 13 1,1 pp
nên trường hợp này không thỏa mãn.
nên trường hợp này không thỏa mãn.
(thỏa mãn).
Nếu p = 12k + 7 thì 13 1,1 pp
(thỏa mãn).
Như vậy cả hai trường hợp sau đều dẫn đến p = 6k + 1. Từ đó có điều phải chứng minh.
Ví dụ 9. Cho p > 3, p là số nguyên tố thỏa mãn p = x2 + 3y2 với x, y nguyên. Chứng minh rằng p ≡ 1 (mod 6).
Chứng minh tương tự ví dụ trên.
Ví dụ 10. Cho p, q là các số nguyên tố lẻ. a không là số chính phương (mod p). Chứng
minh rằng phương trình x2 ≡ a (mod pq) vô nghiệm.
Lời giải
Giả sử phương trình trên có nghiệm y.
Khi đó y2 ≡ a (mod pq) pq | y2 – a p | y2 – a y2 ≡ a (mod p)
Từ đó suy ra a là số chính phương (mod p) (trái với giả thiết).
Vậy có điều phải chứng minh.
Ví dụ 11. Chứng minh rằng 2 không là căn nguyên thủy của một số nguyên tố dạng p = 3.2n + 1
trừ trường hợp p = 13.
Lời giải
TH1 : p = 7 thì 23 = 8 ≡ 1 (mod 7) nên 2 không là căn nguyên thủy (mod 7).
TH2: p = 13 thì dễ dàng kiểm tra được 2 là một căn nguyên thủy của 13.
TH3: n ≥ 3 thì p ≡ 1 (mod 8) 1 2 2 121(mod) p p p = ≡ 2 không là căn nguyên thủy (mod p).
Vậy có điều phải chứng minh.
Ví dụ 11. Cho p là số nguyên tố lẻ, a ∈ ℤ sao cho (a, p) = 1. Chứng minh rằng nếu a là số chính phương (mod p) thì a cũng là số chính phương (mod pn), với mọi n nguyên dương.
Lời giải
Giả sử rằng b2 ≡ a (mod p), chúng ta cần chứng minh tồn tại số x nguyên sao cho
x2 ≡ a (mod pn) (*)
với mọi n nguyên dương.
Ta sẽ chứng minh bằng quy nạp.
Với n = 1, hiển nhiên (*) đúng. Giả sử tồn tại c sao cho c2 ≡ a (mod pn), với n ≥ 1, ta cần chứng minh tồn tại x sao cho
x2 ≡ a (mod pn + 1).
Từ c2 ≡ a (mod pn) suy ra c2 = a + pnk, k nguyên.
Xét tất cả các số có dạng x = c + pni với i nguyên, ta có
x2 = c2 + 2cpni + p2ni2
= a +pn(2ci + k) + p2ni2
≡ a + pn(2ci + k) (mod p)
Ta sẽ chứng minh tồn tại i sao cho 2ci + k ≡ 0 (mod p)
Thật vậy, do p nguyên tố lẻ, c ⋮ p nên (2c, p) = 1 theo định lí Bezout tồn tại các số nguyên y, z sao cho
2cy + pz = 1 2cy ≡ 1 (mod p)
-2cyk ≡ - k (mod p) -2cyk + k ≡ 0 (mod p)
Từ đó chọn i = -yk ta được 2ci + k ≡ 0 (mod p) x2 ≡ a (mod pn + 1)
Vậy có điều phải chứng minh.
Chú ý: Nếu a là số chính phương (mod pn) thì p cũng là số chính phương (mod p)
Ví dụ 12. Cho m là số nguyên dương lẻ, a nguyên dương sao cho (a, m) = 1. Chứng minh rằng a là số chính phương (mod m) khi và chỉ khi a là số chính phương (mod) p với mọi p là ước nguyên tố của m
Lời giải
Giả sử phân tích chính tắc của m là
Ngược lại, nếu 1 a p = thì tồn tại x sao cho x2 ≡ - a (mod p) x2 + a = pt với t
nguyên.
Hay phương trình x2 + py + a = 0 có nghiệm nguyên (x, -t).
Vậy có điều phải chứng minh.
Ví dụ 14. Chứng minh rằng mọi ước nguyên tố của số nguyên có dạng x2 + x + 1, (x ∈
ℤ ) hoặc bằng 3 hoặc có dạng 3k + 1.
Lời giải
Nếu x = 1 thì có p = 3 là ước của x2 + x + 1.
Xét p > 3, ta có x2 + x + 1 ≡ 0 (mod p) ⇔ (2x + 1)2 ≡ -3 ( mod p).
Hay 3 1 p =
Mà 1 2 3133 (1)(). p
= ∏
1 . k i i mp α =
Theo định lí Thặng dư Trung Hoa ta có phương trình x2 ≡ a (mod m) có nghiệm khi và chỉ khi hệ
Nếu p = 12k + 5 thì 3 () p = - 1 1 2
Nếu p = 12k – 1 thì 3 () p = 1 1 2 31333 (1)()()1
Vậy p = 12k + 1 hoặc p = 12k + 7.
Cả hai trường hợp trên đều có dạng 3k + 1.
Từ đó có điều phải chứng minh.
Ví dụ 15. Cho 2 21 n k =+ với n nguyên dương. Chứng minh rằng k là số nguyên tố khi
Mà theo ví dụ trên, phương trình x2 ≡ a (mod pn) có nghiệm khi và chỉ khi phương
trình x2 ≡ a (mod p) có nghiệm. Từ đó có điều phải chứng minh.
Ví dụ 13. Cho p là số nguyên tố lẻ, a là số nguyên sao cho (a, p) = 1. Chứng minh
rằng phương trình x2 + py + a = 0
có nghiệm nguyên (x, y) khi và chỉ khi –a là số chính phương (mod p).
Lời giải
Giả sử phương trình x2 + py + a = 0 có nghiệm nguyên.
Khi đó x2 ≡ -a (mod p) 1 a p =
và chỉ khi k là ước của 1 2 31 k + .
Lời giải
Nếu k là ước của 1 2 31 k + thì ta có 1
2 3 k
Gọi d là bậc của 3 modulo k
≡ -1 (mod k) (1)
⇔ 3k -1 ≡ 1 (mod k) (2)
Từ (1) và (2) ta có d | k – 1 nhưng d lại không chia hết 1 2 k
d = k – 1 k là số nguyên tố
Ngược lại, k là số nguyên tố
Ta có k là số nguyên tố dạng 4l + 1 nên theo luật tương hỗ Gauss ta có
Ví dụ 17. Cho p là một số nguyên tố lẻ. Chứng minh rằng
3 ()()
3 k k =
Mà k ≡ 2 (mod 3) nên 2 ()()1 33 k ==− (do 2 không phải số chính phương mod 3).
Từ đó suy ra
11
22 31(mod)310(mod) kk kk≡−⇔+≡ . đpcm.
Ví dụ 16. Chứng minh rằng với mọi n nguyên dương, số 2n + 1 không có ước nguyên tố dạng
8k + 7.
Lời giải
Giả sử tồn tại số nguyên tố p = 8k + 7 sao cho p | 2n + 1
Nếu n chẵn, ta có
Suy ra -1 là số chính phương mod p
Do đó
2n ≡ -1 (mod p)
1 43 2 1 1()(1)(1)1 p k p + ==−=−=− (vô lí)
Nếu n lẻ, ta có
2n ≡ -1 (mod p) 2n + 1 ≡ -2 (mod p)
Suy ra -2 là số chính phương mod p
Do đó, ta có
212 1()()()1 ppp ===− .1 = -1. (vô lí)
Vậy ta có điều phải chứng minh.
2.2.1 Các ví dụ nâng cao
Trong phần này chúng tôi cố gắng cập nhật các bài toán mới nhất, xuất hiện
trong các đề thi gần đây và hướng giải quyết tinh tế và phức tạp hơn, từ đó giúp học sinh rèn kĩ năng ứng dụng thặng dư bậc hai một cách linh hoạt.
Lời giải Gọi g là một căn nguyên thủy (mod p)
Do đó
i p =
1 1 0. p i
Khi đó ta có {1, 2, ..., p - 1} = {g0 , g1 , ..., gp - 2} (trong mod p)
Từ đó suy ra điều phải chứng minh.
12 10
i pp ii
ig pp =
Dễ thấy 1 g p = nếu i chẵn và i g p = - 1 nếu i lẻ
Ví dụ 18. (IMO Shortlist 2020) Với mỗi số nguyên tố lẻ p cho trước, kí hiệu { }()0;1;2;...;1 p dnp∈− là số dư trong phép chia n cho p . Ta gọi ( ) 012,,,...aaa là một p dãy, nếu ( ) ,1 o ap = và ( ) 1 nnpn aada + =+ với mọi 0 n ≥
a) Tồn tại hay không vô hạn các số nguyên tố p thỏa mãn tìm được hai p dãy ( ) 012,,,...aaa và ( ) 012,,,...bbb sao cho nn ab > với vô hạn n , mm ba > với vô hạn m ?
b) Tồn tại hay không vô hạn các số nguyên tố p thỏa mãn tìm được hai p dãy ( ) 012,,,...aaa và ( ) 012,,,...bbb sao cho 00ab > nhưng nn ab < với mọi 1 n ≥ ?
Lời giải Để thuận tiện cho việc trình bày, ta thay kí hiệu () p dn bởi kí hiệu ()rn .
Vì p là số nguyên tố lẻ nên đặt ord2 p d = thì dễ thấy |1dp .
Với mỗi p dãy ( ) 012,,,...aaa ta thấy ngay ( ) ( ) 0 2n n rara = .
Vì ( ) ( ) 00 2d rara = nên ( ) { } n ra tuần hoàn với chu kì d .
Lại có |1dp nên với mọi n thì () 2 10 0 2 p i npn i aara +− =
−=
Vì 2 là thặng dư bình phương mod p nên
−=
1 0 0 2 d ndn i aara + =
()
là một hằng số với mỗi p dãy ( ) 012,,,...aaa
a) Chọn p là một số nguyên tố thỏa mãn [ ]58 p ≡ . Theo nguyên lý Dirichlet tồn tại vô số số nguyên tố p có dạng này. Ta xét hai p dãy { }n a và { } n b với các số hạng đầu tiên là 0 1 a = , 0 1 b =− và chứng minh chúng thỏa mãn yêu cầu đề bài. Vì 2 không phải thặng dư bình phương mod p nên
1 2 2 21(mod). p p p ≡≡−
1 2 2 21(mod). p p p
Để ý rằng 1 2 p là số lẻ nên không tồn tại n nguyên dương nào thỏa mãn
≡−
Hơn nữa dễ thấy d là ước của 1 2 p nên d là số lẻ
Do đó các số hạng của dãy ( ) { } n ru không trùng với bất cứ số hạng nào của dãy ( ) { } n rv .
+=+=+−=
Do đó () () ()() 1 2 1 2 2 p ipiiii i rarararararap +
với mọi i.
pp rarara + ==
=+=
là một hằng số không phụ thuộc vào 0a . Chứng minh tương tự với dãy { } n b và từ đó suy ra
11 (1) 2 npnnpn pp aabb +−+− −=−= .
Từ đó ta thấy { }nn ab cũng là dãy tuần hoàn với chu kì 1 p . Lại có 00ab > và
11 22 apb =<−= nên (1)(1) kpkpab > với mọi k ∈ ℕ và (1)1(1)1 kpkpab−+−+ > với mọi k ∈ ℕ
b) Chọn p là một số nguyên tố thỏa mãn 7(mod8). p ≡ .
Theo nguyên lý Dirichlet tồn tại vô số số nguyên tố p có dạng này.
Ta xét hai p dãy { }n u và { } n v với các số hạng đầu tiên là 0 1 upA=+ , 0 1 vpB=− với , AB là các số nguyên nào đó.
Ta có
11 10 00 22 dd ii ndn ii Suurur + ==
=−==
()()
;
11 20 00 22. dd ii ndn ii Svvrvr + ==
=−==−
()() 11 12 00 22 dd ii ii SSrrpd ==
là một số lẻ (vì , pd là các số lẻ). Do đó trong hai số 12 , SS có một số lớn hơn số còn lại. Không mất tính tổng quát, giả sử 12SS > .
Thay vào ta được ndnndnndndnn uuvvuvuv ++++ −>− −>− .
Như vậy hiệu { }nn uv là một dãy tăng ngặt.
Bây giờ chọn , AB là các số nguyên sao cho 00uv <
Tồn tại chỉ số M ∈ ℕ thỏa mãn MMuv < và nn uv > với mọi nM > . Xét hai p dãy { }n a và { }n b thỏa mãn 0 Mav = và 0 Mbu = thì đây chính là hai dãy thỏa mãn đề bài.
50
Ví dụ 19 (EMC 2020) Cho p là một số nguyên tố. An và Bình cùng chơi một trò chơi như sau: An viết một số nguyên dương X lên bảng, sau đó đưa cho Bình một dãy số nguyên dương ( ) n n a ∈ℕ . Tới lượt Bình, cậu tiến hành thực hiện một chuỗi các biến đổi theo quy tắc sau: ở lần biến đổi thứ n , xóa số Y đang được viết trên bảng và thay nó
bởi số n Ya + hoặc . n Ya . Bình thắng cuộc nếu như tại một bước nào đó, trên bảng xuất
hiện một số là bội của p . Hỏi rằng bất kể ban đầu An chọn X và ( ) n n a ∈ℕ như thế nào, liệu Bình có thể thắng được hay không nếu:
a. 9 107 p =+
b. 9 109 p =+
Lời giải
a) Câu trả lời là không. Ta sẽ chỉ ra cách An chọn X và ( ) n n a ∈ℕ để Bình không thể thắng.
Kí hiệu ny là số xuất hiện ở trên bảng sau bước thứ n .
Chọn X là một số nguyên dương thỏa mãn 0,1(mod). XXp ≡≡ //
Chọn 1 1 .(1)aXX=− , trong đó ( ) 1 1 X là phần tử nghịch đảo của ( )1 X trong hệ thặng dư thu gọn mod p (lưu ý vì [ ] 10 Xp −≡ / nên 1 X nằm trong hệ thặng dư thu
gọn mod p và do đó có phần tử nghịch đảo 1 (1) X ).
Tại bước thứ nhất: sau khi xóa số X Bình sẽ điền lên bảng số 1Xa + hoặc 1Xa
Ta có 2 1 2 1
Giả sử 2 1(mod). 1 X p X ≡
Suy ra ()2 2 3 11(mod)213(mod)1.XXpXp p −+≡ −≡− =
Điều này vô lý vì p là số nguyên tố chia 3 dư 2.
Do đó 2 1 1(mod). 1 X yp X ≡≡ /
Vì 110,1(mod) yyp ≡≡ // nên có thể chọn ( ) 1 211 1 ayy=−
Chứng minh tương tự như trên thì sau bước thứ hai trên bảng sẽ xuất hiện 2y mà 220,1(mod). yyp ≡≡ //
Tiếp tục quá trình này ta luôn có thể kiểm soát được số dư khi chia cho p của các ny xuất hiện trên bảng sau mỗi bước là duy nhất và đảm bảo rằng 0,1(mod).nn yyp ≡≡ //
Tổng quát hóa quá trình trên chúng ta có dãy ( ) n n a ∈ℕ
c xây dựng như sau:
XX XaXp XX XX XaXp XX
(mod); 11 ..(mod). 11
+≡+= ≡=
Như vậy dù Bình chọn cách nào trong hai cách thì cũng đều thu được số 2 1 1 X y X ≡ Ta
chứng minh 2 0(mod), 1 X p X ≡ / 2 1(mod). 1 X p X ≡ /
Thật vậy
Vì 0,1(mod) XXp ≡≡ // nên hiển nhiên 2 1 0(mod). 1 X yp X ≡≡ /
Trong đó nz là số nguyên được chọn sao cho 2 1 n nn n
a zy a ≡≡
với ny chính là số sẽ xuất hiện trên bảng sau bước thứ n . Sử dụng chứng minh ở trên ta thấy ngay 0(mod). n yp ≡ /
Như vậy Bình sẽ không bao giờ thắng cuộc.
b) Câu trả lời cũng là không.
Ta cũng chỉ ra cách An chọn X và ( ) n n a ∈ℕ để Bình không thể thắng.
Chọn X là số nguyên dương thỏa mãn phương trình đồng dư
2 2210(mod). XXp −+≡
Phương trình này tương đương với 2 (21)1(mod). Xp −≡−
Vì 9 1091(mod4) p =+≡ nên -1 là thặng dư bình phương mod p và do đó phương
trình trên có nghiệm.
Ngoài ra dễ thấy 0,1(mod). XXp ≡≡ //
Chọn 12 , aa giống như phần a. Ta chứng minh 2 (mod).yXp ≡
Tương tự phần a ta có
2 1 (mod). 1 X yp X ≡
Lập luận tương tự để thu được 110,1(mod). yyp ≡≡ // Ta có:
Ví dụ 20. Cho p là số nguyên tố lẻ lớn hơn 3, p ≡ 1 (mod 6). Chứng minh rằng tồn tại
các số nguyên x, y sao cho x2 + 3y2 = p
Lời giải
Vì p ≡ 1 (mod 6) nên 3 1. p =
Khi đó tồn tại r nguyên dương sao cho r2 ≡ -3 (mod p).
Đặt f(u, v) = u + rv và kp =
Khi đó 1 kpk <<+ do p là số nguyên tố.
Xét tất cả các cặp (u, v) với 0 ≤ u ≤ k và 0 ≤ v ≤ k.
Có tất cả (k + 1)2 cặp phân biệt.
2 2 1 2 2 2 1
X yX X a X yXX X
≡≡= −−+
1 11 1 1
Từ đó 4 2 2 (1)(1) X y XXX ≡ −−+ .
Lại có:
Mà (k + 1)2 > p nên theo nguyên lý Dirchlet tồn tại (u1, v1) ≠ (u2, v2) sao cho f(u1, v1) ≡ f(u2, v2) (mod p)
⇔ u1 + rv1 ≡ u2 + rv2 (mod p) ⇔ (u1 – u2) ≡ - r(v1 – v2) (mod p).
Hay tồn tại a, b nguyên không đồng thời bằng 0 sao cho a ≡ -rb (mod p) a2 ≡ r2b2 (mod p) a2 ≡ -3b2 (mod p).
⇔ a2 + 3b2 ≡ 0 (mod p).
Mà 0 ≤ |u1 - u2| ≤ k và 0 ≤ |v1- v2 | ≤ k nên a2 + 3b2 ≤ 4k2 < 4p.
Từ đó suy ra a2 + 3b2 = p hoặc a2 + 3b2 = 2p hoặc a2 + 3b2 = 3p.
Nếu a2 + 3b2 = 2p a và b cùng tính chẵn lẻ
≡ −−+
⇔≡−+−
4 2 332 2
⇔−+≡
(mod) (1)(1) 221(mod) 2210(mod).
X Xp XXX XXXXp XXp
Điều này luôn đúng do cách chọn X . Vậy ta chỉ cần chọn ( ) n n a ∈ℕ thỏa mãn: () () 1 1 1 22
=− =−+ =∀≥ =∀≥
aXX aXXX aak aak +
Nếu a = 2m + 1 và b = 2n + 1 thì a2 + 3b2 ≡ 1 + 3 (mod 4) ≡ (0 mod 4).
Nếu a = 2m, b = 2n thì a2 ≡ b2 ≡ 0 (mod 4) a2 + 3b2 ≡ (0 mod 4).
Mà p lẻ suy ra 2p ≡ 2 (mod 4) vậy trường hợp này không thỏa mãn.
Do đó a2 + 3b2 = p hoặc a2 + 3b2 = 3p.
Nếu a2 + 3b2 = 3p thì a phải chia hết cho 3 hay a = 3c.
Suy ra (3c)2 + 3b2 = 3p ⇔ b2 + 3c2 = p.
2 211 22
1 1 1 2 k k
Thì các số Bình viết trên bảng khi chia cho p sẽ luôn có số dư trùng với số dư của 1y
hoặc 2y và do đó, Bình không thể giành chiến thắng.
Từ đó suy ra điều phải chứng minh.
Ví dụ 21 (MMO 2021) Cho dãy số ( ) * n n x ∈ℕ được xây dựng như sau:
Đặt 2021 a x b = , trong đó , ab là các số nguyên dương nguyên tố cùng nhau. Chứng minh
rằng: nếu p là một ước nguyên tố của a thì 3 p = hoặc 1(mod3) p ≡ .
Lời giải
Ta sẽ chứng minh bằng quy nạp
Thật vậy
1122 221 nn n x =++ với mọi n .
00 22 1 221 x =++
Giả sử mệnh đề đúng tới ( )1 nkk=≥ .
Tức là
1122 221 kk kx =++ .
Ta có
nna Sap p =
+ =
() (,) p n
b) Chứng minh rằng 1 (,)0; p a Sap =
=
c) Chứng minh rằng nếu (a, p) = 1 thì S(a, p) = S(1, p);
d) Chứng minh rằng S(a, p) = -1 nếu (a, p) = 1.
Lời giải
Mà 2 1 n p =
() ( )( ) ( )
111111 2 22222222
1 2221221221221. kkkkkkkk kkk xxx + =−=+++−=+−=++
Vậy mệnh đề trên được chứng minh.
Ta có
20212020
x ++ =++= nên
221 221
20212020 22 221 a =++ .
Đặt 2 1 axx=++ , với 20202 21(mod3) x =≡
Từ đó dễ thấy 3 a⋮ . Gọi 3 p > là một ước nguyên tố của a . Ta có ()
=
=
b) nnanna Sap pp nb nabp p nb pp nbbbp do ppppp
===== == == ====
++ == =+≡ = ===
=++≡ ⇔++≡ ⇔+≡−
2 2 2
axxp xxp xp
10(mod) 4440(mod) 213(mod).
Do đó 3 là số chính phương mod p .
Điều này chỉ xảy ra khi 1(mod12) p ≡ hoặc 7(mod12) p ≡ .
Cả hai trường hợp này ta đều có 13 p ⋮
Ví dụ 22. Cho p là số nguyên tố lẻ, kí hiệu
0 = Mà trong {1, 2, ..., p - 1} có đúng 1 2 p số là số chính phương mod p và 1 2 p không là số chính phương mod p nên 1 1
p b
b p =
= 0. Từ đó có điều phải chứng minh. phần c) và d) xin nhường cho bạn đọc Ví dụ 23. Cho p là một số nguyên tố và p > 3. Chứng minh rằng tổng các thặng dư (mod p) bằng 0 (mod p). Lời giải
Chúng ta biết rằng với mọi p lẻ, luôn có 1 2 p thặng dư (mod p) (chứng minh này dựa
vào căn nguyên thủy).
Đặt 2
+− +
= ℤ
Sk ≤≤− ∃∈≡
. kp aakp
2 11 ,(mod)
Mà k2 ≡ (p - k)2 (mod p) do đó nếu k là một thặng dư (mod p) mà k > 1 2 p thì ta có thể
thay k2 bởi (p - k)2
+ Nếu 3(mod4) p ≡ thì 1 1
Xét hệ thặng dư thu gọn mod p gồm các số { } ,2,3,...,(1) cccpc : trong 1
số này có đúng 1 2 p thặng dư bình phương mod p và 1 2 p số không phải thặng dư
p k
Vậy S ≡ 1 2 2 2 1
pp k =
(1) 24
= (mod p).
Mà p > 3 p2 ≡ 1 (mod 8) và p2 ≡ 1 (mod 3) p2 – 1 ≡ 0 (mod 24)
Vậy 2 1 24 p ∈ ℤ S ≡ 0 (mod p)
Vậy có điều phải chứng minh.
Ví dụ 24 (Baltic Way 2019)
Cho p là một số nguyên tố lẻ. Chứng minh rằng: với mỗi số nguyên c cho trước, tồn tại một số nguyên a thỏa mãn ()
1 1 2 2 (mod). p p aaccp + + ++≡
Lời giải
Nếu cp ⋮ thì ta chỉ cần chọn 0 a =
Nếu cp ⋮ thì ta xét hai trường hợp sau:
TH1: c là thặng dư bình phương mod p , tức 1 c p = , chọn 0 a = .
Khi đó () 111 1 222 2 .(mod). ppp p c aaccccccp p
++− + ++==≡≡
TH2: c không là thặng dư bình phương mod p , tức 1 c p =− , xét hai khả năng:
Chọn 2 i ≥ là số đầu tiên thỏa mãn ic không phải thặng dư bình phương mod p . Khi
đó (1)ic là thặng dư bình phương mod p .
Chọn aic =− thì ta được:
+−− + −+−+≡−−+−−−−
≡−−+−−≡
1 11 1 2 22 2 ()()()(1)(1) 1()(1)(mod).
() () () ()()
p pp p iciccicicicic iciccp
Từ đó ta có điều phải chứng minh.
Ví dụ 25. Tìm x, n nguyên dương sao cho x3 + 2x + 1 = 2n. (1)
Lời giải
Do x nguyên dương nên x3 + 2x + 1 ≥ 4 n ≥ 2.
Nếu n = 2, ta dễ dàng tìm được x = 1.
Xét n ≥ 3
(1) ⇔ x(x2 + 2) = 2n – 1 là số lẻ nên x lẻ
Do đó, x2 + 2 ⋮ 3 2n ≡ (1 mod 3) ⇔ (-1)n ≡ 1 (mod 3).
Từ đó suy ra n chẵn.
Mặt khác
x3 + 2x + 1 = 2n
⇔ x3 + 2x + 3 = 2n + 2.
⇔ (x + 1)(x2 –x + 3) = 2n + 2.
Giả sử p là một ước nguyên tố của x2 – x + 3 p lẻ
Khi đó
2n + 2 ≡ 0 (mod p) ⇔ 2n ≡ -2 (mod p).
Mà n chẵn suy ra 2 ()1 p = .
Ta có 1 = 2 1 1 8 2 212 ()()()(1)(1). p p ppp ==−−
Từ đó suy ra p = 8k + 1 hoặc p = 8k + 3.
Mà n ≥ 3 2n = 8.2n – 3 ≡ 0 (mod 8).
Do đó ta suy ra x3 + 2x + 1 ≡ 0 (mod 8).
Mà x lẻ x = 8k ± 1, x = 8k ± 3.
Nếu x = 8k ± 1 x3 + 2x + 1 ≡ ( ± 1)3 + 2( ± 1) + 1 ≡ 0 (mod 8) (vô lí).
Nếu x = 8k + 3 thì x3 + 2x + 1 cũng không thể chia hết cho 8.
Nếu x = 8k + 5 thì thỏa mãn.
Khi đó x2 – x + 3 ≡ 52 – 5 + 3 ≡ 7 (mod 8).
Hay
x2 – x + 3 = 8l + 7.
Mà số dạng 8l + 7 thì không thể có ước nguyên tố dạng 8k + 1 hoặc 8k + 3.
Do đó (1) vô nghiệm khi n ≥ 3.
Vậy x = 1, n = 2.
Ví dụ 26. Tìm k ∈ℕ sao cho 2 :21|9. k mm ∃−+
Lời giải
Ta có đpcm 2 9( 21) k mmod ⇔≡−− .
Giả sử p| 21 k
*Nếu 3 p ≠ có 1 1 9 pp == nên p4n1 =+ .
Khi p= 4n+1 thì 211(mod4) k −≡ nên k=1
*Nếu p=3 Đặt 2t ks = , s lẻ 21 s không chia hết cho 3. Mà 2 21|9 k m −+ nên
2 21|91 s ms−+ = theo trường hợp trên.
59
Ta sẽ chứng minh với mọi 2t k = thoả mãn (*) bằng cách áp dụng định lý Trung hoa cho bộ số 12 (,,..,) taaa với 1 2 21. i ia =+
Thật vậy: Ta thấy (,)1 uv aa = 1 uvt∀≤≠≤ vì giả sử tồn tại p mà (,). uv aaq =
Giả sử 11 2 2 2. uv k =
11 222 21(2)1 uv k −=− chia hết 1 22 (2)1. v
Mà 111 2222 (2)1(21)(21) vvv −=−+
nên 2 v a chia hết cho u a
2 chia hết cho q suy ra q =1 (mâu thuẫn).
Theo Định lý thặng dư Trung Hoa tồn tại m sao cho
1 0(mod ) ma ≡ 02 2 3.2(mod ) ma ≡ 2 2 3.2(mod ) tma ≡
suy ra 1 229.2(mod ) tma ≡ .
Vậy 221 12 99(21)(mod ...) t tmaaa +≡+
Kết luận 2t k = .
Ví dụ 27. Cho p là số nguyên tố và a, b, c là 3 số nguyên bất kì. Chứng minh rằng tồn tại các số nguyên x, y, z không đồng thời chia hết cho p sao cho
222 axbycz ++
chia hết cho p.
Lời giải
TH1: Nếu một trong ba số a, b, c chia hết cho p thì bài toán hiển nhiên đúng.
Bởi vì, chẳng hạn c ⋮ p, ta chỉ việc chọn (x, y, z) = (p, p, 1) thỏa mãn.
TH2: Cả ba số a, b, c cùng không chia hết cho p, chọn z = 1. Bài toán trở thành
22 (mod)axbycp +≡−
có nghiệm.
Do có 1 2 p + thặng dư bình phương (mod p) nên 22 , axcby đều có 1 2 p + số dư khi
chia cho p khi x và y chạy khắp ℤ .
Nếu các số dư của ax2 luôn khác số dư của –c – by2 khi chia cho p thì vô lý vì khi đó
có p + 1 số dư phân biệt.
Do đó phải tồn tại x, y ∈ ℤ sao cho 22 (mod)axcbyp ≡−− .
Đến đây ta có điều phải chứng minh.
Nhận xét: Thực chất, bài toán này chỉ là một hệ quả trực tiếp của định lý Cauchy –
Davenport!
Ví dụ 28 (Iran TST 2020)
Cho p là một số nguyên tố lẻ. Tìm tất cả các bộ
∈
1 2 121 2 ,,..., p pp xxx
ℤ thỏa mãn:
Xét trường hợp 3 p > :
p p i tTi
≡=
g tgp g ∈=
1 (mod). 1
tTtTtT
111 1 222 2 2 111
ppp p iii iii xxxp ===
≡≡≡
Lời giải (dựa theo lời giải của TheUltimate123)
Với 3 p = thì mỗi tổng trên chỉ có một số hạng nên ta có thể chọn 1x tùy ý.
Kí hiệu { }1;2;3;...;1Tp⊂− là tập hợp các thặng dư bình phương mod p thì
1 2 p T = . Trước hết ta đi chứng minh bổ đề sau:
Bổ đề: nếu với mọi tT ∈ ta có (1)1 1 tx p −+ = thì { }0;1 x ∈ mod p .
Thật vậy: nếu [ ] 0 xp ≡ thì (1)11 1 tx pp −+ == với mọi tT ∈ (thỏa mãn).
Nếu [ ] 0 xp ≡ / , do (1)1 tx−+ là thặng dư bình phương mod p , xét ánh xạ: : ()(1)1 fTT tfttx → =−+ ֏ .
Vì [ ] 0 xp ≡ / nên f là một đơn ánh.
Ngược lại với mỗi ' tt ∈ tồn tại ( ) 1 '11ttx=−+ , trong đó 1 x là phần tử nghịch đảo của x trong hệ thặng dư thu gọn mod p , sao cho ()'ftt =
Do đó f là một toàn ánh.
() () 1 (1)110(mod) 2 1 10(mod) 2 1(mod).
p ttxxtxp p xp xp
ng minh. Quay trở lại bài toán. Giả sử 1 2 121 2 ,,..., p pp xxx ∈ ℤ là một bộ thỏa mãn đầu bài. Đặt: 111 1 222 2 2 111 ...(mod). ppp p iii iii axxxp === =≡≡≡ Với tT ∈ , đặt () 1 2 ()(1)1 p ii Pxtx=−+ Ta có () 1 1 2 2 1 1 2 ()(1)11(1). p p jjj iipi j PxtxCtx = =−+=+− 111 1 222 2 1 1121 111 1 2 2 2 11 ()(1)(1)... 22 ppp p jjjjjjjj ipip p iij j pp PxCtxCtxxx === = =+−=+−+++ 1111 2222 1 1 1110 2 2 11 ()(1)(1)(mod) 22 pppp jjjjj ipip ijij pp PxCtxaCtp ==== =+−≡+− 1 1 2 2 1 1 ().1(mod). 2 p p i i p Pxatp = =+−
Vì tT ∈ nên 1 2 1(mod) p tp ≡ , từ đó suy ra
Chương 3. Hệ số nhị thức và một số định lý cổ điển
p i i
Do đó
p Pxp =
1 ()(mod). 2
3.1 Một số tính chất của hệ số nhị thức
≡
Lại có, với mỗi i thì () 1 2 ()(1)1 p ii Pxtx=−+ chỉ có thể có số dư khi chia p là 0,1,1 (lần lượt tương ứng với các trường hợp (1)1 itx−+ chia hết cho p , là thặng dư bậc hai mod p và không là thặng dư bậc hai mod p ).
≡≡≡≡
3.1.1 Tính chất 1
Cho m, n nguyên dương, m < n và (m, n) = 1. Khi đó ta luôn có 0(mod). m n Cn ≡
Chứng minh
Ta có 11 11 0(mod). mmmm nnnn n CCmCnCn m = =≡
Mà (m.n) = 1 nên m n C ≡ 0 (mod n).
3.1.2 Tính chất 2
Cho p là một số nguyên tố. Khi đó với mọi i = 1, 2, …, p – 1 thì 0(mod) i p Cp ≡ .
3.1.3 Tính chất 3
Cho p là một số nguyên tố, k là một số nguyên dương, 1 ≤ k ≤ pk – 1. Khi đó ta có
0(mod) k i p Cp ≡ .
3.2. Một số định lý cổ điển khác
3.2.1 Định lý Sylvester
Cho các số nguyên dương a, b nguyên tố cùng nhau. Chứng minh rằng
N0 = ab – a – b
là số nguyên lớn nhất không biểu diễn được dưới dạng
ax + by
với x, y là các số nguyên không âm. Hơn nữa, với mọi p, q nguyên, p + q = N0, có đúng một số biểu diễn được dưới dạng ax + by với x, y nguyên không âm.
Chứng minh
Bổ đề
Cho a, b nguyên dương nguyên tố cùng nhau, b > 1. Chứng minh rằng với mỗi số nguyên N, tồn tại duy nhất cặp số (x, y) sao cho N = ax + by, với 0 ≤ x < b.
Chứng minh bổ đề
64
a) Tính tồn tại
Theo định lý Bezout, luôn tồn tại x, y nguyên sao cho ax + by = N.
Chia x cho b ta được x = bq + x0, 0 ≤ x0 < b.
Ta được
a(bq + x0) + by = n Û ax0 + b(aq + y) = N.
Û ax0 + by0 = N, y0 = aq + y. (1)
Tính tồn tại được chứng minh.
b) Tính duy nhất.
Từ 0 ≤ x0 < b, ax0 = N – by0 º N (mod b).
Do (a, b) = 1 Þ {0, a, 2a, …, (b - 1)a} là hệ thặng dư đầy đủ mod b
Do đó tồ tại duy nhất x0 Î {0, 1, …, b - 1} sao cho ax0 º N (mod b).
Suy ra số x0 xác định trong (1) là duy nhất. Do đó (x0, y0) duy nhất.
Trở lại định lý Sylvester
a) Chứng minh với mọi n > N0, luôn biểu diễn được.
Do a, b nguyên tố cùng nhau nên tồn tại x, y sao cho 0 ≤ x < b sao cho
n = ax + by Þ by = n – ax.
Þ by > N – ax = ab – a – b – ax ≥ ab – a – b – a(b - 1) = -b.
Þ b(y + 1) > 0 Þ y > -1 Þ y ≥ 0.
Vây với mọi n > N0 thì luôn tồn tại x, y không âm sao cho n = ax + by.
b) Giả sử tồn tại x, y không âm sao cho N = ax + by
Mà theo trên ta có N = ab –a – b = a(b - 1) + (-1).b là sự biểu diễn duy nhất nên suy ra
x = b – 1; y = -1
Điều đó trái với giả thiết y ≥ 0.
Vậy có điều phải chứng minh.
3.2.2 Định lý Wolstenholme
a) Cho p nguyên tố lẻ. Chứng minh rằng
1
Chứng minh rằng r chia hết cho p2
Chứng minh
a) Ta có 1111()()()() (mod) ()()()()()() papbbpaapbab p abapbbpapapbabpapb −−+−+−+ +++≡
2 0(mod) ()() p p abpapb ≡≡ Do đó 1111
≡ ≡=
11 40(mod)0(mod),(,4)1. ..ijpijpijijppp≤<≤−≤<≤−
1111 (,2)()2()(mod) ppp iiijpi Tp p iiiji ==≤<≤−= =≡−≡
Mặt khác 111 111
≡+≡≡
p p iipiipi ===
Vậy T(p, 2) º 0 (mod p).
b) Ta có 11 1 22 2 111
1111 ()()(mod). () ==+≡
Điều phải chứng minh tương đương với 1 2 1
p i p ipi =
≡
1 0(mod). ()
Thật vậy, ta có 11 22 2 11
pp ii p ipii==
11 (mod). ()
1 (,2) p i Tp i =
= ⋮ p.
2 1
b) Cho p ≥ 5, p nguyên tố Đặt 11 1...,(,)1 21 r sr sp =+++= .
Do i2 º (p - i)2 (modp) nên
111 1 222 2222 1111
ppp p iiii pp ipiiipii ====
111111 ()(mod)0(mod). ()2()
≡−≡−+≡≡
3.2.3 Định lý Legendre
Cho p là một số nguyên tố. Khi đó
Vì nếu m⋮ q thì f(m) – 1 º 0 (mod m) Þ f(m) – 1 º 0 (mod q) Þ -1 º 0 (mod q).
3.2.4 Định lý Lucas
() (!). 1 p p k k
nsn n vn pp
+∞ =
==
1
(Phần chứng minh của định lý này bạn đọc có thể xem trong chuyên đề các bài toán về phân nguyên của cùng tác giả)
Cho m, n nguyên dương, m ≤ n, p là số nguyên tố. Biểu diễn của m, n trong hệ cơ số p là
0101...,... k k k k mmmpmpnnnpnp =+++=+++
trong đó 0 ≤ ni, mi ≤ p – 1. mk và nk không đồng thời bằng 0. Khi đó 0 0 ...(mod) k k
nm m nnn CCCp = .
Chứng minh
Theo định lý Legendre, ta có (1)1(mod) kk pp xxp+≡+
Do đó
01 0 (1)(1)(1) k i k
Dẫn đến điều vô lý.
Do đó (q, m) = 1.
Giả sử m º 1 (mod q).
Khi đó f(m) = mp – 1 + mp – 2 + … + 1 º 1 + 1 + … + 1 º p (mod q).
Þ p º 0 (mod q) Þ p = q.
Mà p | m Þ q | m (vô lý vì m º 1 (mod q)).
Từ đó suy ra với mọi q là ước nguyên tố của f(m) thì (q, m(m - 1)) = 1.
Trở lại chứng minh định lý
Ta giả sử tập các số nguyên tố dạng pn + 1 là hữu hạn. Khi đó gọi tất cả các số nguyên tố đó là p1, p2, …, pk. Xét đa thức fp(x) = xp – 1 + xp -2 + … + 1. Đặt m = p1p2 pkp Þ m ⋮ p
Theo bổ đề trên, ta có q là ước nguyên tố của f(m).
Khi đó (q, m(m - 1)) = 1.
+=+=+∏ 0 (1) i k n p i x =
n k nnpnp n p xxx +++ =
≡+ ∏ (mod p).
Đồng nhất hệ số của xm ta được điều phải chứng minh.
3.2.5 Định lý Dirichlet
Cho p là một số nguyên tố. Chứng minh rằng tập số nguyên tố dạng pn + 1 là vô hạn.
Chứng minh
Bổ đề
Cho p là một số nguyên tố. Đặt fp(x) = xp – 1 + xp – 2 + … + 1. Với mỗi số nguyên dương m chia hết cho p. Tồn tại hay không một số nguyên q sao cho |() (,(1))1. p qfx qmm −=
Chứng minh bổ đề
Gọi q là một ước nguyên tố của fp(m).
Ta có f(m) º 1 (mod m) Þ q không là ước của m
Theo định lý Fermat nhỏ ta có mq – 1 º 1 (mod q).
Ta có (m - 1)fp(m) = mp – 1 º 0 (mod q) Þ mp º 1 (mod q)
Giả sử (p, q - 1) = 1, theo định lý Bezout, tồn tại các số nguyên r, s sao cho (q - 1)r + ps = 1.
Khi đó m = m(q - 1)rmps º 1 (mod q).
Vô lý vì (m – 1, q) = 1
Do đó
(q – 1, p) = p Þ q – 1 ⋮ p Þ q = pn + 1.
Mà (q, m) = (q, p1p2 pkp) = 1 nên q không thuộc {p1, p2, …, pk}
Từ đó suy ra tập các số nguyên tố dạng pn + 1 là vô hạn.
Nhận xét: Thực tế, đây chỉ là một trường hợp đặc biệt của định lý Dirichlet. Định lý
tổng quát được phát biểu như sau:
“Cho a, b là hai số nguyên dương nguyên tố cùng nhau. Khi đó tập số nguyên tố có
dạng an + b là vô hạn”
Tuy nhiên, hiện tại định lý trên chưa có chứng minh sơ cấp!
3.3 Ví dụ áp dụng
3.3.1 Các ví dụ cơ bản
Ví dụ 1. Chứng minh rằng: ∀ m, n ∈ N ta có (2m)!(2n)! ⋮ m!n!(m+n)!
Lời giải
Gọi p là một số nguyên tố bất kì
Số mũ của p trong phân tích chính tắc của m!n!(m+n)! là
Lời giải Ta có
ả
mnmn ppp + ++
Từ đó có đpcm.
∞ =
β = ∞ = + 1 ) 2 2 ( k k k p n p m . Ta dễ dàng chứng minh được + + + ≥ + k k k k k p n m p n p m p n p m 2 2 β ≥ α Từ đó
có điều phải chứng minh. Ví
Lời giải
Ta có n! = 2 m q Trong đó (2, q) = 1.
Ta phải chứng minh n > m.
v
i
++ + + +
+−+=+−+ =+−+ ≡+−+ −+ ≡+−+≡
akCk pppk k k k k k k kp k
(2)(3)...(1) (1)(1) ! (2)(3)...(1) (1)(1) ! (1)(1)! (1)(1)0(mod). !
Ví dụ 4 (VMO 2017). Chứng minh rằng
a) 1008 2 2017 1 0(mod2017); k k kC ≡
−≡−
b) 504 20162 2017 1 (1)3(21)(mod2017). kk k C =
Lời giải Dễ thấy
1 20172016 2017.kk kCC = Do đó
100810072016
100820161008
20172017 2017()(2). 22 kkk kkk kCCCCC ==−=−
20172016201620162016 100
Theo định lý Fermat nhỏ, ta có 22016 – 1 º 0 (mod 2017).
Mặt khác 1008 2016 1009.1010...20161008!(20171)(20172)...(20171008)1008! 1 1008!1008!
C −≡≡ ≡
0(mod2017).
Từ đó ta có điều phải chứng minh.
b) Xét p nguyên tố bất kì ta có
Mặt khác + + + p n n n 2 ... 2 2 2 ≤ + + + p n n n 2 ... 2 2 2 = p n n 2 < n Từ đó suy ra điều phải chứng minh. 70 Ví dụ 3 (Bình Dương 2020) Cho p là một số nguyên tố lẻ. Gi
Ví dụ 8. Cho p nguyên tố lẻ Đặt
111
1 (,) p k Tpk i =
Biện luận số dư của T(p,k) khi chia cho k
Lời giải
Với mọi i Î {1, 2, …, p - 1} ta có (i, p) = 1.
= .
1 1
Do đó luôn tồn tại i’ Î {1, 2, …, p – 1}, i’ ≠ i sao cho ii’ º 1 (mod p).
Mặt khác nếu i’ º j’ (mod p) thì 11 (mod)(mod) pijp ij ≡ ≡ .
Do đó số i’ như trên là xác định duy nhất.
pkp Tpki i pkp ==
=≡≡
Ví dụ 9 (Định lý Wolstenholme) Cho p ≥ 5, p nguyên tố. Đặt 11 1...,(,)1 21 r sr sp =+++=
Chứng minh rằng r chia hết cho p2
Lời giải
Ta có
11
==+≡
1 22 2 111
1111 ()()(mod). ()
Điều phải chứng minh tương đương với
Thật vậy, ta có
Do i2 º (p - i)2 (mod p) nên
1 2 1
p i p ipi =
≡
1 0(mod). ()
11 22
pp ii p ipii==
2 11
11 (mod). ()
1 222 2222 1111
ppp p iiii pp ipiiipii ≡−≡−+≡≡
Từ đó ta có điều phải chứng minh.
Ví dụ 10. Chứng minh rằng số nguyên dương chẵn n là số hoàn hảo khi và chỉ khi n = 2m – 1(2m - 1)
Lời giải
Gọi T(n) là tổng ước dương của n
a) Xét 2m – 1 – 1 là số nguyên tố, chứng minh n là số hoàn hảo.
Ta có
11 21 ()(2(21))(2)(21).(121) 21 m mmmmmTnTTT=−=−=+− = 2n
Vậy n là số hoàn hảo.
b) Xét n là số hoàn hảo.
Viết n = 2s t, (t, 2) = 1.
Do đó ta có
Mà n là số hoàn hảo nên
1 ()(2)()(21)() ss TnTTtTt + ==−
11(21)()22 ssTtnt ++ −==
Vì (2s+1 – 1, 2s +1 ) = 1 Þ T(t) ⋮ 2s+ 1 Þ T(t) = 2s + 1q.
Từ đó ta suy ra (2s +1 - 1)T(t) = 2s + 1t Þ (2s+ 1 - 1)q = t.
Suy ra t ⋮ q.
Nếu t = q Þ 2s + 1 – 1 = 1 Þ s = 0 Þ n lẻ
Vậy t ≠ q.
Mà t + q = 2s + 1q = T(t).
Nếu q ≠ 1, mà t ⋮ q nên t có ít nhất 3 ước là t, q, 1 Þ T(t) ≥ t + q + 1 > t + q (vô lý).
Vậy q = 1 Þ T(t) = t + q = t + 1.
Nếu t không phải số thì t phải có ước số khác t và 1 Þ T(t) > t + 1 (vô lý).
Vậy t = 2s + 1 – 1 và t là số nguyên tố
74
Vậy n = 2s(2s + 1 - 1), 2s + 1 – 1 là số nguyên tố
Từ đó có điều phải chứng minh.
3.3.1 Các ví dụ nâng cao
Ví dụ 11. Cho a là một số nguyên dương. Một số nguyên dương b được gọi là a-good nếu 1(1) b an Can−+ ⋮ với mọi số nguyên dương n sao cho an ≥ b. Chứng minh rằng nếu b là a-good thì b là số chẵn.
Lời giải
Gọi p là số nguyên tố lớn hơn cả a và b. Khi đó (a, p) = 1.
Do đó, tồn tại vô số số nguyên dương m sao cho am + 1 ≡ 0 (mod p).
Chọn m sao cho am ≥ b.
Khi đó |1|1 b ampamC+− .
Ta có
()(1)...(1)(1)(2)...() 1 (1)(mod) !! b b am amamambb Cp bb −−+−−− ≡=≡≡− .
Do đó b là số chẵn.
Ví dụ 12. Cho a là một số nguyên dương. Một số nguyên dương b được gọi là a-good
nếu 1(1) b an Can−+ ⋮ với mọi số nguyên dương n sao cho an ≥ b. Gọi b là một số nguyên dương sao cho b là a-good và b + 1 không là số nguyên tố. Chứng minh rằng mọi ước
nguyên tố của b + 1 đều là ước của a.
Lời giải
Phản chứng.
Giả sử tồn tại p nguyên tố, p là ước của b + 1 nhưng không là ước của a
Do b + 1 không là số nguyên tố nên b + 1 ≠ p, b + 1 – p chia hết cho p.
Đặt 11 (1)1(mod)1() p vbpbprppbprpp αααα α ++ =+− +−≡ +≡+ .
Với 1 ≤ r ≤ p-1.
Ta có (a, p) = 1 nên tồn tạp vô số số nguyên dương m sao cho am +1 ≡ p (mod 1 p α+ ) ⇔ am ≡ p - 1 (mod 1 p α+ ).
Chọn m sao cho am ≥ b.
Mặt khác ta có
1 1 +− = +−− =
amamp pp bbprp pp 11 1 (1)(1)(1)(1) 1. ambampbprpampbprp ppp
++ ++
αα α αα α α αα α ++ + −+−−+−−+−−+−− ==−
ambamb
+++ =+ +1
Ta có với mọi k ≥ 0, ta có kkk
≥+
0(mod). p am Cp ≡
Vậy ta có đpcm.
Nhận xét: Từ hai ví dụ trên, ta dễ dàng chứng minh được bài IMO SL 2019 sau
Ví dụ 13 (IMO SL 2019) Cho số nguyên dương a. Chúng ta nói một số nguyên dương b là a-good nếu 1(1) b an Can−+ ⋮ với mọi số nguyên dương n sao cho an ≥ b. Giả sử rằng b là một số nguyên dương, b là a-good và b + 2 không là a-good. Chứng minh rằng b + 1 là một số nguyên tố.
Lời giải
Dễ dàng suy ra từ ví dụ 11 và ví dụ 12.
Ví dụ 14 (Đề luyện ĐT TST 2020) Cho p là một số nguyên tố. Với mỗi số nguyên dương k , đặt
12...(1) kkk k Ap =+++− .
Chứng minh rằng:
a) Nếu 7 p ≥ và k là một số nguyên dương lẻ sao cho 31 kp <<− thì 1 2 kk kpAA chia hết cho 4 p
b) Nếu p ≥ 5 thì 2 1 2 pp pAA chia hết cho 5 p
Lời giải.
a) Xét k là một số nguyên dương lẻ tùy ý thì: [][]
=+++− =−−+−−++− =−−+−−++−
12...(1)
(1)(2)...(1)
Ap ppppp CppCppCp = = =
kkk k kk k k k k iiikiiiikiiiki k k k i i
(1)(1)(1)(2)...(1)
0 0 0
=−
1 01 (1) p k iikii k ij Cpj ==
332214 321 (mod) kkkkkkk CpACpACpAAp ≡−+− .
22334 1232(mod).kkkkkk kpAACpACpAp −≡− (*)
Ta chứng minh hai bổ đề sau:
Bổ đề 1: nếu k là số nguyên dương nhỏ hơn 1 p thì | kpA
Chứng minh
Giả sử a là một căn nguyên thủy của p . Khi đó { }21 ;;...; p aaa là một hệ thặng dư thu
gọn mod p và do đó:
2(1)12...(1)... 1 pkk kkkkkpk k k aa Apaaa a =+++−≡+++= .
Vì k là số nguyên dương nhỏ hơn 1 p nên [ ] 1 k ap ≡ / .
mà () ( ) 1 1 pk pkkk aaaap −=− ⋮ nên | kpA
Bổ đề 2: nếu k là số nguyên dương lẻ thỏa mãn 31 kp <<− thì 2 | kpA
Chứng minh
Vì k lẻ nên theo (*) thì
22332 123 2(mod) kkkkkk AkpACpACpAp ≡−+ .
Kết hợp với mệnh đề 1 ta có 2 | kpA .
Lại áp dụng mệnh đề 2 cho (*) ta thu được 1 2 kk kpAA chia hết cho 4 p
b) Với p là một số nguyên tố lẻ ta có
44332215 4321 (mod) ppppppppp CpACpACpACpAAp ≡−+−+−
24433225 1432 (mod) pppppppp pAACpACpACpAp −≡−+ .
Vì 234 ,, ppp CCC đều chia hết cho p mà theo mệnh đề 1 thì 3 p Ap ⋮ và theo mệnh đề 2 thì
2 2 p Ap ⋮ nên 4433225 432 pppppp CpACpACpAp −+ ⋮ ; từ đó dẫn tới 2 1 2 pp pAA chia
hết cho 5 p .
Ví dụ 15 (IMO Shortlist 2013). Cho p là một số nguyên tố lẻ. Với mỗi số nguyên dương a , ta đặt 21 ... 121 p a aaa S p =+++ .
Đặt 342 3 m SSS n +−= với * , mn∈ℕ . Chứng minh rằng | pm.
Lời giải
Trước hết ta chứng minh bổ đề
( ) 1 1 (mod). k k p C p pk ≡
với p là một số nguyên tố lẻ, ,11kkp∈≤≤− ℕ .
Thật vậy
k k p C ppkpkk p pkpkpkkk −+−−+−+− ==≡=
( ) 1 1 !(1)...(1)(1)(2)...(1) (mod).
!()!!!
Bổ đề được chứng minh.
Trở lại bài toán
Sử dụng bổ đề trên ta được
() 1 1..(mod) kk k k p aaCp kp ≡− ,,11 kkp ∀∈≤≤− ℕ
Do đó ta có
−−+ ≡−=+−+−=
aa a SCaCap ppp = =
11 1 1..11..(mod). p p k p p k pk kpkk ap p k k
1 1 1 0
Thay vào ta được: 342 2313413(121) 3 pppppp SSS p −++−+−−+ +−≡ ( )2 22 4.244 (mod). p pp p pp ≡≡−
Vì p là số nguyên tố lẻ nên theo định lý Fermat nhỏ thì 22(mod). p p ≡
Do đó ( )2 22 0(mod). p p p ≡
Vậy 34230(mod). m SSSp n +−=≡
Hay | pm.
Ví dụ 16 (CMO 2015)
Cho số nguyên dương k ≥ 2, chứng minh rằng có vô hạn số nguyên dương n thỏa mãn
2 | n n nkC + / .
Lời giải
Xét các trường hợp sau:
Trường hợp 1: k có ước nguyên tố lẻ p Đặt ()0 p tvk=> và chọn m np = với .mt >
Vì
với 2 t ≥ Chọn 2m n= với mt > .
Vì vậy trong trường hợp này các số có dạng 2m n= thỏa mãn bài toán.
Ví dụ 17 (VMO 2016)
a) Chứng minh rằng n là số hoàn hảo lẻ thì n có dạng n= ps.m2
trong đó p là số nguyên tố dạng 4k + 1, s là số nguyên dương có dạng 4h + 1, m nguyên dương không chia hết cho p
b) Tìm tất cả các số nguyên dương n > 1 sao cho n – 1 và
Lời giải
Kí hiệu hàm tổng ước dương của n là T(n).
(1) 2 nn + đều là số hoàn hảo.
Khi đó
t a i np =
a) Xét phân tích chính tắc của n 1 1
= ∏
Nếu a lẻ thì
1 ()2 1 a t i i i
Nhưng p1s º m2 º psm2 º 1 (mod 4) còn 4k + 3 º 3 (mod 4) (vô lý).
==
p Tnn p
+ = ∏
Ta lại có v2(2n) = 1; 1 222 1 (())((1)(1)) t a ii i vTnvpvp + =
v2(pa+1 - 1) – v2(p - 1) = v2(p - 1) + v2(p + 1) + v2(a + 1) – 1 – v2(p - 1) ≥ 1.
Do đó, trong các số ai, i = 1, 2, …, t chỉ có duy nhất một số lẻ
Giả sử số đó là a1
Khi đó
Vậy n lẻ Þ n – 1 chẵn
Theo bài toán 1) mọi số hoàn hảo đều có dạng 2k(2k + 1 - 1).
Do đó ta có n – 1 = 2s(2s + 1 - 1). Suy ra
111 (1) 2(21)12(21)1 2 ssss nn +−+ + =−+−+
+) Nếu s = 1 thì
111 (1) 2(21)12(21)128 2 ssss nn +−+ + =−+−+=
Thoả mãn nên ta được n = 7.
11 2 11 2
a aa i i npppm == ∏
Ta cần chứng minh p1 = 4k + 1 và a1 = 4h + 1.
Thật vậy
+) Nếu s > 1 thì 111 (1) 2(21)12(21)1 2 ssss nn +−+ + =−+−+ là số hoàn hảo lẻ
Do đó
11122(21)12(21)1 ssssa pm +−+ −+−+=
Do đó
1 1 1 2212111 1
+ =+++−≥∀=+
1 ()(1)(1)11,,21. 1 a p vvpvapal p
Do 111 (2(21)1,2(21)1)1 ssss +−+−+−+= nên một trong hai số này phải là số chính phương
Ta sẽ chứng minh cả hai số này đều không là số chính phương.
TH1: 2s(2s + 1 - 1) + 1 = b2 Þ 2s(2s + 1 - 1) = (b - 1)(b + 1)
1 1
211 1 2 211 1
(1)141 1 ()1 (1)141 1 a vppk p v vaah p
Từ đó ta có điều phải chứng minh.
b) Trước tiên, ta chứng minh n là số lẻ.
Thật vậy, giả sử n chẵn ta có n – 1 là số hoàn hảo lẻ.
Theo a), mọi số hoàn hảo đều có dạng psm2 = (4 k + 1)s.(2l + 1)2 º 1 (mod 4).
Do đó
n – 1 º 1 (mod 4) Þ n = 4k + 2 Þ (1) (21)(43) 2 nn kk + =++ lẻ
Suy ra
(1) (21)(43) 2 nn kk + =++ = psm2
Trong đó p º s º 1 (mod 4).
Mà (2k + 1, 4k + 3) = 1 nên 4k + 3 chỉ có thể bằng p1 s hoặc m2 hoặc ps.m2
Do (b – 1, b + 1) | 2 và (b - 1)(b + 1) ⋮ 4 nên b – 1 và b + 1 đều chẵn.
Mà 2s + 1 – 1 là số nguyên tố nên 2(2s + 1 - 1) phải là ước của b – 1 hoặc b + 1
Do s ≥ 2 Þ b > 2 Þ 2(b - 1) > b + 1
Suy ra
2(b - 1)(b + 1) > (b + 1)2 ≥ [2(2s + 1 - 1)]2
Þ (b - 1)(b + 1) > 2(2s +1 - 1)2 > 2s(2s + 1 - 1) = (b - 1)(b + 1).
Dẫn đến vô lý.
Hoàn toàn tương tự cho trường hợp 2s – 1(2s + 1 - 1) + 1 = b2
Kết luận n = 7.
Ví dụ 18. Cho a, b nguyên dương nguyên tố cùng nhau, b > 1.
Chứng minh rằng với mỗi số nguyên N, tồn tại duy nhất cặp số (x, y) sao cho
N = ax + by, 0 ≤ x < b
Lời giải a) Tính tồn tại
82
Theo định lý Bezout, luôn tồn tại x, y nguyên sao cho ax + by = N.
Chia x cho b ta được x = bq + x0, 0 ≤ x0 < b
Ta được
a(bq + x0) + by = n Û ax0 + b(aq + y) = N
Û ax0 + by0 = N, y0 = aq + y. (1)
Tính tồn tại được chứng minh.
b) Tính duy nhất.
Từ 0 ≤ x0 < b, ax0 = N – by0 º N (mod b).
Do (a, b) = 1 Þ {0, a, 2a, …, (b - 1)a} là hệ thặng dư đầy đủ mod b
Do đó tồ tại duy nhất x0 Î {0, 1, …, b - 1} sao cho ax0 º N (mod b).
Suy ra số x0 xác định trong (1) là duy nhất.
Do đó (x0, y0) duy nhất.
Ví dụ 19. Cho a, b, c là các số nguyên dương đôi một nguyên tố cùng nhau. Chứng minh rằng
N0 = 2abc – ab – bc – ca
là số nguyên lớn nhất không biểu diễn được dưới dạng abx + bcy + caz, với x, y, z là các số nguyên không âm.
Lời giải
a) Trước hết ta chứng minh N0 không biểu diễn được dưới dạng abx + bcy + caz.
Thật vậy, giả sử tồn tại x, y, z không âm sao cho
N0 = abx + bcy + caz Þ 2abc = (x + 1)bc + (y + 1)ca + (z + 1)ab
Sử dụng mod a ta suy ra
(x + 1)bc º 0 (mod a) Þ x + 1 º 0 (mod a) Þ x + 1 ≥ a Þ (x + 1)bc ≥ abc.
Tương tự như vậy ta suy ra
(x + 1)bc + (y + 1)ca + (z + 1)ab ≥ 3abc > 2abc (vô lý)
Vậy N0 không thể biểu diễn được dưới dạng abx + bcy + caz
b) Tiếp theo ta chứng minh mọi N > N0 thì đều tồn tại x, y, z không âm sao cho
N = abx + bcy + caz
Điều đó tương đương với mọi N > 2abc, tồn tại x0, y0, z0 > 0 sao cho
N = abx0 + bcy0 + caz0.
Bổ đề: Với a, b là hai số nguyên dương, (a, b) = 1.
Khi đó
A = {ax + by| 1 ≤ x ≤ a; 1 ≤ y ≤ b}
là hệ thặng dư đầy đủ (mod ab).
(Bổ đề này rất đơn giản, xin nhường bạn đọc)
Trở lại bài toán
A = {ax + by| 1 ≤ x ≤ a; 1 ≤ y ≤ b}
là hệ thặng dư đầy đủ (mod ab).
Mà (c, ab) = 1 nên hệ
B = cA = {cax + bcy| 1 ≤ x ≤ a; 1 ≤ y ≤ b}
là hệ thặng dư đầy đủ mod ab.
Điều đó có nghĩa là với mọi N nguyên dương, luôn tồn tại y0, z0 sao cho 1≤ y0 ≤ a, 1 ≤ z0≤ b và
N º bcy0 + caz0 (mod ab) Þ N – bcy0 – caz0 = abx0
Þ N =abx0 + bcy0 + caz0
Hơn nữa
N > 2abc Þ abx0 + bcy0 + caz0 > 2abc Þ abx0 + bc(y0 - a) + ca(z0 - b) > 0.
Mà y0 – a ≤ 0; z0 – b ≤ 0 nên x0 > 0.
Từ đó có điều phải chứng minh.
Nhận xét: Bài toán trên đây gần như được lặp lại ở đề thi TST
Ví dụ 20 (Việt Nam TST 2000). Cho a, b, c là ba số nguyên dương đôi một nguyên tố cùng nhau. Số n gọi là “số bướng bỉnh” nếu n không thể biểu diễn được dưới dạng n = abx + bcy + caz
với x, y, z là các số nguyên dương. Hỏi có tất cả bao nhiêu số bướng bỉnh?
Lời giải
Gọi A là tập hợp các số bướng bỉnh.
a) Theo bài IMO 1983, ta có mọi n > 2abc đều không là số bướng bỉnh.
b) Nếu n = abx + bcy + caz, x, y, z ≥ 1 thì n ≥ ab + bc + ca
Do đó mọi n nguyên dương và n ≤ ab + bc + ca – 1 đều là số bướng bỉnh.
Từ đó suy ra
{1,2,...,1}{,1,...,2} Aabbccaabbccaabbccaabc ⊂++−∪+++++
Đặt B = {,1,...,2} abbccaabbccaabc +++++
Xét f(n) = ab + bc + ca + 2abc – n.
Ta sẽ chứng minh, với mọi n Î B, ta luôn có n Î A Û f(n) ∉A.
Thật vậy
Với mọi n Î B, theo chứng minh bổ đề bài trên thì tồn tại x0, y0, z0 sao cho
1 ≤ y0 ≤ a, 1 ≤ z0 ≤ b, và
n = abx0 + bcy0 + caz0
Do n ∉ A Þ x0 ≤ 0 Þ f(n) = ab(1 – x0) + bc(1 + a – y0) + ca(1 + b – z0) Î A
Hoàn toàn tương tự, nếu n Î A thì f(n) ∉A
Từ đó suy ra trong B chứa đúng một nửa số các phần tử thuộc A.
Vậy
Do đó số 6a2 – a2 – 6 và 36a + 36 – a là các số nguyên dương lớn nhất không biểu diễn được.
Từ đó ta phải có
Suy ra a < 7.
6a2 – a2 – 6 < 250 và 36a + 36 – a < 250
Kết hợp điều kiện (a, 6) = 1 suy ra a = 1 hoặc a = 5.
+) Nếu a = 1 thì phương trình n = x + 6y + 36z luôn có nghiệm (n, 0, 0).
+) Nếu a = 5, ta chứng minh mọi số n ≥ 250 đều biểu diễn được n = 25x + 30y + 36z = 25x + 6(5y + 6z - 20) + 120.
Theo định lý Sylvester, mọi số tự nhiên không nhỏ hơn 20 đều biểu diễn được dưới dạng
5y + 6z
Suy ra mọi số tự nhiên đều biểu diễn được dưới dạng 5y + 6z – 20 = u.
||1
||1(1) 22 B Aabbccaabcabbcca =++−+=+++−
Ví dụ 21 (VMO 2015). Với a, n nguyên dương, xét phương trình a2x + 6ay + 36z = n
trong đó x, y, z là các số nguyên không âm.
a) Tìm tất cả các giá trị của a để với mọi n ≥ 250, phương trình đã cho luôn có nghiệm x, y, z không âm.
b) Biết rằng a > 1, (a, 6) = 1. Tìm giá trị lớn nhất của n theo a sao cho phương trình đã cho không có nghiệm x, y, z nguyên không âm.
Lời giải
Giả sử với mọi n > 250 luôn tồn tại x, y, z nguyên không âm sao cho
n = a2x + 6ay + 36z
Khi đó, (6, a) = 1.
Nếu ngược lại, ta chọn n > 250 sao cho n ⋮ d = (a, 6) thì dẫn đến vô lý.
Ta có
a2x + 6ay + 36z = a(ax + 6y) + 36z = am + 36z.
Theo định lý Sylvester thì số N0 = ab – a – b + 1 là số nguyên nhỏ nhất sao cho
phương trình ax + by = n
có nghiệm nguyên không âm với mọi n ≥ N0.
85
Tương tự, mọi số tự nhiên không nhỏ hơn 120 đều biểu diễn được dưới dạng 25x + 6u.
Do đó, với m ≥ 250 thì n – 120 > 120 nên tồn tại x, u sao cho n – 120 = 25x + 6u Þ n = 25x + 6(u + 20)
Lại chọn, y, z sao cho u = 5y + 6z – 20 ta được n = 25x + 30y + 36z
Từ đó được điều phải chứng minh.
b) Ta chứng minh cho trường hợp tổng quát: Cho a, b nguyên dương, (a,b) = 1. Khi đó N = a2b + ab2 – a2 – b2 – ab + 1
là số nguyên dương nhỏ nhất để phương trình a2x + aby + b2z = n có nghiệm nguyên không âm.
(Phần này tương tự như bài IMO 1983 – xin nhường lại cho bạn đọc)
Ví dụ 22 (DMO 2020) Chứng minh rằng với mọi số nguyên dương k, l cho trước luôn tồn tại vô hạn số nguyên dương m sao cho
i) m ≥ k
ii) gcd(,)1. k m Cl = Lời giải Xét dãy số (un) xác định như sau: a n uknl =+ ,trong đó a là một số nguyên dương sao cho pa > k với mọi p là ước nguyên tố của l.
Ta sẽ chứng minh m = un luôn thỏa mãn bài toán.
Trước hết, dễ thấy un > k.
Ta có ()(1)...(1) (1)...1 n
aaa k u knlknlnl C kk +−++ =
Do với mọi p | l thì pa > k nên vp(i) < vp(nla) nê ta dễ dàng chứng minh được
k a ka pppupp i vinlvjikplvCvinlvi =
+=∀≤≤∀ =+−=
Do đó gcd(,)1. k m Cl =
Từ đó có đpcm.
Nhận xét: Bài này tương tự ý tưởng của bài số chọn đội tuyển Trung Quốc năm 2009, bạn đọc có thể tham khảo trên Aops.
Ví dụ 23 (Gặp gỡ toán học 2020)
.
p kkp nn
k SC
0
(1)
Xét y ∈ ℂ là một nghiệm khác 1 của phương trình 1 p x =
Khi đó
21 1...0 p yyy ++++=
Vì p là số nguyên tố nên dễ thấy ( ) 1;1 k ykp≠∀=
Ta có nhận xét sau
2(1) 1 1...0 1 pk kkpk k y yyykp y ++++==∀ ⋮ .
2(1)1...kkpk yyypkp ++++=∀ ⋮
()
() () ()() 111 1111 11 1(1)1111 111(1)...(1)(1) 1(1)...(1)(1) ... 1(1)...(1)(1)
Ta phải chứng minh 1 1 1 12
n p nnn p xxxp + +++ ⋮ (*).
Ta sẽ làm điều này bằng quy nạp.
Ta có: 11 12 000 ...(1)(1) pp k kkkikjkjji p k iij xxxyCy ===
=−=−≡
.
() [] 1 00 (1)10 p k k jkjji k ji Cypp ==
Bây giờ giả sử (*) đúng với mọi * (1), nkpk≤−∈ ℕ . Ta chứng minh (*) đúng với mọi * (1)(1)(1), kpnkpk −<≤+−∈ ℕ .
thì n Tp ⋮ với mọi n
Vì 12,,..., p xxx là các nghiệm phân biệt của phương trình
88
Nên
( ) 110 p x +−=
11221...0pppp ipipipi xCxCxCx ++++= 1;2;..; ip∀= .
112231...0mpmpmppm ipipipi xCxCxCx +−+−+−− ++++= 1;2;..; ip∀=
và với mọi * m ∈ ℕ .
Cộng các vế của các biểu thức trên và cho i chạy ta thu được:
121 123 ...0 p mppmppmppm TCTCTCT +−+−+−++++=
với mọi * m ∈ ℕ .
Trong đẳng thức trên lần lượt cho (1)(1)1;(1)(1)2;...;(1) mkpkpkp =−−+−−+−
ta thu được 1 1 1 n p n Tp + ⋮ với mọi * (1)(1)(1), kpnkpk −<≤+−∈ ℕ
Bài toán hoàn toàn được chứng minh.
- Trong chuyên đề này, chúng tôi đã đưa ra các dạng toán quan trọng nhất về số nguyên tố, các định lý sơ cấp cổ điển liên quan đến số nguyên tố và ứng dụng. Ở mỗi chương, chúng tôi đã cố gắng đưa ra hệ thống lý thuyết đầy đủ, các ví dụ được lựa chọn mang tính thời sự và được sắp xếp từ dễ đến khó (theo quan điểm cá nhận của nhóm tác giả). Trong mỗi ví dụ, chúng tôi đã cố gắng phân tích, đưa ra ý tưởng dẫn dắt tới lời giải, phù hợp với thực tiễn giảng dạy học sinh chuyên toán trong thời kì hiện nay.
- Chúng tôi cũng đã cố gắng tuyển chọn, hệ thống các ví dụ từ nhiều nguồn tài liệu khác nhau, chủ yếu từ các đề thi học sinh giỏi các nước gần đây nhất, mục đích làm rõ các dạng toán về số nguyên tố và ứng dụng trong các bài toán số học.
- Các ý tưởng về phương pháp giải toán đã được chúng tôi tổng hợp, hình thành trong nhiều năm giảng dạy chuyên toán và một số đã được chúng tôi báo cáo ở những lớp bồi dưỡng giáo viên chuyên hang năm. Hy vọng chuyên đề nhỏ này sẽ góp một phần nhỏ vào quá trình bồi dưỡng đội tuyển học sinh giỏi quốc gia của các đồng nghiệp và chúng tôi cũng rất mong nhận được các ý kiến đóng góp của các thầy cô để chuyên đề ngày càng hoàn thiện hơn.
TÀI LIỆU THAM KHẢO
[1] Đặng Hùng Thắng, Nguyễn Văn Ngọc, Vũ Kim Thuỷ- Bài giảng Số học - NXBGD, 1997.
[2] Nguyễn Vũ Lương, Nguyễn Ngọc Thắng, Nguyễn Lưu Sơn, Phạm Văn Hùng – Các bài giảng Số học - NXB Đại Học Quốc Gia Hà Nội, 2006.
[3] Phan Huy Khải - Các chuyên đề Số học - NXBGD, 2005.
[4] Nguyễn Văn Mậu, Trần Nam Dũng, Đặng Hùng Thắng, Đặng Huy Ruận – Các vấn đề chọn lọc của Số học - NXBGD, 2008.
[6] Titu Andreescu, Dorin Andrica, Zuming Feng - 104 Number theory problems from the training of the USA IMO team - NXB Birkhauser, 2006.
[7] Jemes - Elementary Number Theory in nine chapters – NXB Cambridge.
[8] Đàm Văn Nhỉ, Phan Đức Hiệp, Lưu Bá Thắng, Trần Thị Hồng Dung, Trần Trung Tình, Văn Đức Chín – Lý thuyết số và chuyên đề nâng cao – NXB Thông tin và truyền thông, 2017.
[9] Đàm Văn Nhỉ, Nguyễn Anh Tuấn, Lê Xuân Dũng, Trần Thị Hồng Dung, Trần Trung Tình, Văn Đức Chín, Đặng Xuân Sơn, Đào Ngọc Dũng – Đa thức – chuỗi và chuyên đề nâng cao – NXB Thông tin và truyền thông, 2017.
[10] Lê Anh Vinh, Hoàng Đỗ Kiên, Lê Phúc Lữ, Phạm Đức Hiệp – Định hướng bồi dưỡng học sinh năng khiếu số học – NXB Đại học Quốc gia Hà Nội – 2021.
[11] Trần Nam Dũng, Võ Quốc Bá Cẩn, Trần Quang Hùng, Nguyễn Văn Huyện, Lê Phúc
Lữ - Các phương pháp giải toán qua các kì thi Olympic (các năm).
[12] Nguyễn Duy Liên – Chuyên đề cấp phần tử (Chuyên đề Trại hè Hùng Vương 2014)
[13] Phạm Văn Quốc – Bổ đề LTE – Chuyên đề bồi dưỡng giáo viên chuyên.
[14] Nguyễn Văn Thảo – Chuyên đề số học.
[15] Các đề thi trên diễn đàn Aops.
Chuyên đề: TỪ GÓC NHÌN CỦA ƯỚC SỐ
Phần 1. ĐẶT VẤN ĐỀ
1. Lý do chọn đề tài.
Chúng ta đều biết câu ngạn ngữ ở phương tây: “Toán học là vua của mọi khoa học nhưng Số học là nữ hoàng!”. Ta có cảm giác số học không có “tuổi già”, mỗi lần làm việc với nó đều đem lại cho ta những cảm giác thú vị Điều đó chứng tỏ số học có vẻ đẹp tiềm ẩn bên trong, chỉ có những ai tiếp xúc với số học mới thấy sự hấp dẫn mà nó đem lại.
Trong các đề thi học sinh giỏi Toán từ cấp huyện đến cấp quốc gia và quốc tế luôn có bài toán số học. Vai trò của bài số học thường để phân hóa thí sinh nên dĩ nhiên là khó! Mỗi con số nguyên, tự bản thân nó chứa những tính chất riêng biệt, tùy theo từng điều kiện của bài toán mà khai thác các đặc tính đó. Có thể nói chia hết xuất hiện khắp nơi trên mảnh đất số học, nên vấn đề ước số của một số liên quan khá nhiều đến các tính chất của các số như nguyên tố, chính phương, số các ước số,…. Do đó tôi đặt vấn đề nghiên cứu đặc tính “bên trong” của các số nguyên trong mối quan hệ chia hết và từ “bên trong” nhìn ra để giải quyết một số bài toán số học.
2. Mục đích nghiên cứu.
Bài viết về “Từ góc nhìn của ước số” là những kinh nghiệm được tích lũy trong quá trình giảng dạy. Một số bài toán số học trong các kỳ thi học sinh giỏi tỉnh, thi Olympic, thi học sinh giỏi quốc gia và các nước trên thế giới. Các bài toán sẽ được nhìn dưới con mắt ước số để giải quyết.
Bài viết này hướng đến phục vụ chính cho các học sinh chuyên Toán, góp phần làm sáng thêm vẻ đẹp “nữ hoàng”. Nhằm giúp các em tiếp cận và giải quyết bài toán một cách nhẹ nhàng và hợp lý không khiên cưỡng. Mỗi bài toán số học là mỗi cách giải quyết khác nhau, tùy vào tính chất của nó, khác với Đại số và Giải tích có thể angorit được lớp các vấn đề Đó cũng là lý do tạo nên vẻ đẹp của Số học.
Bài viết không tránh khỏi những sai sót do cách nhìn chủ quan, rất mong được sự trao đổi và góp ý của các bạn và quý thầy cô.
3. Nhiệm vụ nghiên cứu.
Nhằm làm sáng lên vẻ đẹp của suy luận số học thông qua việc giải một số bài toán số học dước góc nhìn của ước số, bội số; đồng thời củng cố một số cách giải toán số học cho các học sinh chuyên toán của nhà trường trong những năm qua; bài viết này góp phần nâng cao và phát triển tư duy cho học sinh chuyên toán chuẩn bị tham dự các kỳ thi học sinh giỏi tỉnh, thi Olympic khu vực và thi học sinh giỏi quốc gia.
4. Đối tượng nghiên cứu.
Đối tượng nghiên cứu là học sinh lớp 10; 11; 12 chuyên Toán trong các năm qua. Đặc biệt là các đội tuyển dự thi Olympic khu vực và thi học sinh giỏi quốc gia.
Ngoài ra, bài viết này còn là tài liệu tham khảo cho học sinh và các đồng nghiệp trong trường.
5. Phạm vi nghiên cứu. Hiện nay trên lĩnh vực môn Số học có rất nhiều sách viết khá đầy đủ, đặc biệt gần đây một số tác giả trẻ đã xuất bản những quyển sách rất công phu và cập nhật các đề thi của thế giới. Vì vậy bài viết này chỉ hạn chế trong các phạm vi sau:
- Về kiến thức: dựa trên các kiến thức của chương trình Số học của Bộ giáo dục cho các trường chuyên.
- Bài viết chỉ đi sâu khai thác các tính chất về ước số trong giải toán số học. Bài viết không giải quyết hết các dạng toán số học.
- Về nội dung: giới hạn trong chương trình chuyên toán, phù hợp với đối tượng học sinh trong đội tuyển.
6. Phương pháp nghiên cứu.
- Phương pháp trao đổi với đồng nghiệp, bạn bè, các sinh viên Toán và đặc biệt là các học sinh chuyên Toán trong đội tuyển.
- Phương pháp nghiên cứu lý luận: từ các tài liệu sách vở, từ các tạp chí và trên internet. Nghiên cứu các kiến thức hổ trợ liên quan đến tính chia hết.
- Phương pháp tổng hợp, phân loại và khái quát hóa vấn đề.
7. Cấu trúc của chuyên đề.
Gồm ba phần:
- Phần đặt vấn đề
- Phần tóm tắt nội dung lý thuyết và các khái niệm liên quan.
- Phần phân dạng bài tập. Mỗi bài toán đều có phần “tiếp cận bài toán”, cuối mỗi bài có vài dòng “lời bàn” (qua góc nhìn chủ quan người viết), bài tập tự luyện.
- Phần hướng phát triển đề tài và kết luận.
I. TÓM TẮT LÝ THUYẾT.
Trong bài viết này chỉ giới thiệu tóm tắt các kiến thức lý thuyết có liên quan.
Xin không trình bày phần chứng minh chỉ nêu định lý và công thức (phần chứng minh đã có trong các giáo trình Số học).
I.1. TÍNH CHIA HẾT TRONG TẬP SỐ NGUYÊN.
Định nghĩa 1: Giả sử a; b là hai số nguyên, ta nói a chia hết cho b nếu tồn tại số nguyên c sao cho abc = . Khi đó ta còn nói b chia hết a; a là bội của b, còn b là ước của a.
Ký hiệu: ab ⋮ hay | ba .
Định nghĩa 2: Số nguyên dương 1 p > chỉ có hai ước số là 1 và p, gọi là số nguyên tố
- Số nguyên dương 1 n > có nhiều hơn hai ước số gọi là hợp số
- Ước số nhỏ nhất lớn hơn 1 của hợp số a không vượt quá a .
- Nếu một số tự nhiên 1 a > không có ước số nguyên tố từ 1 đến a thì a là số nguyên tố.
Định nghĩa 3: Với hai số nguyên không âm a và b (b khác 0) luôn tồn tại duy nhất một cặp số nguyên q và r sao cho ( ) 0 abqrrb =+≤<
Tính chất: Cho các số nguyên ; ; ; abcd
a. Nếu | ab và | bc thì | ac .
b. Nếu | ab và | ac thì ( ) | ambnc + với m; n là các số nguyên.
c. Nếu | ab và | cd thì | acbd
d. Nếu ( ) |1; i bain = ( 1; in = ) thì ( ) 12 |. nbaaa ++…+
I.2. ƯỚC SỐ CHUNG, BỘI SỐ CHUNG
Định nghĩa 4: Số nguyên dương d gọi là ƯCLN của hai số nguyên dương a và b, là số nguyên dương lớn nhất mà cả a và b cùng chia hết cho d, ký hiệu ( ) U; dCLNab = hay ( ) gcd; dab = hay viết gọn là ( ) ;.dab =
Khi d =1 ta nói hai số a và b nguyên tố cùng nhau, viết ( ) ;1ab = .
Định nghĩa 5: Số nguyên dương m gọi là BCNN của hai số nguyên dương a và b là số nguyên dương nhỏ nhất mà chia hết cho cả a và b, ký hiệu ( ) ; mBCNNab = hay ( ) ; mlcmab = hay viết gọn là m =[a;b].
Định nghĩa 6: Cho n số nguyên dương 12;;.; naaa … .
a. Số nguyên dương d gọi là ƯCLN của 12;;.; naaa … nếu thỏa mãn đồng thời hai
điều kiện sau:
+/ i ad ⋮ ( 1; in = )
+/ Nếu có số nguyên dương d’ mà ' i ad ⋮ ( 1; in = ) thì ' dd ⋮
Ta viết ( ) 12;;.; ndaaa =…
b. Số nguyên dương m gọi là BCNN của 12;;.; naaa nếu thỏa mãn đồng thời hai điều kiện:
+/ (1;) i main ∀= ⋮
+/ Nếu có '(1;)ì' i mainthmm ∀= ⋮⋮
Ta viết [ ] 12 ; ;.; nmaaa =…
Các tính chất cơ bản
a. ( ) ( ) 1111;.; . ;1 saocho abdadabdbab =⇔===
b. [ ] ( ) 1111 ; .và. 1 saoh; co mabmabmbaab =⇔===
c. ( ) [ ] ;.;. ababab =
d. ( ) ( ) ( ) ( ) ;;;; ; abaababaab =+=−
e. ( ) ( ) [ ] [ ] .;..;; ;; kakbkabkakbkab ==
f. .và(;)1thì abcacbc = ⋮⋮
g. Nếu ( ) ; abd = luôn tồn tại hai số nguyên x; y sao cho axbyd += .
Thuật Toán Euclide:
Bổ đề cơ sở: Nếu abqr =+ thì ( ) ( ) ;; abbr =
Tiếp tục quá trình đó ta có thuật toán: ( ) ( ) ( ) ( ) 1121;=; ; ; nnn abbrrrrrr ==== … (với 1 nn rkr = )
I.3. ĐỒNG DƯ
Định nghĩa: Giả sử a và b là hai số nguyên. Ta nói a đồng dư với b modulo m nếu ) |( mab .
Ta viết: ( ) mod abm ≡
Tính chất: Cho p là số nguyên tố
- Nếu ( ) mod abm ≡ thì abkm =+ ( k là số nguyên)
- Nếu ( ) mod abm ≡ thì ( ) mod bam ≡
- Nếu ( ) mod abm ≡ và ( ) mod bcm ≡ thì ( ) mod acm ≡
- Nếu ( ) mod abm ≡ và ( ) mod cdm ≡ thì ( ) mod acbdm +≡+
- Nếu p nguyên tố và ( ) .0mod abp ≡ thì ( ) 0mod ap ≡ hoặc ( ) 0mod bp ≡
- Nếu ( ) ( ) ( ) modthìmod;mod abmacbcmacbcm ≡+≡+−≡−
- Nếu ( ) mod acbcm ≡ và ( ) ;1cm = thì ( ) mod abm ≡
- Nếu ( ) mod abm ≡ với k > 0 thì ( ) mod kk abm ≡
- Nếu ( ) mod(1;) i abmin ≡= thì ( ) 12 mod[....] nabmmm ≡
- ()()mod(0) n n abbaa +≡>
I.4. CÁC ĐỊNH LÝ CƠ BẢN CỦA SỐ HỌC
Phân tích chuẩn của số nguyên dương:
Định lý: Cho số nguyên dương n > 1, khi đó n luôn biểu diễn một cách duy nhất ở dạng phân tích chuẩn (chính tắc) 12 12..... k knppp α αα = với pi là các số nguyên tố, i N α ∈ (với 1; in = )
+ Cho hai số nguyên dương 1212 1212 .....à.....(0;0;1;) kk ii kk apppvbpppik ααββαβ αβ ==≥≥= thì () () [] () 12 12 12 12 ;.....(min;)à;.....(ax;) k k x y xx yy iii iii k k abpppxvabpppym ==== αβ αβ
+ Định lý cơ bản về liên hệ giữa chia hết và số nuyên tố: Cho hai số nguyên dương
a; b và số nguyên tố p. Nếu . abp ⋮ thì ap ⋮ hoặc bp ⋮
Định lý Fermat: Nếu p nguyên tố và số nguyên a tùy ý thì ( ) p aap ⋮
Nếu (a; p) = 1 thì ( ) 1 1 p ap ⋮ hay ( ) 1 1mod p ap ≡
Định lý Euler: Cho m là số nguyên dương và ( ) ;1am = thì ( ) () 1mod m am ϕ ≡ ,
trong đó ( )m ϕ là số các số nguyên dương nhỏ hơn m, nguyên tố cùng nhau với m. ( ( ) ( )mhaym ϕ Φ gọi phi hàm Euler)
Định lý phần dư Trung Hoa: Cho n số nguyên dương: 12;;; nmmm … nguyên tố
xam
xam xam
có duy nhất mộ
(mod) (mod) ....................... (mod)nn t
() 12
111 1.1..........1 k nn ppp ϕ =−−− với 1
i k i i np σ =
= ∏
Định lý Wilson: p là số nguyên tố khi và chỉ khi ( ) 1!1 p −+ chia hết cho p (hay ( ) ( ) 1!1mod pp −≡− )
158
I.5. CÁC HÀM SỐ HỌC
a/ Hàm () n σ (tổng các ước số của n)
Cho số nguyên dương n, khi đó () n σ là tổng tất cả các ước tự nhiên của n (kể cả 1 và n)
+ hàm () n σ có tính chất nhân tính: 1212 (.)().() nnnnσσσ = (với (n1; n2) =1)
+ Nếu p nguyên tố thì ()1 pp σ =+
+ Nếu p nguyên tố thì ( ) 2 p τ =
+ Nếu 12 12 k knppp α αα = thì ( ) ( )( ) ( ) 1211....1 k n τααα=+++
e/ Hàm phần nguyên [ ]x
Định nghĩa:
- Cho x là số thực, ta gọi phần nguyên của x là số nguyên lớn nhất không vượt quá x, ký hiệu là [ ]x
+ Nếu 12 12 k knppp α αα = thì () 12 1 11 12 12
b/ Phi‒hàm Euler ( )n ϕ hay ( )n φ
α αα σ + ++ =
ppp n ppp
1 11 ..... 111 k k k
Cho n là số nguyên dương, ta định nghĩa ( )n ϕ là số các số không vượt quá n và nguyên tố cùng nhau với n.
+ hàm phi ( )n ϕ có tính chất nhân tính ( ( ) ( ) ( )ababϕϕϕ = )
+ Nếu 12 12 k knppp α αα = thì () 12
111 11....1 k nn ppp ϕ
+ Đặc biệt nếu p là số nguyên tố thì ( ) 1 pp ϕ =− và ( ) 1 nnn ppp ϕ =−
c/ Hàm S(n) tổng các chữ số của n
Cho n là số nguyên dương viết trong hệ thập phân, S(n) là tổng tất cả các chữ số của n
Các tính chất:
+ ( ) 0 Snn<≤ , với mọi n
+ ( ) 0 9 Snnn=⇔<≤
+ ( ) ( ) ( )SmnSmSn +≤+
+ ( ) ( ) ( ) .. SmnSmSn ≤
d/ Hàm ( )n τ (số lượng các ước số của n)
Cho n là số nguyên dương, ký hiệu ( )n τ là số lượng các ước số nguyên dương của n
Các tính chất:
+ hàm ( )n τ là hàm nhân tính
160
- Cho số thực x, ta gọi phần lẻ hay phần phân của x (ký hiệu là{ }x ) là số được
định nghĩa như sau: { } [ ]xxx =−
- Cho số thực x, ta gọi số ( )x là số nguyên gần x nhất, trong trường hợp có hai số nguyên cùng cách đều x ta quy ước chọn số nguyên lớn hơn.
Các tính chất cơ bản của [ ] { } ( ) ;, xxx
+ [ ] xaxad =⇔=+ với [ ] 01; 1 dxxx ≤<−<≤
+ [ ] xyx += thì x là số nguyên và 01 y ≤<
+ Nếu n là số nguyên thì [ ] [ ]nxnx +=+ và { } { }nxx +=
+ [ ] [ ] [ ]xyxy +≥+
+ nếu n là số nguyên dương thì [ ]x x nn
+ Nếu n là số tự nhiên [ ] [ ]nxnx ≤
+ Với mọi số tự nhiên n và q ≠ 0 thì n qn q
với n là số nguyên dương
+ Trong dãy n số tự nhiên 1;2;3;…;n có đúng n q số tự nhiên chia hết cho số tự nhiên 0 q ≠
+ (Định lý Legendre) Nếu số nguyên tố p có mặt trong phân tích ra thừa số nguyên tố !1.2.3 nn =… thì số mũ cao nhất của p bằng
161
p
=+++=
2 () (!).... 1 p p k nSn nnn vn pp pp
tổng các chữ số của n viết tong hệ cơ số p)
+ () 1 2 xx =+ , với mọi số thực x
I.6. SỐ MŨ ĐÚNG, ƯỚC ĐÚNG
Định nghĩa: Cho số nguyên tố p, số nguyên dương a và αlà số tự nhiên. Ta nói
α là lũy thừa đúng của a và α là số mũ đúng của p trong khai triển của a nếu
1 |à|pavp αα+ a , khi đó ta viết || pa α và ký hiệu ( ) p va α =
Nhận xét: Nếu 12 12...... k knppp α αα = thì ()(1;) i pi vnik α ==
Nếu ( ) ;1ap = thì ( ) 0 p va =
Tính chất: Cho a và b là các số nguyên dương, khi đó ta có:
+ Nếu ( ) p va α = thì . apk α =
+ Nếu và || pa α và || pb β thì || pab αβ + (hay ( ) ( ) ( ) = và ppp vavbvabααβ β = = )
+ Nếu || pa α thì || kk pa α (hay hay () ( ) = k pp vavak α α = )
+ Nếu || pa α và || pb β với αβ ≠ thì min{;} || pab αβ +
Tổng quát: ( ) ( ) ( ) { ; } ppp vabminvavb +≥
+ Nếu ( ) ( ) |thì pp nmvnvm ≤
+ Định lý 1: (định lý 1 về số mũ đúng LTE: Lifting The Exponent)
Cho hai số nguyên x; y (không nhất thiết dương) và n là số nguyên dương, gọi p
là số nguyên tố lẻ:
‒ Nếu ( ),| v||à pxypxpy // . Khi đó: ( ) ()() nn ppp vxyvxyvn =−+ .
‒ Nếu ( ),| v||à pxypxpy + // , với n lẻ. Khi đó: ( ) ()() nn ppp vxyvxyvn +=++ .
+ Định lý 2: (định lý 2 về số mũ đúng LTE)
Cho x; y là hai số nguyên dương
Nếu ( ) ()()222 4 nn xyvxyvxyvn −=−+ ⋮
⋮
+ Định lý Legendre: Nếu số nguyên tố p có mặt trong phân tích ra thừa số nguyên tố
!1.2.3 nn =… thì số mũ cao nhất của p bằng
=+++=
tổng các chữ số của n viết trong hệ cơ số p)
I.7. CẤP (hay BẬC) CỦA MỘT SỐ NGUYÊN – CĂN NGUYÊN THỦ
Y
dương k nhỏ nhất thỏa mãn ( ) 1mod k an ≡ . được gọi là cấp của a modulo n.
Kí hiệu: (). n korda =
Định lý 1: Cho a, n thỏa mãn ( ) ,1. 1; an n = > Khi đó:
( ) ( ) 1mod . x n anxorda ⇔ ≡ ⋮
Định lý 2: Nếu () n korda = thì ( ) mod ij aan ≡ khi và chỉ khi ( ) mod ijk ≡
Hệ quả: Nếu a có cấp k theo modn thì các số 2 ; ;...; k aaa không đồng dư với nhau từng đôi một theo modulo n
Định lý 3: Nếu số nguyên a có cấp k theo mod n và 0 h > thì h a có cấp là (;) k hk theo mod n
Định lý 4: Nếu () n korda = và ( ) n hordb = và ( ) ;1hk = thì ( ) n ordabhk =
Hệ quả: Nếu a;n;m là các số nguyên dương đôi một nguyên tố cùng nhau và ( ) n horda = và ( ) m korda = thì a có cấp [ ] ; hk theo modulo mn .
Hệ quả: Cho a, n thỏa mãn ( ) 1;,1 a nn > = . Khi đó: ϕ(n) ⋮ ( ) ( ). n norda ϕ ⋮
2. Định nghĩa 2: Cho n > 1 và a là một số nguyên dương, ( ) , 1 an = . Nếu ( ) ( ) n norda ϕ = thì a được gọi là một căn nguyên thủy modulo n.
Nhận xét: Từ định nghĩa trên ta dễ dàng suy ra: nếu a là căn nguyên thủy (mod n) thì mọi số cùng lớp với a theo (mod n) đều là căn nguyên thủy (mod n).
Định lí 5: Nếu a là căn nguyên thủy (mod n) thì tập { }211,,,, hAaaa =… là hệ thặng dư thu gọn (mod n) (lúc này ( )hn ϕ= )
163
Định lí 6: Nếu p là một số nguyên tố thì có đúng ( )1 p ϕ căn nguyên thủy (mod p)
Định lí 7: Nếu p là một số nguyên tố lẻ và a là một căn nguyên thủy (mod p2) thì a cũng là căn nguyên thủy (mod pn) với 3 n ≥ .
3. Định lý về sự tồn tại căn nguyên thủy:
Cho m là một số nguyên, m > 1 khi đó m có căn nguyên thủy khi và chỉ khi m có một trong 4 dạng sau: 2,4,,2 a a pp (trong đó p là số nguyên tố lẻ).
II. CÁC DẠNG TOÁN VỀ ƯỚC SỐ
II.1. SỐ NGUYÊN TỐ VÀ ƯỚC NGUYÊN TỐ TRONG GIẢI TOÁN
Từ định nghĩa về số nguyên tố là số tự nhiên lớn hơn 1 có đúng 2 ước số. Hai ước số đó là số 1 và chính số đó. Suy ra số tự nhiên n không có ước số từ 1 đến n là số nguyên tố. Trong các mối quan hệ về chia hết cho số nguyên tố, ta sẽ gặp một số kiến thức liên quan đến định lý Fecma nhỏ, định lý Wilson,…
Một tính chất thường được sử dụng trong giải toán về số nguyên tố là: nếu số nguyên tố p là ước số của tích a.b thì p là ước của a hoặc của b.
Với tính chất đặc biệt của ước số nguyên tố, ta có một số dạng toán thường gặp sau:
II.1.a. Tìm số nguyên tố thỏa điều kiện nào đó.
Ta sử dụng tính chất số nguyên tố chỉ có hai ước số là 1 và chính nó.
Bài toán 1. Tìm các số nguyên tố p để:
a) Các số 10; 14 pp++ cũng là các số nguyên tố.
b) Các số 2; 6; 8; 12; 14 ppppp +++++ cũng là các số nguyên tố
Tiếp cận bài toán: Ta thấy ba số ;10 ;14ppp++ đều nguyên tố mà 10 và 14 chia cho 3 dư 1 và 2, nên xét chia lớp cho 3, ta có lời giải sau:
Bài giải:
a) Với 3 p = : ta có 1013 p += là số nguyên tố; 1417 p += là số nguyên tố
Với 2 p = thì 10; 14 pp++ không nguyên tố
Với p > 3 thì p có 2 dạng 31;32pkpk=+=+ :
+ Nếu 31pk=+ thì 14315pk+=+ có ước số là 3, chứng tỏ số 14 p + có nhiều hơn 2 ước số nên 14 p + không là nguyên tố.
+ Nếu 32pk=+ thì 10312pk+=+ có ước số là 3 nên 10 p + không là số nguyên tố
Vậy 3 p = thỏa mãn điều kiện bài toán.
b) Tương tự câu a ta có số 5 p = thỏa mãn yêu cầu bài toán.
Lời bàn: Dự đoán số nguyên tố 3 p = và kiểm tra đúng,nếu 3 p ≠ trong các số còn lại luôn có một số là hợp số vì có ước thật sự là 3. Sau đây là bài tương tự.
Bài toán 2. Tìm số nguyên tố p để
a) các số 2 8 1 p + và 2 8 1 p đều là số các số nguyên tố
164
b) các số 8p2 + 1 và 2 821 pp++ đều là số các số nguyên tố
Bài giải:
a) Giả sử p là số nguyên tố lớn hơn 3 thì p có dạng 31 k ± ( k ∈ ℕ )
()2 22 3133 2)1 ( 31pkkkt =±=±+=+
+=++=+ + ⋮ là hợp số (trái giả thiết)
( ) 2 2 818311249381 pttp
Do đó: 3 pk = , p nguyên tố 3 p = và
+ 22 8 18.3 ++173 p == (nguyên tố)
+ 22 818.3171 p == (nguyên tố)
Vậy p =3 thì 2 8 1 p + và 2 81 p là các số nguyên tố.
b) Tương tự trên nguyên tố 3 p = thỏa mãn đề bài.
Bài toán 3: Tìm tất cả các số nguyên n sao cho:
a. Số 4 4 n + là số nguyên tố.
b. Số 20212020 1 nn + + là số nguyên tố.
Tiếp cận bài toán: Các đa thức trên có thể phân tích được thành tích nên khai thác tính chất số nguyên tố chỉ có đúng 2 ước số, trong đó có một ước số bằng 1. Bài toán sẽ giải được
Bài giải:
a. Ta có: 422222 4(2)–4 (22)22) ( nnnnnnn +=+=+−++
Nhận xét rằng 22 2–2 22 nnnn +<++ và 22 2–2; 22 nnnn +++ là 2 ước số
của 4 4 n +
+=
Từ ()2 2 2–21–1 0 1. = nnnn += = Khi đó: n4 + 4 = 5 là số nguyên tố.
Vậy n = 1, n = -1 thỏa mãn bài toán.
( ) ( ) ( ) ( )( ) ( )
2021202020122920192019 22 111111 nnnnnnnnnnnnn +=−+−+++=+−+ + ++
6736 2019 3 333372671111 ....... 1 ( ) nnnnnn
()()()
()()()()() 672671 23332 1 1....... 11 + nnnnnnnn = −+++ + +++ ⋮
Do 2 0 11 nnn > ++> , suy ra 2 1 nn++ là ước thực sự khác 1 và khác
() 20212020 1 nn++ .
Vì vậy để 20212020 1 nn++ là số nguyên tố thì
202102022110; 1 nnnnnn +=++ + ==±
Kiểm tra ta thấy khi 1 n = thì 20212020 13 nn + += là số nguyên tố.
Lời bàn: Các đa thức phân tích được thành tích, dĩ nhiên phải có thừa số bằng 1 hoặc chính nó. Bài toán trở nên dễ dàng.
Bài toán 4: Cho 2–1 n là số nguyên tố, chứng minh rằng n là số nguyên tố.
Tiếp cận bài toán: Thử phản chứng! Nếu n là hợp số thì 2–1 n sẽ có dạng tích, và có nhiều hơn hai ước.
Bài giải:
Giả sử n là một hợp số thì ) =. ;1 (,; npqpqpq n >>∈ ℕ
==−=−+++
Do 1211 p p > −> và ( 1 2 ()) 22...11 pqpq+++> 21 n là hợp số. Điều này trái với giả thiết.
Nếu 12–11 n n = = không phải là số nguyên tố.
Vậy 2–1 n là số nguyên tố n là số nguyên tố.
Lời bàn: Chỉ cần xét mệnh đề tương đương, bài toán có hướng đi quen thuộc!
Bài toán 5. Chứng minh rằng nếu: 124() nn n + ++∈ ℤ là số nguyên tố thì 3k n = với k∈ ℕ .
b. Ta có ( ) ( ) ( ) 20212020220212022 0 1 1 + nnnnnnnn +=−+−+++ 167
Tiếp cận bài toán: Như bài toán 4, ta xét mệnh đề tương đương.
166
Nếu n có dạng ( ) 3. k nmk=∈ ℕ , ( ) 1,;31mm≥= , ta cần chứng minh mọi ước
số m của n ( ( ) 1,;31mm≥= ) không làm cho 124() nn n + ++∈ ℤ trở thành số
nguyên tố, tức là nó có ước thực sự khác 1 và chính nó.
Bài giải:
Đặt 3. k nm = với k∈ ℕ, ( ) 1,;31mm≥= và 32 k a = ( 2; aa≥∈ ℕ )
Do ( ) ;31m = , giả sử 1 m > , ta xét 2 trường hợp sau: 31mx=+ và 32mx=+ với x ∈ ℕ
+ Nếu 31mx=+ , ta có:
Vậy các số 123;;;...aaa đều là các số nguyên tố.
Lời bàn: Từ nhận xét ii apd −= không đổi, loại 0 d > nhờ nhận xét không thể xảy ra |mà;2 papaap ≠< khá đơn giản và tinh tế!
Bài toán 7: Tìm 3 số nguyên tố p; q; r sao cho qp pqr += .
Tiếp cận bài toán: Từ giả thiết dễ dàng suy được tính chẵn lẻ của số nguyên tố r, khi đó suy được tính chẵn lẻ của p và q. Do đó biết được 2 là một nghiệm của bài toán. Khi đó bài toán trở nên dễ hơn vì chỉ còn hai biến.
Bài giải:
Giả sử tồn tại 3 số nguyên tố p; q; r sao cho qp pqr += , khi đó 3 r > suy ra r lẻ.
kk nmmxx xx
1124241 1111
+ +−+
+ −+ ⋮
++=++=++ =+++
333162 23262
n aa aaaaaaaa
()()()
Ta có: 31622 111 xx aaaa ++ ++>++> nên 2 1 aa++ là ước số thật sự của
3162 1 xx aa ++++ . Do đó 124nn++ là hợp số, mâu thuẫn giả thiết.
+ Nếu 32mx=+ , tương tự trên cũng dẫn đến 124nn++ là hợp số, mâu thuẫn!
Như vậy 1 m = hay 3k n = với k∈ ℕ .
Lời bàn: Biểu thức 124nn++ có dạng 2 1 xx++ nên khi xét hai trường hợp m không chia hết cho 3 thì biểu thức sẽ có ước thực sự dạng 2 1 xx++
Bài toán 6. (Olympic Liên bang Nga-2018) Giả sử rằng 123;;;...aaa là dãy vô hạn các số nguyên dương tăng nghiêm ngặt và 123;;;...ppp là dãy các số nguyên tố sao cho | kkpa với mọi số nguyên dương k. Biết rằng nknk aapp −=− với mọi số nguyên dương n và k. Chứng minh rằng tất cả các số 123;;;...aaa đều là các số nguyên tố
Tiếp cận bài toán: Từ yêu cầu của đề bài, để 123;;;...aaa đều là các số nguyên tố thì khả năng cao dãy số đó cũng chính là dãy số nguyên tố 123;;;...ppp tức là ii ap = .
Lại có từ giả thiết thì hiệu ii ap không đổi. Bài toán có hướng giải quyết!
Bài giải:
Từ giả thiết ta có: * (;) nnkk apapnkN −=−∀∈
Nên gọi 11 0, nn dapapn =−=−≥∀
Nếu 0 d = thì các ai đều là các số nguyên tố.
Nếu 0 d > khi cho n đủ lớn ta có: nnnn dapapd =− =+
Ta có 2 nnnn papdp <=+< (*). Không thể xảy ra BĐT (*) vì | nnpa
168
Như vậy p; q khác tính chẵn, lẻ nên phải có một số chẵn và là nguyên tố nên số đó là 2, giả sử 2 p = khi đó 2 2q qr+=
Nếu 3 q > thì ( ) 2 1mod3 q ≡ lại có q lẻ nên 3 là ước của 21 q + , suy ra 3 là ước số của 2 2q q + . Khi đó: 3 là ước của r nên r không nguyên tố, mâu thuẫn giả thiết!
Do vậy 3 q⋮ , q nguyên tố nên 3 q = , khi đó: r = 23 + 32 = 17 nguyên tố.
Vì p, q có vai trò như nhau nên có thể 3;2pq==
Vậy có 2 bộ số được tìm là ( ) 2; 3; 17 và ( ) 3; 2; 17 .
Lời bàn: Nhờ đánh giá tính chẵn lẻ ta tìm được một nghiệm p=2, sau đó xét chia lớp mod3 ta có lời giải dễ hiểu và ngắn gọn!
Bài toán 8: (Hy lạp ‒ 2015) Tìm tất cả bộ ba số nguyên dương ( ) ;; xyp với p là số nguyên tố thỏa mãn: () 3 1 xy p xy = +
Tiếp cận bài toán: Ở vế trái của biểu thức (1) là phân thức có thể chưa tối giản, rõ ràng ta phải dùng ước chung của x;y để thu gọn biểu thức lúc đó dễ giải quyết vấn đề hơn!
Bài giải:
Gọi ( ) ; dxy = nên tồn tại các số nguyên dương ; ab sao cho ; xdaydb == và ( ) ;1ab = .
Do đó từ (1) ta nhận được: () 33 2 dab p ab = +
Từ ( ) ;1ab = ta được ( ) ;1aab+= và ( ) 3;1bab+= , nên từ (2) ta có 3 | abd + .
169
Do đó ta viết: 3 d k ab = + với ( ) * 3 k ∈ ℕ
Khi đó (2) trở thành: 3 kabp = và từ đây có: 3 1 | bpb = và kap = .
Ta có các trường hợp sau:
+) Nếu ,1kpa== thì (3) trở nên: 3 33 22|hay8|và8|2 2 d ppdddp = = (vô lý!)
+) Nếu 1 k = và ap = thì (3) trở nên: ()( ) 3321111 = dpdpdddp + −= −++=
Mà 2 11ddd + + >− nên ta được: 11 d −= và 2 1 =2và 7 ddpdp ++=⇔=
Vậy bộ ba số là ( ) ( ) ;;14;2;7.xyp =
Lời bàn: Khai thác tính chất đặc biệt về ước số của số nguyên tố là chỉ có 2 ước là 1 và chính nó, ta giải quyết được lớp các bài toán khá nhẹ nhàng và thú vị!
II.1.b. Dùng tính chất ước của một tích.
Một tính chất thường được sử dụng trong giải toán về số nguyên tố là: nếu số nguyên tố p là ước số của tích nhiều thừa số thì p là ước của ít nhất một trong các thừa số đó. Ta xét một số bài toán sau:
Bài toán 1. (Chọn đội tuyển KHTN Hà nội -2020-2021) Tìm tất cả các cặp số nguyên dương a,b sao cho 32 3 ab += và 2 2()aab ++ là số nguyên tố
Tiếp cận bài toán: Mối liên hệ giữa a và b từ hai dữ kiện của đề ta kết nối thành một biểu thức để khai thác tính chất số nguyên tố
Bài giải:
Đặt 2 2()paab =++ thì 2 22(mod) aabp +≡−
Khi đó: ( ) 2223 (2)443(mod) aabap +≡≡+
Suy ra: 223 (2)4120(mod) aaap +−−≡
Mà ( )( ) 22322 (2)41226 aaaaa +−−=−+ chia hết cho p suy ra p là ước số của
2 2 a hoặc 2 6 a + .
Ta xét hai trường hợp sau:
+ Trường hợp 1. Nếu ( ) 22 |22 paaabp −<++= , chứng tỏ 2 201aa
= ( )|1 p vô lý.
+ Trường hợp 2. Nếu 2 |6pa + , từ 32333132abba += =+≥+=
Mà ( ) 2222()2126paabaa =++≥++=+ , kết hợp với 2 |6pa + ta đượ
2 6 pa=+ . Như vậy các dấu đẳng thức xảy ra nên 1;27abp == = . Cặp số
( ) ( );1;2ab = thỏa mãn đề bài.
Kết luận: ( ) ( );1;2ab =
Lời bàn: Khử b từ hai điều kiện của đề cho ta biểu thức chỉ có a và số nguyên tố p giống phương trình nghiệm nguyên khá thuận lợi cho việc giải.
Bài toán 2. (Đề đề nghị trường THPT chuyên Nguyễn Tất Thành, Yên Bái, lớp 11‒2018) Xác định tất cả các số nguyên tố , pq sao cho 21311 11 n pq pq
+ = với 1 n > , . n ∈ ℤ
Tiếp cận bài toán: Hai phân thức ở hai vế đều rút gọn được, nhưng biểu thức sau rút gọn vế trái khá dài và phức tạp. Nếu trừ 1 vào hai vế thì hai tử số đều có dạng tích, khá thuận lợi để đi tiếp!
Bài giải:
++ =⇔−−=−−
( )( ) ()() 1111nn ppppqq ⇔−+=−+ (1)
Nếu 1 n qp≤− thì các thừa số ở vế trái lớn hơn các thừa số tương ứng ở vế phải của (1), do đó n qp ≥
Vì q nguyên tố, còn n p không nguyên tố nên 1. n qp≥+
Từ (1), số nguyên tố q là ước số của vế trái, mà 1 n qp≥+ , điều đó xảy ra khi 1 n qp=+
Thay vào (1) ta được: ( ) ()( )112nn pppp−=−+ , suy ra 320 n pp−+=
Từ đó 2 p mà p nguyên tố nên 2,2.pn== Suy ra 15. n qp=+=
Lời bàn: Từ việc thêm bớt khá tinh tế ở biểu thức giả thiết, kết hợp với suy luận ước số nguyên tố ta có lời giải đẹp!
Bài toán 3. Cho số nguyên tố lẻ p có dạng 21 t pk=+ với t ∈ ℕ , k là số tự nhiên lẻ. Chứng minh rằng nếu có hai số tự nhiên a;b thỏa mãn p là ước số của 22 tt ab + thì p là ước số đồng thời cả hai số a;b.
Tiếp cận bài toán: Hướng tiếp cận bằng phản chứng có lẽ phù hợp cho bài này. Khi p không là ước của a và b thì ta có thể áp dụng định lý Fermat nhỏ, chắc chắn sẽ dẫn đến mâu thuẩn gì đó. Thử đi hướng này!
Bài giải:
Ta chứng minh bằng phản chứng. Giả sử p không là ước của a, từ giả thiết suy ra p cũng không là ước của b. Theo định lý nhỏ Fermat ta có: ( ) ( ) 111mod;1modpp apbp ≡≡ hay: ()()221mod;1mod tt kk apbp ≡≡
Chứngtỏ p là ướcsố của nnad + hoặc nnbd + .
Mà ta thấy: 1;nnnnnnnn adbdabcdp <++<+++= , điều này không thể xảy ra.
Vậy không tồn tại n để nnnn abcd +++ là số nguyên tố, hay nó luôn là hợp số với mọi n
Lời bàn: Sử dụng điều vô lý để có lời giải hay: “Số nguyên tố p là ước của một trong hai số nnad + ; nnbd + mà hai số đó lại nhỏ hơn p”.
Bài toán 5. (St Petersburg-2001)
a/ Tìm tất cả các cặp số nguyên dương ( ) ; ab với ab ≠ sao cho ( ) 22()baab ++ và 2 ba + làlũythừacủasố nguyêntố.
22222mod2mod ttttkk kk abpabp +≡⇔+≡
suy ra: () ( ) ( ) ()
Mà k lẻ nên ( ) ( ) ( ) ()() 2222 ;2mod tttt kk ababPabp +=+≡ (*)
Theo giả thiết () 22 0mod tt abp +≡
Nên ( ) ()() 22 ;0mod tt abPabp +≡ mâuthuẩnvới(*)
Vậy điềugiả sử trêncủatalàsai.
Do đónếu p là ướccủa a thìtừ giả thiếtsuyra p cũnglà ướccủa b.(đpcm)
Lời bàn: Khai thác yếu tố 22 tt ab + là ước của ap-1 + bp-1 khi k lẻ, ta sử dụng phản chứng là phù hợp và hay nhất.
Bài toán 4. Cho bốn số nguyên dương ;;; abcd thỏa mãn abcd = . Chứng minh rằngkhôngtồntạisố nguyêndương n để nnnn abcd +++ làsố nguyêntố.
Tiếp cận bài toán: Từ giả thiết ta có thể làm xuất hiện nnnn abcd +++ ở dạng tích để khaitháctínhchất đặcbiệtvề ướccủasố nguyêntố.
Bài giải:
Giả sử tồntạisố nguyêndương n saocho nnnn abcdp +++= làsố nguyêntố
Từ giả thiết: abcd = suyra
()() nnnnnn ababcdcd === ( ) 2 nnnnnnnnnnnnn pdabcddadbdcdd =+++=+++
( )( ) 2 ()() nnnnnnnnnnn adabbddadbd =+++=++
b/ Cho ;1ab > là hai số nguyên dương phân biệt thỏa mãn ( ) ( ) 2211baab+−+− . Chứng minh rằng 2 1 ba+− có ít nhất hai ước số nguyêntố phânbiệt.
Tiếp cận bài toán: Ta có 2 ba + có dạng m p , nên m p là ước của cả 2 ba + và
2ba + , ta phải làm xuất hiện m p là ước của biểu thức chỉ có một trong hai biến a;b. Khi đó,dựavàotínhchấtnguyêntố sẽ cóhướnggiảiphùhợp! Bài giải:
a/+Xét 1 b = thì ( ) ()()() 2 (1)|11|112 aaaaa ++=>+−++
Suyra ( ) 1|2 a + do đó 1 a = ,khôngthỏa điềukiện ab ≠ .
+ Xét 1 b > , theo đề bài tồn tại số nguyên tố p và số nguyên dương m sao cho 2 m bap += .
Lạicó: ( ) ( )( ) ( ) 34422222 1 bbbbbaabbabaab +=+=−++=+−++
Tacó: 2 ba + là ướccủavế phải,nên ( ) ( ) ( ) 23311 m babbpbb ++ +
Lại có: ( ) 3 gcd;11 bb += nên suy ra 2 ba + là ước của b hoặc 3 1 b + , mà
2 bba <+ nên ( ) ( ) ()( ) 2322(1)hay11 babbabbb ++++−+
Dễ thấycả haithừasố () ( ) 2 1;1bbb+−+ đềunhỏ hơn 2 ba + nêncả haithừasố
đó đềukhôngchiahếtcho m p .Như vậy ( )|1pb + và ( ) 2 |1pbb−+
Từ đósuyra: ( ) ()() 2 |112 pbbbb −+−+− hay |3 p ,chứngtỏ 3 p = .
Do 1 b > nên 2 5 ba+≥ mà 2 m bap += suyra 1 m >
- Nếu 2 m = thì 22 3 m bap+== suyra 5;2ab==
- Nếu 3 m ≥ , từ ( ) ()() 2 1123bbbb−+−+−= nên trong hai số
() ( ) 2 1;1bbb+−+ có một số chia hết cho 3, số kia chia hếtcho 1 3m . Ta có:
Lại có từ giả thiết: ( ) ( ) 2211baab+−+−
Suy ra: ( ) ( ) ()() 22 0111abbaababab ≤+−−+−=−+− ≥
Như vậy: ab = (mâu thuẩn điều kiện ab ≠ ), chứng tỏ điều giả sử sai.
Vậy 2 1 ba+− có ít nhất hai thừa số nguyên tố.
Lời bàn: Bàitoánnàykhó!Bắt đầutừ biến đổi ( ) ( )( ) ( ) 3222 1 bbbabaab +=+−++
lý!
22
3m bba<+= nên /211313mm b +<+<
Như vậy ( ) 12 3|1 m bb−+
Lại thấy ( ) ()2 22 3|419|213 bbb −+ −+ suy ra ( )3|21 b , dẫn đến 9|3 vô
Vậychỉ cómộtcặp ( ) ( );5;2ab = thỏamãn.
b/ Ta chứng minh bằng phản chứng. Giả sử 2 1 ba+− chỉ có một ước số nguyên tố, như vậy 2 1 ba+− phải là lũy thừa của một số nguyên tố, tức là: 2* 1, m bapm+−=∈ ℕ
Tacó ( )2 222 11 baba −+=−−
Mà: ( ) ( ) 2211baab+−+−
Nên: ( ) ( ) ( ) () 22222222 11111 babaabbb −+−−++−=−++
Suyra: ()( ) 222111babbbb −+−+−
Tanhậnthấy: ( ) 2 gcd;1;11 bbbb−+−= vì ( )( ) 2 1211bbbb+−=+−+
Do đó: 2 1 ba+− là ước của một trong ba thừa số 2 ;1;1 bbbb−+− vì 2 1 ba+− làlũythừacủamộtsố nguyêntố.
Dễ dàngkiểmtra 2 11bbba−<<+− nênchỉ có 2211 babbab +−+− ≤
để có ()( ) 2 11 m pbbb+−+ khi đó nó trở nên quen thuộc hơn! Câu b dựa ý tưởng câu a.
Bài toán 6. Tìm các số nguyên tố p,q thỏa mãn ( )( ) 7575 ppqq chia hết cho pq.
Tiếp cận bài toán: Rõ ràng phải xét p (hoặc q) là ước của một trong hai thừa số và có “bóng dáng” của định lý Fermart nên có hướng giải quyết.
Bài giải:
Ta có nhận xét quen thuộc sau: Cho ( ) ( ) 75;75 xxyypp ⋮⋮ với x nhỏ nhất
thì yx ⋮
Áp dụng:
Ta thấy p, q ≠ 5;7
Trường hợp 1: ( ) 75pp p ⋮
Theo định lí Fermat nhỏ: ( ) ( ) 11 71mod;51mod pppp≡≡
Suy ra: ( ) 1175pp p ⋮ ( ) 1 75.7 pp p ⇔− ⋮ 1 2.522 p ppp = ⋮⋮
Khi đó: ( )( ) () 22 7575 1275 2
qq qq qq ⋮⋮ { } 1 12.2.512.22;3 q qqq ∈ ⋮⋮
Trường hợp 2: Nếu 75pp và p nguyên tố cùng nhau thì 75qq p ⋮
- Nếu 75qq q⋮ thì tương tự như truongf hợp 1: 75pp p ⋮
- Nếu 75qq nguyên tố cùng nhau với q thì 75pp q⋮
Như vậy: 75pp q⋮ và 75qq p ⋮
Gọi 75kk q⋮ với k đạt GTNN khi ấy theo nhận xét trên thì p⋮k nên mà p nguyên tố nên 1 k = hoặc kp =
Khi 1|22kqq = = và dễ tìm ra p.
Khi kp = 75pp q ⋮ với p là số nhỏ nhất thỏa đề, khi đó dẫn đến vô lý vì vẫn còn số nhỏ hơn thỏa mãn là 1175qq q⋮ (theo Fermat nhỏ) nên vô lý.
Do đó trường hợp này có nghiệm giống trường hợp 1.
Vậy (p,q)=(2,3),(3,2),(2,2).
Lời bàn: Một số nguyên tố là ước của một tích hiển nhiên nó phải là ước của ít nhất một trong các thừa số đó. Sự vận dụng khéo léo và linh hoạt cho ta giải quyết một số bài toán về chia hết khá đẹp!
Sau đây là bài toán mà việc chứng tỏ sự tồn tại ước số nguyên tố p trên cơ sở xây dựng
khá công phu dãy các số nguyên phức tạp.
Bài toán 8.( SAUDI ARABIAN-TST -2021)
Cho số nguyên dương k chứng tỏ rằng tồn tại số nguyên tố p mà ta có thể chọn được các số nguyên khác nhau 123 ;;...;{1;2;...;1} k aaap + ∈− sao cho p là ước của
123iiii aaaai +++ với tất cả 1;2;..; ik =
Tiếp cận bài toán: Đây là bài toán quá khó! Không thể định hướng để tiếp cận ngay.Từ p là ước của tích 123iiii aaaai +++ điều đó tương đương với ( ) 123.modiiii aaaaip +++ ≡ .
Với mỗi số nguyên i mà có tích 4 số nguyên khác nhau đồng dư với i modp. Ta bắt đầu với trường hợp đặc biệt tích 4 số nguyên abcd = i ( 1;2;..; ik = ) thì điều này chưa chắc đúng. Nhưng tích 4 số hữu tỉ bằng số i thì có thể! Và ta có thể xây dựng được các số đó , bằng cách cho 3 số nguyên a;b;c còn số hữu tỉ i d abc = . Nhóm này có 4 số như vậy xây dưng các số sau truy hồi theo các số trước, dĩ nhiên chúng có chu kỳ là 4. Khi đó mỗi số hạng của dãy là số hữu tỉ là các phân số tối giản, việc còn lại là xây dựng các số nguyên 3 ;...; ii aa + dựa trên các tử số mẫu số của các phân số đó. Từ đó hình thành vệc chọn số nguyên tố p cho phù hợp.Chúng ta cùng nghiên cứu lời giải sau đây. Bài giả
Trước tiên ta chọn các số hữu tỉ dương khác nhau 123 ;;...; krrr + sao cho
123iiii rrrri +++ = với 1 ik≤≤
Gọi 123 ;; rxryrz === là các số nguyên tố khác nhau lớn hơn k, số hạng còn lại thỏa
mãn 4 123
1 r rrr = và 4 1 ii i rr i + + = . Kéo theo nếu ir là phân số tối giản, tử số là chia hết
cho x nếu ( )1mod4 i ≡ , là chia hết cho y nếu ( )2mod4 i ≡ , là chia hết cho z nếu ( )3mod4 i ≡ và tử không chứa biến x;y;z nếu ( )0mod4 i ≡ . Chú ý rằng 4 ii rr + < do đó dãy
159261037114812 ...;...;...;...rrrrrrrrrrrr <<<<<<<<<<<<
Là dãy tăng và không có số hạng chung, khi đó tất cả các ir đều khác nhau.
Nếu mỗi ir đều biểu diễn ở phân số tối giản i i
uuvuv u rrvvrruvuv vvvv −=−= −=−
u v xét () jijji i ijijijijji ijij
Do đó chọn số nguyên tố p không là ước của ;11 i vik≤≤+ cũng không là ước của
( ) (khi) ijijjiij vvrrvuvuij −=−< và xác định ia bởi ( ) mod iii avup ≡
uuuu iivvvvuuuu vvvv +++ ++++++ +++ =⇔=
Từ 123 . iiii rrrri +++ = ta có 123 123123 123 iiii iiiiiiii iiii
( ) 112233 mod iiiiiiii avavavavp ++++++ ≡
Do đó ( ) 123.modiiii aaaaip +++ ≡ với 1 ik≤≤
Nếu ( ) mod ij aap ≡ thì ( ) mod iiiijji uvavvuvp ≡≡ điều đó mâu thuẫn.
Vậy với mỗi số nguyên dương k luôn chọn được số nguyên tố p sao cho p là ước của tích 123 . iiii aaaai +++ với tất cả 1;2;..; ik =
Lời bàn: Thật là quá khó, từ ý tưởngcho đến lời giải đầy kỹ thuật. Đúng là đề TST!
II.1.c. Chọn một ước nguyên tố phù hợp và kỹ thuật sử dụng ước đúng (bổ đề LTE) để giải toán.
Khi giải một số bài toán về tìm số nguyên n thỏa điều kiện là bội hay ước của biểu thức chứa n nào đó ta thường chọn ước nguyên tố nhỏ nhất của nó sau đó sử dụng ước đúng, số mũ đúng hoặc dùng cấp của số nguyên theo modp,… một cách phù hợp.
Bài toán 1. Cho số nguyên dương n có đúng 12 ước số dương khác nhau kể cả số 1 và chính nó, biết tổng các ước nguyên tố khác nhau của chúng là 20. Tìm giá trị nhỏ nhất có thể có của n.
Tiếp cận bài toán: Rõ ràng kiến thức cần huy động là hàm số các ước của n là ( )n τ . Dĩ nhiên phải xét các khả năng phân tích số 20 thành thừa số nguyên tố. Trên cơ sở số n nhỏ nhất để chọn các ước nguyên tố phù hợp.
Bài giải:
Ta biết số các ước số của số 12 12 k knppp α αα = là ( ) ( )( ) ( ) 1211....1 k n τααα=+++
Do đó: ( )( ) ( ) 12 1211....1 kααα =+++ mà các khả năng phân tích của 12 là 122.63.42.2.3 ===
Suy ra n có tối đa là 3 ước số nguyên tố nên ta có thể đặt 3 12 123.;(1;2;3) i npppi α αα α =∈= ℕ
Theo đề bài ta có: ( )( )( ) 12311112ααα+++= và 123 20 ppp++= .
- Nếu n chỉ có một ước nguyên tố, không thể thỏa yêu cầu đề bài
- Nếu n có hai ước nguyên tố 12 12 12.;;npp αα αα =∈∞ , mà 12 20 pp+= nên có
hai khả năng 123;17pp== hoặc 127;13pp==
Do tìm n nhỏ nhất nên 5 12npp = hoặc 23 12npp = (1)
- Nếu n có đúng 3 ước nguyên tố 3 12 123.;(1;2;3) i npppi α αα α =∈= ∞ thì
123 20 ppp++= do các ip nguyên tố nên phải có một số bằng 2 giả sử
1 2 p = , khi đó: 23 18 pp+= chỉ có 51318 += . Giả sử 235;13pp== . Lại có
( )( )( ) 123111122.2.3ααα+++==
Do cần tìm số nhỏ nhất nên chọn khả năng 2 2.5.13260 n == (2)
So sánh (1) và (2) ta có GTNN có thể của n là 260.
Bài toán 2: (BMO TST - March camp-2021) Gọi ;; xyz là các số nguyên dương lẻ sao cho ( ) gcd;;1 xyz = và tổng 222 xyz ++ chia hết cho xyz ++ . Chứng minh rằng
2 xyz++− không chia hết cho 3.
Tiếp cận bài toán: Ta xét mệnh đề tương đương: “Nếu 2 xyz++− chia hết cho 3 thì xyz ++ có ước 32 k + ”. Như vậy giả sử xyz ++ có ước nguyên tố dạng 32 k +
chắc sẽ dẫn đến điều vô lý, thử xem đi hướng này.
Bài giải:
Giả sử tồn tại ước nguyên tố ( )2mod3 p ≡ là ước số của xyz ++ .
Từ ( )( ) mod zxyp ≡−+ ta có () ( ) () 2 222222220mod xyzxyxyxyxyp ++=+++=++≡
Hay ( ) () 22 20mod xyxyp ++≡ ( ) () 22 0mod(2;)1xyxypdop ++≡=
Nhân hai vế cho ( )xy ta được ( ) 33 mod xyp ≡ nhưng điều này mang lại ( ) mod xyp ≡ ,
Tương tự ( ) mod xzp ≡ khi đó ( ) 30mod xxyzp ≡++≡ chứng tỏ ;; xyz chia hết
cho p mâu thuẫn với ( ) gcd;;1 xyz = .Từ đó xyz ++ không có ước nguyên tố nào có
dạng 32 k + . Vì vậy 2 xyz++− không chia hết cho 3.
Lời bàn: Chỉ việc chọn ước nguyên tố phù hợp đã ẩn ý trong đề ta có lời giải dễ hiểu.
Bài toán 3. (Olympic Balkan- 2018) Tìm tất cả các số nguyên tố p và q thỏa mãn
1 31 qp + là ước của 1117pp + .
Tiếp cận bài toán: Từ tính chẵn lẻ của các biểu thức, ta xét trường hợp đặc biệt của p là 2 p = . Khi 2 p > , giá trị 2 biểu thức chẵn, liệu nó có phải lũy thừa của 2 hay là số chẵn có dạng nào đó? Khi đó, ta hướng đến khai thác ước nguyên tố lẻ của cả 1 31 q p + và 1117pp + . Sử dụng định lý Fermat và cấp sẽ tìm được p;q
Bài giải:
Với 2 p = : thay vào các biểu thức, không tồn tại q thỏa mãn.
Với 2 p > nên p lẻ khi đó ( )11174mod8 pp A =+≡
Lời bàn: Dựa trên sự phân tích của số 12 mà ta dự đoán có tối đa 3 ước nguyên tố phân biệt.Từ tính nhỏ nhất của n để suy ra tính nhỏ nhất các ước nguyên tố. Tính logic trong suy luận khá thú vị! 179
Do đó 8 không là ước của số 1 314 q p +>
Ta xét một ước nguyên tố lẻ t của 1 31 q p + , nên { }3;11;17 t ∉ , do đó tồn tại số nguyên
dương b sao cho ( ) 171mod bt ≡ suy ra ( ) 1710mod ppbt −≡
Vì vậy: ( ) |1mod pp tbAat ≡+ , với 11 ab = .
Nên 2 |1 p ta nhưng t không là ước của 1 p a tức là ( ) or|2 t dap , nhưng
( ) or t da không là ước của p, suy ra ( ) { } or2;2 t dap ∈
Ta biết: nếu ( ) or2 t da = thì ( ) () 2222 |11117mod tabt −≡− nên chỉ có 7 t = .
Lại có: nếu ( ) or2 t dap = suy ra 2|1 pt (do ( ) 1 1mod t at ≡ ), khi đó mọi ước nguyên tố của 1 31 q p + ngoài 2 và 7 đều đồng dư 1 mod(2p).
Vì vậy ta đặt: () 1 1 1 3127...* kr r q kppp αβ += với { }2;7 ip ∉ là các ước nguyên tố với ( ) 1mod2 i pp ≡ .
Ta biết: 2 α ≤ và có: ()
123211 1117 1111171117...17.4mod7 28 pp ppppp p + =−+−+≡ .
Suy ra: 1117pp + không chia hết cho 27 do đó 1 β ≤
Nếu 2 q = thì (*) trở thành 1 1 3127... kr r kppp αβ +=
Lại có: 21 i pp≥+ , do đó chỉ có duy nhất trường hợp 0 ir = với mọi i,
Khi đó: { }31272,4,14.28 p αβ +=∈ các trường hợp này không thỏa.
Do đó: 2 q > suy ra 1 4|312 q p α +=>=
Ta có: vế phải của (*) đồng dư với 4 hoặc 28 modp suy ra 3 p = , thay vào ta có
1 31|6244 q + cho kết quả duy nhất 3 q = .
Vậy nghiệm duy nhất của bài toán: ( ) ( );3;3pq =
Lời bàn: Bài toán rất khó! Việc chọn ước số nguyên tố lẻ t của 1 31 q p + để xét theo modt và sử dụng cấp một cách linh hoạt để có bài giải: Rất kỹ thuật và sâu sắc.
Bài toán 4. (MEMO-2015) Tìm tất cả các cặp số nguyên dương () , ab sao cho !! baabab +=+
Tiếp cận bài toán: Trước tiên đối với bài toán có chứa n! thì nên kiểm tra các số nhỏ và trường hợp đặc biệt. Khi 1 ab<< , sử dụng tính chất |!papa < , nên so sánh số mũ của p trong hai vế của phương trình để đánh giá nghiệm nếu có.
180
Bài giải:
Nếu ab = thì phương trình trở thành ! a aa = . Từ !(2) a aaa>≥ ta có lời giải duy nhất cho trường hợp này là 1 ab==
Nếu 1 a = , phương trình trở thành ! bb = , ta có thêm 1;2ab== . Ta chứng minh 1;1,2;2,1ababab ====== là các nghiệm của phương trình.
Giả sử a;b là các nghiệm khác thỏa mãn 1 ab<< ( trường hợp 1 ba<< tương tự). Suy ra |!ab kéo theo | aab
Gọi p là ước số nguyên tố của a, kéo theo | pb . Ta so sánh số mũ đúng của p ở cả hai vế của phương trình.
aaaa va pppp =+++=
!... p k k
∞ =
Ta có: 23 1
aaaa aa ppppp <+++=≤ .
∞ =
Suy ra số mũ của p trong vế trái nhỏ hơn a.
Ta có ;| aabab < và |;|;() papbba > nên ( ) ba p vaba +> , mâu thuẫn!
Do đó không có nghiệm cho ;2ab ≥ .
Vậy ( ) ( ) ( ) ( );1;1;1;2;2;1ab =
Lời bàn: Bài toán có nghiệm trong các trường hợp đặc biệt. Trường hợp 1 ab<< đã chọn ước nguyên tố p của a và so sánh số mũ của p trong hai vế để loại suy khá hay!
Bài toán 5. (Baltic Way 2015) Tìm các số nguyên dương n thỏa mãn sao cho 1 1 n n chia hết cho 20152
Tiếp cận bài toán: Từ đề cho ta có: 2 là ước của 1 1 n n nên n lẻ, bài toán đi tìm n lẻ để có ( ) 1 2 20 1 15 n vn = . Xem như đã có hướng đi!
Bài giải:
Do 1 21) n n(∣ nên n lẻ, do đó theo Bổ đề về bậc LTE ta có:
181
( ) ()()() 1 2222111112015 n vnvnvnvn−=−+++−−=
Ngoài ra 41) n(∣ hoặc 41) n + (∣ .
- Nếu 41) n(∣ thì ( ) ( ) 2211212015vnvn += −= , vô lý vì VT chẵn, VP lẻ.
- Nếu 41) n + (∣ thì ( ) ( ) 221112014vnvn −= += .
Kết luận: 201412 nm += với m nguyên dương lẻ.
Lời bàn: Bổ đề LTE là “vũ khí” khá tuyệt vời!
Bài toán 6. (Olympic KHTN Hà nội 2019) Tìm số nguyên dương n để 3 31 n n⋮
Tiếp cận bài toán: Nếu chọn ước nguyên tố p nhỏ nhất của n thì từ đề bài đã có bóng dáng của định lý Fermat và cấp! Nên bài toán có hướng giải!
Bài giải:
* Với 1 n = , thỏa mãn yêu cầu bài toán.
* Với 2 n ≥ , gọi p là ước nguyên tố nhỏ nhất của n, từ giả thiết suy ra được: ( ) 31mod n p ≡ . Mặt khác theo định lý Fermat nhỏ, ta cũng có: ( ) 1 31mod p p ≡ .
Gọi ( )or3 p hd = , theo tính chất cấp ta có: | |1 hn hp . Như vậy, ta chỉ ra được một số bé hơn p là ước của n, để p là ước nguyên tố nhỏ nhất thì 1 h = . Khi đó: ( ) 1 31mod2 pp ≡ =
Giả sử ( ) * 2., ntt α α =∈∞ , t lẻ.
Khi đó: ( ) ( ) ()() 2122 22 2 313131.33...31 t tt n αα αα α −=−=−++++
Vì ()() () 2122 2 33...311mod2 tt q αα α ++++≡≡ , nên: ( )( ) ( )( )( ) 12100 22222 313131...313131. n R αα −=++++− (R lẻ).
Vì 121 222 31,31,...,31 αα +++ là 1 α số chẵn và ( ) ( ) 00 22 31.318 +−= , nên: 2 312. n S α+ −= , (S lẻ).
Vì 333312. n nt α −= ⋮ , suy ra 23112nt αααα +≥ ≤ = =
Giả sử n có ước lẻ thực sự, gọi ước lẻ nhỏ nhất của nó là q, và gọi ( )3 qkord = , theo giả thiết có: ( ) 31mod n q ≡ và theo định lý Fermat nhỏ, ta cũng có: ( ) 1 31mod q q ≡
Từ đó suy ra: | |1 kn kq
Nếu k lẻ, ta chỉ ra được một ước lẻ của n mà nhỏ hơn q, trái với cách chọn q, vậy k chẵn.
Mặt khác: 2. nt = (t lẻ), do đó k = 2 và 23|312 n −= , do đó n không có ước lẻ thực sự Vậy 1;2nn==
Lời bàn: Chọn ước nguyên tố p nhỏ nhất của n, qua sử dụng cấp ta có được 2 p = nhờ đó n có dạng 2. nt α = . Khi đó, qua phân tích thành thừa số của 31 n dễ tìm được số mũ đúng của 2. Từ đó tìm được 2 nt = để cuối cùng có 2 n = , khá tự nhiên.
Bài toán 7. Cho ;; abc là các số nguyên thỏa mãn 3 a chia hết cho b; 3b chia hết cho c và 3 c chia hết cho a. Chứng minh tích abc là ước của ()13 Mabc =++ .
Tiếp cận bài toán: Bài toán đã gợi cho ta xét ước nguyên tố p bất kỳ của tích abc và phải chứng tỏ số mũ đúng của p trong tích abc phải nhỏ hơn số mũ đúng của p trong M. Bài giải:
Gọi p là ước nguyên tố bất kỳ của tích abc , khi đó tồn tại các số tự nhiên ;; αβγ sao cho
ββγ ===∈=== ℤ
( ) ( ) ( ) ( ) .;;.;;;,;;;1 apmbpncpkmnkpmpnpk
Do 333|| bapnpm βα mà ( ) ;1pn = nên 3 βα ≤ , tương tự có 3;3 γβαγ ≤≤ .
Ta có: tích ( ) ;;1abcpmnkpmnk αβγ ++ ==
Mà p là ước nguyên tố bất kỳ của tích abc nên ta chỉ cần chứng tỏ số mũ đúng của p trong ()13 abc ++ không nhỏ hơn αβγ ++ . (hay ()13 () p vabc αβγ++≥++ )
Không mất tính tổng quát giả sử min{;;} ααβγ =
Khi đó: số mũ đúng của p trong ()13 abc ++ là 13α.( 13 (())13 p vabc α ++= )
Mà ta có 3913 αβγαααα ++≤++= . Chứng tỏ ()13 | abcabc ++
Lời bàn: Số mũ đúng (LTE) là linh hồn của bài giải!
Bài toán 8. (Olympic Zhautytov 2018) Chứng minh tồn tại vô hạn bộ số ( ) ; mn thỏa mãn mn + là ước của ()()!!1 nm mn++
Tiếp cận bài toán: Yếu tố ước của biểu thức chứa lũy thừa và giai thừa rõ ràng ta phải nghĩ đến ước nguyên tố. Khả năng ước nguyên tố này là lẻ. Có vô hạn số nguyên tố lẻ như vậy trong hai số m;n phải có số chẵn. Trong biểu thức có chứa giai thừa lại cộng thêm 1 chia hết cho p có bóng dáng của định lý Wilson. Thử bắt đầu bởi các ý tưởng đó.
Bài giải: Chúng ta sẽ tìm bộ số ( ) ; mn thỏa mãn mnp += là một số nguyên tố và n là số chẵn. Theo định lý Wilson, ta có:
() ( ) ()()() ()()() () () 1! 11 !! mod 1...211...211!! pn mpnp pnppnnn =−===≡ −+−−−−−−−
Theo định lý Fecma nhỏ, ta có ()() !!mod p nnp ≡ nên
++≡++≡
++
Do đó ta cần chứng minh tồn tại vô số số nguyên dương n chẵn sao cho
Ta chỉ ra điều đó đúng bằng cách lấy ví dụ với mọi số có dạng 2 nq = với 2 q > là số nguyên tố . Thật vậy, xét: ()()()()
2 2 22!2! q q Aqqq =++
Nếu 2 rq < là số nguyên tố và { } 2; rq ∉ thì ()() 2 20mod q Aqr ≡≡ . Số mũ của q trong ( ) 2! q là 2 trong khi đó số mũ của ()2 2 q q và () ()2 2! q q lần lượt là 2q và 4q , suy ra ( ) 2 p vA =
Theo công thức Legendre: () () 2 222222 2!......2 248248 qqqqqq vqq =+++<+++=
Ta có: () () () ()( ) 2 222!2! q vqvq < suy ra ( ) 2 21vAq≤−
Lại có ()2 21222 q q Aqq >> nên A có một ước nguyên tố lẻ 2 pq > (đpcm)
Lời bàn: Bài toán khó, đầy kỹ thuật, nhưng khá hay.
Bài toán 9. (Đề đề nghị của trường Hùng Vương‒Phú Thọ‒lớp 10‒2015)
Chứng minh rằng nếu p là số nguyên tố có dạng 41 k + thì có một số tự nhiên a nhỏ hơn p sao cho 2 1 a + chia hết cho p
Tiếp cận bài toán: trước tiên ta thấy ( ) 2 1mod ap ≡− , gợi cho ta liên tưởng đến định lý Wilson ( ) ( ) 1!1mod pp −≡− khả năng tồn tại số a nhỏ hơn p. Như vậy phải có tích liên tiếp các số nguyên dương, từ ( ) 41mod kp ≡− . Hướng giải đã lộ diện.
Bài giải:
Theo định lý Wilson: ( ) 1!1 pp −+ ⋮ với ( ) * 41 pkk=+∈ ℕ thì ( ) 4!1(1) kp + ⋮
Mặt khác: ( ) 221mod kikip +≡−+− với 1,2,...,2 ik =
Do đó: ( )( ) ( ) ( ) ( ) 2122....42!mod kkkkp ++≡
Suy ra: ()()() 2 4!2!mod(2) kkp ≡
Từ ( )1 và ( )2 ta có () 2 2!1kp +
Lời bàn: Từ ý tưởng dùng định lý Wilson đã rất khéo trong biến đổi 41kp += để có biểu thức ( ) 221mod kikip +≡−+− . Từ ý tưởng đến lời giải khá đẹp!
Bài toán 10. Giả sử , ab là các số nguyên dương, sao cho 21;21; abab −−+ đều là các số nguyên tố. Chứng minh rằng abab + và baab + đều không chia hết cho ab +
Tiếp cận bài toán: Từ giả thiết suy ra ab + nguyên tố lẻ nên a;b khác tính chẵn lẻ. Ta biết nnab + chia hết cho ab + khi n lẻ, nên khi b lẻ thì a+b là ước của bbab + . Do đó kết hợp với giả thiết phản chứng, ta sẽ suy ra biểu thức bội của a+b. Từ đó sẽ có hướng suy luận tiếp.
Nên: () () ()( ) 2 222!2 q vqvq < 185
Bài giải:
đ
Có 21;21 ab là nguyên tố nên 1,12abab >> +> , mà ab + nguyên tố, do
ó ab + là số lẻ, suy ra ()1 abab ϕ +=+− . (“phi hàm Euler của a+b”)
Giả sử a chẵn, b lẻ. Vì b lẻ nên ab + là ước số của bbab + (1)
+) Nếu ab + là ước số của abab + (2)
Từ (1), (2) suy ra ab + là ước số của () ba aa , suy ra: (1)()aba aaab −+ ⋮ , nếu ab < (*) hoặc (1)()bab aaab −+ ⋮ nếu ab >
- Nếu ( )ab + là ước số của a a mà ( )ab + nguyên tố, nên ( )ab + là ước số
của a, vô lý vì aab <+
- Nếu ( )ab + là ước số của b a mà ( )ab + nguyên tố nên ( )ab + là ước số
của a, vô lý vì .aab <+
Từ (*) suy ra ( )ab + là ước số của || 1 ba a hay ( ) || 1mod(). ba aab ≡+
Gọi ( )()||| ab hordahba + =
Mà |()|1 habhab φ + +− |21 |21 ha hb
Do (21),(21) ab là số nguyên tố nên ( )11mod() haab = ≡+
(1)(), aab −+ ⋮ vô lý vì 01. aab<−<+ +) Tương tự, nếu () ba abab ++ ⋮ mà ()()()() bbab ababbbab ++ −+⋮⋮
(1)()aba bbab −+ ⋮ nếu ab < hoặc (1)()bab bbab −+ ⋮ nếu ab > (**)
- Nếu ()() a babbab + + ⋮⋮ do ab + nguyên tố, vô lý vì .bab <+
- Nếu ()() b babbab + + ⋮⋮ do () ab + nguyên tố, vô lý vì .bab <+
Từ (**) có || (1)() ab bab −+ ⋮ ( ) || 1mod(). ab bab ≡+
Gọi ()|(||). ab kordbkab + =
Mà |1() kabab φ +−=+ |21 |21. ka kb
Do 21;21 ab là số nguyên tố 1 k = ( )1mod() bab ≡+ 1(),bab −+ ⋮ vô
lý vì 01. bab<−<+
Lời bàn: Ta thấy vai trò lợi hại của việc dùng cấp để chứng tỏ a+b không phải ước của abba abvàab ++ !
II.1.d. Vai trò ước lẻ và ước lẻ lớn nhất trong giải toán.
Mọi số nguyên dương đều có ước số lẻ, trong các ước lẻ đó luôn tồn tại ước số lẻ lớn nhất. Khai thác tính chất chia hết của số lẻ ta có một số bài giải hay và đẹp.
Trước tiên ta tìm hiểu một số tính chất của ước lẻ lớn nhất.
Ta ký hiệu là ( )Ln là ước lẻ lớn nhất của n.Ta có thể biểu diễn một số nguyên dương n bất kỳ dưới dạng sau: 2. k n = ℓ với k ∈ N và ℓ là ước số lẻ lớn nhất. Khi đó ( )Ln = ℓ .
Nhận xét:
- Mọi số nguyên dương n luôn tồn tại ( )Ln , và ( ) 1 Lnn≤≤
- ( ) * |, Lnnn∀∈∞
Một số tính chất của ước lẻ lớn nhất:
- ( ) ( ) * 2, n Ln Ln = ∀∈∞
- ( ) 2 1 2 1 Lnn+=+
- Mọi ước số lẻ của n đều là ước của ( )Ln
Ta có một số hệ quả sau:
- Khi p nguyên tố lớn hơn 2 thì ( ) Lpp = Đặc biệt ( ) 12Lpp=⇔=
- Với mọi số tự nhiên n luôn có ( ) . nLn là số chính phương hoặc bằng 2 lần của số chính phương. Vì 2. k n = ℓ , nếu k chẵn thì ( ) 22 .2. k nLna == ℓ , nếu k lẻ thì () ( ) 122.2.22 k nLna == ℓ
Sau đây là một số bài toán sử dụng ước lẻ lớn nhất để giải.
Bài toán 1. Tìm tất cả các số nguyên tố p có dạng * 1 (), n pnn=+∈∞ biết p có không nhiều hơn 19 chữ số.
Tiếp cận bài toán: Ta biết tổng hai lũy thừa số mũ n phân tích thành tích khi n lẻ Mà n luôn có ước số lẻ. Từ đó dùng điều kiện để số p là nguyên tố ta sẽ có hướng giải bài toán.
Bài giải:
Ta có: 0 n > , đặt 2. k n = ℓ (k ∈N, ℓ lẻ) ( ) 22211(1) kkk pnnn =+=++ ℓ ℓ ⋮ , do p∈P nên 1 = ℓ
Vậy ; abba abab ++ đều không chia hết cho 21;21. ab 187
Suy ra =2k n , nếu k = 0 thì n = 1, p =2 thỏa.
Nếu k ≥ 1, lại tiếp tục đặt 2. t ka = ( t ∈ N, a lẻ). Khi đó:
( ) ( ) 222 212121 ttt na ann p =+=++ ⋮ , do P p ∈ nên a = 1, khi đó 2t k = ,
2 2, t ntN =∈ .
- Với 0, 25tnp == = .
- Với 4 1, 44 1257 tnp == =+= là số nguyên tố.
- Với 2 t ≥ 16 16161np ≥ ≥+ p ≥ 264 +1 = 16. 10246 +1 >16.1018, suy ra
p có nhiều hơn 19 chữ số
Vậy { }2;5;257 p ∈
Lời bàn: Bài này khai thác tính chất ( ) nn abab ++ ⋮ khi n lẻ nên ước lẻ lớn nhất
đóng vai trò quan trọng ở bài giải này
Sau đây là bài cũng có tính chất như vậy.
Bài toán 2. Cho n là số nguyên dương sao cho 31 n chia hết cho 20212 . Chứng minh 20192 n ≥ Bài giải:
Ta biểu diễn 2. k n = ℓ ( k ∈ N; ∈ λ∞ , ℓ là ước số nguyên lẻ lớn nhất)
Khi đó: ( )
()() 2122 2.22 31313133...31 kk kk k n −=−=−++++
ℓℓ ℓℓ (1)
Ta có 3x là số lẻ với mọi * x ∈∞ và ()() 2122 2 33...31 kk k ++++ ℓℓ có một số lẻ số hạng nên
()() 2122 2 33...31 kk k ++++ ℓℓ là số lẻ
Từ (1) suy ra: (3n ‒ 1) chia hết cho 20212 khi và chỉ khi 22021(31)2 k ⋮ (2)
Lại có hằng đẳng thức: ( ) ( )( ) ( )( ) ()() 11112 2 22222222 313131313131...3131 kkkkkk −=−=+−=+++−
Chú ý mỗi thừa số ( ) 2 31 ki + chia hết cho 2 nhưng không chia hết cho 4 (với
1;1ik=− ) Ngoài ra: 32 ‒1 = 8 = 23. Do vậy ( ) 22312 k k + ⋮ (3)
Từ (2) và (3) suy ra: ( ) 22021312 k ⋮ ⇔ 2 2021 k +≥ ⇔ 2019 k ≥ (4)
Do 2. k n = ℓ , ℓ lẻ nên 2k n ≥ . Vì thế từ (4) suy ra: ( )31 n chia hết cho 20212
⇔ 20192 n ≥ (đpcm)
Bài toán 3. (Czech ‒ Slovak 2013) Gọi L(n) là ký hiệu ước số lẻ lớn nhất của n.
Hãy tính tổng sau:
S = L(1) +L(2) +L(3) +……+L(22013)
Tiếp cận bài toán: Dựa vào tính chất của ước lẻ lớn nhất L(n), ta thử tìm quy luật của S(n) hay tìm số hạng tổng quát của nó.
Bài giải:
Với mỗi * k ∈ ℕ , ta có ( ) ( ) 2 LkLk = và ( )21 21Lkk−=−
Ta gọi ()()()() ( ) 123...2n SnLLLL =++++
Theo tính chất nêu trên, ta có
()()() ( ) ()() ( )24...213...21 nnSnLLLLLL =+++++++−
()()() ( ) () () () 1
=+++−++++−
SnLLL SnSn
+− =−+=−+
nn nn n
12...2113...(21)
Ta được công thức truy hồi: ( ) ( ) 1 14n SnSn=−+ đúng với mọi số nguyên dương n.
Áp dụng công thức truy hồi trên ta có;
( ) ( ) 1 14n SnSn=−+
( ) ( ) 2 124n SnSn−=−+
( ) ( ) 3 234n SnSn−=−+
……………………….
( ) ( ) 1214SS=+
( ) 12 S =
189
Cộng n đẳng thức trên ta được: () 231 4442 2444...42 33 nn n Sn −+ =+++++=+= .
Do đó theo đề bài ta tính được () 2013 42 2013 3 S + =
Lời bàn: Bài này có tính chất của dãy, then chốt của bài toán là tìm công thức truy hồi của S(n). Kết quả đẹp!
Bài toán 4. (IMO Shorlist 2014) Cho trước số nguyên dương 1 n > . Chứng minh rằng tồn tại vô hạn các số hạng của dãy ( ) 1 k k a ≥ là lẻ, trong đó k k n a k =
Tiếp cận bài toán: Điều kiện bài toán tương đối mở nên việc chọn k tùy thuộc n để có ka lẻ. Chú ý bất đẳng thức phần nguyên [ ] 1 xxx −<≤
Bài giải:
Nếu n lẻ, ta cần chọn k sao cho k n k là số nguyên lẻ, đo đó đặt mkn = với
1,2,... m = thì m nm k an = là số lẻ với mọi .m ( thỏa yêu cầu đề bài)
Ta xét n chẵn, đặt 2 nt = với 1 t ≥ . Ta có * k n a k =∈
∞ mà luôn có 2. m k = ℓ ( ℓ là
Vì () .2 2 22
được chứng minh hoàn toàn.
k m n là một số nguyên lẻ nên ka lẻ, bài toán
Lời bàn: Khin lẻ việcchọnktương đốitự nhiên,khinchẵn phảisử dụngkỹ thuật dùng ước lẻ để giải quyết bài toán. Rất tư duy!
Bài toán 5. (Czech ‒ Slovak‒ 25/3/2014) Cho số tự nhiên n, ký hiệu tất cả các ước số của n là 12 ; ; ; kddd giả sử 12 kddd <<…< (với 1 1; k ddn == ). Hãy xác
định tất cả các giá trị của n thỏa mãn hai điều kiện sau: 3 5 50 dd = và
57 11 8 3 ddn +=
Tiếp cận bài toán: Chắc chắn rằng trong các số di sẽ có số này là ước của số kia và sẽ có hai ước có tích bằng n. Dựa vào dữ kiện đề bài sẽ tìm mối quan hệ giữa các ước, bài toán sẽ giải được.
Bài giải:
Ta xét hai trường hợp:
+) Nếu n lẻ thì n là ước số lẻ lớn nhất của n, khi đó tất cả các ước (1;) id ik = đều lẻ và k dn =
Từ 57 11 8 3 ddn += suy ra 75 |11 dd và 57 |8 dd , do các id lẻ nên 57 | dd . Từ giả thiết 75dd > và quan hệ 575 ||11 ddd suy ra 75 11 dd = .
ước số lẻ lớn nhất của k), nên ta phải chọn số α đủ nhỏ để * 2.
k m n a α =∈∞ λ . Như vậy, ta cần chọn ℓ là ước số nguyên tố lẻ lớn nhất của k n α và chọn 2m α =
Khi đó, với mỗi 2 m ≥ thì () ( )22222.1 k kmmmkmk ntt −=−=− , do
2.1 m kmm−=−> ℓ , nên 21 km có ước số nguyên tố, gọi p là ước nguyên tố lớn nhất. Ta đặt .2m kp = , ta có 2 km n có ước nguyên tố p nên 2(mod)km np ≡ .
Mặt khác: từ ( )221kmkkm nnnp −<<+− , suy ra
km kmk mm np nn k pp +− <<
( )21 2 .2.2
Từ 57 11 8 3 ddn += , ta được 5 99 3 dn = hay 5 33dn = , suy ra 1;3;11;33 đều là ước của n và các ước này nhỏ hơn 50.
Từ ước thứ 5 là d5 thỏa 53 50 50 dd=+>
Như vậy ta xác định 1234 1; 3; 11; 33 dddd==== và 53 5061 dd=+= và ta có 5 33 33.61 2013 nd=== . Số 2013 thỏa điều kiện 731111.61dd== +) Nếu n chẵn: từ đẳng thức 575 11 832| ddnd += và 53 2| 50 2| dd
Từ 12 1; 2 dd== và 3 3 d ≠ , nên 3 4 d = hoặc ( ) 3 2 2 dtt=> Nếu 3 2 dt = suy ra t là ước của n, với 23 dtd << , không phù hợp.
Do đó 353 4; 5054 ddd==+= chia hết cho 3, nên 523 3| 3| | 3 dnndd << không xác định, nên n chẵn (loại).
Kết luận 2013 n = .
===
n nn a kp p
190
Lời bàn: Chỉ sử dụng tính chẵn lẻ của ước ta có lời giải rõ ràng dễ hiểu!
Bài toán 6. (Putnam 1969) Cho số nguyên dương n sao cho 1 n + chia hết cho 24. Chứng minh rằng tổng của tất cả các ước của n cũng chia hết cho 24.
Tiếp cận bài toán: Từ giả thiết ta đã thấy tính chẵn lẻ của n, do đó tất cả các ước của n đều lẻ. Để ý nếu d là ước của n thì n d cũng là ước của nó. Hãy xem nó có những dạng ước nào, từ đó có hướng đi. Bài giải:
Từ giả thiết 24|1 n + , suy ra n là số lẻ và n cũng là ước số lẻ lớn nhất của n. Gọi d là ước số của n suy ra d lẻ. Do n không chia hết cho 3 và không chia hết cho 8 nên suy ra ( )1;2mod3 d ≡ và ( )1,3,5;7mod8 d ≡ .
Ta có: n không thể là số chính phương, vì 22 1(1)24(1)3naa +=+ + ⋮⋮ , vô lý!
Ta có: ứng với ước số lẻ d của n thì cũng có ước số lẻ là n d và ta có n dn d = nên để có 24|(1) n + thì ta phải có tích ().2mod3 n d d ≡ và ().7mod8 n d d ≡ , điều đó chỉ có thể xảy ra khi:
+ ()1,2mod3 n d d ≡≡ hoặc ngược lại.
+ ()1,7mod8 n d d ≡≡ hoặc ngược lại.
+ ()3,5mod8 n d d ≡≡ hoặc ngược lại.
Trong tất cả các trường hợp, ta luôn có ()0mod3 n d d +≡ và ()0mod8 n d d +≡ , từ
đó 24() dn d + với , n d d ≠ không ước nào được sử dụng hai lần trong các cặp. Suy ra
dn σ = Chứng tỏ tổng của tất cả các ước của n cũng chia hết cho 24.
() 24. dn
Lời bàn: Bàigiải đãkhaitháctừ số nlẻ vàkhôngchiahếtcho3và8kếthợpvới nhận xét d và n d cùng là ước của n.Suy luận đẹp và quen thuộc!
Bài toán 7. Tìm tất cả các cặp số nguyên dương (,) mn thỏa mãn
Tiếp cận bài toán: Dễ nhận ra tính chẵn lẻ của m và n. Biểu thức đề cho dạng tổng các lũy thừa bậc n, nó phân tích thành tích khi n lẻ. Do
trong tìm ra hướng giải. Bài giải:
nn mn n m
+= = = +
+) Nếu 0 1 |211 1|2 2 |21 n
m nm n m m
+= = = +
+) Nếu 1 0 |211 1 2 |21
+) Nếu 1,1mn>>
Có 1 1 |21 |21
+ + suy ra m, n lẻ.
n m
m n
Giả sử 12.() a mxaN ∗ −=∈ và 12. b ny −= () bN ∗ ∈ với x;y là các ước số lẻ lớn nhất của 1 m và 1 n
Gọi p là ước nguyên tố lẻ bất kỳ của n 12 |21|(2)1 a mx pp + + (1)
Suy ra: () 1122 |(2)1(2)1mod aa xx pp ++ ≡
Gọi ( ) 2x p hord = => 1 |2a h + => ( )21 t hta=≤+ . Nếu ta ≤ thì () () 2 2|221mod a tax p =>≡
Suy ra 2 |(2)1 a x p (2)
Từ (1) và (2) ta có: |2 p mâu thuẫn p lẻ. Do đó 1 ta=+ nên ( ) 11 2|2|(1) aa hpp ++ ==>− ϕ 11 1(mod2)21. aa ppk ++ ≡ =+
Như vậy mọi ước lẻ của n đều có dạng 1 2.1. a k + + Do tích của các số có dạng 1 2.1 a k + + cũng là số có dạng 1 2.1 a k + + nên 1 1(mod2). a n + ≡
Ta có 1 11 12.(y) a nyN +∗ −=∈ mà 12. b ny −= ( y lẻ) 1 ba ≥+ (3)
Gọi q là ước nguyên tố bất kỳ của .m
12 ||21|(2)1 b nx qmqq + + tương tự trên ta cũng có 1 1(mod2) b q + ≡
1 2.1 b qt + =+
1 11 2.1() b mxxN +∗=+∈
1 11 12.(), b mxxN +∗ −=∈ mà 12. a mx −= ( x lẻ ) 1(4) ab ≥+
Từ (3) và (4) vô lý. Vậy không có 1,1mn>> thỏa mãn đề bài.
Kết luận: 1,1;1,2;2,1.mnmnmn ======
Lời bàn: Cấp và ước lẻ đóng vai trò chính chobài giải:
Bài toán 8. (Chọn đội tuyển ĐH Vinh-2012). Giả sử p là một số nguyên tố sao cho 1 21 p chia hết cho 2 . p Chứng minh rằng với số nguyên dương n tùy ý, số (1)(!2) n pp−+ có ít nhất 3 ước nguyên tố phân biệt.
Tiếp cận bài toán: Từ giả thiết đã cho ta p lẻ và cả hai (1);(!2) n pp−+ đều chẵn, như vậy chỉ cần chứng tỏ mỗi số đều có ít nhất một ước lẻ là xong!
Bài giải:
Vì 1 p là một ước của !p nên ước chung lớn nhất của 1 p và !2n p + là một lũy thừa của 2. Bài toán quy về chứng minh 1 p và !2n p + đều có ít nhất một ước lẻ (vì khi đó 2 ước lẻ của chúng khác nhau, vì thế suy ra đpcm).
Giả sử 12, k p −= hay 21. k p =+
Nếu 3 s ≥ là một ước lẻ của k thì 21(21), stt pA =+=+ nghĩa là p không phải số nguyên tố. Do đó 2. t k = Từ đó suy ra
Do đó 21 t chia hết cho 2 p Điều này kéo theo 21 m chia hết cho 2 p hay !p chia hết cho 2 , p mâu thuẫn. Nên !2n p + có ít nhất một ước lẻ
Vậy mỗi số 1 p và !2n p + đều có ít nhất một ước lẻ. Do đó số (1)(!2) n pp−+ có ít nhất 3 ước nguyên tố phân biệt.
Lời bàn: Một số không có dạng 2n thì nó có ít nhất một ước lẻ (khác 1). Nghệ thuật phản chứng đã chỉ ra sự tồn tại ước lẻ khá hay.
Tổng quan: Qua các bài toán đã nêu ta thấy vai trò của ước số, ước nguyên tố, ước lẻ và ước lẻ lớn nhất đã cho ta một góc nhìn khá thú vị trong giải quyết vấn đề Đôi lúc nhìn từ “bêntrong” ta khám phá được vẽ đẹp tiềm ẩn, những nét rất“duyên” và rất “riêng” của số học. Sự thưởng thức đó chỉ có những cư dân của “vương quốc Toán” mới cảm nhận được!
kkk tttk
11 11
1222 2222 2121(21)(21) (21)(21)(21)...(21).
p +−
−=−=−+ =−+++
Rõ ràng 2 p không phải là ước của tích trên vì
22 (21,21) tl ++ với lt > , và 221 212. tt p + −<=
Do đó: 1 p không phải là một lũy thừa của 2, nên 1 p có ít nhất một ước lẻ
Tương tự, giả sử !22,, nk pkn +=> hay !2(21). nkn p =− Khi đó p là một ước của 21, m ở đây .mkn =− Giả sử t là số nguyên dương bé nhất sao cho p là ước của
21. t Khi đó t là một ước của m và t là một ước của 1. p Nếu 1 plt −= thì 1(1) 21(21)(2...21). pttlt −=−+++
Vì 21(mod) t p ≡ nên chúng ta có (1) 2...210(mod). tlt lp+++≡≡ /
II.1.e. Bài tập tự luyện
Bài 1. (Hungary-2000) Tìm tất cả các số nguyên tố p sao cho tồn tại bộ ba các số nguyên dương ( ) ;; xyn thỏa mãn 33 n xyp += .
Bài 2. Cho a;b là các số nguyên dương nguyên tố cùng nhau. Chứng minh rằng tồn tại các số nguyên dương m;n sao cho ab là ước số của 1 mnab+−
Bài 3. (Bulgary Olympic Mathematic) Tìm tất cả bộ ba số nguyên dương ( ) ;; abc sao cho 333 abc ++ chia hết cho 222 ;; abbcca
Bài 4. (2002 Baltic Mathematics Competition) Tìm tất cả các số nguyên không âm n sao cho số ( )2 21 211 m+ ++ chia hết cho nhiều nhất hai số nguyên tố
Bài 5. (IMO lần thứ 40) Tìm tất cả các cặp số nguyên dương (;) np sao cho p nguyên tố 2 np ≤ và ()11 n p −+ chia hết cho 1 p n
Bài 6. Chứng minh rằng với mọi số nguyên 2 n ≥ , tồn tại n số nguyên dương 12;;...; naaa sao cho ji aa là ước của ijaa + với 1 ijn≤<≤
Bài 7. Chứng minh rằng với mọi số nguyên dương 2 n ≥ , số !n biểu diễn thành tổng của n ước số khác nhau của nó.
Bài 8. Gọi 2 p > là số nguyên tố sao cho 3|(2) p .Gọi 23 {y1|;,0;1}Sxxyxyp =−−∈≤≤− ℕ . Chứng minh có nhiều nhất p phần tử của S chia hết cho p.
111 11...1 k mn ppp
. Chứng minh rằng tồn tại số nguyên tố p sao cho p là ước số của 21 m nhưng không là ước số của m.
Bài 10. (Đài Loan 1997) Gọi X là tập hợp các số nguyên có dạng
a/ Có 10 số nguyên tố nhỏ hơn 3000 chúng lập thành cấp số cộng. Tìm các số này.
b/ Chứng minh không có 11 số nguyên tố nhỏ hơn 20000 chúng có thể lập thành cấp số cộng.
Bài 14. Có n số 12;;...; nxxx mỗi số bằng 1+ hoặc 1.
Chứng minh rằng nếu 12231 ......0 n xxxxxx+++= thì n là bội của 4.
Bài 15. (Hy lạp 2014) Tìm tất cả các số nguyên n sao cho số 825 5 n A n = + bằng lập phương của số số hữu tỷ
Bài 16. (Tuymaada Olympiad 2014, C.A Grimm USA) Cho m và n là hai số tự nhiên sao cho 1 n mn > và các số 1;2 ; ..; mmmn +++ . Chứng minh rằng tồn tại các số nguyên tố khác nhau 12 ; ; .; nppp … sao cho mk + là chia hết cho kp với bất kỳ 1;2;3;.; kn =… .
Bài 17. (Austrian ‒ MO 2016) Gọi a; b; c là các số nguyên sao cho abacbc cba ++
là số nguyên. Chứng minh các số: ;à abacbc v cba cũng là các số nguyên. (của Gerhard J. Woeginger)
Bài 18. (Thổ Nhĩ Kỳ - 2018) Với mỗi số nguyên a ký hiệu ( )da là số các ước nguyên tố của a. Chứng minh rằng với mỗi số nguyên dương n cho trước luôn tồn tại hai số nguyên dương m,k sao cho ( ) ( ) ––1 kmnvàdkdm== .
Bài 19. (Nga -2018 lớp 9) Giả sử 12 ; ;; naaa … là dãy vô hạn các số nguyên dương tăng nghiêm ngặt và 12 ; ;; nppp là dãy các số nguyên tố sao cho |(1;)kk pakn ∀= . Biết rằng * (;) nknk aappnkN −=−∀∈ Chứng minh rằng tất cả các số (i1;) ia n = đều là các số nguyên tố.
Bài 20. Tìm số nguyên dương n nhỏ nhất sao cho 171 n chia hết cho 2020 2.
2222
22220 1010...10 kk kk aaaa ++++ , với k là số nguyên không âm và
2 {1;2;...;9} i a ∈ cho 0;1;2;...; ik = . Chứng tỏ rằng mọi số nguyên có dạng 23pq với p;q là các số nguyên không âm là ước số của một số phần tử của X
Bài 11. (Nga-2001) Tìm tất cả các số nguyên dương lẻ n lớn hơn 1 sao cho mọi
ước số nguyên tố a và b của n, số 1 ab+− cũng là một ước số của n.
Bài 12. (IMO lần thứ 39) Xác định tất cả các cặp số nguyên dương ( ) ; ab sao cho
2 7 abb++ là ước của 2 abab ++
Bài 13.
Bài 21. (Đề đề nghị trường HVT-Hòa bình-2014) Tìm tất cả các số tự nhiên a sao cho tồn tại số tự nhiên 1 n > mà 1 n a + chia hết cho 2 n .
Bài 22. Tồn tại hay không các số nguyên dương phân biệt 12,,..., naaa thỏa mãn: 3 2 1 121 |21;|21;...;|21;|21. naa a a nn aaaa
Bài 23. Chứng minh rằng nếu p là ước nguyên tố lẻ của 2 1(,) n aaZnN ∗ +∈∈ thì 1(mod2), n p ≡ hay p có dạng 1 2.1,. n pkkN +∗=+∈
197
Bài 24. Cho n chẵn, 2 n > và gọi p là một ước nguyên tố của 1 n . Chứng minh
1 i p + là ước của 1 i p n với mọi 0,1,... i =
Bài 25. Chứng minh nếu n là số nguyên dương có k ước nguyên tố lẻ phân biệt thì
( ) 2k n ⋮ ϕ
Bài 26.
a) Chứng minh rằng với mọi n lẻ, n>1 thì ( )22 n n / + ⋮
b) Chứng minh rằng có vô số số n chẵn để ( )22 n n + ⋮
Bài 27. Cho k là số nguyên dương, p là số nguyên tố thỏa mãn 2 |21 k p + . Chứng minh rằng ( ) 1 12k p + ⋮
Bài 28.( JBMO TST - April camp-2021).
Có 2 n > số nguyên khác 0, sao cho mỗi số trong chúng chia hết cho tổng 1 n số còn lại. Chứng minh rằng tổng của n số đúng bằng 0.
Bài 29. (Bungari MO2006) Cho p là số nguyên tố với 2121. p p Chứng minh rằng với mọi số nguyên dương n thì số ()( ) 1!2n pp−+ có ít nhất ba ước số nguyên tố phân biệt.
II.2. SỐ CHÍNH PHƯƠNG NHÌN TỪ ƯỚC SỐ CỦA NÓ.
Ta biết rằng số chính phương là số bằng bình phương đúng của số tự nhiên, như vậy trong phân tích chuẩn của số chính phương thì các số mũ của các thừa số nguyên tố đều chẵn. Từ đó ta dễ dàng suy ra số các ước số của số chính phương luôn luôn lẻ Do đó ta có các khẳng định sau: Nếu n là số chính phương thì theo bổ đề ước đúng ta có:
+ ( ) ;|2 p ppnvn∀=> ⋮ .
+ ( ) 21nk=+ τ
II.2.1. Số chính phương có số mũ của các thừa số nguyên tố đều chẵn.
Từ nhận xét trong phân tích chuẩn của số chính phương các số mũ đều chẵn ta có một số bài toán sau.
Bài toán 1. Tìm số chính phương nhỏ nhất có đúng 2021 ước số sao cho nó có nhiều ước số nguyên tố nhất.
Tiếp cận bài toán: Từ đề bài ta có số các ước số của n là ( ) 2021 n = τ . Số các ước nguyên tố của n bằng số thừa số của 2021 trong phân tích chuẩn, từ đó sẽ tìm được lời giải!
Bài giải:s
Gọi A là số cần tìm, gọi các ước nguyên tố của A là 12;;...; kppp
Ta có A viết dạng chuẩn 12 12....; k i kAppp=∈ ℕ α αα α
Theo đề bài, số các ước của A là 2021, do đó ( ) ( )( ) ( ) 1211...12021 k A =+++=τααα
Mà 20211.202143.47 == nên tối đa có 2 ước nguyên tố
Do yêu cầu bài toán tìm số chính phương nhỏ nhất nên chọn hai số nguyên tố 122;3pp==
So sánh 42462.3 và 46422.3 , dễ dàng kết luận 4642 min 2.3 A = .
Lời bàn: Yêucầubàitoáncóvẽ như khó,nhưngchỉ suyluậntừ hàm ( )n τ tacó lời giải khá đẹp! Từ đó dễ dàngthay đổisố 2021 bởisố kháctùy ý, tacũngcó bài giải tương tự
Bài toán 2. Chứng minh rằng nếu a;b là các số nguyên dương thỏa mãn
22 | ababa ++ thì a là số chính phương.
Tiếp cận bài toán: Từ điều kiện ab là ước của biểu thức nên hướng xét ước nguyên tố có thể sử dụng được. Để a là số chính phương thì mọi ước số nguyên tố của a đều có lũy thừa chẵn.
Bài giải:
Từ 22 | ababa ++ 222|| aabaab ++
Gọi p là ước nguyên số bất kỳ của a, theo bổ đề LTE ta giả sử () p va là số lẻ
Đặt 2 1 l apx + = với x nguyên dương và ();1gcdxp =
Từ giả thiết ta có: 2121 |;||ll paabpb ++ => . Ta đặt 1 l bpy + = với y nguyên dương.
Ta có:
()()
22 211211211212 || lllllll pxpypxpypxppxpyx +++++++ ++ ++
212 |.| l ppxpyxpx + ++
Mâu thuẫn với ();1gcdxp = . Do đó () p va là số chẵn. Vậy a chính phương.
Lời bàn: Chỉ khaithácsố mũ đúng của số nguyên tố trong số chính phương phải chẵn, cho ta lời giải hay,ngắn gọn!
Bài toán 3. Cho 17 số tự nhiên có ước số nguyên tố không vượt quá số 7. Chứng minh luôn tồn tại ít nhất hai số có tích là số chính phương.
Tiếp cận bài toán: Rõ ràng các ước số nguyên tố chỉ cố thể là 2,3,5,7. Cần phải chỉ ra tồn tại hai số mà tích của nó có lũy thừa bậc chẵn của các thừa số nguyên tố.
Tồn tại hai số trong 17 số phải chăng có bóng dáng của nguyên lý Dỉichlet.?!
Bài giải:
Gọi 17 số tự nhiên đó là 121617 ;;.;; nnnn … . Do ước số nguyên tố không vượt quá 7nên trong phân tích chuẩn ra thừa số nguyên tố của các số đó đều có dạng: với 1;2;.,17 i =…
Mỗi số ;;; iiii abcd nhận các giá trị là chẵn hoặc lẻ (xem số 0 là số chẵn). Do đó có 4 216 = dãy bốn số ( ;;; iiii abcd ) có tính chất chẵn lẻ khác nhau trong 4 vị trí của bộ số. Chẳng hạn như (ch; ch; ch; ch); (ch; ch; ch; lẻ); ….; (lẻ; lẻ; lẻ; lẻ) (kí hiệu: ch là chẵn)
Mà có 17 số tự nhiên khác nhau nên theo nguyên lý Dirichlet tồn tại 2 bộ số ( ;;; iiii abcd ) có cùng tính chẵn lẻ của từng vị trí trong bộ số, giả sử đó là 2 số nk và nl khi đó tích hai số
200
.23.5.72.3.5.7(2.3.5.7)
222222
klklklkl aabbccdd abcdabcd klnn n ++++ ==== là số chính phương.
Nhận xét: Ta có thể nêu bài toán tổng quát
Cho số tự nhiêncó ướcsố nguyêntố không vượtquásố nguyêntố thứ kgọilàpk (trongdãycácsố nguyêntố sắptheothứ tự tăng dần).Chứngminhrằngtrongnsố luôntồntạiítnhấthaisố cótíchlàsố chính phương.
Chứng minh: Các số tự nhiên n có dạng phân tích chuẩn là: 12 12 kc ab kppp . Mỗi số mũ của p nhận một trong hai giá trị chẵn hoặc lẻ. Nên có dãy gồm 2k bộ số có tính chất chẵn lẻ khác nhau. Mà 2k <n nên theo nguyên lý Dirichlet, tồn tại ít nhất 2 số có cùng tính chất chẵn lẻ của các số mũ, khi đó tích của hai số đó sẽ có số mũ chẵn, chứng tỏ tích hai số đó chính phương.
Bài toán 4. Cho hai số nguyên dương a;b thỏa mãn 2223aabb +=+ . Chứng minh ;221abab−++ là các số chính phương.
Tiếp cận bài toán: Cần chứng minh ;221abab−++ là các số chính phương, như vậy tích hai số đó sẽ có liên hệ với giả thiết. Từ mối quan hệ đó ta sẽ có hướng đi. Bài giải:
Từ () 222222 2322 ()221. aabbababbababb +=+⇔−+−=⇔−++=
Ta gọi p là ước số nguyên tố chung của ab và () 221 1 abp++>
Khi đó theo bổ đề LTE, ta có: ()1 p Vab−≥ và () 2211 p Vab++≥
suy ra: ()() 2 21 2212 (())() ppp VababVabVab−++=−+++≥
Do đó: () ()() 2 2 22 1 ppp VbVbVb ≥ ≥⇔≥
Suy ra: | pb , mà () 1 hay ()| p Vababppab −≥ ⋮
nên ta cũng có () || papab + |(221)| 1 pabp ++ vô lý vì 1 p >
Chứng tỏ ab và 2 2 1 ab++ không có ước nguyên tố chung hay chúng nguyên tố cùng nhau, mà () 2 (221 ) ababb −++= nên tồn tại các số nguyên tố 1p là ước của ab và 2p là ước của 221 ab++ , mà () () 1 1 2 22 [()]1 pp VababVb −++=
Suy ra ()()111 221 () 2 ppp VabVabVb −+++= () 11 ()02 pp
VabVb −+= là số chẵn, do đó ab chính phương.
+ Tương tự 2 2 1 ab++ chính phương.
201
Lời bàn: Vai trò rất lớn của bổ đề LTE trong việc chứng tỏ số mũ đúng của các
ước số nguyên tố là chẵn! Từ ý tưởng đến lời giải khá hợp lý và thuyết phục!
Bài toán 5. Cho 3 số nguyên dương a; b; c thỏa mãn điều kiện:
()()() 2 123 caccbcb + = ++
Chứng minh c là số chính phương.
Tiếp cận bài toán: Từ biểu thức đề bài cho sẽ xuất hiện mối quan hệ giữa c và các số a,b. Số c chính phương thì số mũ của mọi ước nguyên tố p của nó đều chẵn.
Bài giải:
Ta cần chứng minh trong phân tích chuẩn, tất cả các số mũ của các thừa số nguyên tố đều là các số chẵn.
Từ giả thiết ta có: ()22216+5 caccbcb +=+ suy ra 2b chia hết cho c.
Ta gọi p là ước số nguyên tố bất kỳ của c.
Ta gọi số mũ đúng của p trong phân tích chuẩn của số tự nhiên n, ký hiệu là ( ) p Vn Với ký hiệu đó, ta có: 2 | cb suy ra ( ) ()()() 2 2 pppp VbVcVbVc ≥ ≥ (1)
+ Ta giả sử: ( ) ( ) ( ) ,do 1;1 = pp VbVcacc ≥+ nên ( ) 10 p Vac +=
Do đó từ giả thiết suy ra: ( ) ( ) ( ) ( ) ( ) 23 ppppp VcVcbVcbVcVc =+++≥+
Suy ra: ( ) 0 p Vc ≤ mà ( ) 0 p Vc > dẫn đến vô lý. Tức là điều giả sử: ( ) ( ) pp VbVc ≥ không xảy ra.
+ Như vậy: ( ) ( ) pp VbVc < . Từ giả thiết ta cũng có:
( ) ( ) ( ) ( ) ( ) ( ) 23 2 pppppp VcVcbVcbVbVbVb =+++≥+= (2)
Từ (1) và (2) ta có: ( ) ( ) 2 pp VbVc = . Chứng tỏ số mũ của các thừa số nguyên tố p trong phân tích chuẩn là số chẵn. Vậy c là số chính phương.
Lời bàn: Qua các bài toán trên, thấy được chứng minh số chính phương bằng cách số mũ đúng của các thừa số nguyên tố là khá hay và dễ sử dụng.
II.2.2. M.ột số dạng khác nhìn từ ước số để giải toán số chính phương.
1. Sử dụng số chính phương xét theo lớp đồng dư modulo p
Ta thấy các số chính phương khi chia theo lớp đồng dư nào đó thì nó không thỏa đầy đủ cả hệ thặng dư, mà nó chỉ có một số lớp nào đó và sẽ có những lớp nó không
bao giờ xuất hiện. Chẳng hạn như không bao giờ có số chính phương dạng 32 k + .
Các bài toán sau thể hiện quan điểm đó.
202
Bài toán 1. (Turkish‒ 2013)
a/ Tìm tất cả bộ ba số nguyên tố ( ) ;; pqr sao cho pqr ++ không chia hết cho 3 và cả hai pqr ++ và 3 pqqrrp+++ đều chính phương.
b/ Có phải bất kỳ bộ ba số nguyên tố ( ) ;; pqr sao cho 3 pqr ++ ⋮ và cả hai pqr ++ và 3 pqqrrp+++ đều chính phương.
Tiếp cận bài toán: Với bài này, hướng tiếp cận đầu tiên là xét tính chẵn lẻ của các số nguyên tố. Khi đó sẽ có một số nguyên tố chẵn là 2, lúc này biểu thức sẽ gọn hơn, sẽ có hướng đi tiếp.
Bài giải: (Đáp số: Các hoán vị của ( )2;3;11 )
Giả sử 2 pqrx ++= và 2 3 pqqrrpy +++= với ; xy nguyên.
Chúng ta chứng tỏ rằng một trong các số nguyên tố đó là 2. Thật vậy: nếu tất cả các số nguyên tố đều lẻ, xét theo modulo 4 các số nguyên tố lẻ chi có hai dạng 41 k ± nên p,q,r đều là hoán vị của ( ) ( ) ( ) ( ) ( ) ( ) ;;1;1;1;1;1;3;1;3;3;3;3;3 mod4. pqr ≡
Trong đó các trường hợp không xảy ra là: (1;1;1); (1;3;3) từ ( ) 2 3mod4 xpqr=++≡ và trường hợp (1;1;3); (3;3;3) từ ( ) 2 33mod4ypqqrrp−=++≡ ( vô lý).
Do đó phải có ít nhất một trong ba số p; q; r bằng 2, không mất tính tổng quát giả sử
2 p = và qr ≤ . Khi đó: 2 2 qrx += và 2221qryx = + .
Bây giờ nếu 3| y thì ( )( ) ( ) 2 2211mod3qry++=+≡
Tới đây ( )2mod3 qr≡≡ hoặc ( )0mod3 qr≡≡ . Nhưng khi ( )0mod3 qr≡≡ , ta được điều vô lý ( ) 2 2mod3 x ≡ . Khi ( )2mod3 qr≡≡ , ta được ( ) 2 21mod3 x −≡ và 3| x mâu thuẩn giả thiết 3 x / ⋮ , như vây 3| y không xảy ra.
Bây giờ, từ 3 x / ⋮ ta được ( ) 22 1mod3 xy≡≡ và như vây: ( ) 22210mod3qryx=−+≡
Đó là 3 q = , bây giờ 2 5 rx=− và 22 321 ryx=−+ . Do đó: ( )( ) 2 5 933ryyy = =+−
Cho 2;3;5 r = không có số nguyên x nào. Do đó 5 r > , từ 31 y −= không có
nghiệm,
35 3 yry−==+ và 11 r = . Khi 4; 8 xy== ta được: ( ) ( ) ;; 2;3;11 pqr =
b/ ( ) ( );;2;11;23pqr = thỏa mãn điều kiện bài toán.
Lời bàn: Từ nhận xét số chính phương sẽ không xuất hiện ở một số dạng (theo lớp đồng dư) đã giải quyết bài toán nhẹ nhàng linh hoạt.
Bài toán 2. Chứng minh rằng số 2n (với n là số tự nhiên 2 n > ) biểu diễn được thành tổng của 4 số chính phương khi và chỉ khi n chẵn.
Tiếp cận bài toán: Rõ ràng ta phải biểu diễn 2n thành tổng 4 số bình phương, từ đó nên xét ước đúng của 2 ở vế phải và cần thể hiện tồn tại bộ số sao cho đẳng thức đó xảy ra.
Bài giải:
Giả sử 2222 2n abcd=+++ (1) (với a;b;c;d là các số tự nhiên)
Gọi ( ) ( ) ( ) 222 ;; xVayVbzVc === và ( ) 2 ) ,;;(; tVdxyzt =∈ ℕ
Theo tính chất của số mũ đúng, ta có 2.’;2.’; 2.’; 2.’xyzt aabbccdd ==== với a’ ; b’ ; c’ và d’ là các số tự nhiên lẻ. Do vai trò của các số như nhau nên không mất tính tổng quát giả sử xyzt ≤≤≤ . Khi đó:
Nếu 210nx−−> thì vế phải (4) lẻ, vế trái (4) chẵn, vô lý!
Nên 210nx−−= lúc đó:
( ) 2222 32 2. 2 ()()( ) .2 yxzxtx abcd ′′′′ ⇔=+++ , điều này cũng không thể
xảy ra trên tập ℕ
+ Vế phải của (3) có 4 số hạng lẻ: Cũng từ nhận xét trên ta có (3) có dạng:
222284 221nxnxKK =+⇔=+ , tương tự trên ta có:
( )22021nxnx −−=⇔=+
Ta nhận thấy: 212222 () 22222 xxxxx + =+++ .
Vậy 2n biểu diễn được thành tổng của 4 số chính phương khi và chỉ khi n chẵn.
Lời bàn: Chỉ sử dụng lý luận chẵn lẻ của dạng các số chính phương để giải.
2. Đưa về phương trình nghiệm nguyên dạng tích
Bài toán 3. (BMO‒2015) Ba số nguyên dương p; a và b thỏa mãn phương trình
222 pab += . Chứng minh rằng nếu p là số nguyên tố lớn hơn 3 thì a là bội của 12 và ( )21 pa++ là số chính phương. (của Gerry Leversha)
Tiếp cận bài toán: Từ giả thiết đã xuất hiện phương trình tích mà một vế là lũy thừa của số nguyên tố nên khá thuận lợi cho việc xét các yêu cầu của bài toán.
Bài giải: ( )( ) 222 Pt pbababa ⇔==−+
1 ) 2 2.’ 2.’ 2.’ 2.’ (2 nxyzt abcd⇔=+++
( ) 22222222
Do ( ) ( ) ( ) { } 2222
222 22.’; ; 2.’2nxt VminVaVdx ≥…= nên
Các ước của 2 p là 2 1; ; pp không có trường hợp babap +=−= , vì vậy phải xét trường hợp 2bap += và 1 ba−= . Khi đó: 2211 và 22 bapp+− ==
( ) 2222222222
2 2 ’ 2.’ 2.’ 2.’.nxyxzxtx abcd⇔=+++
( )( )211 app =−+
( ) 22222
2 ’ 2.’ ()()()2.’2.’.3nxyxzxtx abcd⇔=+++
Ta thấy vế phải của (3) là tổng của 4 số chính phương, vế trái (3) là số chẵn, mà đã có a ′ lẻ nên trong vế phải của (3) có 2 số hạng lẻ và
2 số hạng chẵn hoặc cả 4 số hạng đều lẻ.
Từ nhận xét: ()2224kk = và ()() 2 2141181 kkkk′ +=++=+ nên ta xét hai trường hợp:
+ Vế phải của (3) có 2 số hạng lẻ và 2 số hạng chẵn: Theo nhận xét trên thì (3) có dạng:
( ) 221 2422214 nxnxKK =+⇔=+
204
Từ p lẻ 1 p + và 1 p đều chẵn, hơn nữa trong hai số đó có số chia hết cho 4 8|2a (1)
Từ p không chia hết cho 3 một trong hai số 1; 1 pp+− chia hết cho 3 3|2a (2)
Từ (1) và (2) 24|2a hay 12| a (đpcm)
Xét () () 2 2 2 1 2121211 2 p papppp
++=++=++=+
là số chính phương.
Lời bàn: Từ tích hai thừa số khác nhau bằng p2 nên chỉ có một số bằng 1 số kia bằng p2 . Đó là then chốt của bài này.
Bài toán 4. (Thailand‒2015) Gọi m, n là hai số nguyên dương sao cho mn là
số lẻ. Chứng minh ( )( ) 357 mnmn ++ không thể là số chính phương.
Tiếp cận bài toán: Ta thử chứng minh phản chứng! Từ mn là số lẻ và
( )( ) 357 mnmn ++ là số chính phương nên ta cần xét quan hệ hai thừa số
( ) ( ) 357 ; mnmn ++ . Rõ ràng là cần xét ước chung của chúng
Bài giải:
Ta chứng minh bằng phản chứng, giả sử ( )( ) 357 mnmn ++ là số chính phương.
Gọi ( ) ; dgcdmn = thì ; mdxndy == với ( ) ;1gcdxy = và
( )( ) ( )( ) 2 357357 mnmndxyxy ++=++ (1)
Từ 2|()2|() mnxy //
Gọi c là ước số chung lớn nhất của 3 xy + và 35xy + . Dễ thấy 3 xy + và 35xy + đều lẻ, vì vậy c cũng là số nguyên lẻ. Từ |3 cxy + và |35 cxy + ta có: ( ) ( ) ( ) 5357 cxyxy +−+ và ( ) ( ) ( ) 35773 cxyxy +−+
Vì vậy ta có: | 8 cx và 8 | cy , từ c là số lẻ ta có: | cx và | cy , nhưng ( ) ;1gcdxy = nên 1 c = . Do đó ( ) 3;571gcdxyxy++=
Từ (1) và giả thiết ( )( ) 357 mnmn ++ là số chính phương, nên phải có: ( )( ) 357 xyxy ++ là số chính phương, mà ( ) 3;571gcdxyxy++= , do đo phải tồn tại hai số nguyên a; b sao cho: 2 3 xya += và 2 57xyb += (2)
Từ 3 xy + và 35xy + đều lẻ nên a; b đều lẻ. Do đó ( ) 22 0mod8 ab−≡
Nhưng từ (2) ta có: ( ) 22 4 y abx=− và 2|() xy / .Vìvậy: 22 ) 8|(ab ,mâu thuẫn!
Vậy ( )( ) 357 mnmn ++ không thể là số chính phương.
Lời bàn: Tích hai số nguyên tố cùng nhau là số chính phương chỉ khi cả hai cùng chính phương, bài toán trở nên quen thuộc.
Bài toán 5. Tìm tấtcả các số nguyêndương 1 n ≥ sao cho 2 3n n + là số chính phương.
Tiếp cận bài toán: Từ điều kiện đề bài đã ẩn chứaphương trình nghiệm nguyên.
Sau khiphân tích thành tích,một vế sẽ là lũy thừa của số nguyên tố.Hướng giải đã xuấthiệ
Gọi m là số nguyêndương thỏa 22 3n nm +=
Từ ( )( ) 3n mnmn−+= suyra 3 là ước của cả mn và mn + ,do đótồn tạisố nguyên 0 k ≥ sao cho: 3;3 knk mnmn−=+=
Từ 3321 knk mnmnknknk −<+ < <− −≥
Nếu 21nk−= thì ()() ( ) ( ) 21 2333313312.3 nkkknkkk nmnmn =+−−=−=−=−=
Suy ra: 321 k nk==+
Ta có: () 22 321122...21 m mm m mCm =+=+++>+
Do đó 0;1kk== ,như vậy: 1;3nn==
Nếu 21nk−> thì 22nk−≥ và 2 knk≤−− 2 33knk < ,như vậy: 223 2 3333 3231 82 81 22 161624 () [()]
nkknknknknk n nknk =−≥−=−−=− ≥+−−=−−
Kéo theo: 8 127 kn +≥
Mặtkhác: 2 2 714 14 nknk ≥+ ≥+ ,mâu thuẫn!
Vậy chỉ có 1;3nn==
Lời bàn: Đưa bài toán về dạng tích bằng 3n , rõ ràng mỗi thừa số cũng có dạng lũy thừa của 3 là bước quan trọng trong bài này.
Bài toán 6.( JBMO TST - April camp-2021).
Cho dãy ( ) na được xác định bởi 2 111 45;15nnn aaaa ==+ với 1 n > .Chứng minh trong dãy không chứa số chính phương nào.
Tiếp cận bài toán:Từ công thức truy hồidễ thấy số hạng đứng sau là bộicủa số liền trước.Từ đó dễ su được 45 là ước của mọisố hạngtrong dãy. Đó cũng là manh mối tìm ra lờigiảicho bàitoán!
Bài giải.
Bằng quy nạp ta chứng minh được 0 na > vớimọisố nguyên dương n.
Giả sử trong dãy tồn tạisố só n đề na là số chính phương.
Ta có ( ) 2 11111515nnnnn aaaaa=+=+
Từ đó suy ra .Suyra 1 45| naa = vớimọisố nguyên dương n.
Gọi 1 k > là số nguyên dươngnhỏ nhấtthỏa ka là số chính phương và 1 45 k ax = với x là số nguyên dương. Tacó
()() 2 45451515.331 k axxxx =+=+ là số chính phương
Suy ra () 3.31 xx + phảilàsố chính phương,mà ta thấy () 3;311 xx += nên phảicó
3và31 xx + phảilà các số chính phương,vìvậy 30và310 xx=+= . Điều đó không
thể vì 0 na > vớimọisố nguyên dương n
Vậy không tồn tại số chính phương nào trong dãy đã cho.
Lờibàn: Chỉ cần phản chứng, khai thác yếu tố 45 là ước của mọi số hạng trong dãy, ta có lời giải đẹp và “dễ thương”!
II.2.3. Bài tập rèn luyện
Bài 1.
a/Tìm số tự nhiên có 4 chữ số biếthai chữ số đầu giống nhau và hai chữ số cuối giống nhau.
b/Tìm số tự nhiên có 4 chữ số biếtnó bằng bình phương của tổng haisố có hai chữ số được tạo bởihaichữ số đầu và haichữ số cuốicủa số cần tìm.
Bài 2. Tìm tấtcả các số chínhphương có 4 chữ số:
a/biết4 chữ số đều chẵn.
b/biết4 chữ số đều lẻ.
Bài 3. (Belarusian ‒2015) Cho 3 số nguyên a; b; c thỏa mãn:
22
2222 2 acc abacbc += + ++
Chứng minh rằng tích bc là số chính phương.
Bài 4. (Hy ‒ lạp 2015) Chứng minh rằng không có số nguyên dương n nào để cả hai số sau đều là chính phương ()12n n + và () 2 32n n + +
Bài 5. (Slovenia‒2014) Tìm tất cả các số nguyên dương n sao cho
() 2 114!33.134 n nnn+−++ là số chính phương.
Bài 6.( TS lớp 10 trường THPT Chuyên Thành phố Hà Nội năm học 2016 –2017 )
Cho số nguyên dương n thỏa mãn 2 22121 n ++ là số nguyên. Chứng minh 2 22121 n ++ là số chính phương.
Bài 7.( TS lớp 10 trường THPT Chuyên Tỉnh Hải Dương năm học 2015 –2016)
Tìm các số nguyên k để 4328232610kkkk −+−+ là số chính phương.
Bài 8.( TS lớp 10 trường THPT Chuyên Tỉnh Vĩnh Phúc năm học 2012 –2013)
Tìm tất cả bộ hai số chính phương (n;m) mà mỗi số có đúng 4 chữ số, biết rằng mỗi chữ số của m bằng chữ số tương ứng của n cọng thêm với d (với d là số nguyên dương nào đó cho trước)
Bài 9.( TS lớp 10 trường THPT Chuyên Tỉnh Thanh Hóa năm học 2012 –2013)
Tìm số nguyên dương n sao cho ( )21 26 nn là số chính phương.
Bài 10.( TS lớp 10 trường THPT Chuyên Quốc Học Huế năm học 2011 – 2012)
Tìm các số nguyên tố p sao cho sao cho hai số ( )21 p + và ( ) 2 21 p + đều là các số chính phương.
n i i
a =
là số chính phương với mọi n. Tìm số số hạng nhỏ nhất bằng không.
II.3. DÙNG ƯỚC SỐ ĐỂ GIẢI PHƯƠNG TRÌNH NGHIỆM NGUYÊN
Phương trình nghiệm nguyên là một dạng toán thường xuyên xuất hiện trong các
đề thi ở phần Số học. Chỉ có một số dạng phương trình nghiệm nguyên có phương pháp giải cụ thể chẳng hạn như: phương trình Diophant, phương trình Pell, phương trình Pitago, …Ngoài các dạng đó thì hầu như không có phương pháp angorit cụ thể được, khi đó tùy vào từng bài mà có hướng tiếp cận khác nhau. Lúc đó sẽ xuất hiện các lời giải độc đáo, lời giải đẹp và thú vị Đó cũng là lý do để Số học là nữ hoàng!
Một trong những hướng giải phương trình nghiệm nguyên là đánh giá chia hết, một số nguyên tố là ước của vế này thì nó cũng là ước của vế kia. Tùy theo mối quan hệ giữa các biểu thức trong đề mà ta có hướng tiếp cận riên. Vì vậy trong phần này chúng tôi sẽ chọn một số bài mà hướng tiếp cận được nhìn từ ước số, chọn ước số phù hợp để “công phá từ bên trong đánh ra” để giải quyết bài toán.
II.3.1. Bài tập về phương trình nghiệm nguyên qua góc nhìn ước số.
Bài toán 1. (Olympic Balkan -2018) Tìm tất cả các cặp số nguyên () ; mn thỏa
mãn phương trình 55 16 mnmn −=
Tiếp cận bài toán: Rõ ràng nếu d là ước của một trong hai số m hoặc n thì nó sẽ là ước của số kia. Do đó ta tiếp cận bài toán từ hướng ước chung của m và n
Bài giải:
Nếu 0 m = thì 0 n = và ngược lại nên ()() ;0;0mn = là nghiệm của phương trình.
Xét trường hợp 0 mn ≠ :
Gọi ()gcd; dmn = , khi đó
Phương trình trở thành: 3535 16 dadbab −= .
Suy ra: 353|| adbad , do ();1ab = . Tương tự: 3 | bd .
Suy ra: . Do đó phương trình được viết lại: ()5555 ..1616 abtaabtbabtab −= −=
Như vậy: 55ab phải là ước của 16 hay {} 55 1;2;4;8;16 ab−∈±±±±±
- Do 0 ab ≠ nên 55 1 ab−≠
- Nếu 55 2 ab−= thì 1;1ab==− hoặc 1;1ab=−= suy ra
()() ()() ;;;1;1;8;2 ;;;1;1;8;2 abtd abtd =−− =−−
Khi đó: ()() ;2;2mn =− .
- Nếu 55 2 ab−> không mất tinh tổng quát giả sử ;2aba>>
Đặt 1(1)axx=+≥ ta được () () 55 55554321151010513116abxbxbxxxx −=+−≥+−=++++≥>
Nên 55 2 ab−> không xảy ra.
Vậy phương trình đã cho có hai nghiệm là ()() 0;0;2;2 .
Lời bàn: Đi từ hướng ước số bài toán trở nêndễ dàng.
Bài 2. (Đề đề nghị của trường Amstesdam ‒lớp 10‒2017) Tìm nghiệm nguyên dương của phương trình 2 221xy z += (1)
Tiếp cận bài toán: Nếu x là số chẵn thì 2 2x z sẽ phân tích thành tích khi đó dễ dàng xét ước số 3 và 7 của hai vế!
Bài giải:
Nếu x là số lẻ hay 2 1(mod2)2212(mod3) xy xz ≡ =+≡ , vô lý. Vậy x chẵn.
Từ (2) ta suy ra có hai trường hợp:
+ Cả hai số 2,2mm zz+− cùng có ước nguyên tố là 3 hoặc 7 1 20(mod3) m+ ≡ hoặc 1 20(mod7) m+ ≡ , điều này không xảy ra.
+ (2,2)1 mm zz+−= .Vì 122mm zz ≤−<+ nên chỉ có hai khả năng sau:
−= += hoặc
m my z z
21 (3) 221
−= +=
my my z z
23 (4)
27
- Xét hệ (3) từ hai phương trình ta nhận được phương trình:
11 212126(mod7) mym ++ += ≡ . Điều này không xảy ra bởi vì với 1 m ≥ thì {} 1 2(mod7)1,2,4 m rr + ≡ ∈ . Nên hệ (3) không có nghiệm nguyên dương.
- Xét hệ (4), từ hai phương trình của hệ, ta nhận được: 1 732(5) yym+ −=
Nếu 1125ymxz = = = = vậy (2,1,5) là nghiệm nguyên dương của (1).
- Nếu 1 2273404730(mod8)2 myyyy ymyn + ≥ =−≥ > −≡ = Khi đó, ta có: 1 (5)(73)(73)2 nnnnm+ ⇔+−= (6)
+ Ta có: * 73(1)(1)2(mod4) nnnn nN∀∈ +≡−+−≡ 73nn + có ước nguyên tố lẻ p, từ (6) ta suy ra p là ước của 1 2m+ điều này vô lý, vậy nếu 2 y ≥ thì phương trình (5) không có nghiệm nguyên dương. Vậy (1) có nghiệm duy nhất một nghiệm nguyên dương (2,1,5).
Lời bàn: Sau khilý luận được x chẵnbàitrở về dạng phương trình tích, mỗithừa số đều dạng lũy thừa củasố nguyên tố, hướng giải quyết kháhay!
Bài toán 3: Tìm số nguyên dương n để 2 3n n + là số chính phương.
Tiếp cận bài toán: Thực chất bài này là phương trình nghiệm nguyên tương tự bài toán trên. Do đó hướng tiếp cận dùng ước số 3 để giải quyết bài toán.
Bài giải:
Giả sử () () 3n knkn⇔=−+ (1)
Ta thấy: vế trái là lũy thừa của 3 nên 3 là ước của tích ()()knkn −+
Ta xét số mũ đúng của số nguyên tố 3 trong biểu thức trên, theo bổ đề LTE ta có: () ()() () () 3333 3 () n VVknknnVknVkn =−+⇔=−++
Gọi ()() 33 0; 0 2 () aVknbVknabn =−≥=+≥ +=
Ta thấy: trong (1) vế trái là lũy thừa của 3 nên vế phải cũng phải là lũy thừa của 3,
do đó từ (2) ta phải có: () 3và 30ab knknab −=+=≤<
Suy ra: () 332 3312baaba nn−=⇔−=
Nếu 1 ba−= thì 33321 aaanaba =⇔=+⇔=+ suy ra 0 a = hoặc 1 a = , khi đó:
+ 01an = =
+ 13an = = . Thử lại cả hai trường hợp đều đúng
Nếu 2 2 baba −≥ −≥ thì () 222 ) 233338.38.1 ( 2 8.12(.)2 b babbb n b =−≥−==+≥+− (BĐT Becnuli)
812 8128127812722 do 22 () nbnaanaana ≥−=−−⇔+≥ +≥+>+
8121414 aa ⇔+≥+ , vô lý vì 0 a ≥
Vậy chỉ có 1; 3 nn== thỏa mãn yêu cầu bài toán.
( ) ( )
Lời bàn: Đưa bài toánvề dạng phương trìnhtích, mỗi thừa số làlũythừa của số 3, rất quen thuộc!
Bài toán 4. Tìm tất cả các cặp số nguyên ( , ) thỏa mãn 212122xx y + ++=
Tiếp cận bài toán: Để ý rằng 212 221 xx pty + ⇔+=− , khi đó cả hai vế đều có dạng tích và 2 là ước số của tích, nên khả năng xét tính chẵn lẻ, do đó ta cần xét theo modulo 2.
Bài giải:
Giả sử ( ) ; xy là một cặp số nguyên thỏa mãn phương trình. Dễ thấy 0;0xy≥≠ và ( ) ; xy cũng là một cặp nghiệm của phương trình. Do đó ta chỉ cần xét trường hợp 0 y >
Nếu 0 x = , ta có : 2 y =± .
Nếu 0 x > , ta có: y là số lẻ.
Viết lại phương trình đã cho dưới dạng: ()() ( ) 1 11212xx yy + −+=+ (*)
Ta có: 1;1yy−+ là hai số chẵn liên tiếp nên luôn có một số chia hết cho 4, số kia chỉ chia hết cho 2, do đó 2 x > và phải xảy ra một trong hai trường hợp sau:
Trường hợp 1: Nếu 1 12. x yn −= , với là số nguyên dương lẻ, thay vào (*) và biến đổi ta được: ( ) 22 281 x nn −=− (1)
Do 10 n −≤ suy ra 2 80 n −≤ , do vậy 1 n = . Thay vào (1), ta thấy không thỏa mãn.
Trường hợp 2: Nếu 1 12. x yn += , với là số nguyên dương lẻ.
Từ (*) ta có: ( ) 22 281 x nn −=+ (2)
Suy ra: 2 80 n −> và 2 12(8) nn +≥− . Do đó 3 n = . Thay vào (2), ta được 4 x = , bởi vậy 23 y = Thử lại thấy đúng.
Vậy các cặp số nguyên ( ) ; xy phải tìm là: ( ) ( ) ( ) ( )0;2,0;2,4;23,2;23 .
Lời bàn: Đây là bài toán không khó, hướng giải đã có bóng dáng trong biểu thức của đề, tính chất của tích hai số chẵn liêntiếp được phát huy!
213
Bài toán 5. Tìm tất cả các bộ ba () ;; xyz các số tự nhiên thỏa mãn
222 2 xyzxyz ++=
Tiếp cận bài toán: Dễ thấy VP là số chẵn nên dễ dàng xét tính chẵn lẻ của bộ
() ;; xyz . Khi đó sẽ xuất hiện số 4 là ước của VP.
Bài giải:
Giả sử () ;; xyz là một bộ ba số tự nhiên thỏa mãn đề bài.
Nếu ;; xyz là các số lẻ thì 222 xyz ++ là số lẻ, suy ra 2xyz là số lẻ, vô lý!
Do đó tồn tại ít nhất một số chẵn, như vậy 2xyz phải chia hết cho 4, suy ra 4 là ước
của 222 xyz ++ . Suy ra ;; xyz phải là các số chẵn.
Phương trình trở thành 222 111111 4 xyzxyz ++= .
Tương tự phương pháp ta sẽ chứng minh được 111 ;; xyz là các số tự nhiên chẵn.
Tiếp tục quá trình này ta được dãy các bộ ba các số tự nhiên chẵn (;;) nnnxyz thỏa mãn
() 000111 (;;);;;(;;)2(;;) nnnnnn xyzxyzxyzxyz+++ == với mọi số tự nhiên n.
Như vậy với mỗi số nguyên dương k, cả ba số ;; xyz đều chia hết cho 2k
Do đó 0 xyz=== là nghiệm của phương trình. Thử lại thấy đúng.
Vậy bộ ba phải tìm là () 0;0;0 .
Lời bàn: Một số nguyên có vô hạn ước ( 2k ước) chỉ có duy nhất làsố 0. Bài này thực chất giải bằng nguyên tắc cựchạn sẽ rõ ràng hơn.
Bài toán 6. (Chọn đội tuyển Belarusian‒ 2015) Giải phương trình nghiệm nguyên không âm ;; abc : 3220153! ab c ++= (1) (của I. Gorodnin)
Tiếp cận bài toán: Ta thấy 3c! bị chặn dưới bởi 2015 nên sẽ biết được vùng nhận giá trị của c. Hơn nữa, mọi số nguyên không lớn hơn c đều là ước của c!. Do đó ta xét những ước đặc biệt của hai vế để tìm các giá trị thích hợp.
Bài giải:
+ Giả sử 1 a = , nếu 6 c = thì 2142 b = , vô nghiệm.
+ Nếu 7 c ≥ , từ (1) suy ra: 22018 b + chia hết cho 7 hoặc ( )220mod7 b +≡ , vô
+ Nếu 2 a ≥ , thì 3a chia hết cho 9, từ (1) ta có ( ) 21mod9, b ≡ do đó 6 b⋮
Nếu 0 b = thì ta có: 320163! a c += không thể được với 6 c = , với 7 c ≥ ta thu được 37 a ⋮ , điều này vô lý.
Do đó 6 b ≥ và từ (1) suy ra ( )31mod16 a a ≡ chia hết cho 4, giả sử 4, 6 atbq ==
Khi đó (1) trở thành 81 64 2015 3! tq c ++= (2)
Trường hợp 6 c = kéo theo 1 tq== , khi đó: 4, 6 ab==
Trường hợp 7 c ≥ ta được ( ) 4160mod7. tq++≡ Đó là điều không thể.
Cuối cùng chỉ có trường hợp ( ) ( );;4;6;6abc =
Lời bàn: Từ tính chất giai thừa đã giới hạn vùng của c, bài toán trở nên quen thuộc khi xét ước và chia theo lớp đồng dư.
Bài 7. (Chọn đội tuyển Belarusian ‒ 2015) Tìm tất cả các số nguyên dương n sao cho ( ) () 2 123nqqqrr =−−=+ với q và r là số nguyên tố. (của B. Gilevich)
Tiếp cận bài toán: Rõ ràng q,r đều là ước của n và là ước của hai vế, do đó có thể biểu diễn các biểu thức theo q hoặc r. Khi đó khả năng phương trình trở thành bậc hai theo một biến nên có hướng giải.
Bài giải:
Từ đề: ( ) () 2 123qqqrr−−=+ (1)
Nếu qr = thì 2 123rrr−−=+ , cho ta 4 r = , không nguyên tố, nên qr ≠
Từ (1) suy ra: ( ) 2 1 rqq suy ra 2 1 qqkr −−= và 23rkq += (với * k ∈ ℕ )
Thay 23 rkq=− vào (1) ta được phương trình: ( ) 22 22320 qkqk−++−= (2)
Từ 3!2015 c > , ta có: 6 c ≥ .
+ Khi 0 a = từ (1) ta có: ()23!672 b c =− không thể xảy ra!
214
Biệt số 42 4 2420kkk ∆=+−+ và Δ phải chính phương.
Ta thấy: ( ) ( ) 2222 2 kk<∆<+ với 5 k >
Nên ( )2 221224190kkk ∆=+⇔−+= . Nhưng phương trình cuối không có nghiệm nguyên.
Nếu 1 k = thì (2) trở thành 2 2310,1 kqq−+== không nguyên tố
Chú ý k là lẻ, từ 2 1 krqq=−− là lẻ. Nếu 3 k = thì (2) không có lời giải
Nếu 5 k = thì (2) trở thành 2 227130 qq−+= , khi đó: 13; 31 qr== và ( ) 232015nrr=+= .
Lời bàn: Từ nhậnxét rlà ước của n,nên biểudiễn được quar, khi đócó phương trình bậc hai ẩn q. Đãquy lạ về quen!
Bài toán 8. (Hy lạp ‒ 2015) Tìm tất cả bộ ba số nguyên dương ( ) ;; xyp với p là số nguyên tố thỏa mãn phương trình sau: 3 xy p xy = + (1)
Tiếp cận bài toán: Vế trái là phân thức đối với hai biến, khả năng phân thức sẽ còn rút gọn được và rút cho ước chung lớn nhất của x;y. Sau khi rút gọn biểu thức sẽ đưa về dạng tích lúc đó sẽ sử dụng ước phù hợp để tìm hướng đi.
Bài giải:
Gọi ( ) ; dxy = nên tồn tại các số nguyên dương ; absao cho ; xdaydb == và ( ) ;1ab =
Do đó từ (1) ta nhận được: 33dab abp = + (2)
Từ ( ) ;1ab = , ta được: ( ) ;1aab+= và ( ) 3; 1 bab+= , nên từ (2) ta có: 3 | abd + .
Do đó ta viết: 3 d k ab = + với * k ∈ ℕ (3)
Khi đó, (2) trở thành: 3 kabp = và từ đây có 3 |1bpb = và kap = . Ta có các trường hợp sau:
+) Nếu , 1 kpa== thì (3) trở nên: 3 33 22|hay8|và8|2 2 d ppdddp = = , vô lý!
+) Nếu 1 k = và ap = thì (3) trở nên:
()( ) 3321111 dpdpdddp + −++ = ==
Mà 2 11ddd++>− nên ta được: 11 d −= và 2 1 2 ddpd++=⇔= và 7 p =
Vậy nghiệm phương trình là ( ) ( ) ;; 14;2;7. xyp =
Lời bàn: Chỉ dùng ướcsố một cách linhhoạt ta có lời giải gọn và đẹp!
Bài toán 9. (Turkish ‒15/11/2014) Tìm tất cả các số nguyên dương x; y; z thỏa mãn:
3 3.78 yz x =+ (1)
Tiếp cận bài toán: Từ 382 = nên chuyển 8 sang vế trái để có 33 2 x xuất hiện tích ở hai vế. Khi đó chọn ước phù hợp để xét hai vế. Bài toán có hướng giải quyết! Bài giải:
VP của (1) lẻ suy ra x lẻ, từ ()()( ) 2 14223. 7 yz xxx−++= ⇔
Gọi 2 2 2 |2|44 (2;24)|6 |24 dxdxx dxxxdx dxx −+ =−++ ++
Mà |12 2612 dxdxd , do 2 x lẻ nên d lẻ { }1;3 d ∈
Ta có 4 trường hợp sau:
+) Nếu 23y x −= và 2 247z xx++= . Ta suy ra ( )2mod3 x ≡ , thay vào 2 244447(mod3)1111(mod3),vôlý z xx++≡++≡⇔++≡
−=≡
−=
+) Nếu 21 23.7 243
++≡
x xx
++=
3.72 z x =+ , thay vào phương trình sau ( ) ( ) 2 12 1 3.7223.72439.7 18.71232zzyzzy y ++++=⇔++= > Chia hai vế cho 3 223.76.743 zzy ++= , vế trái không chia hết cho 3 nên loại.
−= ++=
+) Nếu 1 2 23 243.7
x xx
y z
-1 3 2 y x =+ , thay vào phương trình sau ta được: ( ) ( ) 2 11 3 22.3 243.7 yyz ++++= ( ) () 12 3 3 3 3.7 2 yz⇔++=
Từ (2) với 1;2yy== không xảy ra.
Nên 2 y > thì 112 2|334|(33) yy+=>+ , do đó từ (2) có ()() 033.1mod4 z +≡−
z chẵn 2 zt = , ta được: ( ) ( ) 12 2 ( 33 7) 272 t yyt+=−+ , suy ra 3 là ước của vế phải.
nên | xyxy pppp < , như vậy: 2 3|49nnn++ . Mà ( )( ) 2 493627nnnn+=++
{ }33;9;27 n ⇔+∈
‒ Trường hợp 24 n = là không thể, vì ( ) 33336 2724 273813.19 n +=+=+=
không là lũy thừa của một số nguyên tố. ‒ Trường hợp 0; 6 nn== ta được: ( ) ( ) ( ) { } ;; 3;3;0;3;5;6 pmn ∈
Lời bàn: Tích haisố bằng lũy thừa của số p,phương phápgiải quen thuộc. Có thể thay số 27 bởi số khácVD: 64; 125….
+=>
ty y t
qq q
+ −=
Trừ vế theo vế ta được: 22 113232 43.2.3 22 yy yy q ++ =−≥− , từ 2 y > ta
được: 2 2 11.3 415.314 2 y y ≥−>−≥
Do đó chỉ có một khả năng 12 1;4332 yy q ==−− . Ta được 11; 3; 2 xyz=== .
Lời bàn: Khi phươngtrình đưa được về dạng tích ở một vế, vế kia làtích của hai lũy thừa củahai số nguyên tố. Khi đó vai trò của ước số và lớp đồngdư phát huythế mạnh!
Bài toán 10. (Ireland‒2015) Xác định tất cả các bộ số ( ) ;; pmn với p nguyên tố, m và n là hai số nguyên không âm thỏa mãn phương trình: 3 27 m pn−= .
Tiếp cận bài toán: Dễ thấy trong phương trình xuất hiện 33ab + nên sẽ có dạng tích trong khi VT là m p , như vậy mỗi thừa số là lũy thừa của p. Bài toán có hướng đi!
Bài giải: ()( ) 3227339 m pnnnn =+=++
Suy ra tồn tại x, y nguyên dương sao cho: 2 3; 39 xy pnpnn =+=+
Bài toán 11. (Ireland‒ 2015) Tìm tất cả các nghiệm nguyên dương của phương trình: 444222222 1008222 xyzxyyzzx +++=++ (*)
Tiếp cận bài toán: Nếu ta chuyển các biến về cùng vế trái và số 1008 sang vế phải, khi đó biểu thức ở VT là đẳng cấp bậc 4, nên có thể xem là phương trình trùng phương theo một biến. Ta xét thêm biệt số ∆ có chính phương không, nếu có thì VT phân tích được thành nhân tử, khi đó bài toán sẽ giải quyết được!
Bài giải: () ( ) ( ) 4222222*2 1008 xyzxyz ⇔−++−=− (**)
VT là tam thức bậc hai theo 2 x , ta có:
( ) ( ) () 22 2 22222242 yzyzyzyz∆=+−−==
Ta được: 2222 2() xyzyzyz =+±=±
Do đó ()() ( ) () ( ) 2222 ** 1008 xyzxyz ⇔−+−−=−
( )( )( )( ) 1008... xyzxyzxyzxyztuvw ⇔++−+++−−+==
Với ( ) ( ) ( ) ( ) ; ; ; txyzuxyzvxyzwxyz =++=−++=−+=+−
Ta thấy: tuvw =++ và 42 ...2.3.7tuvw = và 2; 2; 2 vwxwuyuvz +=+=+= (***)
Suy ra: ; ; ; tuvwcó cùng tính chẵn lẻ. Mà 1008 chẵn nên cả 4 số ;;; tuvw có cùng tính chẵn.
Suy ra ()() 222 39346220nnnnnn+−+=+−+=> 219
218
Khi đó: tuvw ′′′′ =++ và 2 ...3.7tuvw ′′′′ = . Nhưng t ′ không thể bằng 1 hoặc 3, bởi vì trong trường hợp này một trong các số ,, uvw ′′′ phải là bội của 7 có tổng lớn hơn t ′ .
Mặt khác, t ′ không thể bằng 63 hoặc 21 vì tổng các số còn lại không quá 5. Vì vậy, chỉ có hai khả năng: 7 t ′ = và 9 t ′ = .
Nếu 7 t ′ = , ta có: .. 9 uvw ′′′ = và các thừa số này là 1.1.9 hoặc 1.3.3 mà có tổng bằng 7, nên: ()() ;;;7;1;3;3 uvw t ′′′ = ′ hoặc các tổ hợp của 3 phần tử sau.
Nếu 9 t ′ = , ta có ..7uvw ′′′ = và thu được ()() ;;; 9;1;1;7 tuvw ′′′′ = hoặc các tổ hợp của 3 phần tử sau.
Từ (***), ta có ; và xwvywuzuv ′′′′′′ =+=+=+
Vậy ()()()()()() { } ;;8;8;2;8;2;8;2;8;8;6;4;4;4;6;4xyz ∈ .
Lời bàn: Khi phương trình (*) biến đổi về dược dạng trùng phươngvà phương trình có nghiệm nên bài toán trở nên bình thường quen thuộc!
Bài toán 12. (Serbian MO ‒2015) Tìm tất cả các số nguyên không âm thỏa mãn phương trình: ()20152015 21221 x y ++=+ . (của BojanBasíc)
Tiếp cận bài toán: Dễ thấy 3 là ước của 2015 21 + nên 9 là ước của 2015 (21) x + với 1 x > . Mà 3 cũng là ước của 21 y + khi y lẻ, do đó từ phương trình suy ra y phải chẵn. Ta thử tiếp cận theo hướng này. Bài giải:
Chỉ có lời giải với 1 x ≤ , phương trình có nghiệm là: ()()() ; 0,2015;1;2016 xy =
Giả sử 1 x > . Từ 2015 21 + chia hết cho 3, ta có
201520152015 21225mod9 x ++≡≡
Nên: ()24mod 9 y ≡ hay () 2 21mod 9 y ≡ . Gọi () 9 2 hord = hay ()21mod 9 h ≡ mà () 6 21mod 9 ≡ nên 6|2 y , suy ra 62yk=+ với k ∈ ℕ .
Từ () 2015 27mod 13 ≡ () 2015 218mod 13 +≡ , do đó () () 20152015 2128741mod13 x x ++≡+≡+ . Đây là điều không thể, vì 8x luôn luôn
có một số dư là: 1;5;8;12modulo13!
Cách 2: Với 02015xy = = , với 12016xy = =
152015 212(mod2 2212112 ) x ii x i
x y C xx =
=++−=++≡+
Nhưng 2019 y > , vì vậy ta có: 16|1 x + . Bây giờ ta xét phương trình modulo 17 và dùng () 2015 29mod 17 ≡ , ta được () 15 1081089mod 13 x +≡+≡ , không thể có!
Lời bàn: Ước số và đồngdư là linh hồn của bài giải!
Bài toán 13. (Chọn tuyển ĐH Vinh 2019-2020) Tìm tất cả các bộ ba số nguyên dương () ;; xyz thoả mãn: 1232.4 xyz+=+
Tiếp cận bài toán: Từ phương trình đã cho biến đổi về dạng tích, khi đó dễ thấy 2 là ước chung hai vế. Manh mối bài toán đã xuất hiện. Bài giải:
Phương trình 21 12321232 xyzxyt + ⇔+=+⇔+=+ (1), với t nguyên dương lẻ, 3 t ≥
Vì 1 y ≥ nên 31 y > , kéo theo 22 xt xt > > . Khi đó:
() () 122131 txty ⇔−=− (2)
Vì vế phải không chia hết cho 3 nên xt là số lẻ. (3)
Xét các trường hợp sau:
Trường hợp 1. Nếu y là số lẻ.
Theo định lí LTE, ta có: () () 2231311 y vv−=−= . Suy ra: () 2 211 t vt = = , không thoả mãn.
Trường hợp 2. Nếu y là số chẵn.
Ta lại có: () 6 21mod 13 ≡− nên () () 62212.2 141mod13 k y +=+≡+ và () 2015 27mod 13 ≡ 221
Đặt . Khi đó (2) trở thành: () 22191 txtk −=−
Xét hai khả năng:
+ Khả năng 1. Nếu k là số lẻ thì () () 2291913 k vv−=−= kéo theo 3 t = hay 1 z =
Thay vào (1) ta được: 297 xk −= .
()() 49723237 lklklk −=⇔+−=
Suy ra 237371 lkk k +≤ ≤ = . Từ đó ta tìm được: 2 l = hay 4;2xy==
Vậy ()() ;;4;2;1xyz =
+ Khả năng 2. Nếu k chẵn, đặt * 2; kmm=∈ ℕ thì 4 ym = ta có
()() 911mod5 k k ≡−≡ nên 5|21 xt . Cũng theo (3) ta đặt: * 21; xtnn−=+∈ ℕ thì
5|2.41 n suy ra ()43mod5 n ≡ , nhưng điều này vô lý, do ()41mod5 n ≡± . Vì vậy khả năng này không có nghiệm.
Vậy tất cả các bộ ba số nguyên dương là: ()() ;;4;2;1xyz =
Lời bàn: Từ 2 là ước chung nên nhờ bổ đề LTE làm công cụ, bài toán giải quyết khá ngắn gọn!
Bài toán 14. (Chọn đội tuyển ĐH Vinh-2016) Tìm các số nguyên dương ,,, abcd thỏa mãn: 233!1 bc ad++=+ , biết rằng tồn tại các số nguyên tố , pq sao cho
()()()()2 12111appqq =++=+− .
Tiếp cận bài toán: Rõ ràng việc đầu tiên phải tìm a, trong phương trình theo hai biến nguyên tố p;q ở hai vế dùng ước số lý luận để đưa về phương trình bậc hai theo một biến, sẽ tìm được a. Thay a vào phương trình ban đầu có chứa giai thừa (giống bài 6 đã nêu trên), bài toán quen thuộc.
Bài giải:
Trước hết ta tìm a
Ta có: ()()()()2 12111ppqq ++=+− () () 2 231ppqqq ⇔+=−− (1)
Nếu pq = thì (1) trở thành: 2 2314 pppp +=−−⇔= , không thỏa mãn là số nguyên tố.
Suy ra pq ≠ , khi đó 2 1 qqp ⋮ Đặt 2 1 qqkp −−= với + ∈ Z k Từ (1) ta có 23pkq += và k là số nguyên lẻ. Thay vào (1) ta được: () ()() 222 32122320 kqkqqqqqkqk −=−−⇔−++−= (2)
Xét phương trình bậc 2 ẩn ,q ta có ∆=+−−=+−+ 2242 (2)8(32)42420. kkkkk
Nếu > 5 k thì <∆<+ 2222 ()(2) kk nên phương trình (2) có nghiệm nguyên q khi và chỉ khi ∆=+⇔−+= 222 (1)224190, kkk không tồn tại k nguyên.
Do đó ≤≤ 15. k Vì k lẻ nên xét các trường hợp:
* = 1, k suy ra = 1 q không thỏa mãn.
* = 3, k không tồn tại q
* = 5, k suy ra ==3,31qp là các số nguyên tố. Khi đó = 2016. a
Tiếp theo ta tìm ,, bcd thỏa mãn ++= 2015233!. bc d (3)
Vì > 3!2015 d nên ≥ 6. d
* Xét = 1. c Nếu = 6 d thì = 2142 b không thỏa mãn.
Nếu ≥ 7 d thì từ (3) ta có + ⋮ 220187 b hay +≡ 220(mod7), b không thỏa mãn.
* Xét ≥ 2. c Ta có ⋮ 39 c nên từ (3) suy ra ≡ 21(mod9), b điều này chỉ xảy ra khi ⋮6. b Do đó ≥ 6. b Khi đó từ (3) suy ra ≡ 31(mod16), c do đó ⋮4. c Đặt ==6,4, bmcn ta có (3) trở thành: 201564813! mn d ++= (4)
Với = 6, d thì == 1, mn hay ==6,4,bc thỏa mãn.
Với ≥ 7 d thì từ (4) suy ra ++≡ 6140(mod7), mn hay không thể xảy ra.
≡ 40(mod7), n
abcd
==== 2016,6,4,6.
Vậy Lời bàn: Bài toán hay vì đã lồng ghép được hai ý tưởng giải quyết khá đẹp!
Bài toán 15.( JBMO TST - April camp-2021).
Tìm tất cả các số nguyên dương a;b;c và số nguyên tố p thỏa mãn:
2222 73691717 pabc +=++ (1)
Tiếp cận bài toán: Từ các số đã cho trong đề ta thấy đa phần có dạng 81 k + , lại có mọi nguyên tố lẻ đều có dạng 81 k + nên ta thử đi theo hướng xét đồng dư mod8.
Bài giải:
Giả sử bc ≤ , ta phân biệt hai trường hợp liên quan đến p
223
TH1: Nếu 2 p ≠ => p lẻ, vì vậy ( ) 2 1mod8 p ≡ . Khi đó ( ) 2222222917177367mod8abcabcp ++≡++≡+≡ (2)
Mặt khác ( ) 222 0;1;2;3;4;5;6mod8 abc++≡ mâu thuẫn với (2) nên 2 p > không xảy ra.
TH2: Nếu 2 p = thì 2222 91717736298 abcp++=+= , từ ;;1abc ≥
Ta có 2222 298917172617 abcc=++≥+ suy ra 4 c ≤ , ta có 2 khả năng sau:
+ Nếu 4 bc≤= thì 22 91726 ab+= suy ra 1 ab==
+ Nếu 3 bc≤≤ thì dễ dàng kiểm tra không có số nào thỏa mãn.
Vậy (1) có duy nhất một nghiệm ( ) ( );;;1;1;2;2abcp = .
Lời bàn: Chỉ việc xét hai vế theo mod8 ta đã loại tất cả trường hợp lẻ. Khi p = 2 bài toán trở nên dễ dàng thông qua phương pháp kẹp và bị chặn. Khá hay!
II.3.2. Bài tập rèn luyện
Bài 1. (Australian MO -2016- ngày1) Tìm tất cả các số nguyên dương n thỏa mãn
27nn + là số chính phương.
Bài 2. (Australian MO -2016- ngày 2) Cho ba số nguyên dương thỏa mãn
33 2c ab+=
Chứng minh ab = .
Bài 3. Chứng minh rằng phương trình ( )3 2211yb−=+ không có nghiệm nguyên dương.
Bài 4. Chứng minh rằng phương trình 442 4 xyz += không có nghiệm nguyên dương.
Bài 5. Giải phương trình nghiệm tự nhiên sau:
a/ 2 1!2!3!...!xy++++=
b/ 1!2!3!...! z xy++++=
Bài 6.
a/ Tìm số tự nhiên có ba chữ số abc biết: !!!abcabc =++
b/ Tìm tất cả các số tự nhiên bằng tổng các bình phương các chữ số của nó.
Bài 7. (IMO‒2014 Shortlist) Xác định tất cả các cặp (x;y) của các số nguyên dương thỏa mãn
22 3 7137||1 xxyyxy−+=−+
Bài 8. (Chọn đội tuyển Turkish‒30/1/2013) Tìm các cặp số nguyên dương (m; n) thỏa mãn:
2(()1)!1 nm nnn ϕ +−−=+ (với ϕ(n) là hàm Euler)
Bài 9. (Ireland‒ 2015) Tìm tất cả các số nguyên dương n sao cho cả hai số 837 + n và 837 ‒ n là lập phương của số nguyên dương.
Bài 10. (Austrian MO ‒ 2016) Hãy xác định tất cả các số nguyên dương k và n thỏa mãn phương trình: 2 20163n k −= . (của Stephan Wagnet)
Bài 11. (Chọn đội tuyển ĐH Vinh -2014) Giả sử p là số nguyên tố thỏa mãn 2(1) 51 p không chia hết cho 2 p . Tìm tất cả các cặp số nguyên dương ( ) , xy thỏa mãn 5 xppy +=
Bài 12. Xác định tất cả các số nguyên dương x;y và z sao cho 5 42013 yz x += .
Bài 13. (Romanian MO ‒2015) Tìm các số nguyên tố khác nhau p; q;r thỏa mãn:
11111 1 pqrspqrs −−−−= (của Merceca Franu)
II.4. ƯỚC SỐ TRONG CÁC BÀI TOÁN VỀ DÃY SỐ.
Ta biết rằng dãy số là hàm với biến số là các số nguyên không âm, nên giá trị của các số hạng trong dãy số cũng sẽ có những số nguyên. Khi dãy các số nguyên được xây dựng theo công thức hay quy luật nào đó nó sẽ có những tính chất đặc trưng của dãy số đó. Lúc đó, những tính chất mang đậm tính “số học” sẽ xuất hiện. Đây cũng là vấn đề lý thú về dãy số khi “nhúng” nó vào số học. Vấn đề Số học trong dãy số đã có nhiều bài viết, nhiều sách viết khá phong phú và hấp dẫn, bài viết này chỉ khai thác xung quanh vấn đề vai trò ước số trong các dãy số nguyên.
Những kiến thức lý thuyết về dãy số xin không trình bày ở đây (xem như các bạn đã biết)
II.4.1. Bài tập về dãy số qua góc nhìn ước số
Bài toán 1. Cho dãy số ( ) n u được xác định như sau: 1230,14,18uuu===− và ( ) 112763;4;...nnn uuun +−− =−= . Chứng minh rằng với mọi số nguyên tố p thì | p pu .
Tiếp cận bài toán: Dễ thấy dãy số đã cho bởi công thức truy hồi nên trước tiên ta thử tìm số hạng tổng quát của nó. Từ yêu cầu chứng minh | p pu nên có khả năng vận dụng đồng dư và các định lý liên quan. Bài giải:
Phương trình dặc trưng của dãy là: 3 760xx−+= có nghiệm: 1231;2;3xxx===− , suy ra ().1.2.3 n nn n uabc=++− . Thay các giá trị tương ứng ta tìm được số hạng tổng quát là ( )123 n n n u =++−
Với p là số nguyên tố thì theo định lý Fermat nhỏ, ta có:
( ) 22mod p p ≡ và ( ) ( ) 33mod p p −≡−
Do đó: ( )( ) ( ) 123mod 0mod pp upup ≡++− ≡
Chứng tỏ | p pu với mọi số nguyên tố p.
Lời bàn: Từ công thức số hạng tổng quát của dãy chỉ sử dụng định lý Fermat ta có lời giải cho bàitoán!
Bài toán 2. (Olympic Hylap 2014) Cho dãy (xn):
= = =+−
Tiếp cận bài toán: Dãy đã cho bởi công thác truy hồi, cần chứng minh mọi số hạng của dãy đều nguyên điều đó gợi ý cho ta phải tìm số hạng tổng quát của dãy. Công thức truy hồi có chứa căn, khả năng phải khử căn để tìm quy luật của các số hạng. Ta có 2011 là số nguyên tố nên chú ý khai thác tính chất này cho câu b. Bài giải:
a/ Từ giả thiết ta có: ()2 222 1 11 235441244 nnnnnnn xxxxxxx + ++ −=−⇔−+=− 22 1131 nnnn xxxx ++ ⇔−+=− (1)
Thay n bởi n+1 vào (1) ta được: 22 212131nnnn xxxx++++−+=− (2)
Ta xét phương trình bậc hai: 22 11 3.10 nn xxxx++ −++=
Từ (1) và (2) ta có xn và xn+2 là hai nghiệm của phương trình trên nên 21 2 21
1 ;1;2;3;....
354 nnn
1 2 1
x n xxx +
a/ Chứng minh tất cả các số hạng của dãy là số tự nhiên.
b/ Xem xét sự tồn tại số hạng của dãy chia hết cho 2011.
3(3) .1(4) nnn nnn
xxx xxx ++ ++
+= =+ (định lý Viet)
Từ (3) suy ra: 21 3 nnn xxx ++ =− và từ 1 1 x = , ta có mọi số hạng của dãy đều là số tự nhiên.
b/ Giả sử rằng tồn tại số hạng thứ k của dãy mà 2011| kx . Từ (4) suy ra:
2 1 2011|1 kx + +
Mà tất cả các số hạng của dãy là số nguyên và 2011| kx nên 2 1 2011|1 kx + + 2 1 1(mod2011) kx + ≡− ( ) ()()() 1005 1005 2 2010 1 1 1mod20111mod2011 k kxx + + ≡−=>≡− (5)
Mà 2011 là số nguyên tố nên ( ) 1; 20111 kx + = (thật vậy: giả sử ( ) 1; 20111 k xd + => thì 1 1 | | | |2011 k k k
dx dx dx d + + => từ (4) 2 21 . 1 kkk xxx++=+ ta có |1 d vô lý)
Theo định lý Fecma, ta có ( ) ( ) 2010 1 1mod20116 kx + ≡
Từ (5) và (6) mâu thuẫn. Vậy không tồn tại số hạng nào của dãy chia hết cho 2011.
Lời bàn: Sự khéo léo xem biểu thức chứa lũy thừa bậc hai là phương trình bậchai theo một ẩn và sự tồn tại nghiệm nên từ hệ thức Viet ta có được công thức tổng quát của dãy. Khi đó dễ dàng nhận thấy mọi số hạng đều nguyên. Phần còn lại là sử dụng định lý Fermat!
Bài toán 3. Cho dãy số ( ) n u được xác định: 12 ; uaub == và
2 1153;(1;2;...)nnn uuun +− =−= Chứng minh rằng với mọi cách chọn cặp số nguyên a;b
thì dãy trên hoặc không có số nào chia hết cho 2021, hoặc có vô số số chia hết cho 2021.
Tiếp cận bài toán: Dãy số này cho bởi công thức truy hồi nhưng phụ thuộc tham số a;b. Từ yêu cầu bài toán ta không thể xét trên số hạng tổng quát mà xét phép chia cho 2021 của các số hạng.
Bài giải:
Xét cặp số nguyên a;b tùy ý, với mỗi số i ∈ ℕ , ta có: 2021. iii uqr =+ với
;;02021iii qrr∈∈≤< ℤℕ .
Ta xét các cặp số: ( ) ( ) ( ) 011223 ;;;;;;... rrrrrr các cặp giá trị này là hữu hạn (do 02021 ir ≤< ) nên sẽ tồn tại hai số nguyên ( ) ;0 ijij ≤< sao cho hai cặp
( ) ( ) 11;và; iijj rrrr++ trùng nhau, nghĩa là có 11 và ijij rrrr++ == , điều đó chứng tỏ ( ) ( ) 11 mod2021vàmod2021ijij rrrr++ ≡≡
Từ cách xác định dãy ta chứng minh bằng quy nạp được: ( ) mod2021; ikjk uuk ++≡∀∈ ℕ (1)
Từ công thức truy hồi của dãy ta suy ra: ( ) ( ) 1111 33mod2021mod2021 ijij uuuu ≡ ≡
Tiếp tục quá trình đó ta có: ( ) mod2021;0 ikjk uuki ≡∀≤≤ (2)
Từ (1) và (2) suy ra: () ( ) mod2021; jk ikji uuki + ++− ≡∀≥
Hay ( ) mod2021;; nnT uunTji + ≡∀∈=− ℕ
- Nếu tập hợp 011 {;;...;} Tuuu không có số nào là bội của 2021 thì với mọi n ta có ( )0mod2021 n u ≡ / (điều này suy từ ( )mod2021 nr uu ≡ nếu ;01nkTrrT =+≤≤− )
- Nếu tồn tại 01 rT ≤≤− để ( )0mod2021 r u ≡ thì ( ) 0mod2021;, n unkTrk ≡∀=+∈ ℕ tức là có vô số số hạng của dãy là bội của 2021.
Do vai trò của hai số nguyên a;b không tham gia trong quá trình chứng minh nên mọi cách chọn cặp số nguyên a;b cũng không ảnh hưởng đến kết quả bài toán.
Lời bàn: Lợi dụng tính hữu hạn của tập số dư so với vô hạn của dãy để xét tính tuần hoàn của số dư, khi đó dễ xét vai trò của tham số a;b.
Bài toán 4. (Olympic vùng Vịnh2016) Cho số thực x thỏa mãn 1 3 x x += . Chứng minh rằng số 3030 33 xx + có ít nhất 352 ước nguyên dương phân biệt.
Tiếp cận bài toán: Ta thấy hai số 303033 ; xx là nghịch đảo của nhau nên số 3030 33 xx + có thể biểu diễn qua các số đồng dạng có số mũ thấp hơn. Điều đó cho phép ta nghỉ đến xét dãy 33 nn n uxx =+ Bài giải:
Đặt 33 nn n uxx =+ . Suy ra: ( ) ( ) ( ) 11 11 22 33333333333333 ()333 nnnnnnnnnnnn n uxxxxxxxxxxxx ++ ++ =+=+++=+++
Do đó: 3 3 113hay3 nnnnnn uuuuuu ++ =+=− , ta có 0 1 3 ux x =+= , suy ra mọi số hạng của dãy đều là số nguyên.
3 33 3 1 11 333.318uxxxx
=+=+−+=−=
hay 2 1 183.2 u ==
Và ( ) 2 1 3 nnn uuu + =− . Từ đó ta thấy 3|;1;2;3... n un∀=
Ta xét ước đúng của 3 cho công thức truy hồi trên. Theo bổ đề LTE ta có: ( ) ( ) 2 313 ()3 nnn vuvuu + =− , suy ra: ( ) ( ) ( ) ( ) 2 3133331 nnnn vuvuvuvu + =+−=+
Do 3| n u nên ( ) 22 gcd;33và3nnnn uuuu −=−> nên 2 3 n u phải có một ước nguyên tố khác tất cả các ước của n u . Như vậy 30u có ít nhất 30 ước nguyên tố phân biệt và ( ) 330 31 vu = .
Giả sử ít nhất 30 ước nguyên tố phân biệt của 30u là 1230 ;;...; ppp nên số các ước ( ) 30 1230...2ppp τ ≥ . Suy ra số các ước nguyên dương của 30u có ít nhất là ( ) 3035 23112 += .
Lời bàn: Từ xây dựng được dãy truy hồi, mỗi số hạng đều có ước là 3. Đã chỉ ra được 2 3 n u phải có một ước nguyên tố khác tất cả các ước của n u ,để 30u có ít nhất 30 ước nguyên tố phân biệt.
Bài toán 5. Cho dãy số ( ) m a biết rằng n m amn =+ Tồn tại hay không hai số nguyên dương phân biệt p, q sao cho p a là ước số của q a với mọi số nguyên dương n?
Tiếp cận bài toán: Khả năng giải bài toán bằng phản chứng, giả sử tồn tại hai số p;q thỏa đề ta cần chỉ ra số n(liên quan đến p;q) để p a không là ước số của q a .
Bài giải:
Giả sử tồn tại hai số p, q nguyên dương phân biệt sao cho n pn + là ước số của n qn + với mọi số nguyên dương n, thế thì nn qnpnqp +>+=>> .
Giả sử a là một số nguyên tố lớn hơn q và n là số tự nhiên thỏa mãn
(1)(1)1npa=+−+ . Khi đó n = (p+1)a –p (mod) a p n ≡ (1)
Vì pqa << nên ( ) ( ) , , 1 paqa== . Theo định lý nhỏ Fermat, ta có: ) 1(mod ) 1(mod ( 1 a p a p a p a ≡ ≡ + ). (mod 1 ( a p p a p ≡ + +
Do đó: ) (mod a p p n ≡ (2)
Từ (1) và (2) suy ra: ( ) 0(mod) n n pnahaypna +≡+ ⋮ (4)
Chứng minh tương tự, ta được: ( ) (mod)(3) nn qqavàqna ≡+ ⋮
Từ (1) và (3) suy ra: ) (mod a p q n q n ≡ + (5)
Từ (4) và (5) suy ra: ()qpa ⋮ . Điều này không thể xảy ra vì pqa <<
Vậy không tồn tại hai số nguyên dương phân biệt p, q sao cho n qn + chia hết cho n pn + với mọi số nguyên dương n
Lời bàn: Việc chọn số nguyên tố aqp >> để xây dựng số n sao cho a là ước của np + là kỹ thuật hay!
Bài toán 6. (Chọn ĐT KHTN Hà nội-2020) Cho dãy số nguyên ( ) na thỏa mãn
=== =−+∀≥
kk kk n xy xy a + =−=
Khi đó () 20182018 20192019 2 22 22 1 1 22
Mà x;y là nghiệm của phương trình 2 410xx−+= nên ta đưa về xét dãy () n b
được xác định bởi: 0121 2;4và 4 nnn bbbbb ++ ===− , khi đó ta có nn n bxy =+
Do đó: () 20182018 2017 22 2 2 kkkk kk k xyxybbb −=−
kk n axybbb =−
2 222 1 1 .... 2 kk k
Suy ra: () 22017 2
Ta có: ()() 1221...23.kkkkkk kxyxyxxyxyyT −=−++++=
Từ công thức truy hồi của dãy () n b ta suy ra kb chẵn với mọi số nguyên dương k.
- Nếu k lẻ thì ()() () 1221...4.,kkkkkk kkxyxyxxyxyyUU +=+−+−+=∈ ℤ
Do đó: chứng tỏ 4039 1 2 na
() () 2201722017 22
2 2 2222225220172
1 1 23.4....96....2.3.(2). 2 kkkk nkkkkk aTUbbTUbbA ===
aaa aaaan ++−
1;6;25 55;1nnnn
012 211
Chứng minh rằng nếu 20192 n thì 4039 1 2 na
Tiếp cận bài toán: Từ sự xác định dãy ta suy được số hạng tổng quát của dãy là
lũy thừa của các số vô tỷ liên hiệp 23;23xy=+=− . Do đó xây dựng dãy
nn n bxy =+ sẽ là các số nguyên nên có điều kiện xét ước số chứa lũy thừa của 2.
Bài giải:
Ta có phương trình đặc trưng của dãy:
- Nếu k chẵn thì 2 kl = (l nguyên dương) khi đó ()().23..kkllll llxyxyxyTb −=+−=
Do đó: () 22017 2
2017 222222 1 6..b...2.3.4.2. k k nll kaTbbB == . Chứng tỏ 4039 1 2 na
Vậy nếu 20192 n thì 4039 1 2 na .
Lời bàn: đâylà bài toán khó,chứa đầy kỹ thuật biến đổi.
Bài toán 7. Cho dãy số () n u được xác định
32
= ⇔ =±
325510xxx−+−= ()( ) 2 1410xxx ⇔−−+= 1 23 x x
Ta tìm được số hạng tổng quát: ( ) ( ) 1111 23231 22 nn na ++ =++−−
Đặt 23;23xy=+=− , suy ra 1;4xyxy=+= , khi đó 11 1 2 nn n xy a ++ + =−
112;32993,2 nn uuunnnn ==+−+−∀≥ . Chứng minh rằng với mỗi số nguyên tố p thì p luôn là ước số của tổng 1 1 2021 p iu = .
Tiếp cận bài toán: Dãy này có ẩn dạng cấp số nhân nên dễ tìm được số hạng tổng quát. Từ đó việc xét ước nguyên tố trở nên thuần số học!
Bài giải:
Ta có: () 1 3 nn uufn =+ , do đó cần biểu diễn ()()() 31fngngn=−−
Dễ thấy: ()()() 3 323 299331 fnnnnnn =−+−=−−−−
Từ giả thiết nếu 20192 n suy ra 2019 2; nkk=∈ ℤ 231
Nên () ( ) () 3 3 3 3 1 1 3313(1)nn nn uunnunun =−−−−⇔+=+−
Đặt ( ) nn vufn =+ , khi đó ta có: 1 3 nn vv = 1 1 3n n vv⇔= và 1 3 v = nên 3n n v =
Do đó số hạng tổng quát của dãy ( ) n u là 3 3n n un =−
a) Ta có b1=1; b2=1; b3 = 2; b4=3
Dễ thấy bn = Fn với n=1; 2; 3; 4. Bằng quy nạp ta chứng minh dãy ( ) n b trùng với dãy ( ) n F . Thật vậy:
Mệnh đề đúng với n=1; 2; 3; 4.
Như vậy: () 11 3 11 202120213 pp i i ii Sui ==
==−
=−=+++−+++−
Khi 2 p = , ta có: ( ) 1 20212021.2Su== suy ra 2|hay|SpS
Khi 2 p > , p nguyên tố lẻ. Ta có:
()() () 1 2122 311 33...33133...33.33 22 p ppp +++=++++==−
mà 33 p chẵn và theo định lý Fecmart nhỏ: ( ) 330mod p p −≡
Và ( )3 3322 33 ipippipi +−=−+ suy ra ( )3 3 |;1;1 pipiip +−∀=− nên
() ( ) 3 33 12...1pp+++−
Do đó 1 1 |hay 2021 p ipSpu
Lời bàn: Từ số hạng tổng quát suy racác tổng quen thuộc, việc xét ước khá quen thuộc!
Bài toán 8. Cho dãy (an) với n > 0 được xác định bởi:
Giả sử mệnh đề đúng đến n+3. Khi đó ta có: ( ) ( ) ( ) ( ) 4321423221 nnnnn nbnFnFnFnF ++++ +=+++−+−
Dùng công thức của dãy Fibonaci : Fm+2 = Fm+1 + Fm ta dễ dàng biến đổi vế phải thành (n+4)Fn+4 suy ra bn+4 = Fn+4
Vậy mệnh đề đúng với n+4, do đó nó đúng với mọi n nguyên dương.
Điều đó chứng tỏ an luôn chia hết cho n với mọi n nguyên dương.
b) Gọi rn là số dư của bn cho 2020 với n=1; 2; 3;....
Trước tiên ta chứng minh (rn) là một dãy tuần hoàn. Thật vậy:
Ta có ( ) 2121 mod2020 nnnnnn bbbrrr ++++ =+ =+
Vì có vô hạn các cặp (r1; r2), (r2; r3),..., (rn; rn+1) nhưng chỉ nhận hữu hạn giá trị khác nhau nên tồn tại ít nhất hai phần tử của dãy trùng nhau. Ta giả sử là (rm; rm+1) =( rm+T; rm+T+1) (với T là một số nguyên dương).
Ta chứng minh (rn) tuần hoàn với chu kỳ T
Ta có: ( ) ( ) () 21 21 2222
rrrrrr rrrr +++++++ ++++++
≡+≡+ ≡ =
mod2020; mod2020 mod2020 mmmmTmTmT mmTmmT
Tiếp tục như vậy ta chứng minh được: rm+k =rm+T+k với mọi 0 k ≥ (1)
==== =+−−∀≥
1; 2; 6; 12; 22 1. nnnnn
1234
aaaa aaaaan ++++
4321
a) Chứng minh an chia hết cho n với mọi giá trị nguyên dương của n.
b) Đặt n n a b n = . Chứng minh tồn tại vô số số nguyên dương n để 2020 là một ước của bn.
Tiếp cận bài toán: Yêu cầu câu a tương đương với chứng minh dãy ( ) n b là dãy số nguyên. Khi tính các số hạng đầu của dãy ( ) n b dễ nhận ra đây là dãy Fibanoxi.Con
đường quy nạp là khả dĩ nhất!
Ta có: ( ) ( )
mod2020; mod2020 mod2020 mmmmTmTmT mmTmmT
() 1111 1111
rrrrrr rrrr −++−+++ −+−−+−
≡−≡− ≡ =
Bằng quy nạp ta chứng minh được: rm-k =rm+T-k với k=1; 2;...; m-1 (2)
Từ (1) và (2) suy ra (rn)n>0 là một dãy tuần hoàn.
Bổ sung vào dãy (bn) phần tử b0 = 0 thỏa mãn b0 +b1 =b2 suy ra r0 =0. Khi đó dãy (rn) là dãy tuần hoàn bắt đầu từ phần tử đầu tiên r0 =0. Do đó tồn tại vô số phần tử trong dãy (rn) bằng 0. Như vậy câu b được chứng minh xong.
Bài giải: 233
Lời bàn: Chỉ việc nhậnra đây là dãy Fibanoxi quen thuộc,bài toán trở nên dễ
dàng hơn!
Do p là ước của n b , n b là ước của n B nên ()2 1 1 5 4 nnn pBpnBa +− .
Vậy tồn tại số nguyên m thỏa mãn 2 5 pm (đpcm).
=+++−++++∀≥
=−=−
aa aaaanannann +
5,6 (1)(21)...((1)1)(()21),2 nn nn
luôn tồn tại số nguyên m sao cho 2 5(mod) mp ≡ .
Tiếp cận bài toán: Công thức truy hồi của 1 n a + khá phức tạp, việc đầu tiên phải tìm quy luật của nó. Phải tạo dãy mới từ quy luật đó, ngay trong đề đã gợi ý quy luật
1 n na + . Việc còn lại là xét sự chia hết trong dãy mới đó.
Bài giải:
Lời bàn: Khi đặt 1 nn bna=+ và 1
= ∏ dãy mới xuất hiện, việc xét ước
nguyên tố p để xuất hiện dạng 2 5 m là bội của p là thuật biến đổi khéo léo!
Ta biết dãy số và đa thức có mối liên hệ với nhau, nên tính chia hết và ước số của các số trong dãy đa thức hệ số nguyên được kế thừa.
Bài toán 10. (HV-Phú Thọ-2019) Cho dãy các đa thức ( ) n Px với hệ số thực được xác định bởi: ( )
=−
4 111,.nnn
3 0 1
()()()
Chứng minh rằng ( ) 2020 Px chia hết cho 2020 x
Đặt 1 nn bna=+ và 1
n ni i Bb =
= ∏
Dễ thấy: 1 2(1) n BBn∀> . Vì vậy, 2,1 n an∀> .
Ta có: 11 1 11
Tiếp cận bài toán: Từ công thức truy hồi có thẻ đânh giá được tính chẵn lẻ của đa thức ( ) n Px . Qua công thức truy hồi cũng tìm được sự chia hết trong dãy, từ đó sẽ có hướng đi tiếp. Bài giải:
aaBnbnanBnBn nananBnB +− +−
=+++=+++∀≥
++=+++++
((1))(1),2 (1)1(1)((1))1 nnnnnnn nnnn
22 1 1 (1)1()(1) nnnn bnannBnB + =++++++
222 1 1 (1).().(1) nnnnnnnn BbnaBBnnBBnB +
=++++−+
Với 1, n ≥ theo công thức truy hồi ta có n P là hàm chẵn.
Mặt khác: (
) (
=+−−++−
) ( ) ()()()() ()()()()()() 211 2 111 21211 2222 nnn nnnn nnnnnn PxPxPx PxPxPxPx PxPxPxPxPxPx +++=+−− =+−−−−−
Vì vậy: () () ()
−++=−+++
BnBaBnBanB na BnBBBnBa
+++ +−−
=−+++=−+
nnnnnnn n nnnnnn
11 (1)2(1) 44 1 (1)224
Thật vậy, nếu ( ) n Px chia hết cho ( )2 x thì thay x bởi 2 x + ta có ( )2 n Px + chia hết cho ,x suy ra ( ) 2 n Px chia hết cho x . Suy ra ( ) ( ) 22 nn PxPx ++− chia hết cho .x
Suy ra: () () ()
−+=−+=−∀≥
2 2 1212
( ) 2 2. k xx +
=+−∀≥
2 1
11 25,2 44 1 5,2 4
( ) 3 0 4 Pxxx =− nên 0P là hàm lẻ và ( ) ( )( ) 0 22Pxxxx=−+ chia hết cho ( )2 xx
( ) ( ) ( ) ( ) ( ) ( ) ( ) 2 2000000 2222 PxPxPxPxPxPxPx =+−+++−
( ) ( )( ) 0 22Pxxxx=−+ chia hết cho ( )2 xx nên ( ) 2 0 Px chia hết cho ( ) 2 2 xx và ( ) ( ) 0022 PxPx ++− chia hết cho x (theo chứng minh trên).
Từ đó ta có ( ) 2 Px chia hết cho ( ) 2 2. xx
Do đó ( ) 2020 Px chia hết cho 2020 x
II.4.2. Bài tập rèn luyện
Bài 1. (Thi Học sinh nữ Châu Âu EGMO-2020) Cho m là số nguyên, một dãy các số 123;;;...aaa thỏa mãn 12 1 aa== ; 3 4;4an=∀≥ , ( ) 123nnnn amaaa =+− . Hãy xác định số nguyên m sao cho mọi số hạng của dãy đều là số chính phương.
Bài 2. (Thi học sinh nữ Châu Âu EGMO-2020) Cho dãy các số nguyên 01233030 ;;;;....; aaaaa thỏa mãn 21 24 nnn aaa ++=+ với 0;1;2;...;3028 n = . Chứng minh rằng có ít nhất một trong các số 01233030 ;;;;....; aaaaa chia hết cho 20202 .
Bài 3. Cho dãy các số nguyên dương ( ) n a ( 1;2;... n = ) thỏa mãn điều kiện: * 1 02001; nn aan + <−≤∀∈ ℕ . Chứng minh rằng tồn tại vô số cặp số ( ) ; pq nguyên dương phân biệt mà | pq aa
Bài 4. Cho dãy các số nguyên ( ) n a ( 1;2;... n = ) được xác định bởi
01212;1;; nnn aaaaan ++ ===+∀∈ ℕ
Chứng minh rằng: nếu p là một ước nguyên tố của 2 2 k a thì p cũng là ước nguyên t ố của 21 1 k a + .
Bài 5.( JBMO TST - April camp-2021).
Xét dãy 12;;...aa biết 1 9 a = và ( ) 1 522 3 n n na a n + ++ = + với 1 n ≥ . Tìm tất cả các số nguyên dương n sao cho na là số chính phương.
Bài 6. (Gặp gỡ đồng hành cùng toán học 2014- đề thử sức) Cho dãy số ( ) n a được xác định bởi công thức 01210;1;2014; nnn aaaaan ++ ===−∀∈ ℕ . Chứng minh với mọi số 2 n ≥ thì n a là hợp số
Bài 7. (Trường Đông toán học Bắc Trung Bộ-2018) Cho cặp số nguyên dương ( ) ; ab với 1 ab>> . Xét dãy ( ) n x được xác định bởi công thức: 1 ;1;2;... 1 n n n
a xn b ==
Trong dãy ( ) n x tồn tại nhiều nhất bao nhiêu số hạng liên tiếp đều là số nguyên tố?
Bài 8. (Trường hè Toán học 2015)
a/ Tìm tất cả các số nguyên dương lẻ t thỏa mãn tính chất: với mọi số nguyên dương k tồn tại số nguyên dương ka sao cho 2 k at + chia hết cho 2k
b/ Chứng minh rằng tồn tại dãy số nguyên dương ( ) ka sao cho 2 7 ka + chia hết cho 2k và 2 1 1 7 2 k k a + + + chia hết cho 2 7 2 k k a + với mọi 1;2;3;... k =
Bài 9. Cho dãy ( ) n a được xác định như sau:
12115;11;23;2;3;... nnn aaaaan +− ===−=
Chứng minh rằng 2022a chia hết cho 11.
* Tổng quan: Nhìn chung về những dãy số nguyên, các số hạng có những tính chất đặc biệt mà ta có thể giải quyết trên cơ sở chia hết, ước số, bội số. Ta nhận được những lời giải hay và đẹp! Đồng hành cùng dãy số nguyên là các đa thức có hệ số nguyên cũng có nhiều vấn đề liên quan đến ước số, vấn đề này được nêu trong phần sau.
II.5. ƯỚC SỐ TRONG ĐA THỨC
Đa thức ( )Px với hệ số nguyên khi ta thay xa=∈ℤ thì giá trị ( )Pa là số nguyên. Khi đó các số nguyên a và ( )Pa có những tính chất số học liên quan đến nó.
Dĩ nhiên sự kết hợp giữa tính chất của đa thức với tính chất số học sẽ hấp dẫn và thú vị tạo nên những bài toán đa dạng và phong phú. Bài viết này không khai thác mối quan hệ giữa đa thức và số học vì đã có nhiều sách và chuyên đề viết về nó. Bài này chỉ viết về những bài toán đa thức hệ số nguyên dưới góc nhìn ước số của nó.
II.5.1.Bổ sung kiến thức lý thuyết của đa thức liên quan đến số học
Bài viết này không nêu lại lý thuyết về Đa thức (các bạn tham khảo lại trên các chuyên đề về đa thức) mà chỉ nêu các tính chất có liên quan đến số học vận dụng trong giải toán đa thức.
Tính chất: Cho đa thức hệ số nguyên ( )Px , với mọi cặp số nguyên a;b ta luôn có ( ) ( ) ( ) ( ) ( ) ( ) ( ) hay| PaPbababPaPb ⋮
Hệ quả: Cho đa thức hệ số nguyên P và các số nguyên a, b, n. Nếu ( ) mod abn ≡ thì
( ) ( )( ) mod PaPbn ≡ Định lý Schur: Cho P là đa thức hệ số nguyên, khác đa thức hằng, luôn có vô hạn số nguyên tố p thỏa mãn: ứng với mỗi p tồn tại số nguyên n sao cho ( ) | pPn .
Chứng minh:
- Nếu ( ) 00 P = thì ( ) | pPp với mọi số nguyên tố p, điều kiện bài toán được
thỏa mãn.
Nếu ( ) 01 P = , giả sử chỉ có hữu hạn số nguyên tố thỏa mãn bài toán là 12 ... kppp <<< . Vì P khác đa thức hằng nên ta chọn được số nguyên dương n đủ lớn để knp > và ( ) |!|2 Pn ≥ . Ta có ( ) ( )!|!0nPnP nên
( ) ( )!1mod! Pnn ≡ . Gọi q là một ước nguyên tố của (!)Pn , nếu kqp ≤ thì
( ) !1! Pnnq ⋮⋮ , suy ra 1 q⋮ , vô lý, do đó kqp > , nhưng điều này lại mâu thuẫn
với điều giả sử. Do đó, có vô hạn số nguyên tố thỏa điều kiện bài toán.
- Nếu ( ) 00Pa=≠ , đặt () ( )Pax Qx a = . Khi đó, Q là một đa thức hệ số nguyên
với ( ) 01 Q = . Theo chứng minh trên, tồn tại vô hạn số nguyên tố p mà ứng với
II.5.2. Các bài toán về đa thức qua góc nhìn ước số
Bài toán 1. Cho các đa thức ( ) ( ) [] ; x PxQx ∈ ℤ và a ∈ℤ thỏa mãn ( ) ( ) 20190 PaPa=+= , ( ) 20182020 Q = . Chứng minh rằng phương trình
( ) ( ) 2021 QPx = không có nghiệm nguyên.
Tiếp cận bài toán: Từ giả thiết ta thấy tính chẵn lẻ của các các nghiệm và các giá trị của nó. Nên tư tưởng bài toán xét trên tính chẵn lẻ (bội của 2).
Bài giải:
Từ giả thiết ta có và2019xaxa==+ là các nghiệm của ( )Px và 2018 x = là nghiệm của ( ) 2018 Qx nên ta có:
+ ( ) ( )( ) ( ) 2019 Pxxaxagx =−−− ,suy ra ( )Px chẵn với x ∀∈ ℤ
+ ( ) ( ) ( ) 20182020Qxxhx=−+ ( ) ( ) ( ) ( ) ( ) 20182020QPxPxhx=−+ chia hết cho 2
Nên ( ) ( ) 2021; QPxx≠∀∈ ℤ . Chứng tỏ ( ) ( ) 2021 QPx = không có nghiệm nguyên.
Lời bàn: Chỉ việc xét tính chẵn lẻ ta có lời giải đẹp!
Bài toán sau đây cũng chỉ xét tính chẵn lẻ ta có giải hay!
Bài toán 2. Chứng minh rằng với mọi số nguyên a, phương trình: ( ) 4322021202020190 xxaxxa −++−+= không thể có hai nghiệm nguyên phân biệt.
Tiếp cận bài toán: Các hệ số chứa các số nguyên liên tiếp nên từ ( )1 f và nghiệm của đa thức sẽ xét đợc tính chẵn lẻ của nó.
Bài giải:
Đặt ( ) ( ) 432202120202019 fxxxaxxa =−++−+
Gọi α là nghiệm nguyên của ( )fx , ta có ( ) 0 f α =
Vì ( ) 122019fa=− là số lẻ nên ( ) ( ) 122019ffa α −=− là số lẻ
Do ( ) ( ) ( ) 11ff αα ⋮ nên 1 α là số lẻ, suy ra α là số chẵn
Giả sử αα 12 , là nghiệm nguyên phân biệt của phương trình ( ) 0 fx = thì αα 12 , là các số chẵn.
mỗi p tồn tại số nguyên n sao cho ( ) | pQn . Khi đó ( ) | pPan . (điều phải chứng minh) 239
238
Ta có: ( ) (
32322 12 1122112212 12 0 202120202019 ff a αα αααααααααα αα ==++−+++++−
Đẳng thức trên không xảy ra vì αα12 , là các số chẵn (mâu thuẫn)
Vậy phương trình ( ) 0 fx = không thể có hai nghiệm nguyên phân biệt.
Bài toán 3. Cho ( )fx là một đa thức với hệ số nguyên. Chứng minh rằng nếu các số ( ) ( ) ( )0;1;...;1fffm đều không chia hết cho m (m là số nguyên dương cho
trước 2 m ≥ ) thì phương trình ( ) 0 fx = không có nghiệm nguyên.
Tiếp cận bài toán: Giá trị của đa thức tại n số nguyên liên tiếp liệu có tồn tại số chia hết cho n không?
Bài giải:
Giả sử ( )fx có một nghiệm nguyên ,{0;1;...;} xccm =≠
Suy ra ( ) ( ) ( )fxxcgx =−
( ) ( ) ( )000fcg =−
( ) ( ) ( )111fcg =−
………………..
( ) ( ) ( )111fmmcgm −=−−−
Do 0;1;...;1ccmc là m số nguyên liên tiếp tồn tại một số chia hết cho m (mâu thuẫn với giả thiết)
Vậy nếu các số ( ) ( ) ( )0;1;...;1fffm đều không chia hết cho m (m là số nguyên dương cho trước 2 m ≥ ) thì phương trình ( ) 0 fx = không có nghiệm nguyên.
Lời bàn: Chỉ sử dụng định lý Bezout xét cho ta đã chỉ ra tồn tại một số chia hết cho n trong số liên tiếp!
Bài toán 4. Cho a,b,c là ba số nguyên phân biệt và đa thức ( ) [] x Px ∈ ℤ sao cho ( ) ( ) ( ) 2020 PaPbPc=== Chứng minh rằng phương trình ( ) 20210 Px −= không có nghiệm nguyên.
Tiếp cận bài toán: Dễ nhận ra a;b;c là nghiệm của phương trình ( ) 20200 Px −= .
Nếu phương trình ( ) 20210 Px −= có nghiệm ta nên xét theo hướng ước số! Bài giải:
Từ giả thiết ta có ( ) ( )( )( ) ( ) 2020 PxxaxbxcQx =−−−+ trong đó ( )
Giả sử phương trình ( ) 20210 Px −= có nghiệm nguyên là d
Khi đó ( )( )( ) ( ) 1 dadbdcQd −−−= . Do đó ;;{1;1}dadbdc−−−∈− vô lý do ;; dadbdc phân biệt. Từ đó ta có đpcm.
Lời bàn: Chỉ ra số 1 không thể là tích của 3 số nguyên phân biệt, ta có lời giải đẹp! Sau đây hai bài có cùng hướng giải quyết như trên.
Bài toán 5. Cho ( )fx là một đa thức bậc 5 với hệ số nguyên, nhận giá trị 2020 với 4 giá trị nguyên khác nhau của biến x. Chứng minh rằng phương trình ( ) 2037 fx = không thể có nghiệm nguyên.
Bài giải:
Giả sử ( ) 2020 fx = tại bốn giá trị phân biệt là 1234 ;;; xxxx và phương trình ( ) 2037 fx = có nghiệm nguyên a. Khi đó ta có: ( ) ( )( )( )( ) ( ) ( ) 1234[] 2020; x fxxxxxxxxxHxHx =−−−−+∈ ℤ
và ( ) ( )( )( )( ) ( ) 1234 2037 17 faaxaxaxaxHa =⇔−−−−= .
Do 17 là số nguyên tố nên không thể phân tích thành tích của 4 số nguyên phân biệt điều này mâu thuẫn suy ra đpcm.
Bài toán 6. Cho [] x f ∈ ℤ .
a/ Chứng minh rằng nếu ()1fx = có quá 3 nghiệm nguyên thì phương trình
()1fx =− không có nghiệm nguyên.
b/ Giả sử a,b,c là 3 nghiệm nguyên khác nhau của phương trình ()1fx = . Chứng minh rằng nghiệm nguyên của phương trình ()1fx =− lớn hơn min{;;} abc
Bài giải:
a/ Giả sử phương trình ()1fx = có bốn nghiệm ;;; abcd ∈ℤ khác nhau.
Suy ra ( ) ( )( )( )( ) ( ) 1 fxxaxbxcxdQx =−−−−+ với ( ) [] x Qx ∈ ℤ . Do đó phương trình ()1fx =− ( )( )( )( ) ( ) 2 xaxbxcxdQx ⇔−−−−=− , chứng tỏ -2 có 4 ước số phân biệt.
Do -2 không thể phân tích thành tích của bốn số nguyên phân biệt nên phương trình ()1fx =− không thể có nghiệm nguyên.
b/ Giả sử phương trình ()1fx = có 3 nghiệm nguyên a,b,c và phương trình
()1fx =− có nghiệm nguyên d. Theo câu a ta suy ra phương trình ()1fx = có đúng 3 nghiệm.
Khi đó ( ) ( )( )( ) ( ) 1 fxxaxbxcQx =−−−+ với ( ) 0; Qxx≠∀∈ ℤ .Và
( )( )( ) ( ) ( )( )( ) ( ) 112 dadbdcQdadbdcdQd −=−−−+⇔−−−=
suy ra ba số ;; adbdcd phân biệt và đều là ước của 2 nên tồn tại một số âm, giả sử 0min{;;} addadabc −<⇔>⇔> (đpcm)
Bài toán 7. Cho P(x) và Q(x) là hai đa thức với hệ số nguyên. Biết rằng đa thức ( ) ( ) 33xPxQx + chia hết cho 2 1 xx++ . Gọi d là ƯCLN của ( ) ( )2021và2021PQ
.Chứng minh rằng 2020 d ≥
Tiếp cận bài toán: Ta có 2 1 xx++ là ước của 3 1 x , trong đề có yếu tố lũy thừa bậc ba. Nên chăng dùng tính chất ( ) ( ) ( ) | abPaPb , chỉ còn việc thêm bớt biến đổi đưa về đơn giản hơn.
Bài giải:
Ta có: ( ) ( ) ( ) () ( ) ( ) () ( ) ()() 33331111 xPxQxxPxPQxQxPQ +=−+−++
mà ( ) ()
3 (1) xPxP và ( ) () 3 1 QxQ cùng chia hết cho 3 1 x nên chia hết cho
2 1 xx++ .
Do đó ( ) ( )11xPQ + chia hết cho 2 1 xx++ . Vì deg ( ) ( )11xPQ + =1 ≤deg(
2 1 xx++ )=2 nên ( ) ( ) (11)0 xPQ+≡ ( ) ( ) 110PQ == . Theo định lý Bezout ta có ( ) ( ) ( )
1 1 PxxPx QxxQx =− =− , với ( ) ( ) 11 ;Q Pxx là các đa thức với hệ số nguyên.
()()()
1 1
Do đó ( ) ( )2021và2021PQ cùng chia hết cho 2020. Vì ( ) ( ) (2021;2021)dPQ = suy ra 2020 d ≥ .
Lời bàn: Từ nhận xét đa thức bậc nhỏ hơn chia hết cho đa thức bậc lớn hơn chỉ xảy ra khi đa thức đồng nhất 0. Nhờ định lý Bezout ta có đa thức 1 ()(1)() PxxPx =− thuận lợi cho việc chứng minh
Bài toán 8. Cho đa thức 1012 () fxxaxbxc =+++ với ,, abc ∈ℤ có ba nghiệm nguyên 123 ,, xxx . Chứng minh rằng 2 10111 101 1233 0 1 (1)()()() abcxxxxxx +++−−− chia hết cho 101.
242
101|
Tiếp cận bài toán: Nhận xét đầu tiên 101 là số nguyên tố nên có (
có ( ) ( )0mod101 igx ≡ . Lúc đó sẽ có hướng đi tiếp!
Bài giải:
- Ta có ( ) 2 101 ()1 fxxaxbxc x ++ =++ . Đặt 2 ()(1) gxaxbxc =+++ .
Do 101 là số nguyên tố nên theo định lý Fecma nhỏ ta có ( ) 101 mod101 xx ≡
Từ giả thiết suy ra ()0 igx ≡ (mod 101).
- Nếu 122331 ()()() xxxxxx chia hết cho 101 thì bài toán chứng minh xong.
- Nếu 122331 ()()() xxxxxx không chia hết cho 101.
Ta có 12 ()()101gxgx ⋮ 1212 ()(()1)101 xxaxxb −+++ ⋮ 12 ()1101axxb +++ ⋮ (1)
Tương tự 23 ()1101axxb +++ ⋮ (2)
Từ (1) và (2) suy ra 13 ()1011011101 axxab + ⋮⋮⋮ . Mà 1 ()101101gxc ⋮⋮ .
Do đó 1101 abc+++ ⋮ mà ( ) 1 101101 01 1(1)mod101abcabc +++≡+++
nên 101101101 1101 abc+++ ⋮ .
Lời bàn: Nhờ định lý Fermat và sử dụng tính chất ( ) ( ) ( ) | abPaPb cho ta lời giải đẹp!
Bài toán 9. (Ukraina MO 2016) Cho P, Q là hai đa thức hệ số nguyên không âm, khác đa thức hằng. Xét dãy số () 2016(), 1 Pn n xQnn =+≥ . Chứng minh rằng tồn tại vô hạn số nguyên tố p thỏa mãn: ứng với mỗi p, tồn tại số nguyên dương m sao cho mpx .
Tiếp cận bài toán: Yêu cầu bài toán làm ta liên tưởng đến định lý Schur, từ dãy n x ta phải làm xuất hiện đa thức để sử dụng định lý Schur, phần dư còn lại chắc phải dùng đến Fermat!
Bài giải:
Ta có ()()(1)(1)()(1) 2016()20162016()201620162016() PnPnPPPnP n xQnQnRn =+=−++=−+ trong đó ( ) ( ) (1) 2016P RnQn=+ là một đa thức hệ số nguyên, khác đa thức hằng .
Theo định lý Schur, tồn tại vô hạn số nguyên tố p, 2016 p > thỏa mãn: ứng với mỗi p, tồn tại số nguyên dương n sao cho ()pRn
Vì ( ) ,11pp −= nên theo định lý thặng dư Trung Hoa, tồn tại số nguyên m sao cho (mod); 1(mod1)mnpmp ≡≡− .
Vì ()pRn và (mod)mnp ≡ nên ()pRm .
Hơn nữa, 1(mod1)mp≡− nên ()(1)(mod(1)) PmPp≡− , do đó ()(1)20162016 PmP p (định lý Fermat nhỏ)
Suy ra mpx . Ta có điều phải chứng minh.
Lời bàn: Xuất hiện da thức R(n) để dùng định lý Schur, phần còn lại có bóng dáng của định lý Fermat nhưng phải bổ sung cho hoàn thiện điều kiện nguyên tố cùng nhau!
Mạch kiến thức khá rõ ràng, lời giải khá tự nhiên !
Bài toán 10. (Mathematical Reflections 5- 2017) Tìm tất cả các cặp số nguyên dương phân biệt ( ) , ab sao cho tồn tại đa thức ( )Px với hệ số nguyên thỏa mãn :
( ) ( ) ( ) ( ) 3232 77. PaabPbba ++=++
Tiếp cận bài toán: Rõ ràng áp dụng tính chất ( ) ( ) ( ) | abPaPb , phần còn lại cũng có thừa số ab ,nên sẽ có hướng giải quyết tiếp.
Bài giải: Từ giả thiết ( ) ( ) ( ) ( ) 3232 77 PaabPbba ++=++ suy ra ( ) ( ) ( ) ( )( ) 3322771 PaPbbaababab −=+−−=−+−
Mà ta có ( ) ( ) 3333 abPaPb nên
( )( ) 33 71, abababab −−+−≠
( ) 22 71aabbab +++−
Không mất tính tổng quát ta giả sử ab > . Ta thấy ( ) ( )( ) ( )( ) 222 7111111716 abaabbababbabaaa +−≥++=+−++−+>+−+ >+⇔>
Vì vậy 6 ba<< . Thử trực tiếp ta được 1,2ba== và 3,5ba==
• Nếu 1,2ba== và 1,2ab== , ta được ( ) 2.Pxx =
• Nếu 3,5ba== và 3,5ab== , ta được ( ) Pxx =
Vậy các cặp ( ) , ab thỏa mãn là ( ) ( ) ( ) ( ) 2,1,1,2,5,3,3,5.
Lời bàn: Chỉ sư dụng tính chất chia hết của hiệu ( ) ( )PaPb , và ước số phải bé hơn số đó ta đã thu hẹp phạm vi của các biến a;b. Lời giải khá tự nhiên và đẹp!
II.5.3. Xây dựng đa thức hệ số nguyên để chứng minh chia hết
Bài toán 1. (IMO Shortlist 2005) Cho ,,,,, abcdef là các số nguyên dương . Giả sử
rằng Sabcdef =+++++ là ước của các số abcdef + và abbccadeeffd ++−−−
Chứng minh rằng S là hợp số
Tiếp cận bài toán: Từ giả thiết thấy có tín hiệu của định lý Viet, chú ý tổng abcdef + và abbccadeeffd ++−−− xây dựng hiệu hai đa thức bậc ba có chứa các hệ số là các số đã cho.
Bài giải:
Xét đa thức ( ) ( )( )( ) ( )( )( ) 2 PxxaxbxcxdxexfSxQxR =+++−−−−=++ .
Trong đó ;; SabcdefQabbccadeeffdRabcdef =+++++=++−−−=+
Vì theo giả thiết ; SQSR nên ( ), SPxx∀∈ Z
Nói riêng ( ) ( )( )( )SPddadbdc =+++
Nếu S là số nguyên tố thì S là ước của một trong 3 số ,, dadbdc +++ , nhưng điều đó là vô lý, vì { } ,, Smaxdadbdc >+++ . Do đó S là hợp số (đpcm)
Lời bàn: Việc xây dựng được đa thức P(x) là linh hồn của lời giải!
Bài toán 2. (THTT 2000) Cho số tự nhiên lẻ p và các số nguyên ,,,, abcde thỏa mãn các điều kiện abcde ++++ và tổng 22222 abcde ++++ đều chia hết cho p . Chứng minh rằng 55555 5 abcdeabcde ++++− cũng chia hết cho p .
Tiếp cận bài toán: Như bài trên, tín hiệu của định lý Viet và định lý Bezout khá rõ, ta cần xây dựng đa thức bậc 5 có hệ số thỏa các số đã cho, việc còn lại là của “số học”!
Bài giải:
Xét đa thức ( ) ( )( )( )( )( )fxxaxbxcxdxe =−−−−− .
Ta đặt ( ) ( ) 5432 ,1 fxxAxBxCxDxabcde =−+−+− . Với ,e AabcdeBabacadabcbdbecdcede =++++=+++++++++ , , CD∈∈ZZ
Ta có p là ước của () ( ) 2 22222 2Babcdeabcde =++++−++++ , mà p lẻ nên | pB
Trong ( )1 , lần lượt thay x bởi ,,,, abcde rồi cộng các kết quả với nhau, để ý rằng
( ) ( ) ( ) ( ) ( ) 0 fafbfcfdfe ===== , ta được :
555554444433333 5 abcdeabcdeAabcdeBabcde ++++−=++++−+++++
244
( ) ( ) 22222 CabcdeDabcde ++++−++++ chia hết cho p
( do p đều là ước của 22222 ,,, ABabcdeabcde ++++++++ ).
Suy ra vế trái cũng chia hết cho p (đpcm)
Lời bàn: Kết hợp giữa đại số và số học khá nhẹ nhàng, cho lời giải đẹp!
Bài toán 3. Cho số nguyên dương 2 m > . Chứng minh rằng với mọi số tự nhiên 3 n ≥
thì số 21 1 1
n n m m m , luôn có một ước số có dạng 1 a m + với a ∈ N
* Các bài toán đã nêu trên đã dùng ý tưởng đa thức để giải, còn sau đây là các
bài toán yêu cầu xây dựng hoặc tìm đa thức thỏa điều kiện cho trước.
Bài toán 4. (Thailand MO 2014) Tìm tất cả các đa thức P với hệ số nguyên thỏa mãn *
()2557213.2014, n Pnn+∀∈ℕ .
Tiếp cận bài toán: Ta luôn có 1 là ước của mọi số nguyên nên dễ có nghiệm trong trường hợp này.
Tiếp cận bài toán: Ta thấy số 1 m là ước của 21 1 n m nên 21 1 1
n n m m m luôn là số nguyên, mà có ước dạng 1 a m + với a là số tự nhiên hoặc là biểu thức chứa số tự nhiên.Như vậy ta phải xây dựng (thêm, bớt) để xuất hiện các bội của số dạng 1 a m +
Bài giải:
Ta luôn có 21 1 1
n m m là số nguyên nên 21 1 1
n n m m m là số nguyên.
Ta nên chọn số nguyên tố p là ước của ( )Pn khi đó p cũng là ước của vế phải. Ta dùng tính chất ( ) ( ) ( ) | abPaPb để thu gọn biểu thức có ước là p. Khi đó sẽ có hướng giải tiếp! Bài giải:
Dễ thấy * ()1, Pnn=∀∈ ℕ và * ()1, Pnn=−∀∈ ℕ là những đa thức thỏa mãn điều kiện bài toán.
Xét trường hợp ( ) 1 Pn ≠±
Giả sử P là đa thức thỏa mãn bài toán và * 00:()2nPn ∃∈≥ ℕ .
nn nn n mmmm m mm
−−+ −−+− −=
Ta có 2121111 11
Đặt 12r ns += ( , rs∈ℕ, s lẻ và rn < )
Xét đa thức: () ) ( ) ( ) 22 211 2 111 rnr n r s nnns Pmmmmmmm −+ =−+−=+−+ .
Do s lẻ nên 2nr s lẻ , ta suy ra : () 2 (1)| mPm +
Khi đó () ( ) ()() [] ( ) 2 1, r PmmQmQmx =+∈Z (1)
Vì ( ) ( ) 1010PQ = = nên ( ) ( ) ( ) 1 QmmHm =− với ( ) [ ]Hmx ∈ Z (2)
Gọi p là ước nguyên tố của 0 ()Pn .
Ta có: 0 0 ()2557213.2014 n Pn + và 0 0 ()2557213.2014 np Pnp + ++ .
Do đó, 0 00 ()()2557(25571) n p pPnpPn+−− .
Mặt khác, vì 0 0 ()2557213.2014 n pPn + nên { }2,3,19,53,71,2557 p ∉
Do đó, (25571) p p . Hơn nữa, theo định lý Fermat nhỏ, (25572557) p p , nên 2556 p . Suy ra { }2,3,71 p ∈ (vô lý).
Vậy chỉ có hai đa thức * ()1, Pnn=∀∈ ℕ và * ()1, Pnn=−∀∈ ℕ thỏa mãn bài toán.
Từ (1) và (2) () () 21 2 1 1 1
n r n m mmHm m −=+
Hay số 21 1 1
n n m m m luôn có một ước số có dạng 1 a m + với a ∈ N . (đpcm)
Lời bàn: Từ 00 |()() pPnpPn +− và với ( ) 0 | pPn ta so sánh ước để có lời giải
Bài toán sau cũng có hướng giải quyết tương tự.
Bài toán 5. (Chọn ĐTQG- Phú Thọ- 2017) Tìm tất cả các đa thức ( )Px với hệ số nguyên khác đa thức không sao cho 1032016 n n chia hết cho ( )Pn với mọi số
nguyên dương .n
Lời bàn: Xây dựng được đa thức ( )Pm là giải quyết được nửa bài toán rồi! Còn lại phải xuất hiện số mũ lẻ là việc biểu diễn ước lẻ lớn nhất của số học, khá nhẹ nhàng. 247
Bài giải:
Trước hết ta chứng minh ()Px đa thức hằng. Thật vậy giả sử ()Px đa thức khác hằng .
Suy ra tồn tại số tự nhiên N đủ lớn sao cho với mọi nN ≥ , ta có () 1 Pn >
Gọi p là ước nguyên tố bất kỳ của ()Pn .
Ta có ()() PnpPnp +− ⋮ . Suy ra ()() (103()2016)(1032016)10(101)3 npnnp PnpPnnpnpp + +−=−+−−−−=−− ⋮
Suy ra () 10101 np p ⋮
Nếu 2;5 p ≠ thì ta có () 101 p p ⋮ , mà theo định lý Fermat’s ta có ()1010mod p p ≡ ()1019mod p p −≡
Từ đó suy ra 3 p =
Do đó 2;3 p = hoặc 5 .
Bài toán 6. Tìm tất cả các đa thức () [ ] Pxx ∈ℤ thỏa mãn ()|2 p Ppp , với mọi số
nguyên tố .p
Tiếp cận bài toán: Từ đề bài đã có bóng dáng của định lý Schur, nên ứng dụng
tính chia hết của đa thức và định lý Schur ta sẽ có hướng đi.
Bài giải:
Dễ thấy nếu ()Px là đa thức hằng thì 1 P =± thỏa mãn điều kiện bài toán.
Xét ()Px không phải là đa thức hằng.
Theo định lí Schur, tồn tại vô hạn số nguyên tố p thỏa mãn: ứng với mỗi p tồn tại số nguyên n sao cho |().pPn
Nếu | pn thì |()() pPnPp nên |()|2 p pPpp , vô lí nên (,)1 pn = .
( ) 1033016,312 n np−−= = 5
Ta thấy rằng hoặc .
Theo định lí Dirichlet về số nguyên tố, ta có thể chọn số nguyên k sao cho qnkp =+ là số nguyên tố. Khi đó, |()() pPqPn nên |()|22(). qnkp pPqqnkp + −=−+
Nếu thì ta có suy ra chẵn .
2 p = 21032016 n n n
Nếu thì ta có suy ra
Chọn được không thỏa mãn cả hai điều kiện trên và , khi đó ta có điều vô lý.Vậy đa thức hằng.
Đặt . Ta có
5 p = 51032016 n n ( )3mod5 n ≡ n ( ) ( ) 1mod10 n ≡ nN ≥ ( )Px ( ) , Pnaaconst == ( ) * 1032016, n nan −−∀∈ ⋮ N
Kết hợp định lí Fermat nhỏ suy ra, |2. nk pn + Đặt (2). p hord = Nếu có các số nguyên 12 , kk thỏa mãn 1 |2nk pn + và 2 |2nk pn + thì 12(mod).kkh ≡ Do đó, ta chỉ có thể chọn k theo một lớp thặng dư nào đó đối với modulo h Nhưng, theo định lí Dirichlet, ta có thể chọn k sao cho số nguyên tố qnkp =+ nguyên tố cùng nhau với h Khi đó, vì |1hp nên (,)1 nkh+= , nghĩa là ta có () h φ cách chọn lớp thặng dư đối với modulo h cho k Vô lí.
1 n = a
Với , suy ra lẻ.
Với , suy ra
5 n = ( ) ,51a =
Vậy chỉ có 1 P =± là những đa thức thỏa mãn bài toán Lời bàn: Với đa thức khác hằng, đã kết hợp tính chia hết và định lý Schur loại trường hợp | pn , còn trường hợp ( ) ;1pn = phải dùng đến những công cụ nặng như cấp và các định lý Euler; Fermat để giải quyết.
Chọn ta có , mà
1 a > q a ( ) 10,1 q = nq = ( )1032016 q qq ⋮ ( ) 1010mod2006 q qq ≡
Nếu thì gọi là một ước nguyên tố bất ký của . Ta có .
II.5.4. Bài tập rèn luyện
Chọn ta có , mà
1 nq=− ( ) ( ) 1 10312016 q qq ⋮ ( ) 1 101mod2012 q qq ≡
Từ hai điều trên suy ra vô lý do lẻ.
( ) 2006,20122 q = a
Vậy . Suy ra , là hai đa thức cần tìm.
Bài 1.(BMO- 2016) Tìm tất cả các đa thức hệ số nguyên thỏa mãn điều kiện: tồn tại số nguyên dương N sao cho p là ước của ( ) ( ) 2!1 fp + với mỗi số nguyên tố pN > mà ( )fp là số nguyên dương.
1 a = ( ) ( ) 1,1PxPx≡≡− 249
Bài 2: (Balkan MO ‒ 2016) Tìm tất cả các đa thức monic f ( hệ số cao nhất bằng 1) thỏa mãn các điều kiện sau: tồn tại số nguyên dương N sao cho p là ước số của 2(f(p)!) + 1 với mỗi số nguyên tố p > N mà f(p) là số nguyên dương.
248
Bài 3: Cho ( )Px là đa thức với hệ số nguyên. Chứng minh rằng không tồn tại ba số a, b, c phân biệt sao cho ( ) ( ) ( ) , , PabPbcPca === .
Bài 4: P(x) là đa thức với hệ số nguyên và P(0), P(1) là các số lẻ. Chứng minh rằng P(x) không có nghiệm nguyên.
Bài 5: Cho P(x) là đa thức với hệ số nguyên sao cho ()()()1 PaPbPc=== với a, b, c là ba số nguyên phân biệt. Chứng minh rằng P(x) không có nghiệm nguyên.
Bài 6: (HSGQG 2014). Cho đa thức ( )2 2 ()7613 n Pxxx=−++ với n là số nguyên dương. Chứng minh rằng P(x) không thể biểu diễn được dưới dạng tích của n + 1 đa thức khác hằng số với hệ số nguyên.
Bài 7: (IMO Shortlist 2005)
Biết rằng các số nguyên dương abc ≥≥ và d thỏa mãn đồng thời các điều kiện sau
i. 3 abcd =
ii. Số abcd ++− là một ước nguyên tố của số 2 abbccad ++−
Chứng minh rằng bd =
Bài 8. (Mathemattica Excalibur 5- 1996)
Tìm tất cả các số nguyên a sao cho 2 xxa −+ là ước của 13 90 xx++
Bài 9.( Mathematical Reflections 2017)
Cho đa thức ( )Px với hệ số nguyên và có nghiệm nguyên. Chứng minh rằng nếu , pq là hai số nguyên tố lẻ phân biệt thỏa mãn ( ) 21Pppq=<− và ( ) 21Pqqp=<− , thì p và q là hai số nguyên tố sinh đôi.
Bài 10. (Thailand MO 2014)
Tìm tất cả các đa thức ( )Px với hệ số là các số nguyên thỏa mãn: ( ) * 2557213.2014, n Pnn+∀∈N
III. HƯỚNG PHÁT TRIỂN ĐỀ TÀI:
Đề tài này còn hai hướng phát triển sau:
1/ Phương trình hàm trên tập ℤ , qua cách nhìn về ước số, bội số.
2/ Dùng ước số, bội số trong các bài toán tổ hợp rời rạc.
Đó là hai vấn đề tôi (hoặc các bạn) sẽ tiếp tục trong bài viết sau.
Thí dụ bài toán sau về Rời rạc trong thời gian gần đây, các bạn tham khảo:
Đề (BMO TST - March camp-2021)
Có 2 n ≥ số nguyên dương được viết trên bảng trắng. Một nước đi gồm ba bước: tính bội số chung nhỏ nhất N của tất cả các số, sau đó chọn một số a và thay a bởi số N a .
Chứng minh rằng sử dụng một số lần hữu hạn ta luôn có thể thực hiện được tất cả các số trên bảng đều bằng 1.
Bài giải. Gọi P là tập hợp tất cả các ước nguyên tố của các số trên bảng. Rõ ràng ta không thể thêm bất kỳ số nguyên tố mới nào vào tập P. Ta chọn số nguyên tố bất kỳ pP ∈ và sắp xếp tất cả các số theo lũy thừa giảm dần của cố mũ của p.
Theo bổ đề LTE ta có ( ) ( ) ( ) 12 ... pppn vavava
Chú ý rằng ( ) ( ) ( ) 112;;...; ppn vavlcmaaa = ở đây 1 1 p N v a =
Do đó sau khi ta chọn liên tiếp các số 12;;...; naaa cuối cùng không còn số nào trên bảng là bội của p. Nên ta luôn có thể giảm || P xuống ít nhất, sau một số lần thực hiện như vậy đến lúc ||0 P = nghĩa là tất cả các số trên bảng đều bằng 1.
Khá hấp dẫn phải không các bạn.!
250
IV. KẾT LUẬN
Bài viết này tôi hướng đến các em học sinh tự học, sau đó làm tài liệu tham khảo cho các bạn đồng nghiệp mới bước vào phân môn Số học. Với mục đích giúp các em có hướng tìm tòi giải quyết bài toán một cách tự nhiên nên mỗi bài tôi đều có gợi ý hướng tiếp cận bài toán. Dĩ nhiên đó không phải là hướng duy nhất mà còn có nhiều cách tiếp cận hay hơn của các bạn. Cũng có những bài toán thuộc dạng khó, ta không thể “thấy” liền mối liên hệ giữa giả thiết và kết luận mà nó “ẩn” quá sâu. Những bài đó đòi hỏi ta phải tập trung cao độ tư duy “quán tưởng” về nó, dĩ nhiên “ánh sáng” cũng sẽ xuất hiện! Khi đó ta có những cảm giác thú vị! Cũng là lúc ta thấy được “vẻ đẹp của số học”. Chính vẻ đẹp “nội tâm” đó mà Số học bước lên ngôi “nữ hoàng” của toán học!
Tôi cũng học cách viết của người xưa là sau mỗi vấn đề, mỗi bài toán đều có vài lời bàn, đó cũng là cách nhận định chủ quan của người viết, bạn có thể không đồng tình thì bỏ qua, chỉ đọc phần nội dung bài giải.
Bài viết là chỉ khai thác cách nhìn từ “bên trong” của bài toán thông qua sử dụng
ước số. Mục đích làm cho người học cảm nhận được vẻ đẹp của Số học từ lối tiếp cận đến tư duy giải toán khá độc đáo và thú vị. Bài viết không giải quyết hết các vấn đề
của môn Số học mà chỉ dừng lại ở một số bài toán phù hợp với nội dung của chủ đề
Nhân đây tôi cũng xin cảm ơn quý bạn đồng nghiệp trong tổ đã cung cấp tài liệu, chỉnh sửa và đóng góp nhiều ý kiến hay cho tôi hoàn thành bài viết này. Chắc chắn rằng còn nhiều sai sót không tránh khỏi trong lúc viết, rất mong nhận được sự góp ý chân tình của các bạn.
Xin gởi nơi đây tấm lòng tri ân!
Trường THPT Chuyên Lê Thánh Tông- Quảng Nam.
Tháng 8 năm 2021.
“Năm Covid thứ hai”
Giáo viên: Nguyễn Văn Thời.
V. TÀI LIỆU THAM KHẢO. [1]. Định hướng bồi dưỡng học sinh năng khiếu Toán Số học. (Lê Anh Vinh chủ biên) [2]. Tuyển tập các đề thi Olympic Toán vòng quanh thế giới (Lê Phúc Lữ chủ biên) [3]. Các phương pháp giải Toán qua các kỳ thi Olympic. (Trần Nam Dũng chủ biên)
[4]. Các chuyên đề Số học bồi dưỡng học sinh giỏi toán trung học -5 tập (của Phan Huy Khải) [5]. Tạp chi Pi. [6]. Tạp chí Toán học và tuổi trẻ. [7]. Các chuyên đề hội thảo của Duyên hải và đồng bằng Bắc bộ và các chuyên đề của các
trường Hè, trường Đông của viên Toán tổ chức. [8]. Số học. ( của Hà Huy Khoái) [9] Chuyên đề Số học. ( Diễn đàn Mathscope) [10]. Problems on number theory. (của Titu Andreescu) [11]. Olympiad problems number theory.
(của Doãn Quang Tiến-Nguyễn Minh Tuấn- Nguyễn Nhất Huy-Huỳnh Kim Linh) [12]. Selected Problems and Theorems in Elementary Mathematics Arithmetic and Algebra
(D.O. Shklyarsky –N.Nchenstov – I.M Yalom)
CHUYÊN ĐỀ
VẬN DỤNG BỔ ĐỀ VỀ MŨ ĐÚNG TRONG CÁC BÀI TOÁN SỐ HỌC
A. PHẦN MỞ ĐẦU
1. Lý do chọn đề tài
Số học có thể nói là một lĩnh vực xuất hiện sớm nhất trong lịch sử toán học. Khi những con số được con người sử dụng thì cũng là lúc Số học ra đời. Ngạn ngữ Pháp có câu: “Toán học là vua của các khoa học nhưng Số học là Nữ hoàng”. Điều đó đã cho thấy tầm quan trọng của Số học trong đời sống và khoa học. Trải qua một quá trình phát triển lâu dài nhưng Số học vẫn giữ được vẻ đẹp sơ khai của nó. Vẻ đẹp ấy được thể hiện qua cách phát biểu đơn giản của một bài toán. Chính vì cách phát biểu đơn giản nhưng cần những suy luận sâu sắc và tinh tế nên những bài toán Số học trong các kì thi học sinh giỏi thường là những thách thức thực sự và thường được dùng để phân loại học sinh.
Đối với phân môn Số học trong trương trình toán THPT chuyên thì các kiến thức về Bổ đề nâng lũy thừa là một kết quả hay và có nhiều ứng dụng. Việc sử dụng thành thạo các tính chất về số mũ đúng cùng sự kết hợp khéo léo với các định lý cơ bản của số học như định lý Euler, định lý Fecmat, kiến thức về cấp của một số…sẽ giúp chúng ta có thể giải quyết được các phương trình Diophante hoặc các bài toán chia hết liên quan đến số mũ. Chính vì vậy nhóm tác giả đã quyết định viết chuyên đề: “Vận dụng bổ đề về mũ đúng trong các bài toán Số học”.
2. Mục đích của đề tài
Trong chuyên đề này, tác giả trình bày một hệ thống lý tuyết về Bổ đề nâng lũy thừa và một số kiến thức hỗ trợ cho phương pháp LTE trong việc giải quyết các bài toán Số học. Các bài toán trong chuyên đề đều được nhóm tác giả tìm kiếm, chọn lọc, tổng hợp, trình bày lời giải một cách chi tiết kèm theo những phân tích, bình luận và đặt nó trong sự kết nối với các bài tập khác. Chuyên đề được viết với mong muốn phục vụ trực tiếp cho công tác giảng dạy của nhóm tác giả đồng thời cũng hy vọng sẽ là tài liệu có ích cho các thầy cô, các em học sinh tham khảo và học tâp.
3. Cấu trúc của đề tài
Chuyên đề bao gồm 3 phần chính như sau:
A. Phần mở đầu.
B. Phần nội dung.
C. Phần kết luận.
I. KIẾN THỨC CƠ BẢN
B. PHẦN NỘI DUNG
1. Định nghĩa. Cho p là số nguyên tố, a là số nguyên và α là số tự nhiên. Ta nói
p
α là ước đúng của a (exact power) và α là số mũ đúng của a nếu | pa α và
1 pa α+ . Khi đó ta viết || pa α và kí hiệu ( ) p va α =
2. Tính chất. Cho ,, abc là các số nguyên, khi đó ta có
o ( ) ( ) ( ) ppp vabvavb =+
o ( ) ( ) . n pp vanva =
o ( ) ( ) ( ) { } min, ppp vabvavb +≥ . Đẳng thức xảy ra khi ( ) ( ) pp vavb ≠
o () 1 ! n p i i
=
n vn p =
3. Hai bổ đề
(Định lý Legendre)
3.1. Bổ đề 1. Cho , xy là hai số nguyên (không nhất thiết phải nguyên dương) và n là số nguyên dương. Cho p là số nguyên tố bất kỳ sao cho
( ) ,|, 1,, y np ppxypx =− . Ta có ( ) ( ) nn pp vxyvxy −=−
Chứng minh.
Ta có ( )( ) 1221 ... nnnnnn xyxyxxyxyy −=−++++
Do | pxy nên ( ) mod xyp ≡ ( ) 12211...0modnnnnn xxyxyynxp ++++≡≡ /
Ta có đpcm.
3.2. Bổ đề 2. Cho , xy là các số nguyên (không nhất thiết phải nguyên dương), n là
số nguyên dương lẻ và p là số nguyên tố bất kỳ thỏa mãn ( ) ,|, 1,, y np ppxypx =+ . Ta có ( ) ( ) nn pp vxyvxy +=+
Chứng minh.
Sử dụng Bổ đề 1 ta có ( ) () ( ) () ( ) () n nnn pppp vxyvxyvxyvxy +=−−=−−=+
4. Lifting The Exponent Lemma (LTE)
4.1. Định lý 1. Cho , xy là hai số nguyên (không nhất thiết phải nguyên dương), n là một số nguyên dương và p là một số nguyên tố lẻ thỏa mãn | pxy và , p xpy .
Ta có ( ) ( ) ( ) nn ppp vxyvxyvn −=−+
Chứng minh.
Ta sẽ chứng minh quy nạp theo ( ) p vn . Trước hết, ta chứng minh khẳng định sau:
( ) ( ) 1 pp pp vxyvxy−=−+ ( )1
Để chứng minh ( )1 , ta cần chỉ ra rằng
1221|...pppp pxxyxyy ++++ ( )2
−=−=−
αααα αααα αα αα αα
=−=− =−+=−+ =−+−=−+ =−+
vxyvxy vxyvn
Định lý được chứng minh.
4.2. Định lý 2. Cho , xy là hai số nguyên, n là một số nguyên dương lẻ và p là một số nguyên tố lẻ thỏa mãn | pxy + và , p xpy . Ta có
Chứng minh.
Áp dụng Định lý 1 và điều kiện n lẻ ta có:
n nnn pp pp pp
( ) () ( ) () () () ()()
+=−− =−−+ =++
vxyvxy vxyvn vxyvn
4.3. Định lý 3. (Cho trường hợp 2 p = ) Cho , xy là hai số nguyên lẻ sao cho
4| xy . Ta có ( ) ( ) ( ) 222 nn vxyvxyvn −=−+
Chứng minh.
Theo Bổ đề 1, nếu ( ) ,|, 1,, y np ppxypx =− thì ( ) ( ) nn pp vxyvxy −=−
Tức là nếu 2 p = , n là số lẻ và 2| xy thì
( ) ( ) 22 nn vxyvxy −=−
Do đó ta chỉ cần chứng minh Định lý trong trường hợp n là lũy thừa của 2 . Tức là: ( ) () 22 22 nn vxyvxyn −=−+
Thật vậy, ta có: ( )( ) ( )()()
1122 22222222 .... nnnnnn xyxyxyxyxyxy −=++++−
Vì ( ) ( ) 22 1mod41mod4 kk xyxy ≡≡± ≡≡ ( ) 22 2mod4 kk xy +≡ ( ) ( ) ( ) 1122 222222 222...1 nnnn vxyvxyvxy +=+==+=
Lại có 4|, xy , xy là số lẻ nên ( )2mod4 xy+≡ ( ) () 22 22 nn vxyvxyn +=−+
4.4. Định lý 4. (Cho trường hợp 2 p = ) Cho , xy là hai số nguyên lẻ và n là một số nguyên dương chẵn. Ta có
( ) ( ) ( ) ( ) 2222 1 nn vxyvxyvxyvn −=−+++−
Chứng minh.
kk kkkk nnmm vxyvxyvxyvxy −=−=−=−
258
( ) ( ) ( )
=−+−=−+−+− =−+−+−
11 1 vxykvxyvxyk vxyvxyvn
()()()
22 222 222
5. Một số nhận xét.
5.1. Sai lầm thường gặp nhất khi sử dụng bổ đề nâng lũy thừa là việc không kiểm tra điều kiện | pxy ±
5.2. Với n là số nguyên dương thì ( ) log pp nvnn ≤< . Thật vậy Đặt npk α = với ( ) ,1pk = , khi đó ( ) p vn α = ( ) p vnppkn αα α =<≤=
Ta cũng có loglog pp nrr α =+≥ .
5.3. Nếu , ab là hai số tự nhiên, p là một số nguyên tố, ta có ( ) ( ) | pp abvavb ⇔≤
5.4. Với p là một ước nguyên tố của a . Đặt m apk = với * , mk ∈ ℕ , ta có
i) ( ) p va ap ≥
ii) m pkm α ≥+ với . m β ≥ Do đó ( ) p ava α≥+ với ( ) p va β ≥ hay
apβ ≥ .
II. VẬN DỤNG BỔ ĐỀ VỀ MŨ ĐÚNG TRONG GIẢI TOÁN
Trước hết ta xét một số bài toán mà việc lựa chọn Bổ đề LTE và chọn số p là điều dễ dàng quan sát thấy. Bài toán 1. (AIME 2020 I- Problem 12) Cho n là số nguyên dương nhỏ nhất thỏa mãn 35714923.5.7 nn ⋮ . Tìm số các ước nguyên dương của n Phân tích bài toán
Đề bài xuất hiện 1492nn , ta nghĩ ngay đến sử dụng bổ đề LTE. Kiểm tra điều kiện bổ đề đều thỏa mãn trong các trường hợp 3,7pp== . Tuy nhiên 14925 / ⋮ nên ta chưa thể áp dụng ngay bổ để LTE trong trường hợp 5 p = Để giải quyết vấn đề này ta nghĩ đến việc xét các trường hợp của n
Lời giải Áp dụng Định lý 1, ta có
( ) ( ) ( ) ( ) 3333 149214921 nn vvvnvn −=−+=+
Để 35714923.5.7 nn ⋮ thì
( ) ( ) ( ) ( ) 357 3333 3.5.71492312 nn vvvnvn ≤− ≤+ ≥ 23 n⋮ .
Lại áp dụng Định lý 1, ta có
( ) ( ) ( ) ( ) 7777 149214922 nn vvvnvn −=−+=+ .
Để 35714923.5.7 nn ⋮ thì ( ) ( ) ( ) ( ) 357 7777 3.5.71492725 nn vvvnvn ≤− ≤+ ≥ , ta được 57 n⋮
Ta có 149254, nn nss −⇔=∈⋮ℕ
Xét ( ) ( ) 4 44 14921160mod5 −≡−−≡ , hay 44 14925 ⋮ .
Áp dụng Định lý 1, ta có ( ) ( ) ( ) ( ) 4444 5555 149214921492 nnss vvvvs −=−=−+ ,
Mặt khác, ta cũng có ( ) ( ) 4 44 149211615mod25 −≡−−≡− nên 44 149225 ⋮
Vậy ( ) 44 5 14921 v −=
Để 35714923.5.7 nn ⋮ thì ( ) ( ) ( ) ( ) 357 5555 3.5.71492514 nn vvvsvs ≤− ≤+ ≥ ,
ta được 44 54.5sn ⋮⋮
Do 254 3,7,4.5 nguyên tố cùng nhau nên 2345 2.3.5.7 n⋮ , do n là số nguyên dương nhỏ
nhất nên 2345 2.3.5.7 n =
Vậy số các ước của n là ( )( )( )( ) 21314151270 ++++= .
Lời giải khác
Theo Công thức khai triển Nhị thức Newton, ta có ( ) 149214722 n nnn −=+−
11222333 .147.2.147.2.147.2147 nnnn nnn CCC =++++ ⋯
Ta kiểm tra điều kiện để 3 1492 3 nn ⋮
Do 3||147 nên các số hạng chứa 34 147,147,⋯ sẽ đều chia hết cho 33 , tức là ngoại
trừ hai số hạng đầu, thì các số hạng còn lại đã chia hết cho 33 .
Do đó ta cần ( ) 2 31222 1 12 3 1 |..147147.22.7..1 14.2 47.2 2 nn n nn n nn Cn C = + +
( ) ( ) 1233 .147.21.147.2147.2147143 nnn nnnnn =+−=− ,
tức là 23 n∣ (do 1471433 n ⋮ ).
Ta kiểm tra điều kiện để 7 1492 7 nn ⋮
Vì 2 7||147 nên các số hạng chứa 45 147,147, đều chia hết cho 7 7 , tức là ngoại trừ
ba số hạng đầu thì các số hạng còn lại đều chia hết cho 7 7
Do đó, ta cần
711222333 7|.147.2.147.2.147.2 nnn nnn CCC ++
260
nn n nnnnnn =+−+−−
()()() 3 123 4 147 .147.21.147.212..2
( ) ( ) ( ) 42 147.281147.2327203 n nnnn =+−+−+ ( ) 42 147.272032190314120 n nnn =−+ ,
tức là 5 7| n (do 2 720321903141207 nn−+ ⋮ ).
Ta kiểm tra điều kiện để 5 1492 5 nn ⋮ .
Ta có ( )( )149242221mod5 nnnnnn −≡−=− ,
nên để 5 1492 5 nn ⋮ thì 4 n∣ . Đặt 4 nm = và đặt ( )( ) 442222 149214921492147.151.22205 c =−=−+= .
Kiểm tra được rằng 5|| c
Theo Công thức khai triển Nhị thức Newton, ta có
( ) ( ) 44 1492(16)(16) mm mm c −=+−
11222333444 16161616 mmmmm mmmm CcCcCcCcc =⋅⋅+⋅⋅+⋅⋅+⋅⋅++ ⋯
Do 5|| c nên các số hạng chứa 56 ,,cc đều chia hết cho 55 , tức là ngoại trừ bốn số hạng đầu thì các số hạng còn lại đều chia hết cho 55 .
Bằng cách lập luận tương tự như trên, ta cũng có 55 cm∣ , và do 5|| c nên 45 m∣
Theo cách đặt 4 nm = nên 44.5 n∣ .
Tổng hợp các điều kiện thì ( ) 2245 32.5.7 | n .
Với n là số nguyên dương nhỏ nhất thì ( ) 2245 32.5.7 n = . Số các ước nguyên dương của n là 270
Bài toán 2. (Balkan MO 2013) Tìm các số nguyên dương ,, xyz sao cho
5 42013 yz x +=
Phân tích bài toán
Từ giả thiết, không khó để nhận ra có thể giải quyết bài toán bằng cách sử dụng Bổ đề
LTE với 3,11,61 p = là các ước của 2013. Tuy nhiên, ban đầu số mũ của x và 4
khác nhau nên ta chưa thể áp dụng Bổ đề LTE ngay được. Dễ dàng kiểm tra được rằng 5 yt = , khi đó phương trình có dạng 5542013 t z x +=
Lời giải.
Xét modulo 11, ta có 11|2013 nên ( ) 5 40mod11 y x +≡ . Mà ( ) 5 0,1,1mod11 x ≡−
nên ( )41,0,1mod11 y ≡− . Do đó, các nghiệm trong modulo 11 là ( ) ( ) ( ) ( ) { }( ) 5 ,40,0,1,1,1,1mod11 y x ∈−−
Ta có ( )44,5,9,3,1mod11 y ≡ , nên ta cần 41(mod11) y ≡ .
Mặt khác, vì ( ) 11 45 ord = nên 5 y∣ . Khi đó, đặt 5 yt = , ta có
5542013 t z x += , hay ( )5 5 42013 tz x +=
Vì ( )5 5 442013 tz t xx++= ∣ , mà 20133.11.61 = và 41 t x +≠ nên
431161 tijk x += với ,, ijk + ∈ℤ .
Ta kiểm tra được rằng 43,11,61 t ⋮ nên 3,11,61 x ⋮
Áp dụng Định lý 2, ta có ( ) ( ) ( ) () 5 5 333445 tt vxvxv +=++
do đó ( ) ( ) 3320134 zt vvx=+ , hay ( ) 3 4t zvx=+ nên 34 || zt x +
Tương tự, ta chứng minh được | 14 | 1zt x + và | 64 | 1zt x +
Do 431161 tijk x += nên 4 x + chỉ có các ước nguyên tố trong tập { }3,11,61
Lại thêm với kết quả vừa chứng minh được, 34 || zt x + , | 14 | 1zt x + , | 64 | 1zt x + nên
42013 tz x +=
Điều này dẫn đến mâu thuẫn, vì ( )5 5 2013442013 zttz xx =+>+= .
Vậy phương trình này không có nghiệm.
Lời giải khác.
Ta vẫn sử dụng kết quả 5| y như chứng minh trên. Đặt 5 yt = .
Khi đó, vế trái trở thành ( ) ( )( ) 55432234 444444 tttttt xxxxxx +=+−+−+ .
Hơn nữa, vì ( ) ( )( ) 432234432234 444442.43.44.45.4 ttttttttt xxxxxxxxx −+−+=+−+−+
nên 4t x + và 432234 4444 tttt xxxx −+−+ nguyên tố cùng nhau. Thật vậy, mọi ước chung d của chúng phải là ước của 2013 và 45.4 t , mà ( ) 4 gcd2013,5.41 t = nên
1 d = .
Từ phương trình 5 42013 yz x += , thì ( ) 5 10mod3 y x +≡ nên 5 3 x , do đó 3 x Điều này dẫn đến ( ) 432234444411120mod3 tttt xxxxxxx −+−+≡−+−+≡−≡
Từ phương trình 5 42013 yz x += thì ( ) 5 01mod2 x +≡ nên x là lẻ Điều này dẫn đến
( ) 43223444441mod4 tttt xxxx−+−+≡ .
262
Ta có ( ) ( )( ) 55432234 2013444444 ztttttt xxxxxx =+=+−+−+ và
20133.11.61 =
Vì 432234 34444 tttt xxxx −+−+ nên 3|4t x + , kéo theo ( )2mod3 x ≡ . Do đó ( ) 432234444422mod3 tttt xxxxx −+−+≡−≡
Vì 2 n và ta chứng minh ở trên thì 4 x + và 432234 4444 tttt xxxx −+−+ nguyên
tố cùng nhau, nên 432234 4444 tttt xxxx −+−+ phải có dạng ( )11,61,11.61 z zz .
Ta lập bảng kiểm tra ( ) 43223444442mod3 tttt xxxx−+−+≡ và ( ) 43223444441mod4 tttt xxxx−+−+≡ .
xx xx −+ −+
4 444
t ttt
43 2234
modul o 3 Điều kiện của z để
xxx x −+ −+≡
() 4322 34
44 442mod3
tt tt
modul o 4 Điều kiện của z để
xxx x −+ −+≡
44 441mod4
tt tt
() 4322 34
11z 2 z Lẻ 3z Chẵn 61z 1z Không thỏa mãn 1z Luôn thỏa mãn ( )11.61 z 2 z Lẻ 3z Chẵn
Nếu 432234 4444 tttt xxxx −+−+ có dạng 11z hoặc ( )11.61 z thì z không thể vừa là số lẻ, vừa là số chẵn.
Nếu 432234 4444 tttt xxxx −+−+ có dạng 61z thì không có số nguyên z thỏa mãn.
Vậy phương trình này không có nghiệm.
Bài toán 3. Tìm tất cả các bộ số nguyên dương ( ) ,, xyz thỏa mãn phương trình
20092009 7 z xy+=
Lời giải.
Dễ thấy xy ≠ (vì nếu xy = thì 20092009200927z xyx+=≠ )
Ta có 20092009 | xyxy ++ đồng thời 17| xyxy +> + Đến đây ta xét hai trường hợp sau
TH1: Nếu 7 x , 7 y , Áp dụng Định lý 2, ta có
( ) ( ) ( )
20092009 777 2009 vxyvxyv +=++
( ) ( )
20092009 77 2 vxyvxy +=++
( )
20092009 49.. xykxy +=+ với 7 k điều này mâu thuẫn với giả thiết
20092009 xy + chỉ có ước là 7
TH2: Nếu một trong hai số x hoặc y chia hết cho 7 thì số còn lại cũng chia hết cho
7 hay ,7xy⋮ .
Không giảm tổng quát, giả sử ( ) ( ) 77 vxvyk ≥= 1 7. k xx = và 1 7. k yy = với
* 11111 ,,7, xyyyx / ∈> ℕ⋮ do 11xy ≠
Khi đó ta có ( ) 20092009200920092009 11 77 kz xyxy+=+= ( ) 200920092009 11 7 zk xy += là
lũy thừa của 7 . Mà 1177yx// ⋮⋮ quay về TH1 ta có điều mâu thuẫn.
Vậy phương trình trên không có nghiệm nguyên dương.
Tiếp theo ta xét bài toán ở dạng Tổng quát hơn
Bài toán 4. (European Mathematical Cup 2012, Senior Division). Tìm số nguyên dương ,, abn và số nguyên tố p thỏa mãn 20132013 nabp +=
Phân tích bài toán.
Ở đây, a và b có cùng số mũ 2013, tuy nhiên đề bài không có dữ kiện , apbp ⋮⋮ và abp + ⋮ .
Bằng cách đặt () () , ,, abab abab ′′ == thì ta có pa ′ và pb′ và
20132013 abp ab ′′ + + ⋮⋮ .
Lời giải.
Đặt ( ) , dab = và , ab xydd == . Ta có ( )
( ) ( ) ( ) 20112009220102012 xxyxyxyxyxyy =−+−++−+
Biểu thức này lớn hơn 61 khi ( ) ( ),1,1xy ≠ và xy ≥ .
Đến đây, ta xét hai trường hợp.
TH1: Nếu ( ) 20131 p v = thì ( ) 201220112010220112012 1 p vxxyxyxyy−+−−+= và { }3,11,61 p ∈ nên { } 201220112010220112012 3,11,61 xxyxyxyy−+−−+∈ ⋯ , điều này
không thể xảy ra vì 201220112010220112012 61 xxyxyxyy−+−−+> ⋯ theo chứng minh
trên.
TH2: Nếu ( ) 20130 p v = thì ( ) ( ),1,1xy = . Do đó ab = .
Vì 20132013 n abp += nên 2013 2. n a p = , do đó 2 p =
Ta có nghiệm * 2,20131,2, k abnkpk ===+=∀∈ ℕ .
Lời giải khác.
Đặt (),,, dabxyab dd === . Khi đó, ta có ( ) 201320132013 ndabp += , nên d cũng phải là lũy thừa của p , do đó , k dpk=∈ ℕ .
Do đó ( ) 201320132013kn pabp += , nên 20132013 mabp += (với 2013 mnk =− ).
Đặt 671Ax = và 671By = thì ta có 33 m ABp += , nên ( )( ) 22 mABAABBp +−+=
201320132013 n y d xp += , nên
20132013 m xyp += , hay ( )( ) 201220112010220112012 m xyxxyxyxyyp +−+−−+= ⋯ .
Do , xy là các số tự nhiên nên 1 xy+> nên pxy + ∣ . Mà theo cách đặt , ab xydd ==
với ( ) , dab = thì , xy nguyên tố cùng nhau, kéo theo px và py
Áp dụng Định lý 2, ta có ( ) ( ) ( ) 20132013 2013 ppp vxyvxyv +=++ .
Mà ( ) ( ) ( ) 20132013201220112010220112012 ppp vxyvxyvxxyxyxyy +=++−+−−+
nên ( ) ( ) 2012201120102201120122013... pp vvxxyxyxyy =−+−−+ .
Xét 20133.11.61 = nên ( ) 20130 p v = nếu p là số nguyên tố khác 3,11,61, và
( ) 20131 p v = nếu p là một trong các số nguyên tố 3,11,61
+) Nếu ( ) ( ),1,1xy = thì 201220112010220112012 1 xxyxyxyy−+−−+= ⋯ +) Nếu ( ) ( ),1,1xy ≠ , không giảm tổng quát, giả sử xy ≥ ta có
201220112010220112012 xxyxyxyy −+−−+
Theo cách đặt (),,, ab dabxydd === thì , xy nguyên tố cùng nhau nên , AB cũng
nguyên tố cùng nhau.
Ta xét ba trường hợp.
TH1: Nếu 1 AB+= . Điều này không xảy ra do A và B là các số nguyên dương.
TH2: Nếu 22 1 AABB−+= hay 2 ()1 ABAB−+= , xảy ra khi 1 AB== . Do đó ab = .
Vì 20132013 n abp += nên 2013 2. n a p = , do đó 2 p =
Ta có nghiệm * 2,20131,2, k abnkpk ===+=∀∈ ℕ
TH3: Nếu cả AB + và 22 AABB −+ đều lớn hơn 1, thì pAB + ∣ và ( )2 22 3 pAABBABAB −+=+− ∣ nên 3 pAB ∣
+) Nếu pAB ∣ , kết hợp với pAB + ∣ thì ta có pA ∣ và pB ∣ , mâu thuẫn với điều kiện A và B nguyên tố cùng nhau.
+) Nếu 3 p∣ thì 3 p = . Ta có hai trường hợp nhỏ.
o ( )2 22 33AABBABAB −+=⇔−+= , nên 2,1AB== hoặc 1,2AB==
Nhưng theo cách đặt 671Ax = và 671By = thì 671 2 x = không có nghiệm nguyên dương.
265
o 22 3 AABB −+ > thì 2223 AABB −+ ∣ . Ta có 3| AB + nên ( )2 2 3| AB + và ( )2 222 33AABBABAB −+=+− ∣ . Điều này dẫn đến 2 33AB ∣ , hay 3 AB ∣ .
Như chứng minh trên, ta có nếu pAB ∣ , kết hợp với pAB + ∣ thì ta có pA ∣ và pB ∣ , mâu thuẫn với điều kiện A và B nguyên tố cùng nhau. Do đó, trường hợp này cũng
không cho ta nghiệm của phương trình.
Vậy tất cả các nghiệm của phương trình là
* 2,20131,2, k abnkpk ===+=∀∈ ℕ
Bài toán 5. (Romania 2019) Tìm tất cả các số nguyên dương k sao cho tồn tại các số nguyên dương n và m với 2 m ≥ sao cho 35kkm n += .
Lời giải.
Hiển nhiên ta thấy rằng n là số chẵn.
Ta xét các trường hợp sau:
TH1: Nếu k chẵn thì ( )mo2 5 d 30 kk ≡ + và ( )mo4 5 d 32 kk ≡ + nên ( ) 2 351 kk v += , kéo theo 1 m = Điều này không thỏa mãn 2 m ≥
TH2: Nếu k lẻ. Áp dụng Định lý 2, ta có ( ) ( ) ( ) ( ) ( ) 222223535803 kk mvnvvvkv=+=++=+= |3 m
với 2 m ≥ 3 m =
+) Nếu 1 k = , ta có ( ) ( ),,1,2,3knm =
+) Nếu 1 k > ( ) 3 5mod9 k n ≡ ⇔ 3 k∣ .
Đặt * 3, ktt=∈ ℕ . Khi đó ta có ( ) ( ) 33 3 35 tt n += , mà ta biết rằng phương trình này có
dạng 333 abn += nên không có nghiệm nguyên dương.
Vậy 1 k = .
Bài toán 6. (Italy TST 2003) Tìm các số nguyên dương , xy và số nguyên tố p sao
cho 219 xyx p += .
Lời giải.
Ta có 219192 xyxxxy pp +=⇔−= . Mà 17|192xx VT =− nên 1717 y pp = ⋮
Do ( ) 21719217217 x xyxxx +==+>+ với 2 x ≥ . Do đó ta xét hai trường hợp sau
TH1: Nếu 2 x ≥ thì yx > . Áp dụng Định lý 1 thì
( ) ( ) ( ) ( ) 17171717 17192192 yxx vvvvx =−=−+ ( ) 17 1 yvx =+ .
Mà ( ) 17 11 yvxx =+≤+ . Do đó 1 yx =+ và ( ) 17 vxx = . Điều này không xảy ra.
TH2: Nếu 1 x = thì 1 y = .
Vậy ( ) ( ),,1,1,17xyp = là nghiệm duy nhất của bài toán này.
Bài toán 7. (Russia MO – 1996) Tìm tất cả các số nguyên dương n sao cho tồn tại
các số nguyên dương ,, xyk thỏa mãn ( ) ,1,2xyk=≥ và 3nkk xy=+ .
Lời giải.
Nếu một trong hai số x hoặc y chia hết cho 3 thì số còn lại cũng chia hết cho 3 ,
điều này mâu thuẫn. Do đó cả x và y đều không chia hết cho 3.
Nếu k chẵn thì ( )1mod3 kk xy≡≡ nên kk xy + không chia hết cho lũy thừa của 3 .
Do đó k lẻ.
Gọi p là ước nguyên tố của xy + .
Vì k lẻ kk xyxyp ++⋮⋮ hay 33 n pp = ⋮
Do đó 3m xy+= với m là số nguyên dương.
Áp dụng Định lý 2, ta có ( ) ( ) ( ) ( ) 3333 3nkk vvxyvxyvk =+=++
( ) 3 nmvk =+ Đến đây ta xét các trường hợp sau
TH1: Nếu 1 m > . Dễ dàng chứng minh bằng quy nạp được rằng 32 a a ≥+ với mọi số nguyên 1 a ≥ , và ( ) 3 2 vkk≤−
Đặt ( ) max, Mxy = , do 39 m xy+=≥ nên 5 M ≥ . Khi đó
+≥=>
() 1 5
1 .3.5 2 m k
kkkkmk xy xyMMM + ≥ ≥ =
11 1 .3 22
( ) 3 22 3.5333 mv mkmkn k + −+− >≥≥= mâu thuẫn giả thiết.
TH2: Nếu 1 m = 3 xy+= nên 1,2xy== hoặc 2,1xy== . Kéo theo
( ) 3 1 312 vk k + =+ . Nhưng ta có ( ) ( ) 33 3 3 vkvkkk ≤ ∣ . Do đó
( ) 3 3 1 () 1233.33213 v vk k k k k kk ≤
+ +==≤ +≤ mà k là số lẻ 3 k =
Khi 3,2,1kxy=== hoặc 3,1,2kxy=== thì 2 n = .
Bài toán 8. (Balkan 1993) Cho p là số nguyên tố và 1 m > là số nguyên dương.
Chứng minh rằng nếu tồn tại các số nguyên dương 1,1xy>> mà 22
m pp xyxy ++ = thì mp =
Lời giải.
Ta có bất đẳng thức quen thuộc 22
p pp xyxy ++ ≥ với mọi số nguyên dương p .
Kết hợp giả thiết 22
m pp xyxy ++ = mp ≥ .
Đặt ( ) , dxy = , 11 , xdxydy == với ( ) 11,1xy = , 11,1xy ≥ và không đồng thời bằng
Ta có 22
hợp
m pp xyxy ++ = ⇔ ( ) ( ) 1 1111 2 m mppmpxydxy +=+ . Ta có hai trường
TH1: Nếu p lẻ
Gọi q là ước nguyên tố bất kì của 11xy + . Đặt ( ) 11 1 q vvxy=+≥
o Nếu q lẻ, áp dụng Định lý 2, ta có ( ) ( ) ( ) 1 11 2mpp qq vxyvvp +=+
Mà () ( ) 11 q m mp vx m y v d +≥ ( ) q vvpmv +≥ 1 mvv ≤+ (Vì ( ) () 1 0 q q
vpkhiqp vpkhiqp == =≠ ) ( ) 11 vm −≤
Do đó 22mpm ≤ ≤≤ mâu thuẫn.
o Nếu 2 q = thì 1 mvmv −+≥ , nên 1 v ≤ và 11 2 xy+= , hay xy = , suy ra
mp =
TH2: Nếu 2 p =
o Nếu ( ) ( ),1,2xy = hoặc ( ) ( ),2,1xy = thay vào ta thấy không thỏa mãn
o Nếu ,24 xyxy ≥ +≥
m xyxyxyxy xy ++++
Vậy 2 mp==
Bài toán 9. (Romanian Junior Balkan TST 2008) Cho số nguyên tố 3 p ≠ và các số
nguyên , ab thỏa mãn | pab + và 233 | pab + . Chứng minh rằng 2 | pab + hoặc
333 | pab +
Lời giải.
Do | pab + , nên nếu | pa thì | pb và khi đó 333 | pab + .
Nếu pa thì pb , lại có | pab +
Áp dụng Định lý 2, ta có ( ) ( ) ( ) ( ) ( ) 33 33 3 ppppp vabvabvvabvab +=++ +=+ .
Mà 233 | pab + nên ( ) 33 2 p vab+≥ , kéo theo ( ) 2 p vab+≥ , hay 2 | pab +
Cách giải khác.
Nếu pa thì pb . Ta có ()() ( ) 2 233|3 pabababab +=++− , mà , pab nên
()( ) 2 3 pabab +− .
Ta có ( ) ()() ( ) ( ) 2 33 3 pp vabvababab +=++−
()() ( ) () 2 3 ppp vabvababvab =+++−=+ .
(vì ()( ) 2 3 pabab +− nên ()( ) 2 30 p vabab+−= )
Mà 233 | pab + nên ( ) 33 2 p vab+≥ , kéo theo ( ) 2 p vab+≥ , hay 2 | pab +
Vậy ta có điều phải chứng minh.
Bài toán 10. Tìm tất cả các số nguyên dương ,, xyp thỏa mãn 1 xppy−= với p là
một số nguyên tố
Lời giải.
Ta xét các trường hợp sau
TH1: Nếu 2 221 x py = −=
y là số lẻ. Khi đó ( ) 2 212mod4 x y =+≡
1,1xy ==
TH2: Nếu p là số nguyên tố lẻ, ta có 1 px yp +=
Áp dụng Định lý Fecmat nhỏ ta có ( ) 11mod xp pyyp =+≡+
|1py + . Khi đó áp dụng Định lý 2 ta có
( ) ( ) ( ) ( ) 1111 p pppp vyvyvpvy+=++=++
( )11 p xvy −=+
Mặt khác ( )( ) 12111...1 pppx yyyyyp +=+−+−+=
Do đó 1 12
x pp
1 ...1
yp yyyp += −+−+=
o Nếu 1 y = thì 2 p = (loại)
o Nếu 1 233;2 x yppx = = == (thỏa mãn)
o Nếu 2 y > , ta có ( ) ( ) ( ) 1224...111...11pppp yyyyyyyyy −+−+=−+−++−+ 24 ...11 pp yyyy >++++>+
1 x pp >
269
( ) 110 p xvy = += (vô lý vì |1py + )
Vậy bộ ( ) ,, xyp thỏa mãn yêu cầu bài toán là ( ) ( )1,1,2;2,2,3
Bài toán 11. (Trung Quốc 2009) Tìm cặp số nguyên tố ( ) , pq thỏa mãn |55pq pq +
Lời giải.
Ta xét các trường hợp sau
TH1: Nếu 5 pq== thỏa mãn.
TH2: Nếu một trong hai số bằng 5 và số còn lại khác 5 . Không giảm tổng quát, giả
s
ử 5,5pq=≠ . Ta có
5415|55|55 qqqq+⇔+
Nhưng theo Định lý Fecmat bé ta có ( ) 1 51mod q q ≡
4 |51626 q += 2 q = hoặc 313 q = .
Trường hợp này ta có bốn nghiệm ( ) ( ) ( ) ( )5,2;5,313;2,5;313,5
TH3: Nếu một trong hai số bằng 2 và số còn lại khác 5 . Không giảm tổng quát giả sử
2,5pq=≠
( ) 1550mod2 q q +≡
Mà theo Định lý Fecmat bé ta thu được ( ) 55mod q q ≡ và ( )55mod2 q ≡ do đó
( ) 300mod2q ≡ 3 q =
Trường hợp này ta có hai nghiệm ( ) ( )2,3;3,2
TH4: Nếu , pq khác 2 và 5
Ta có ( ) ( ) 55mod,55mod pqpq≡≡ , kết hợp với giả thiết
Giả sử tồn tại , xy thỏa mãn yêu cầu bài toán.
TH1: Nếu | px mà || pp pxypy + . Mà p là số nguyên tố lẻ nên 3 p ≥ , ta có
() 222 |,||1! pp p pp x pxpypypp = + . Mâu thuẫn.
TH2: Nếu px mà () 1 | ! p pp p pxyp+=− nên py .
Theo Định lý Fermat nhỏ, ta có ( ) mod pp xyxypxyp +≡+ + ⋮ .
Áp dụng Bổ đề 2, ta có ( ) ( ) ( ) 2 2 p p p p ppp vxyvxyvpxyp +=++≥ + ⋮ (Mâu thuẫn)
Vậy không tồn tại các số nguyên , xy thỏa mãn đề bài.
Bài toán 13. (IMO shortlist 2014) Tìm tất cả các bộ ba số nguyên dương ( ) ,, xyp , thỏa mãn 1 p xy + và 1 p xy + đều là lũy thừa của p , trong đó p là số nguyên tố Phân tích bài toán.
Ý tưởng là xét bài toán theo p . Ta sẽ tìm cách giới hạn lại p
Dễ dàng xét với trường hợp 2 p =
Với trường hợp 3 p ≥ , vì 1 p xy + và 1 p xy + đều chứa các số hạng có số mũ là
1 p nên ta nghĩ đến sử dụng Định lý Fecmat, do đó cần xét các trường hợp p có là
ước của , xy không. Đồng thời giả thiết 1 p xy + và 1 p xy + đều là lũy thừa của p gợi ý cho ta nghĩ đến việc sử dụng Bổ đề LTE.
Lời giải.
Ta xét hai trường hợp sau
p p q q +− + ⋮⋮ ⋮ ⋮
21 1 1 21
5151 51 51
qq p p
Đặt 5 q mord = ( ) ( ) ( ) 22 |2111 mpvmvp ≤+−
Mặt khác ( ) 1 510mod p q +≡ ( ) 1 51mod p q ≡ / 1 p m
( ) ( ) 2211vmvp =+−
Lại có ( ) ( ) 1 51mod|1 q qmq ≡ ( ) ( ) 22 1 vmvq ≤−
Do đó ( ) ( ) 22 111 vpvq +−≤− ( ) ( ) 2211vpvq −<−
Tương tự khi xét theo modulo p ta lại có ( ) ( ) 2211vqvp−<− (Vô lý).
Vậy các cặp số thỏa mãn là ( ) ( ) ( ) ( ) ( ) ( ),2,5;5,2;5,5;5,313;313,5pq =
Bài toán 12. Cho p là số nguyên tố lẻ. Chứng minh rằng không tồn tại các số , xy ∈ℤ
TH1: 2 p = , dễ dàng thấy bộ ( ),2,2 k xx thỏa mãn bài toán
TH2: 3 P ≥
+) Ta thấy rằng nếu p là ước của một trong hai số x hoặc y thì p cũng là ước của số còn lại Giả sử ( ) ( ) , pp vxavyb == , ,0ab >
Không giảm tổng quát giả sử ab ≥
Ta có ( ) 11pp p vxab => ( ) 1 p p vxyb += 1 pb xyp += điều này vô lý vì
1 pb xyyp +>>
+) Nếu p không là ước của x và y
Theo định lý Fecmat, ta có 111,1pp xpyp ⋮⋮
+ .
270
Trước hết ta chứng minh xy ≠ . Thật vậy giả sử 1 p xyxx = + là lũy thừa của p ( ) 11 ppa p vxxaxxp += += mà 1 pa xxxp +>= (vô lý)
Không giảm tổng quát giả sử 11pp xyxyxy > +>+
Đặt 11 , pmpn xypyxpmn +=+= > . Ta có ( ) ( ) 11211 || nppnppp pxyxypyxyxy =++ +−+
( ) 22 |1npp pxxy
( ) 2 |1 p n pxy
Mà ( ) ( )( ) ( ) 12 12modmod pp pnpn xypxyp ≡− ≡−
( ) () ( ) 2 2 11mod p pp n xyyp −≡−−
Hay ( )2 1 pp n yp + ⋮
Lại có ( ) ( ) 1 1modmodpp ypyyp ≡ ≡
( ) ( ) 2 2 11mod pp p yyp +≡+ ( ) 2 10mod p yp +≡
Áp dụng Định lý 2, ta có () ( ) ( ) 2 2 111 pp p pp vyvy+=++ 21 1 pn yp + ⋮ ( ) ( ) 2112 11 pnppp pypxyyyyy +≥=+>+=+
py > 1 py ≥+
Lại có 1 1 p xp ⋮ ( ) 1 1 p xyyp +−+ ⋮ 11 ypyp + +≥ ⋮
Do đó 1 yp +=
Ta có ( ) ( )( ) 122|111pnpp xyppyyy +=+=++ ( ) 1121 |122 pppp xyyyyyx ++++<+
( ) 12 1 ppn xypyp +=+= ( ) 2 1 11 p n pp −+=
+) Nếu 32;5pyx = ==
+) Nếu 3 p > Ta có ( ) 2 11 p pP −+> và
Phân tích bài toán.
Với bài toán này việc tìm ra các yếu tố ,, pab đề áp dụng Bổ đề LTE không phải là điều dễ dàng. Nó đòi hỏi ở người làm toán phải có Kinh nghiệm và vận dụng một cách linh hoạt các định lý số học. Trong bài toán này việc sử dụng Định lý Fecmat là điều mà chúng ta có thể nghĩ đến.
Lời giải.
Nếu 1 n = thì p là số nguyên tố bất kỳ thỏa mãn yêu cầu bài toán.
Nếu 2 n ≥ , ta xét các trường hợp sau
TH1: Nếu 2 p = thì 22 nn = ⋮
TH2: Nếu 2 p > . Gọi q là ước nguyên tố nhỏ nhất của n , q là số lẻ
Ta có ( ) ( ) ( ) ( ) 2 11mod11mod n n pqpq −≡− −≡ và ( ) 1,1pq−=
Gọi ( )1 q tordp=− |2 tn
Áp dụng Định lý Fecmat nhỏ, ta có ( ) ( ) 1 11mod q pq −≡ |1tq
Nếu ( ) ,1tn > , gọi d là một ước nguyên tố của t và n | dn Mà |1 dtdtqdq ≤≤− < . Do đó , dq đều là ước của n và dq < mâu thuẫn với cách chọn q .
Vậy ( ) ,1tn = . Khi đó |22tt = hay ( ) ( ) 2 11mod pq −≡ ( ) ( ) 20mod ppq −≡ . Ta có 2 khả năng sau
+) Nếu |2qp thì ( ) ( ) 11112mod n n pq −+≡+≡ 2 q = ( ) 112 n pp −+ ⋮
là số chẵn (mâu thuẫn).
+) Nếu | qp . Khi đó 3 p ≥ và ( ) ( ) ( ) 1111mod nn pq −+≡−+
p pi ipi p i pp p p
2 2 2 2 0 32 3 2 2
−=− ≡−+− ≡−−
11 11 21mod
=
pCp Cp ppp
( ) ( ) ( ) 2 2 112112mod p ppppp −+≡−−+≡− (vô lý)
Vậy tất cả các bộ cần tìm là ( ) ( ) ( ) ( ),,,2,2,2,5,3,5,2,3 k xypxx =− .
Nếu n chẵn thì ( ) ( ) ( ) 11112mod nn pq −+≡−+≡ 2 q = 2 q = ( ) 112 n pp −+ ⋮ là số chẵn (mâu thuẫn). Vậy n là số lẻ
Áp dụng Định lý 2, ta có ()( )
( ) ( ) ( ) 2 qq vppvn ≥−
()()()()111
n qqqq vpvpvnpvn −+=+≥−
Đặt a pqb = , với * , ab ∈ ℕ . Ta chứng minh quy nạp được rằng 2 a qba≥+ với 3 q ≥ ( ) 2 q pvp ≥+ Đẳng thức xảy ra khi 1,3abq=== ( ) ( ) ( ) 22 qq pvppvn −≥≥− ( ) 1 q vn = và 1,3 abqp ====
( ) 3 1 vn = Đặt 3 nk = , với ( ) *,,31kk∈= ℕ , k là số lẻ
273
Từ giả thiết ta có 2 819 k k + ⋮
Nhận xét 819 k + ⋮ . Do đó ta cần tìm k sao cho 2 81 k k + ⋮
+) Với 13kn = = thỏa mãn
+) Với 2 k ≥ , tương tự cách chứng minh trên, ta gọi r là ước nguyên tố nhỏ nhất của
k và 8 r sord = . Khi đó |22ss =
2 |81 r hay 7 r = (vô lý vì ( )812mod7 k +≡ )
Vậy cặp số ( ) , np thỏa mãn là ( ) ( ) ( )1,,2,2,3,3 p
Bài toán 15. ( TST Romania 2009) Cho ,3an ≥ là hai số nguyên dương sao cho tồn
tại số tự nhiên 2 k ≥ thỏa mãn ( )|1 k na . Chứng minh rằng n là ước của
12 ...1 nn aaa++++
Lời giải.
Ta có 12 1 ...1 1 n nn a aaa a ++++=
Gọi p là một ước nguyên tố của n . Ta có ( )||1 k pna và ( ) ,1ap =
Áp dụng Định lý 2, Ta có ( ) ( ) ( ) 11 n ppp vavavn −=−+ () 1 1 n pp a vvn a = 1 | 1 n a n a (đpcm).
Bài toán 16. Cho , an là hai số nguyên dương và p là một số nguyên tố lẻ thỏa mãn ( ) 1mod pn ap ≡ . Chứng minh rằng ( ) 1 1mod n ap ≡ .
Lời giải.
Vì ( ) 1mod pn ap ≡ nên ( ) ,1ap = .
Theo Định lý Fermat nhỏ, ta có ( ) 1 1mod p ap = .
Do đó ( ) 1 mod pp aap = mà ( ) ,1ap = nên 1 pa ∣ .
Áp dụng Định lý 2, ta có ( ) ( ) ( ) ( ) 1111 p ppp p vavavpnvan −=−+≥⇔−≥− .
Vậy ( ) 1 1mod n ap ≡
Bài toán 17. Tìm tất cả các số nguyên dương ,1ab > thỏa mãn 1 abba ∣
Lời giải.
Gọi p là ước nguyên tố nhỏ nhất của b .
Đặt p morda = . Trước tiên, ta chỉ ra 1 m = . Thật vậy
+) Nếu 2 p = hoặc ap ⋮ thì ta có 1 m = .
+) Nếu p là số nguyên tố lẻ và pa .
Do |||1|1 abb pbbapa nên | mb .
Do ap ⋮ , theo Định lí Fermat nhỏ, ta có 1 |1 p pa nên |1mp
Do đó ( )|,1mbp
Vì p là ước nguyên tố nhỏ nhất của b nên ( ) 1,1pb−= . Vậy 1 m =
Trở lại bài toán, ta xét hai trường hợp sau
TH1: Nếu p lẻ Áp dụng Định lý 1, ta có ( ) ( ) ( ) ( ) ( ) 11. b a ppppp vavavbvbavb −=−+≥=
nên ( ) ( ) ( ) 111 pp vaavba −≥−≥− , điều này vô lí.
TH2: Nếu 2 p = . Ta suy ra b chẵn, 1 b a cũng chẵn nên a lẻ
Áp dụng Định lý 4, ta có ( ) ( ) ( ) ( ) ( ) 22222 1111 b vavavavbavb −=−+++−≥ ( ) ( ) ( ) ( ) ( ) 222111111 vavaavbaa −++≥−+≥−+=
Đẳng thức chỉ xảy ra khi ( ) 2 3,1avb==
Đặt 2 bs = với s là lẻ, yêu cầu đề bài trở thành 332s 2|31 s
o Nếu 1 s = thì 2,3ba== thỏa mãn yêu cầu.
o Nếu 1 s > . Đặt q là ước nguyên tố nhỏ nhất của s nên q cũng là lẻ.
Đặt 3 q nord = . Tương tự như trên, ta cũng chỉ ra 1 n = . Thật vậy:
+) Nếu 3 q = thì 1 n = .
+) Nếu 3 q ≠ thì ||| nqsns và theo định lí Fermat nhỏ, ta có 1 |31 q q nên
|1nq .
Từ đó ( )|,1nsq , mà q là ước nguyên tố nhỏ nhất của s nên ( ) ,111 sqn −= = .
Khi đó |312 q −= , điều này mâu thuẫn với q lẻ do s lẻ.
Vậy cặp số thỏa mãn yêu cầu bài toán là ( ) ( ),3,2ab =
Bài toán 18. Cho số nguyên dương 1 a > . Tìm số nguyên dương n nhỏ nhất sao cho 2000 2|1 n a
Lời giải.
Từ giả thiết ta có ( ) 2 12000 n va −≥
Ta có ( ) 20001mod2 n a ≡ . Theo Định lý Euler kết hợp giả thiết n nhỏ nhất suy ra
( ) 20001999|22 n ϕ =
n là lũy thừa của 222k = với 0 k ≥ . Khi đó
()()( ) ( )
1 222 11111...1 k k n aaaaaa −=−=−+++ ( ) ( ) ( ) ( ) 1 222 2222111...1 k n vavavava−=−+++++
Mà () ( ) 22 2 12mod412 i ava +≡ += với 1,1ik=−
Vậy ( ) ( ) 2 221112000 n vavak−=−+−≥
( ) 2 2 20011kva≥−−
Ta có ( ) 32|32|32 qqqrqrqq p
Áp dụng Định lý 2, ta có ( ) ( ) ( ) 3232 qrqrqq ppp kvvvr =−=−+ . Đến đây ta xét hai
trường hợp sau
TH1: pr ( ) ( ) 3232 qrqrqq pp vvk −=−=
32322 qrqrqqk −=−= điều này không xảy ra vì ( ) ()()() ( ) 121 3232332...232 qrqrqr qrqrqq qq−=−+++>−
Vậy chọn k thỏa mãn ( ) ( ) () 22 22 2 2
=−−−≤
kvakhiva kkhiva
=−>
2001112001 012001
Khi đó 2k n = thỏa mãn yêu cầu bài toán.
Bài toán 19. (China TST 2009) Cho hai số nguyên dương 1 ab>> và b lẻ, n là số
nguyên dương. Chứng minh rằng nếu |1nnba thì 3n b a n >
Phân tích bài toán.
Nếu gọi p là một ước nguyên tố của b khi đó ( )||1nnnpba và ( ) ,1pa = theo Định lý Fecmat bé, ta có ( ) 111mod|1pp appa ≡ . Lúc này ta đủ điều
kiện sử dụng Bổ đề LTE với nhận xét ( ) ( ) 1 1|1 n npaa
Lời giải
Gọi p là ước nguyên tố nhỏ nhất của b . Do b lẻ nên p là số nguyên tố lẻ
Do ( ) ( )||11 nnnnn pp pbanvbva ≤≤−
Lại có ( ) ( ) ( ) ( ) () ( ) ( ) 111 1111 n nppp ppppp vavavavnvna −≤−=−+=−
()1111 n npp p pnaa n ≤− ≥+ 1 3 nn bp p aa nn >>≥
Vậy 3n b a n >
Bài toán 20. (Bulgaria 1997) Cho n là số nguyên dương thỏa mãn 32nn là lũy thừa của một số nguyên tố. Chứng minh rằng n là số nguyên tố
Lời giải.
Đặt 32nnk p −= với p là một số nguyên tố
Ta chứng minh bằng phản chứng
Giả sử n không phải là số nguyên tố. Suy ra tồn tại số nguyên tố p sao cho nqr = , ,1rr∈> ℕ
276
TH2: | pr 32|32|32 pprrnn
|32pp p
Hay ( ) 320mod pp p −≡
Áp dụng Định lý Euler, ta có ( ) ( ) 33mod,22mod pppp≡≡
( ) 32321mod pp p −≡−≡ (mâu thuẫn)
Vậy n thỏa mãn điều kiện phải là một số nguyên tố.
Bài toán 21. Chứng minh rằng không tồn tại cặp số ( ) , an nguyên dương 2 n > sao cho ( )1 n n aa +− là lũy thừa của 5
Lời giải. Giả sử tồn tại số nguyên dương m sao cho ( ) 15 n nm aa+−=
Nhận xét rằng nếu 1 a + chia hết cho 5 thì số còn lại cũng chia hết cho 5 (vô lý). Do đó cả hai số 1 a + và a đều không chia hết cho 5 . Ta xét các trường hợp sau
TH1: ( ) ( ) ( )1mod50121mod5 n nn aaa ≡ ≡+−≡− 4| n
TH2: ( ) ( ) ( )2mod50132mod5 n nnn aaa ≡ ≡+−≡− 2| n
TH3: ( ) ( ) ( )3mod50143mod5 n nnn aaa ≡ ≡+−≡− 4| n
Do đó n luôn là số chẵn, đặt 111 2,,2nnnn=∈≥ ℕ . Ta có
()() ( ) () () () ()() ( ) 1 1 11 1 22212 2221 22 5111...1 n n nn n m aaaaaaaa =+−=+−+++++
()() () () ()() ( ) 1 11 212 2221 211...1 n nn aaaaa =++++++
5|21 a + và
1 1 11 1 212 2 2221 2 5 55 1...1121 n n nn n vaaaavaava +++++=+−−+
() () () ()() ( ) ()( ) ()
Mặt khác, áp dụng Định lý 1, ta có ()( ) ()()()()
1 1 2 2 5551551 121221 n n vaavavnvavn +−=++=++
Do đó () () () ()() ( ) () 1 11 212 2221 5 51 1...1 n nn vaaaavn +++++=
Mà () ( ) () ()() 1 11 212 2221 1...1 n nn aaaa +++++ là lũy thừa của 5 nên () ( ) () ()() 1 11 212 2221 1 1...1 n nn naaaa +++++ ⋮ (Vô lý vì () ( ) () ()() 1 11 212 2221 1 1...1 n nn aaaan +++++> với * 1 12, an+≥∈ℕ )
Vậy không tồn tại cặp số ( ) , an thỏa mãn yêu cầu bài toán.
Bài toán 22. Cho ,, bmn là các số nguyên dương thỏa mãn 1; mmn >≠ . Biết 1 mb
và 1 nb có cùng tập hợp các ước nguyên tố. Chứng minh rằng 1 b + là lũy thừa của 2
Lời giải.
Từ giả thiết, gọi p là ước nguyên tố bất kì của 1 mb và 1 nb
Áp dụng một kết quả quen thuộc ( ) ( ) , 1,11 mn mn bbb−−=− , đặt ( ) , mn α =
Ta suy ra |1pbα
Với ( ) , mn α = * , kl ∃∈ℕ sao cho , mknl αα== với ( ) ,1kl =
Vậy 11abα +=+ là lũy thừa của 2
1 bα + là lũy thừa của 2 nên b là số lẻ
+) Nếu α là số chẵn thì ( ) () 2 112mod4bbαα+=+≡ là lũy thừa của 21 b =
+) Nếu α là số lẻ thì ( )( ) 1 11...1bbbb αα+=++++ nên 1 b + cũng là lũy thừa của 2
Vậy trong mọi trường hợp thì 1 b + là lũy thừa của 2
Tiếp theo ta xét một số bài toán sử dụng Bổ đề mũ đúng dưới dạng Định lý Lengedre.
Bài toán 23. (IMO 2019) Tìm các cặp số nguyên dương ( ) , nk sao cho ( )( )( ) ( ) 1 !212224....22 nnnnn k =−−−−
Phân tích bài toán.
Ý tưởng chính của bài toán là cố gắng giới hạn lại ( ) , nk , chỉ xét tại hữu hạn trường hợp.
Vế phải chứa các thừa số dạng ( ) 22 ni , vế trái là !k nên ta nghĩ
Đặt ab
α = , từ giả thiết ta suy ra tập hợp các ước nguyên tố của 1,1,1 kl aaa là trùng nhau.
Do mn ≠ suy ra trong hai số k và l phải có ít nhất một số lớn hơn 1. Giả sử 1 k > .
Ta sẽ chứng minh 1 a + là lũy thừa của 2 . Thật vậy, ta xét các trường hợp sau
TH1: Nếu k là số chẵn, đặt 1 2 kk β = với 1k lẻ
Ta có ( )( )( ) ( ) 1 1111 22 1111...1 kkkk k aaaaa β −=−+++
Do đó mọi ước nguyên tố q của 1 1 k a + cũng là ước của 1 a
Mà với 1k lẻ thì 1 11 k aa++ ⋮ , ( ) 1,11aa−+= hoặc 2 22 qq = ⋮
Do đó 1 1 k a + là lũy thừa của 2 1 a + cũng là lũy thừa của 2
TH2: Nếu k là số lẻ, ta có ( )( ) 1 11...1kk aaaa −=−+++
Gọi q là ước nguyên tố bất kì của 1 ...1 k aa+++ . Do 1 ... k aa ++ là tổng của chẵn số tự nhiên nên nó là số chẵn 1 ...1 k aa+++ là số lẻ q là số lẻ và là ước của
1 k a q cũng là ước của 1 a
Ta có ( ) ( ) ( ) 1 11...1kk qqq vavavaa −=−++++
Áp dụng Định lý 1, ta có ( ) ( ) ( ) 11 k qqq vavavk −=−+
Suy ra ( ) ( ) ( ) ( ) 1 ...111kk qqqq vaavavavk +++=−−−=
( ) 1 ...1 k kaa +++ ⋮ (vô lý vì 1 ...1 k aak +++> )
Bài toán 24. (HSG Hà Nam 2016) Cho q là số nguyên tố lẻ và đặt
()()() ( )2 2 22!2! q q Aqqq =++ . Chứng minh rằng A có một ước nguyên tố 2 pq >
Phân tích bài toán.
Ý tưởng của bài toán là chứng minh bằng phương pháp phản chứng. Tìm dạng phân
tích tiêu chuẩn của A Để ý vế phải các số hạng đều là lũy thừa của 2q với cơ số là
2q và ( ) 2! q từ đó nghĩ đến sử dụng Bổ đề LTE và Định lý Legendre với 2, pq =
Lời giải.
Giả sử ngược lại, mọi ước nguyên tố p của A đều không vượt quá 2q . Khi đó ta có:
( ) |2!pq và ()( )2 |2! q pq do đó |2 pq
Từ đó suy ra 2 p = hoặc pq =
A có dạng 2.mn q với , mn∈ℕ
Ta có ()( ) () ()( ) 2 2 2222;2!2 q q vqqvqq =>
Và ()( ) () ()( ) 2 2 22;2!2 q q qq vqqvqq =>>
Mặt khác, theo Định lý Legendre suy ra () ()
2 22 2211 2!....2...2 2222 qq vqqq ≤++≤++< và ( ) ( ) 2!2 q vq = vì 2 q >
Từ đó suy ra ( ) ( ) 2 2,2 q mvAqnvA =<=<
( )2 22 22 q q Aqq << vô lý.
Vậy A có ước nguyên tố lớn hơn 2q .
Bài toán 25. (Rumani 2010) Cho , an là các số nguyên dương sao cho tất cả các ước
nguyên tố của a đều lớn hơn n . Chứng minh rằng ( )( ) ( ) 21 11....1 n aaa chia
hết cho !n Phân tích bài toán.
Ta sẽ chỉ ra rằng nếu p là một ước nguyên tố của n thì
( ) ( )( ) ( ) ( ) 21 !11....1 n pp vnvaaa≤−−−
Lời giải.
Vì tất cả các ước nguyên tố của a đều lớn hơn n nên ( ) ,1an = . Gọi p là ước
nguyên tố của !n . Khi đó ( ) ,1ap =
Áp dụng Định lý Fecmat nhỏ, ta có ( ) 1 1mod p ap ≡ ( ) ( ) 1 1mod kp ap ≡ với 1 k ∀≥
280
() 111 ! k p iii iii
nnn vn ppp
∞∞ ===
11 11 1 1 1 !.. 11 1 11
≤=≤= () () 1 !1 1 p n vn p ≤
pp nn pn vnnn pp pp =
≥=≥
( ) () 1 1
−≤ =
nn pp
Từ ( )1 và ( )2 ta suy ra ( ) ( )( ) ( ) ( ) 21 !11....1 n pp vnvaaa≤−−− (đpcm)
Bài toán 26. Tìm tất cả các số nguyên dương n sao cho ( ) 2 1! n chia hết cho ( )! n n .
Trước hết ta thử một vài giá trị ban đầu của n và nhận thấy 1 n = thỏa mãn, 2,3,4 n = không thỏa mãn bài toán. Đồng thời ta nhận thấy nếu np = là một số
Lời giải.
Trước tiên ta chứng minh rằng 4 n = và np = là số nguyên tố thì ( ) ( ) 2 1!! n nn / ⋮
Thật vậy o Nếu 4 n = thì ( ) ( ) ( ) 4 2 222 4!44!121141vvv==>=− , do đó ( ) ( ) 2 1!! n nn ⋮ o Nếu np = là một số nguyên tố thì ( ) ( ) ( ) ( ) 2 !.!11! p ppp vppvpppvp ==>−=− do đó ( ) ( ) 2 1!! n nn / ⋮
Tiếp theo, ta chứng minh rằng khi 4 n ≠ và n không phải là số nguyên tố thì ( ) ( ) 2 1!! n nn ⋮ . Thật vậy
Gọi p là một ước nguyên tố bất kì của n
Ta sẽ chứng minh ( ) ( ) 2 !1! pp nvnvn≤−
Đặt d npk = , với ( ) * ,,,1 kdkd∈= ℕ
( ) ( ) 2 !1! pp nvnvn≤− ( ) ( ) ( ) ( ) 222 !!!2 pppp nvnvnvnvnd ⇔≤−=−
Theo Định lý Lengendre, suy ra 281
Ta cần chứng minh ( ) ( ) 1 1...2* d kppd +++≥
Ta chứng minh ( )* bằng quy nạp theo d
o Nếu 0 d = thì ( )* đúng
o Nếu 1 d = , vì d npk = và n là số nguyên tố nên 2 k ≥ , BĐT (*) đúng.
o Nếu 2 d = ta cần chứng minh ( ) 14kp+≥
2 2242pnkk = =≠ ≥ do đó BĐT đúng
3 p ≥ thì BĐT luôn đúng
o Nếu 3 d ≥ ta có ( ) 11 1...12...2212 ddd kppd +++≥+++=−≥
Do đó ( )* được chứng minh
Vậy 4 n ≠ và n không phải là số nguyên tố là tất cả các giá trị cần tìm thỏa mãn yêu
cầu bài toán.
Ngoài việc Bổ đề số mũ đúng được vận dụng trực tiếp vào lời giải thì phương pháp này còn được dùng để tìm dạng vô hạn trong các bài toán chia hết. Ta xét một số
bài toán sau
Bài toán 27. Chứng minh rằng tồn tại vô số số nguyên dương n sao cho
( ) 1 2 12015 n vn −=
Lời giải.
Nếu n là số chẵn thì 1 12 n n ⋮ , không thỏa mãn với đề bài ( ) 1 2 12015 n vn −= . Vậy
n là số lẻ
Áp dụng Định lý 4 ta có
( ) ( ) ( ) ( ) 1 222211111 n vnvnvnvn−=−+++−−
( ) ( ) 22 2111 vnvn =−++− .
Do đó ( ) ( ) 22 2112016 vnvn−++= .Ta xét các trường hợp sau
TH1: Nếu ( ) ( )1mod4,12mod4nn≡+≡ thì ( ) 2 11 vn += , nên ( ) 2 21201612015 vn −=−= , vô lí.
TH2: Nếu ( ) ( )3mod4,12mod4nn≡−≡ thì ( ) 2 11 vn −= , ta có ( ) 2 212016 vn ++= ( ) 2 12014 vn += . Khi đó 20142014 1221nknk += =−
với mọi số nguyên dương k . Đặt 21km=+ thì ( ) 20152014 221 nmm =+−∈ℕ là các số thỏa mãn đề bài.
Bài toán 28. Cho k là số nguyên dương. Tìm tất cả các số nguyên dương n thỏa mãn 3|21 kn .
Lời giải.
Ta xét các trường hợp sau:
TH1: Nếu n lẻ thì ( ) 211mod3213 nnk −≡ ⋮ (loại).
TH2: Nếu n chẵn, ta đặt * 2, nmm=∈ ℕ . Khi đó 2141 m n −=−
Áp dụng Định lý 1, ta có ( ) ( ) ( ) ( )
333 3 41411 m vvvmvm −=−+=+ .
Để 341 km ∣ thì ( ) 3 41 m vk −≥ hay ( ) 3 1 vmk≥−
Do đó 1*3, k mss=∈ ℕ
Vậy 1 2.3k ns = với * s ∈ ℕ thỏa mãn yêu cầu bài toán.
Bài toán 29. (Romania 2012) Chứng minh tồn tại vô hạn số nguyên dương n thỏa mãn 21 21 n n + + ⋮ Phân tích bài toán.
Nhận thấy 3 n = thỏa mãn bài toán. Ta tiếp tục thử và thấy 9,27 n = đều thỏa mãn.
Từ đây ta dự đoán 3k n = sẽ là một dạng của n thỏa mãn yêu cầu bài toán.
Lời giải.
Ta sẽ chứng minh 3, n n an=∈ℕ thỏa mãn yêu cầu bài toán.
Áp dụng Định lý 2, ta có ( ) ( ) ( ) 333 21211 na n vvvan +=++=+
Và ( ) () ( ) () 3 21 33 3 2121212 an na n vvvnva + +=+++=+>
2 |21 an n a + .
Lời giải khác.
Ta sẽ chứng minh số nguyên dương
Áp dụng Định lý 2, ta có ( ) ( ) ( ) 3333321 n n vavvn =+−=−
n n a + = thỏa mãn yêu cầu bài toán
3 21 9
1 3. n n am = , với m∈ℕ ( ) 3,1 m = . Ta có ( ) ( ) () 21 333 21121211 an na n vvvann + +=++=+=+>−
Do đó 121 3|21 an n−+ +
Lại áp dụng Định lý 2, ta có ( ) ( ) ( ) 333 2121 na n vvvan +=++=
21 3| n n a + 321 21|21 a n n + ++ . Mà 21 |21 an m + + và ( ) 3,1 m =
21 |21 an n a + +
Bài toán 30. Chứng minh rằng tồn tại vô hạn số tự nhiên n thỏa mãn |31 n n +
Phân tích bài toán.
Với dạng toán này, ta chỉ cần tìm một trong các dạng của n thỏa mãn yêu cầu bài toán
Bằng cách thử một vài trường hợp đầu của n , ta có 225|31 + , đồng thời để ý đến các
điều kiện ,, abp trong Định lý 2 ta thấy 12.52.55|31 nn n+ + . Do đó ta chỉ cần chứng
minh 2.5n n a = là một dạng cần tìm.
Lời giải. Áp dụng Định lý 2, ta có ( ) ( ) ( ) 2.52 555 313151 n n vvvnn +=++=+>
2.5 5|31 n n +
Mặt khác 2.5 2|31 n + 2.5 2.5|31 n n +
Do đó tồn tại vô hạn số dạng 2.5n n a = thỏa mãn |31 na n a +
Bài toán 31. Cho số nguyên 1 k > . Chứng minh rằng tồn tại vô số số nguyên dương n sao cho 123nnnn nk ++++ ∣ .
Lời giải.
Ta xét các trường hợp sau
TH1: k không là lũy thừa của 2
Ta nhận xét với p là ước nguyên tố lẻ bất kì của k thì m np = với * m ∈ ℕ bất kỳ
thỏa mãn yêu cầu bài toán, tức là 12 mnnn pk +++ ⋯ ∣ . Thật vậy
Xét ( )n n ipi +− , với 1,2,3,1ip=…− . Theo Định lý 2, ta có
Do đó ( ) 1 n mn pipi + +− ∣ .
Do 1,2,3,,1ip =…− nên () ( ) 1 2121 n mnn pp + +++− ⋯ ∣ , suy ra ( ) ( ) 1 1211 nn mnnnn ppppk + +++−+++++ ∣ 12 mnnn pk + ++ ⋯ ∣
TH2: k là lũy thừa của 2.
Với p là ước nguyên tố lẻ của 1 k + . Chứng minh tương tự phần trên, ta có m np =
với mọi số nguyên dương m .
Bài toán 32. (Romanian MO 2005) Giải phương trình nghiệm nguyên dương 321 xx y =+
Phân tích bài toán.
Ý tưởng của bài toán là sử dụng Bổ đề LTE trong trường hợp 2 p = và sử dụng nhận xét sau
Với p là một ước nguyên tố của m apk = với * , mk ∈ ℕ thì
o ( ) p va ap ≥
o m pkm α ≥+ với . m β ≥ Kéo theo ( ) p ava α ≥+ với ( ) p va β ≥ hay
apβ ≥ .
Lời giải
Xét hai trường hợp saus
TH1: Nếu x là số lẻ
Áp dụng Bổ đề 1, ta có ( ) ( ) 2231311 x vv−=−= ( ) 2 21 x vy =
1 x = . Khi đó 1 y =
TH2: Nếu x là số chẵn
Áp dụng Định lý 4, Ta có ( ) ( ) ( ) ( ) 2222 3131311 x vvvvx −=−+++−
( ) ( ) 22 312 x vvx −=+
Do đó ( ) ( ) 2222 x vyvx=+
Hay ( ) ( ) ( ) 22222xvyvxxvx +=+ ≤+ (*)
Đến đây ta tìm cách giới hạn lại x
Đặt 2m xk = , với * , mk ∈ ℕ và ( ) ,21k =
+) Với 38mx≥⇔≥ , ta chứng minh quy nạp được 22 m km>+ với mọi ,3mm∈≥ ℕ
Do đó ( ) 2 2 xvx>+ Điều này mâu thuẫn với (*)
+) Vậy 38mx < < mà x là số chẵn nên { }2,4,6 x ∈ . Thử các trường hợp trên ta tìm được ( ) ( ) ( ) ( ),1,1,2,2,4,5xy =
() ( ) ()() 1 n n ppp vipivpvnm +−=+=+ . 285
Sau đây, ta xét thêm một số bài toán có ý tưởng tương tự
Bài toán 33. (Iran 2008) Chứng minh rằng nếu a là số nguyên dương thỏa mãn
( )41 n a + là số lập phương đúng với mọi n thì 1 a = .
Phân tích bài toán.
Nhận xét: Số ( )41 n a + là số lập phương đúng với mọi n nếu với mọi số nguyên tố p
ta đều có ( ) ( ) ( ) ( ) 2213 41 n ppp n vvvaa + +=+ ⋮
Nếu a là số chẵn dễ dàng chọn 2 p =
Nếu a là số lẻ, chọn 2 p ≠ , khi đó ( ) ( ) ( ) ( ) ( ) 1 2211 4 p nnn ppp vvvavaa =+= + ++
Lời giải.
Ta xét các trường hợp sau
TH1: Nếu a chẵn, chọn 1 n = . Ta có ( )841 a + nên ( )41 a + không thể là số lập phương đúng.
TH2: Nếu a lẻ và 1 a > thì ( ) 2 12mod4 a +≡ và 2 12 a +> nên 2 12,1 s as+≠>
Do đó 2 1 a + có ước nguyên tố lẻ. Gọi p là số nguyên tố lẻ là ước của 2 1 a + . Áp dụng bổ đề LTE, ta có ( ) ( ) ( ) ( ) 2221111 p pppp vavavpva+=++=++ .
Do ( ) 2 41 p a + và ( ) 2 41 a + đều là các số lập phương đúng (ứng với 2 np = và
2 n = ) nên ( ) 2 13 p p va + ⋮ và ( ) 2 13 p va + ⋮ , điều này dẫn đến ( ) ( ) 22111 p pp vava+≠++
Trong trường hợp này, ta không có số a thỏa mãn.
TH3: 1 a = , ta kiểm tra được rằng ( ) 418 n a += là số lập phương đúng với mọi n .
Vậy nếu a là số nguyên dương thỏa mãn ( )41 n a + là số lập phương đúng với mọi n
thì 1 a =
Bài toán 34. Chứng minh rằng với mỗi số tự nhiên n , tồn tại số nguyên m sao cho 7231 |4 nmmm++−
Lời giải.
Xét 1 3.7n m = . Ta sẽ chứng minh 1113.73.73.7 23417 nnn n ++− ⋮ .
1111111 373737773.737 23418134 nnnnnnn ++−=−++
Ta có ( ) ( )
Áp dụng Định lý 1, ta có ( ) () ( ) 11 771 777 81817 nn n vvvn −=−+= ,
( ) () ( )
113737 1 777 348343.7 nn n vvvn +=++= .
Do đó 1113.73.73.7 23417 nnn n ++− ⋮ .
Bài toán 35. Romanian Junior Balkan TST 2008 Cho số nguyên tố 3 p ≠ và các số nguyên , ab thỏa mãn | pab + và 233 | pab + . Chứng minh rằng 2 | pab + hoặc 333 | pab +
Lời giải, Do | pab + , nên nếu | pa thì | pb và khi đó 333 | pab +
Nếu pa thì pb , lại có | pab +
Áp dụng Định lý 2, ta có ( ) ( ) ( ) ( ) ( ) 33 33 3 ppppp vabvabvvabvab +=++ +=+
Mà 233 | pab + nên ( ) 33 2 p vab+≥ , kéo theo ( ) 2 p vab+≥ , hay 2 | pab +
Lời giải khác. Do | pab + , nên nếu | pa thì | pb kéo theo 333 | pab + .
Nếu pa thì pb . Ta có
()( ) 2 3 pabab +−
()() ( ) 2 233|3 pabababab +=++− , mà , pab nên
Ta có ( ) ()() ( ) ( ) 2 33 3 pp vabvababab +=++−
()() ( ) () 2 3 ppp vabvababvab =+++−=+ .
(vì ()( ) 2 3 pabab +− nên ()( ) 2 30 p vabab+−= )
Mà 233 | pab + nên ( ) 33 2 p vab+≥ , kéo theo ( ) 2 p vab+≥ , hay 2 | pab + .
Vậy ta có điều phải chứng minh.
Bài toán 36. (Chuyên KHTN, ĐHKHTN, ĐHQG Hà Nội 2016) Cho n là số nguyên dương. Chứng minh rằng tồn tại số nguyên dương m có n chữ số, chỉ gồm các chữ số 1,2,3 và chia hết cho ( )Sm (với ( )Sm là tổng các chữ số của m ).
Lời giải.
Trường hợp 1. n là lũy thừa của 3 Đặt 3t n =
Theo Định lý 1, ta có ( ) () ( ) 3 333 10110132 t vvvt −=−+=+ ( ) 23 3||101 t t+ 3 101 3|| 9
t t
287
Ta chọn 101 111..11 9 n m == ( n số 1) 101 9 n mn = ⋮
Trường hợp 2. n không là lũy thừa của 3 . Khi đó tồn tại số x sao cho 1 33 xx n + <<
Ta chọn ( ) 1 3x Sm + = . Ta sẽ chỉ ra tồn tại số m có tổng các chữ số là 1 3x+ và 1 3x m + ⋮ .
Thật vậy, o Nếu 2.3x n ≤ , đặt 3x nt =+ , * t ∈ ℕ . Ta chọn 111...1333...3222...2 m = , trong đó gồm t chữ số 1, t chữ số 2 và 3x t chữ số 3
C. PHẦN KẾT LUẬN
1. Rút ra những vấn đề quang trọng của đề tài
Chuyên đề đã đưa ra hệ thống lý thuyết và bài tập liên quan đến bổ đề về mũ đúng giúp học sinh có thể tiếp cận từng bước, theo từng mức độ từ dễ đến khó. Đồng thời trong mỗi bài toán, tác giả đã đưa ra những phân tích, bình luận cụ thể để học sinh có thể thấy được cơ sở và lối tư duy tự nhiên nhất trong lời giải của từng bài toán.
2. Những đề xuất, ý kiến hợp lý
Khi đó ( )( ) 3 102101 9
x t m +− =
Ta có ( )( ) ( ) ( ) ( ) 3 3 333 1021011021013 x x tt vvvx +−=++−=+ ( )( )
= ⋮
Chuyên đề “Vận dụng bổ đề về mũ đúng trong các bài toán Số học” chỉ là một mảng nhỏ trong phân môn Số học đa dạng, phong phú. Tuy nhiên, nếu biết vận dụng kiến thức về bổ đề LTE, thì việc giải quyết một số bài toán Số học sẽ cho lời giải đẹp và đôi khi rất gọn gàng. Chuyên đề này hy vọng sẽ là một tài liệu tốt cho học sinh giỏi, giúp các em tiếp cận với những ứng dụng của Bổ đề LTE, hiểu sâu bản chất vấn đề và tư duy trong giải toán.
x x m + +−
3 1 102101 3 9
o Nếu 2.3x n > . Đặt 2.3x nt =+ , * t ∈ ℕ . Ta chọn 111...1222...2111...1 m =
trong đó theo thứ tự từ trái sang phải gồm 3x t + số 1, 3x t số 2 , t số 1
Tuy đã có nhiều cố gắng nhưng chuyên đề không thể tránh khỏi những sai sót. Nhóm tác giả rất mong nhận được sự góp ý quý báu của các thầy cô để chuyên đề được hoàn thiện hơn. Xin chân thành cảm ơn!
Khi đó ( )( ) 33 10101101 9
xx tt m + ++− =
Ta có ( )( ) ( ) ( ) ( ) 3333 3 33 101011011010110112 xxx x tt tt vvvx + + ++−=+++−=++
= ⋮
xx tt x m + + ++−
( )( ) 33 1 10101101 3 9
Vậy ta có đpcm.
288
289
Tài liệu tham khảo
[1] Amir Hossein Parvardi. Lifting The Exponent Lemma (LTE), 2011.
[2] Văn Phú Quốc. Đột phá đỉnh cao bồi dưỡng học sinh giỏi chuyên đề số học, 2015.
[3] Trường đông miền Nam 2019- Hướng tới kỳ thi VMO 2020.
[4] Các diễn đàn toán học: Diendantoanhoc.net/forum Forum.mathscope.org
Mathlinks.ro.
THẶNG DƯ BÌNH PHƯƠNG
PHẦN A. ĐẶT VẤN ĐỀ
1. Lý do chọn đề tài
Luật tương hỗ là một công cụ sắc bén trong nhiều bài toán số học, đặc biệt trong các kì thi Olympic trên toàn thế giới, nhờ nó mà chúng ta có thể giải các bài toán khó một cách thật dễ dàng và dễ hiểu. Đây là một công cụ được nhiều sách lí thuyết số học nhắc tới, bài viết nhỏ này muốn trình bày một vài bài tập Olympic để minh họa cho công cụ tuyệt vời này.
2. Mục đích và nhiệm vụ nghiên cứu Kí hiệu toán học có thể ví như những chương trình con trong một thuật toán. Kí toán học giúp cho việc diễn đạt vấn đề được ngắn gọn, mạch lạc và dễ hiểu hơn (vấn đề này đã được thể hiện trong lịch sử phát triển của toán học). Chính vì vậy với kí hiệu Legendre và kí hiệu Jacobi liên quan đến khái niệm đồng dư thức trong số học, giúp cho việc trình bày nhiều vấn đề trong số học liên quan đến quan hệ đồng dư một cách ngắn ngọn và dễ hiểu hơn.
Rèn luyện kĩ năng vận dụng khái niệm thặng dư bình phương theo modulo n vào giải toán thông qua các bài thi học sinh giỏi, Olympic.
3. Phương pháp nghiên cứu
Dựa trên dữ liệu các đề thi học sinh giỏi, Olympic trong nước và nước ngoài.
Cơ sở lí luận dựa trên các phương các phương pháp phân tích, tổng hợp, tọa đàm (với học sinh và giáo viên dạy chuyên toán).
4. Giả thuyết khoa học Nếu học sinh được học chuyên sâu theo chuyên đề sẽ phát triển năng lực tư duy Toán học, đặc biệt là có phương pháp để giải quyết được một lớp các bài toán về số học liên quan đến quan hệ đồng dư. Đây là phần khó với học sinh các lớp chuyên toán.
5 . Bố cục Phần A. Đặt vấn đề Phần B. Nội dung
I. Thặng dư bình phương
II. Kí hiệu Jacobi
III. Bài tập áp dụng
IV. Bài tập đề nghị
Phần C. Kết luận và kiến nghị. PHẦN B. NỘI DUNG
Trước hết ta định nghĩa thặng dư bình phương của một số theo modulo n, trong đó n là số nguyên dương lớn hơn 1.
I. Thặng dư bình phương
Định nghĩa 1. Cho số nguyên dương n số nguyên dương a nguyên tố cùng nhau với n được gọi là một thặng dư bình phương modulo n (chính phương modulo n) nếu phương trình 2 (mod)xan ≡ có nghiệm, trường hợp trái lại ta nói a không là thặng dư bình phương (không chính phương) modulo n
Nhận xét.
Rõ ràng một số chính phương sẽ là một số chính phương (mod) n với mội số nguyên dương n Nếu a là một thặng dư bình phương modulo n và (mod),abn ≡ thì b cũng là một thặng dư bình phương modulo n
1. Trường hợp modulo np = là số nguyên tố.
Định nghĩa 2. (kí hiệu Legendre): Cho p là một số nguyên tố lẻ, số nguyên a nguyên tố cùng nhau với p. Kí hiệu Legendre a p xác định như sau:
1 = a p nếu a là một thặng dư bình phương modunlo p
=−
1
a p nếu a là bất thặng dư bình phương modunlo p.
Chú ý: Trong tài liệu còn định nghĩa cho trường hợp ⋮ ap là 0 = a p Định lí 1. Cho số nguyên tố lẻ p, số nguyên a nguyên tố cùng nhau với p khi đó:
i) Phương trình 2 (mod) ≡ xap hoặc vô nghiệm, hoặc có đúng 2 nghiệm đồng dư modunlo p
ii) Trong số các số 1,2, ...,1 p có chính xác số thặng dư bình phương và số bất thặng dư bình phương theo modulo p cùng là 1 2 p
Chứng minh.
Giả sử a là một thặng dư bình phương mod. p Ta thấy rằng nếu b là một trong các số thuộc { }1,2,...,1 p thì 22()(mod).≡− bpbp Do đó phương trình 2 (mod) ≡ xap có đúng 2 nghiệm mà bình phương có cùng thặng dư a theo modulo p Đặt { } { } 1 1,2,...,1,1,2,...,(1)/2.SpSp =−=− Với mỗi 1, iS ∈ gọi ir là số (duy nhất) mà 2 (mod). i irp ≡ Ta thấy rằng ijrr ≠ nếu ij ≠ Thật vây, nếu 22 0(mod)()()0(mod). ij rrijpijijp = −≡ −+≡ Nhưng điềunàykhôngxảyravì
, ijij −+ điều không chia hết cho p do 1,. ijijp≤−<+< Vậy { } 2 1 |(mod),(1)/2.ii ArSripiSp =∈≡∈=− Ta chứng minh A là tập tất các cố chính
phương modulo p trongS.Thậtvậy,mỗisố thuộc A đềulàsố chínhphương (mod). p
Ngượclại,giả sử aS ∈ saocho 2 (mod)akp ≡ với .kS ∈ Nếu 1kS ∈ thì . k arA =∈ Nếu
1kS ≠ thì 1bpkS =−∈ và ta có 2 (mod)abp ≡ (do 22()(mod) kpkp ≡− ) do đó . b arA =∈
Định lý được chứng minh.
p p
Như vây, phương trình đồng dư 2 1(mod)≡− xp có nghiệm khi và chỉ khi 2 = p hoặc 1(mod4) ≡ p
Phương trình đồng dư 2 1(mod)≡− xp vô nghiệm khi và chỉ khi 1 (mod4). p ≡−
= p r
Hệ quả. Vớimỗisố nguyêntố lẻ p ,tacó 1 1 0. =
r p
Định lí 2. (Tiêu chuẩn Euler) Với p là số nguyên tố lẻ và số nguyên a thỏa mãn
(,)1, = ap khi đó
1
Định lí 3. (Kí hiệu của Lagrange có tính chất nhân) Giả sử p là số nguyên tố lẻ, a và b là những số nguyên. Khi đó:
=
2 (mod). ≡
p a ap p
Chứng minh. Xét trường hợp 1. = a p Khi đó đồng dư 2 (mod) ≡ xap có nghiệm
0 = xx Theo định lý Fermat bé, ta có
() 11 1 2 22 00 ()1(mod). ≡≡≡ pp p axxp
Do đó
1
Trường hợp 1. =− a p
= abab ppp
Chứng minh. i) Nếu (mod),abp ≡ thì phương trình 2 (mod)xap ≡ có nghiệm khi và chỉ
=
iii) Nếu a hoặc b chia hết cho p thì khẳng định của định lý hiển nhiên đúng.
Bây giờ ta xét trường hợp cả a và b không chia hết cho p
2 (mod). ≡
p a ap p
Khi đó phương trình đồng dư 2 (mod) ≡ xap vô nghiệm.
Với mỗi { }1,2,...,1∈− ip tồn tại duy nhất ,11 ≤≤−jjp sao cho .(mod), ≡ ijap rõ ràng
≠ ij (vì phương trình 2 (mod) ≡ xap vô nghiệm), nên các số thuộc tập { }1,2,...,1 p
phân thành 1 2 p cặp, sao cho tích hai số trong mỗi cặp đều đồng dư với a theo modulo p.
Từ đó 1 2 (1)!(mod), −≡ p pap mặt khác theo định lý Wilson thì (1)!1(mod). −≡− pp
≡≡≡
ppp abab ababp ppp
Vì ab p và , ab pp chỉ nhận các giá trị 1 hoặc -1, mà 2 ≠ p nên không thể xảy ra
= abab ppp
iv) Vì 1, =±
.1.
p a ap p
Do đó 1 2 (mod).
nh lí 4. (Bổ đề Gauss) Giả sử p là số nguyên tố lẻ, a là số nguyên dương không chia
hết cho p. Nếu trong các số thặng dư dương bé nhất của các số nguyên 1 ,2,3,..., 2 p aaaa
có s thặng dư lớn hơn 2 p thì (1). s a p =−
Chứng minh. Trong số các thặng dư dương bé nhất của các số nguyên 1 ,2,3,...,, 2 p aaaa
giả sử 12,,..., s uuu là các thặng dư lớn hơn 2 p và 12,,..., tvvv là các thặng dư bé hơn 2 p
Vì ( ) ,1jap = với mọi 1 1, 2 p j ≤≤ nên mọi , ijuv đều khác 0, tức là thuộc tập hợp
{ } 1,2,...,1. p Ta sẽ chứng minh tập hợp
{} 1212 1 ,,...,;,,...,1,2,...,. 2 st p pupupuvvv −−−=
Thật vậy, rõ ràng không có hai số u nào, cũng như không có hai số tv nào đồng dư với nhau theo modulo, (nếu trái lại ta sẽ hai số tự nhiên phân biệt m, n thuộc tập 1 1,2,..., 2 p
thỏa mãn (mod), manapmn ≡ = mâu thuẫn) đồng thời các số ipu cũng không đồng dư với số v nào theo modulo. Như vậy ta có
1212 1 ()()...().....!(mod). 2 st p pupupuvvvp −−−=
Mặt khác, 1212 ,,...,;,,...,st pupupuvvv là các thặng dư dương bé nhất của 1 ,2,3,..., 2 p aaaa theo modulo p, nên 1 2 1212 1 ,,...,.,,...,.!(mod). 2 p st p uuuvvvap ≡
Như vậy, là số 2 là một thặng dư bình phương modulo p khi và chỉ khi 1(mod8). p ≡±
Chứng minh.
Áp dụng bổ đề Gauss, ta cần đếm số các thặng dư dương bé nhất lơn hơn 2 p của dãy số 1 1.2,2.2,...,.2 2 p
vì các số này bé hơn p nên chúng trùng với các thặng dư dương bé nhất của chúng. Như
vậy, ta chỉ cần đếm các số của dãy lớn hơn . 2 p
Số chẵn 2, j với 1 1, 2jp≤≤ không vượt quá 2 p khi . 4 jp ≤
Vậy số các số trong dãy lớn hơn 2 p là 1 . 24 pp s =−
Như vậy, ta có
=−
1 24 2 (1). pp p
Bằng cách xét các trường hợp 1(mod4), p ≡± ta được 2 11 (mod2). 248 ppp −≡
Khi đó được lí được suy ra từ bổ đề Gauss.
Định lí 6. (Luật thuận nghịch Gauss) Với mỗi cặp các số nguyên tố lẻ khác nhau (,) pq . Ta có:
11 . 22 (-1) . pq pq qp =
Chứng minh. Trước hết ta chứng minh bổ đề: Giả sử p là số nguyên tố lẻ và a là số
−≡ −≡ ≡−
Như vậy ta có 1 11 2 22 11 (1)!.!(mod)(1)1()(1)(). 22 p pp s s s ppapamodpamodp
Áp dụng tiêu chuẩn Euler ta được (1)(mod), s a p p ≡− mà a p chỉ nhận giá trị 1 hay
nguyên không chia hết cho p. Khi đó (1), k a p =− trong đó
(Ở đây kí hiệu x là phần nguyên của só thực x ).
Chứng minh bổ đề.
, nên (1). s a p
=−
p j
ja k p =
=
1 2 1
Xét các thặng dư dương bé nhất của các số trong dãy 1 ,2,3,...,, 2 p aaaa theo (mod). p
Gọi 12,,..., s uuu là các thặng dư lớn hơn 2 p và 12,,..., tvvv là các thặng dư bé hơn . 2 p
Theo thuật toán chia Euclide ta có (mod), ja jajap p ≡− với 01. ja jap p ≤−≤−
=≤<≤≤=≤≤≤≤
ja japrs p ====
=++
pp st ij jjij
(1)
Vì 12 , SSS =+ nên ta có
xy SSqxpy pq== ==
Mặt khác, từ chứng minh định lí 4, ta có {} 1212 1 ,,...,;,,...,1,2,...,. 2 st p pupupuvvv −−−=
Định lí 7. Cho 2 p > là một số nguyên tố. Khi đó
i)
−=−+
Trừ từng vế các đẳng thức (1) và (2) ta được (1)/2(1)/2 111 (1)2. pp s i jji
ja ajpsr p ===
3 (mod8) ≡ p
3 là thặng dư bình phương mod p nếu và chỉ nếu 1 (mod12), p ≡± (với 3 p ≠ )
Do a , p lẻ và (1)2 2 1
= nên (3) suy ra (1)/2 2 1
iv) 3 là thặng dư bình phương mod p nếu và chỉ nếu 1 (mod6) ≡ p
p j =
Suy ra (1)/2 1 (mod2). p j
ja s p =
có bổ đề được chứng minh.
Trở lại bài toán.
≡−≡−
pja as p =
1 0(1)(mod2). 8 p j
v) 5 là thặng dư bình phương mod p nếu và chỉ nếu 1 (mod5), p ≡± (với 5 p ≠ )
≡=
ja sk p =
Do đó (1)/2 1 (mod2). p j
vi) 7 là thặng dư bình phương mod p nếu và chỉ nếu 1,3,9 (mod28), p ≡±±± (với 7 p ≠ )
+
vi) 1 4 2 (1). p p =−
Chứng minh.
Theo bổ đề định lí sẽ được chứng minh nếu ta chứng minh được (1)/2(1)/2 11
11 22 pq jj
+=
jqjppq pq==
Khi đó tập S có 11 . 22 pq phần tử và không có phần tử (,)xyS ⊂ thỏa mãn ,qxpy =
vì p, q là hai số nguyên tố phân biệt.
i) Theo định lí 5, ta có 2 1 8 2 (1). p p
=−
Do đó 2 là thặng dư bình phương modulo2 khi
và chỉ khi 2 1 2 8 (1)11611(mod8). p pkp −=⇔=+⇔=±
ii) 2 là thặng dư bình phương mod p nếu và chỉ nếu
1
2 1
- Nếu 83,pk=± thì chỉ thỏa mãn khi 83pk=+ thỏa mãn (1).
298
Vậy 2 là thặng dư bình phương mod p nếu và chỉ nếu 1 (mod8) ≡ p hoặc
3 (mod8) ≡ p
iii) Giả sử số nguyên tố lẻ p thỏa mãn (3,)1. p = Goi s là số các thặng dư dương nhỏ nhất
theo modulo p của số 1 3,2.3,...,.3 2 p mà lớn hơn 2 p . Khi đó theo bổ dề Gauss ta có
3 (1). s p =−
Khi chia các số của dãy số trên cho p được số dư lớn hơn 2 p là các số 3 j
thuộc khoảng ,. 2 p p
Do đó ta cần đánh giá số cách chọn số j thỏa mãn 3. 2 pjp <<
Đặt 12, kr + với { } * 1,5,7,11,.rk ∈∈ ℕ Ta có 324. 263 prr jpkjk <<⇔+<<+ (1)
Tính chẵn lẻ của số các số j thỏa mãn (1) cùng tính chẵn lẻ của số các số j thỏa mãn ,(2) 63 rr j <<
Với 1, r = có 0 số j thỏa mãn (2), suy ra 0, s = nên theo bổ đề Gaus 3 là thặng dư bình phương modulo p.
Với Với 5 r = có 1 số j thỏa mãn (2), suy ra 1, s = nên theo bổ đề Gaus 3 không là
thặng dư bình phương modulo p
Với Với 7 r = có 1 số j thỏa mãn (2), suy ra 1, s = nên theo bổ đề Gaus 3 không là thặng dư bình phương modulo p
Với Với 11 r = có 2 số j thỏa mãn (2), suy ra 2, s = nên theo bổ đề Gaus 3 là thặng dư bình phương modulo p.
Vậy 3 là thặng dư bình phương mod p nếu và chỉ nếu 1(mod12), p ≡± (với 3 p ≠ ).
iv) Theo luật thuận tương hỗ, ta có 3 là thặng dư bình phương mod p nếu và chỉ nếu
11311
Do đó 5 là số chính phương modulo p khi và chỉ khi p là số chính phương mod5, ulo khi
và chỉ khi (51)/22 1(mod5)1(mod5)51. 5 ppppk
vi)
7 p pppp
1(mod4),1(mod7)1(mod28)popp ≡≡⇔≡
1(mod4),2(mod7)9(mod28)popp ≡≡⇔≡
1(mod4),4(mod7)253(mod28)popp ≡≡⇔≡≡−
vii) Nếu 1(mod4). p ≡−
Theo luật tương hỗ, ta có 7 là số chính phương (mod) p khi và chỉ khi p không là số chính phương (71)/23 (mod7)1(mod)3,5,6(mod7). 7 p pppp
1(mod4),1(mod7)3(mod28)popp ≡−≡⇔≡
1(mod4),5(mod7)199(mod28)popp ≡−≡⇔≡≡−
1(mod4),6(mod7)1(mod28).popp ≡−≡⇔≡−
Tóm lại 7 là số chính phương (mod) p khi và chỉ khi 1,3,9(mod28). p ≡±±±
+
do đó 1 4 2 (1). p p
Nhận xét. Rõ ràng một số chính phương sẽ là số chính phương (mod) p với mọi số nguyên tố p. Tuy nhiên một số không chính phương có thể là một số chính phương theo một (mod) p nào đó. Chẳng hạn 6 là số chính phương (mod19), vì 2 56(mod19). ≡
==⇔=−⇔=−−⇔=−
ppp p pp pppp
1 22223133 11(1)(1).(1)(1)(1) 33
Đặt 6,pkr =+ với { } * 1,5,.rk∈∈ℕ
Với 61pk=+ thì (1) thỏa mãn.
Với 65pk=+ thì (1) không thỏa mãn.
Vậy 3 là thặng dư bình phương mod p nếu và chỉ nếu 1(mod6) ≡ p .
Vậy có tồn tại số không chính phương nhưng là số chính phương (mod) p với mọi số nguyên tố p. Định lí sau đây sẽ chứng tỏ không tồn tại số như vậy.
Định lí 8. Cho a là số không chính phương. Khi đó tồn tại số nguyên tố lẻ p sao cho a không chính phương (mod) p
Chứng minh. Giả sử 2 ,abc = ở đó 12...... mcqqq = với 12... mqqqq <<< là các số nguyên
v) Với p là số nguyên tố lẻ khác 5, theo luật tương hỗ ta có 511 . 22 555 (1)1 555
=−⇔=⇔= .
p ppp ppp
tố. Ta sẽ chứng minh tồn tại số nguyên tố lẻ p sao cho 1. a p =−
Từ 2 , ac abc pp = =
nên ta cần chứng minh 1. c p =−
a) Trường hợp 1 2. q > Giả sử 1r là số không chính phương 1 (mod) q và
( ) ,,1,1,,iii rrqim == là các số chính phương (mod). q Theo định lí thặng dư trung hoa
tồn tại số nguyên B thỏa mãn 1(mod4) (1) (mod),1,. ii
B Brqim ≡ ≡=
Đặt 12 4.... mAqqq = Dễ thấy (,)(,4)1,1,, i BqBim === suy ra (,)1. AB = Nên theo định lí
Đi rich lê tồn tại số nguyên tố p dạng 12 4.... m pAkBqqqkB =+=+ và p cũng thỏa mãn (1).
Hay (mod4),(mod) ipBpBq ≡≡ 1(mod4),(mod). ii pprq ≡≡ Vậy thì
nêui r p nêui qq > == −=
1 1 1 1. i ii
2. Trường hợp modulo n là hợp số
Giả sử hợp số n có phân tích tiêu chuẩn * 1 ,, k iii i npp α α = =∈∈℘
ℕ (℘ là tập các số
nguyên tố).
Định lí 9. Cho a là một số nguyên và và hợp số n có phân tích tiêu chuẩn * 1 ,,. k iii i npp α α = =∈∈℘
ℕ Khi đó a là thặng dư bình phương (mod) n khi và chỉ khi a là thặng dư bình phương mod αi ip , với mỗi 1,2,...,. ik =
Chứng minh.
Thành thử
qp im pq
Vì 1(mod4), p ≡ nên theo luật tương hỗ Gauss ta có ,1,. i i
==−
q c pp =
≡=
Đặt 12 8.... mAqqq = Khi đó (,)1. AB = Nên theo định lí Đi rich lê tồn tại số nguyên tố p dạng 12 8.... m pAkBqqqkB =+=+ và p cũng thỏa mãn (2).
tồn tại số nguyên ix ∈ ℕ sao cho 2 (mod),1,. i i i xapik α ≡= Theo định lí thặng dư Trung
Hoa tồn tại số nguyên x sao cho (mod),1,. i i i xxpik α ≡= Suy ra 22(mod),1,. i i i xxpik α ≡= Vậy a là thặng dư bình phương (mod). n
Nhận xét. Từ định lí 9, suy ra xét sự chính phương của một số theo (mod) n quy về xét hợp số n có dạng lũy thừa của một số nguyên tố. Định lí 10. Giả sử 2,1, s ns=> và a là số lẻ. khi đó a là số chính phương (mod) n khi và chỉ khi:
a) 1(mod4), a ≡ nếu 2. s =
qr p im pqq
nên theo luật tương hỗ Gauss, ta có 1,2,. ii ii ====
Vì 1 2 5(mod8)1. q p pp
==−
1
b) 1(mod8), a ≡ nếu 3. s ≥ Chứng minh.
a) Với 2. s= Tức là 4. n = Nếu tồn tại nguyên x∈ℕ sao cho 2 (mod4)1(mod4),xaa ≡ ≡ vì mọi số lẻ 2 1(mod4). xx ≡
Đảo lại, nếu 2 1(mod4)1(mod4).aa ≡ ≡ vậy a là số chính phương (mod). n
b) Với 3. s ≥ Nếu tồn tại x ∈ ℕ sao cho 22(mod)1(mod8),xanax ≡ ≡≡ vì mọi số lẻ 2 1(mod8). xx ≡
Đảo lại. Giả sử 1(mod8), a ≡ tức là 81.at=+ Với
là số chính phương (mod8). Với 3. s > Đặt 3 2. s m =
2 3811(mod8) snaa = = ≡≡
Ta xét tập { }21 1 , m i AT = = ở đó (1)/2. k Tkk=+ Ta có A là HTDĐĐ (mod), m vì nếu
2121(mod)()(221) ij TTmijil ≡ −+− chia hết cho .m Vì m không có ước lẻ nên ij
chia hết cho m Suy ra ,ij = vì 1,. ijm≤≤
Vậy tồn tại k ∈ ℕ sao cho
3
aaaa bppp
12 12 ..., r r
Nh
Legendre.
aa bb ∈−= khi gcd(,)1. ab ≠
≡ −≡ −≡
21 (mod)(21)(mod2)8(21)8(mod) s k Ttmkktkktn
2 2 (41)81(mod)61(41)(mod). ktnatkn −≡+ =+≡−
Vậy a là số chính phương (mod). n
Định lí được minh.
Định lí 11. Gỉa sử , s np = với p là số nguyên tố lẻ. Khi đó a là số chính phương (mod) n khi và chỉ khi a là số chính phương (mod). p
Chứng minh.
Hiển nhiên a là số chính phương (mod) n thì a là số chính phương (mod). p
Giả sử a là số chính phương (mod). p Ta chứng minh bằng quy nạp khẳng định sau: Với mỗi số nguyên dương k tồn tại kx ∈ ℕ sao cho 2 (mod). k k xap ≡ Thật vậy.
Với 1, k = thì khẳng định đúng vì a là số chính phương (mod). p
Giả sử khảng định đúng với 1, k ≥ tức là tồn tại kx ∈ ℕ sao cho 2 , k k xatp =+ với t ∈ℤ
Đặt 1 ., k kk xxhp + =+ với h là số nguyên dương thỏa mãn 2(mod). k hxtp ≡− Số h này tồn tại vì (2,)1. k xp = Vậy 22221 1 2(2)(mod. kkkkk kkkk xxhxppahxtppap + + =++=+++≡
Vậy khẳng định đúng với 1. k + Do đó khẳng định đúng với mọi số nguyên dương k Nói riêng tồn tại s x để 2 (mod). s s xap ≡ hay a là số chính phương (mod). n
Định lí được chứng minh.
Hề quả. Giả sử n là số lẻ. Khi đó a là số chính phương (mod). n Khi và chỉ khi với mọi
ước nguyên tố p của ,n a là số chính phương (mod). p
II. Kí hiệu Jacobi
Định nghĩa 3. Cho 1 b > là số nguyên lẻ, và gọi
cho biết phương trình 2 (mod)xab ≡ có nghiệm hay không (ví dụ ta có kí hiệu Jacobi 2 1, 15 = nhưng phương trình 2 2(mod15) x ≡ lại không có nghiệm).
Định lý 12. Số các thặng dư bình phương mod(0) > n pn bằng
1 21 2 3 n + nếu 2 = p và bằng 1 1 1 2(1) n p p
nếu 2 > p
không
12 αα α = r rbppp là phân tích ra thừa
12
Định lý 14 Với mỗi số nguyên lẻ a , ta có 1 1, a = =−=−=−
2 1 1 1 4 8 2 12 (1),(1)(1). a a a
Định lý 15. (Luật tương hỗ) Với hai số nguyên lẻ nguyên tố cùng nhau , ab bất kì, ta có (1)(1) 4 .(1). ab ab ba =−
(mod) 2 << p axpp , thì (1) =−
Chú ý: Các định lí ở phần này chủ yếu được nêu ra không chứng minh, bạn đọc quan tâm có thể tham khảo chứng minh ở phần tài liệu tham khảo của bài viết này. Cũng nên
chú ý rằng việc chứng minh lại chúng rất bổ ích, cho chúng ta thấy chặng đường phát triển của phương pháp, cũng là cách học xây dựng phương pháp mới.
III. Bài tập áp dụng
Phần tiếp theo này chúng ta sẽ khảo sát một vài ứng dụng quen thuộc của thặng dư bình phương trong bài toán các thi Olympic. Ứng dụng đầu tiên mà chúng ta nghĩ tới chắc chắn là về các bài toán về đồng dư, chia hết. Dưới đây là tập hợp các bài toán đó.
1. Các bài toán chia hết, ước số.
Bài 1. Chứng minh rằng tồn tại số tự nhiên 1<+ap không là thặng dư bình phương mod p.
Giải. Xét số tự nhiên a nhỏ nhất không là thặng dư bình phương mod p và đặt
1 bp a =+ . Suy ra 0 <−<abpa nên theo định nghĩa của a thì abp phải là một thặng dư bình phương mod p . Do đó
1. ===− abpabb pppp
Vậy b không thặng dư bình phương mod p , suy ra 11. ≤<+ <+ p abap a Đpcm.
Bài 2. (Polish Olympiad) Cho 5 ≥ p là một số nguyên tố. Chứng minh rằng tồn tại
∈ x Z sao cho 2 |3 −+pxx khi và chỉ khi tồn tại ∈ y Z sao cho 2 |25 −+pyy
Giải.
Ta có 2 |3 −+pxx tương đương với 22 |4(3)(21)11 −+=−+pxxx , từ đó ta thấy rằng
tồn tại x khi và chỉ khi 11 là thặng dư bình phương mod p . Tương tự 2 |25 −+pyy
tương đương với 22 |4(25)(21)99, pyyy−+=−+ nên tồn tại y khi và chỉ khi 99 là thặng dư bình phương mod p . Mặt khác ta thấy:
2 9911311
Bài toán được chứng minh.
Bài 3. Chứng minh rằng
+ x y không phải là một số nguyên, với mỗi số nguyên
Giải. Giả sử 2 2 1 5 + x y là số nguyên. Ta phân biệt hai trường hợp:
=−=−=−
vô lí.
Trường hợp 1. y chẵn thì 2 –5 y có dạng 43 + k do đó nó có ít nhất một ước nguyên tố p có dạng 43 + m , và ta có 2 1(mod)−≡ xp , tức là 1 thặng dư bình phương theo mod, p mặt khác theo tiêu chuẩn Euler ta có 1 21 2 1 (1)(1)1, p m p +
Trường hợp 2. y lẻ, thì 2 5 y chia hết cho 4, nên 2 1+ x chia hết cho 4, vô lí.
Bài toán được chứng minh.
Bài 4. Chứng minh rằng tồn tại vô số số nguyên tố dạng:
i) 41 + k ;
ii) 109 + k .
Giải. i) Cách 2. Xét số 2 12 (2...)1. nAppp=+ Rõ ràng mỗi ước nguyên tố của A phải là số lẻ. Xét một ước nguyên tố p của A.
Suy ra
2 12 1 (2...)10(mod)1. n pppp p +≡ =
Suy ra 1 2 1 (1)121(mod4) 2
p p p −= ≡ ⋮ 41,pk =+ nhưng nó lại không chia hết
cho số nào trong các số 12,,..., nppp . Vô lí.
Tỏng quát: Cho , ab là các số nguyên khác không và (,)1. ab = Chứng minh mọi ước nguyên tối của 22ab + phải có dạng 41. k +
Lời giải. Gọi p là một ước tố lẻ của 22ab + , suy ra 2 2222 (mod)1 b abpabp p + ≡− =
⋮ (vì (,)1(,)(.)1 abapbp = == ). (1)
Mặt khác, theo định lý 2, thì 111 2 21222 ()(1).(1) ≡−≡−≡−
ppp p b bb p (mod p)
Nếu p = 4k + 3 2 1 ≡− b p (mod p), mâu thuẫn với (1)
Vậy p có dạng 41. k + ii) Tương tự, xét với số 2 12 5(2...)1 =− nBppp .
Bài 5. Chứng minh rằng với ∈ ℕ n , mỗi ước số q của số 42 1−+nn có dạng 121 + k
Giải. Chú ý hai đẳng thức sau: 42222 1(1),−+=−+ nnnn và 42222 1(1)3−+=+− nnnn
Vì 42 1−+nn lẻ, nên mội ước nguyên tố của của 42 1−+nn đều lẻ nà nguyên tố cùng nhau với n.
Xét p là một ước nguyên tố bất kì của 42 1 nn−+ 222 (1)(mod) nnp −≡− 1 11(mod4). p p = ≡
Từ 42222222 3 1(1)30(mod)(1)3(mod)1,nnpnnpnnp p
−+ +−≡ +≡ = ⋮ suy ra 1(mod12) p ≡± (theo các các kết quả mở rộng). Do đó 1(mod12) ≡ p , đpcm.
Bài 6. Chứng minh rằng với ∈ ℕ n , mỗi ước số p của số 84 1−+nn có dạng 241, + k
∈ ℕ k .
Giải. Theo bài trên, với mỗi ước số nguyên tố p của 84 1−+nn thì 1(mod12) ≡ p , suy ra p đồng dư với 1, hoặc 13 theo mod24 . Nhưng ta lại thấy: 8442232 1(1)2() −+=++−+ nnnnnn
Suy ra 2 là thặng dư bình phương theo mod p , suy ra 1(mod8). p ≡±
Do đó không thể có 13(mod24) ≡ p . Nên 1(mod24). p ≡ Đpcm.
Bài 7. (Korea TST 2000) Giả sử , mn là các số nguyên dương sao cho (51)51ϕ −=− mn , chứng minh rằng gcd(,)1 > mn .
Giải. Giả sử gcd(,)1 = mn . Viết 1 1 512...... α α α −= k m kpp (1)
là phân tích ra thừa số nguyên tố của 51 m , ở đây 2 > ip với 1,2,.., = ik . Từ điều kiện
bài ra
Suy ra
1 1 1 1 1 1 51(51)2.....(1)...(1). α α α ϕ −=−=−− k nm k k pppp (2)
1 1 1 1 1 2.....|51 α α α k n kpp , mà gcd(,) gcd(51,51)514 −−=−= mnmn nên 1 α = i với
mỗi 1,2,.., = ik , và 2 α = . Do 3 2|51 x với mỗi x chẵn, nên m là số lẻ, 2'1=+mm . Do '2 |5.(5)1 m ip với mỗi 1,2,.., = ik , nên 5 là thặng dư bình phương modulo ip , suy ra 1(mod5) ip ≡± . Ngoài ra, từ (2) suy ra không tồn tại ip sao cho 1(mod5) ≡ ip , vậy
307
1(mod5)≡− ip với mọi 1,2,.., = ik . Lấy hai vế của (1) theo mod5, suy ra
(1)1(mod5) −≡ k , do đó k là số chẵn. Lấy (2) theo mod 5, suy ra 1 (2)1(mod5) + −≡ k , do
đó 3(mod4) ≡ k , mâu thuẫn với chứng minh trước.
Bài 8. (Bulgaria MO 1998) Cho m, n là các số tự nhiên sao cho:
(3)1 3 ++ = n Am m
là một số nguyên. Chứng minh rằng A là số lẻ.
Giải. Tất cả các biến xét dưới đây đều nhận giá trị nguyên.
Giả sử ngược lại A là số nguyên chẵn, khi đó
(3)16(1) ++= n mkm
Suy ra m là số nguyên chẵn ( xét (1) theo mod 2).
Từ (1) suy ra 0(3)11(mod3) nn mmn≡++≡+ là số lẻ (vì số chính phương chia cho 3 dư 0 hoặc 1), và = m 32. t +
Đặt 1 2. mm α = , với 1 α ≥ và 1m là số lẻ. Khi đó 2α chia hết 31, n + suy ra 2 α ≤ (vì n lẻ, nên 1 2 313.914(mod8) n n +=+≡ ).
Nếu 2, α = thì 11 4.321(mod3)1(mod3). mmtm ==+≡− ≡− do vậy tồn tại số nguyên tố p là ước của 1m sao cho 1(mod3) p ≡− . (2)
Đặt 21,nk=+ do 2121 0(3)13131(mod)310(mod) nnkk Ammp ++ ∈ ≡++≡+≡+ +≡ ℤ
( )2 1 33(mod), k p + ≡− suy ra 3 là thặng dư bình phương (mod)1(mod6) pp ≡ ,
mâu thuẫn với (2).
Nên 1 α = . Khi đó 1 2 mm = và từ (1) suy ra ()21 1111 2311224 k mkmmm + ++= ⋮
chẵn. Mâu thuẫn.
Vậy m lẻ thì (3)1 n m ++ và 3m lẻ, suy ra A là số lẻ. Đpcm.
Bài 9. Tìm tất cả các số nguyên dương n thoã mãn 21|31 nn .
Giải. Giả sử tồn tại 1 > n thoã mãn điều kiện bài toán, thì 21 n không phải là bội số của 3, suy ra n lẻ. Do đó 28(mod12) ≡ n . Chúng ta thấy rằng mỗi số nguyên tố p khác 3 thì
có dạng 121,125 ±±kk và do 217(mod12) −≡ n nên 21 n có ít nhất một ước số nguyên
308
tố p dạng 125 ± k . Ta có 3 1 = p suy ra 1 2 (1) 3 =− p p ( từ luật tương hỗ). Nhưng lại
có 2 (1) 33 ± ==−± p . Suy ra (1)(1) ±=−± , vô lí.
Thử với 1 = n ta được đây là nghiệm duy nhất của bài toán.
Bài 10. (AMM, Romania TST 2008) Với các số nguyên dương lẻ , mn lớn hơn 1, thì
21 m không chia hết 31 n
Giải. Giả sử ngược lại, tồn tại các số tự nhiên lẻ , mn lớn hơn 1, sao cho 21=− m M 31 n
Ta có: 1 2 2 13(mod)3(3)(mod). n n MM + ≡⇔≡
Suy ra 3 là thặng dư bình phương mod M
Suy ra: 3 1 = M (ở đây ta sử dụng kí hiệu Jacobi).
Mặt khác do: 1(mod3) ≡ M
Nên: 1. 3 M =
Do đó theo luật thuận nghịch Gauss ta có
Giải. Do (),()[] PxQxZx ∈ và nguyên tố cùng nhau trên [], x ℚ nên tồn tại các đa thức (),()[] uxvxZx ∈ và số nguyên dương d sao cho ( ) ().()().()(),(),. PxuxQxvxdPnQndn += ≤∀∈ ℤ
Giả sử ()Qx không là đa thức hằng số. Khi đó dãy số ( )() n Qn ∈ℤ không bị chặn, mà ta lại có * (), Qnn∈∀∈ℕℤ nên deg() Q chẵn và hệ số của số hạng bậc cao nhất của ()Qx
dương. Từ đó ta có thể chọn m ∈ ℤ sao cho { }max(1),(2),...,() () 213. PPPd Qm M =−≥
Theo giả thiết, ta lại có ()1()1 2|3,(1) QmPm M =
nên (,2)(,3)1. MM== Gọi , ab lần lượt là cấp của 2,3 theo (mod). M
Từ (1) ta có ( ) |(),|()(,)(),(). aQmbPmabQmPmd ≤≤
Đặt 00(,), sabxy = ∃∈ ℤ sao cho 00. saxby =+ Do 1,0sdkkd≤≤ ∃∈≤< ℕ sao cho sdk =− Ta sẽ chứng minh rằng tồn tại các số nguyên , xy sao cho 0. maxbyd≤++≤ Đặt 0,0xtxyty == (số nguyên t xác định sau), ta có 00 00()0(). mdm maxbydmtaxbydmtdkdtdkdk ≤++≤⇔≤++⇔≤+−≤⇔≤≤
Do 1, dmmd dkdkdk −=> nên tồn tại số nguyên t từ đó tòn tại các sso nguyên , xy
Với , x ∈ ℤ ta có
()()()() ()()(mod)221(mod)|21|31. QmaxQmQmaxPmaxQmaxQmaMM + ++ +≡ ≡≡
Tương tự, ta có
131 . 22 33 .(1)1
M M MM (do 2(mod4) ≡ M )
Điều này là mâu thuẫn. Vậy giả sử là sai, bài toán được chứng minh.
Nhận xét. Từ Bài Toán 10, ta có kết quả tổng quát sau cho Bài Toán 9, 10:
Bài toán tổng tuát: Chứng minh rằng nếu 1 > m và 0 > n có cùng tính chẵn lẻ, thì 21 m không chia hết 31 n .
Giải: Nếu , mn cùng lẻ, thì ta có Bài Toán 10, mà chúng ta vừa mới giải quyết.
Nếu , mn cùng chẵn, thì 21 m chia hết cho 3, bài toán được chứng minh.
Bài 11. (Shortlist 2011) Cho (),()PxQx là hai đa thức hệ số nguyên, nguyên tố cùng nhau trên []. x ℚ Giả sử rằng với mọi n ∈ ℤ thì (),() PnQn nguyên dương và () 21 Qn chia hết () 31. Pn Chứng minh rằng ()Qx là đa thức hằng.
()() ()()(mod)331(mod) PmaxbyPmax PmaxbyPmaxbM +++ ++≡+ ≡≡
{ }max(0),(1),...,() () () 3131221, PPPd Pmaxby Qm M M ++ ≤−≤−≤−=− điều này vô lí.
Vậy ()Qx là hằng số.
Bài 12. Cho n là số nguyên dương lẻ và m là một ước dương lẻ của 31. n + Chứng minh rằng 1 m chia hết cho 3.
Giải. Gọi p là một ước nguyen tố lẻ của 31, n + suy ra 3. p ≠ Ta có 1 310(mod)33(mod)(1) nnpkp + +≡ =−
Vì n lẻ, nên 1 n + chẵn, từ đó (1) suy ra 3 là số chính phương modulo p, hay 3 1(2) p =
Theo tiêu chuẩn Euler ta có (1)/2
3133 (1).(3) p pppp
ppp p p pp
=−
1 3(1).(5)
=−
33 p
2 1 21 8 1(1)(1)83 p pp =−− = = , mâu thẫn. Vậy 1(mod3)1(mod3)13.pmm ≡ ≡ ⋮
Bài 13. (Tài liệu Bồi dưỡng HSGQG Việt Nam, 2004) Cho 3 p > là số nguyên tố và p có
∏
Giải. Trước hết ta chứng minh mệnh đề sau: Với số nguyên tố 31pn=+ thì 3 là số chính phương mod. ulop Thật vây, vì p nguyên tố nên n phải chẵn 2 nl = suy ra
61.pl=+ Nếu 2,lt = thì 121(1)/26 ptpt =+ −= là số chẵn nên (1) là số chính phương (modp) và 3 là số chính phương (modp). Do đó (3)(1).3 −=− là só chính phương (mod). p .
Nếu 21127(1)/263 ltptpt =+ =+ −=+ là số lẻ nên (1) khong là số chính phương (mod) p và 3 cũng không là số chính phương (mod) p . Do đó (3)(1).3 −=− là số chính phương (mod). p
Vậy luôn tốn tại , x ∈ ℕ x lẻ để 2 3(mod) xp ≡− (ta có thể giả sử x bởi vì nếu x chẵn thì thay x bởi xp + ).
Giả sử 222 2134(1)0(mod)10(mod). xkxkkpkkp =+ +=++≡ ++≡
∏
p i
() 2 1
ppk iippk pp = ∏
=+ ++≡=+ =
Bài toán 14. (VMO 2004) Với mỗi số nguyên dương n ta kí hiệu ()Sn là tổng các chữ số của n trong biểu diễn thập phân. Xét các số tự nhiên n là bội số của 2003, tìm min() Sn
Giải. Đặt 2003, p = thì p là số nguyên tố. Rõ ràng ()1Sn > vì 10k không chia hết cho p Giả sử tồn tại n là bội của p và ()2.Sn = Suy ra tồn tại k để 101(mod). k p ≡− Chú ý rằng 107 2102410(mod), p =≡ nên ( ) ( ) 510727 2210101(mod). kkkk p =≡≡≡−
Vậy 1 là số chính phương (mod). p Mâu thuẫn vì p không có dạng 41. k +
Do đó ()3.Sn ≥ Tiếp theo ta sẽ chứng minh tồn tại n là bội của p sao cho ()3.Sn =
Ta có 7107001001(1)/2 1022.10221(mod), p p ≡ ≡=≡− vì 200381.pk=≠±
Ta chọn 700 2.101 n =+ thì n là bội của p và ()3.Sn =
Vậy giá trị nhỏ nhất của ()Sn khi n chạy trên các bội của 2003 là 3.
Nhận xét. Tồn tại vô hạn số tự nhiên n là bội của 2003 sao cho ()3.Sn =
Bài 15. Cho số nguyên dương k và số 41pk=+ là số nguyên tố. Chứng minh rằng
1 k k chia hết cho p
Giải. Ta có 22 1111 41 4242 pppp pkkpkp
=+ ==−+=−+
Suy ra 2211 (mod)(mod), 22 pp kpkp
modulo p, suy ra 2 2 ()1(mod). p kp−≡ (1)
Mặt khác 1111 2222 ()(1).(mod) pppp kkkp −≡−≡ (2)
Từ (1) và (2) suy ra 1 2 1(mod). p kp ≡
Ta lại có 2 1 2 p kkp =−+ và 1 2. 2 p k = ,
312
k pp k kkk pp kkpkp
suy ra 11 22 22 11 22(1).(1).2.(mod).
≡−+≡−≡−
Nếu k chẵn, * 2,,ktt=∈ ℕ suy ra 81,pt=+ suy ra 2 là số chính phương modulo p, do đó
1 2 21(mod), p p ≡ và (1)1. k −= Vậy 1(mod)1.kk kpkp ≡ ⋮
Nếu k lẻ, * 21,,ktt=+∈ ℕ suy ra 85,pt=+ suy ra -2 không là số chính phương modulo p, do đó 1 2 21(mod), p p ≡− và (1)1. k −=− Vậy 1(mod)1.kk kpkp ≡ ⋮
Từ các kết quả trên suy ra bài toán được chứng minh.
Bài 16. Chứng minh rằng với mọi số tự nhiên 2, n ≥ ước số nguyên tố của 2 21 n nF =+ (số Fecma) có dạng 2 .21, n mm + + là số nguyên dương.
Giải. Gọi p là một ước nguyên tố của .nF Suy ra 1 22 21(mod)21(mod). nn pp + ≡− ≡
Gọi 1 (2)21(mod)|22,,1. hnk p hordphhkkn + = ≡ =∈≤+ ℕ
Nếu 22 21(mod)21(mod), kn knpp ≤ ≡ ≡ mâu thuẫn với 2 21(mod). n p ≡−
Vậy 1 2. n k + = Mặt khác theo định lí Fecma ta có 1 21(mod), p p ≡ suy ra
11 2|1.21, nn ppmm ++ =+ nguyên dương.
Do 2, n ≥ suy ra
2 1 8 00 2 81(1)1,(,)1 p pkxxp p =+ =−= ∃∈= ℕ sao cho 2 0 2(mod) xp ≡
p p nnn pp xpmpm +++ =≡ = =+
1 1 112 2 0 11 21(mod)2|.2.21. 22
Bài 17. Cho 2 21 n k =+ với n là số nguyên dương, giả sử k là số nguyên tố. Chứng minh
rằng k là ước của 1 2 31. k +
Giải. Vì 2 21 n k =+ là số nguyên tố, nên theo luật luật tương hỗ Gauss, ta có 1 311 22 22 3321412 (1)1
nn k kk kk ++
=−= =====−
1. 33333
1111 2222 3 3(mod)13(mod)310(mod)31. kkkk
phương modulo n.
t 21 . 3 k n = Với giá trị nào của k thì 1 là số chính
Giải. Giả sử tồn tại k sao cho 1 là số chính phương modulo n.
Tức là tồn tại số nguyên m sao cho 2 1.mn + ⋮ Đặt 2.,1 s kbs=≥ và b lẻ. Thì 2121 kb ⋮ (vì | bk ) và (21,3)1 b −= (do b lẻ), nên 21 k chia hết cho 3(21), b tức là 21|. b n
Nếu 1, b > thì 211 b −> và 21 b có dạng 43. l + Do đó 21 b có ước nguyên tố p dạng 43.pt=+ vậy 2 1.mp + ⋮ Vô lí.
Do đó 1. b = Tức là 2,1. s ks=>
Đảo lại, giả sử 2,1. s ks=> Ta có 2 1 2 1
s s i n =
21 (21). 3
==+ ∏ Với mỗi i thì 2 21 i + chỉ có
ước nguyên tố dạng 41. t + Do đó 1 là số chính phương là số chính phương (mod), p nên theo hệ quả của định lí 11, suy ra 1 là số chính phương (mod). n
Bài 19. Cho số nguyên đa thức 8 ()16.fxx=− Chứng minh rằng với mọi số nguyên tố p , đều tìm được số nguyên dương n sao cho (). fnp ⋮
Giải. Ta có 442222 ()(4)(4)(2)(2)(22)(22). fxxxxxxxxx =−+=−+−+++
Suy ra 2222 ()(2)(2)[(-1)1][(1)1]. fxxxxx=−++++
- Nếu 2 là số chính phương modulo p thì tồn tại * n ∈ ℕ sao cho ()0(mod). pnp ≡
- Nếu 2 là số chính phương modulo p thì tồn tại * n ∈ ℕ sao cho ()0(mod). pnp ≡
- Nếu 1 là số chính phương modulo p thì tồn tại * n ∈ ℕ sao cho ()0(mod). pnp ≡
- Nếu 2,2,1 đều không là số chính phương modulo p thì
111 222 21(mod),(2)1(mod),(1)1(mod). ppp ppp≡−−≡−−≡−
Nhưng ta lại có 111 222 2(2)(1)(1)(1)1(mod), ppp p ≡−−≡−−≡ suy ra mâu thuẫn.
Vậy bài toán được chứng minh.
Nhận xét.
- Từ bài toán trên ta xây dựng được bài toán sau (với cách giải tương tự): Cho đa thức 222 ()()()(), fxxaxbxab =−−− trong đó , ab là hai số nguyên tố phân biệt. Chứng minh
rằng với mọi số nguyên tố p đều tìm được số nguyên dương n sao cho (). fnp ⋮
- Đa thức dạng trên “rất hữu ích” trong ứng dụng vào việc tìm ước số nguyên tố của số nguyên (bằng thuật toán Rho-phương pháp)
Bài 20. (IMO Shortlist, 1998) Xác định tất cả các số nguyên dương n sao cho với n này tồn tại số nguyên m thỏa mãn 2 21|9. n m −+
Giải. Ta viết 2.;,, s ntstt −∈ ℕ là số lẻ.
Nếu 3, t ≥ thì 2 21|2121|9. tnt m −+ Ta có 211(mod4), t −≡− tức là 21 t có dạng 43, k + do đó nó có ước nguyên tố p mà 1(mod4). p ≡− Hiển nhiên 3, p ≠ vì
3|21, t t−∀ / lẻ
Từ đó suy ra 22 |99(mod). pmmp + ≡− Vậy 9 là thặng dư bậc hai theo modulo p, suy ra () 1 2 1 2 2 913 1(1).11. p p ppp
Suy ra 1 2 p là số chẵn, tức là 1(mod4). p ≡ Mâu thuẫn.
Từ đây ta suy ra 1 t = hay 2. s n = Ta sẽ chứng minh rằng 2, s n = (với s ∈ ℕ ) là tất cả các
giá trị của n cần tìm. Thật vậy, ta sẽ chỉ ra số nguyên m thỏa mãn 2 21|9. n m −+
Ta có 1 222 2121(21)(21)(21)...(21). s s n −=−=−+++
Từ đó để 2 21|9, n m −+ ta cần có 22 21|9,0,1,...,1. k mks ++∀=−
Mặt khác dễ chứng minh được các số Fecma có tính chất: 22 21,211,. mk mk ++=∀≠
Do đó theo định lí thặng dư Trung Hoa thì tồn tại số nguyên 0x thỏa mãn hệ đồng dư
3.2(mod21)9.29(mod21)
3.2(mod21)9.29(mod21)
3.2(mod21)9.29(mod21)
Từ đó suy ra 22 0 90(mod21),0,1,...,1. i xis +≡+∀=−
Từ đây suy ra 222 00 21|3(1)21|921|9, nnn xxm−+ −+ −+ với 0mx = đây chính là giá trị m cầm tìm.
Vậy tất cả giá trị của số tư nhiên n cần tím là 2, s n = với s ∈ ℕ
Bài 21. (APMO, 1997) Tìm một số n nằm giữa 100 và 1997 sao cho |22 + n n . Giải. Hiển nhiên n thoã mãn điều kiện thì n là số chẵn. Không thể xảy ra 2 = np , với p là số nguyên tố ( từ Định lí Fermat). Bây giờ chúng ta tìm 2 = npq , với , pq là các số nguyên tố khác nhau. Chúng ta cần 21 22 |211
pq pq pq . Theo Định lí
Fermat ta lại có 2121 |21,|21 ++qppq , dễ thấy 3,5,7 = q không thoã mãn, ta thử chọn 11 = q xem sao. Trong trường hợp này chúng ta tìm thấy 43 = p . Chúng ta còn lại chỉ cần chứng minh với 43,11==pq . Thật vậy, chỉ cần chứng minh rằng
2121 |21,|21 ++qppq . Điều này là rất dễ dàng kiểm tra.
Vậy chúng ta chọn 2.11.43 = n
Bài 22. (IMO Longlits, 1992, ROM 2) Cho , ab là các số nguyên. Chứng minh rằng
2
2 21 , 2 a b +
không phải là một số nguyên. Giải. Ta xét hai trường hợp:
a) Trường hợp 1: b chẵn thì 2 21 a chia hết cho 2, vô lí.
b) Trường hợp 2: b lẻ thì 2 23(mod8)+≡ b , do đó 2 2 + b có ước số nguyên tố lẻ dạng 83 + k , gọi một trong số đó là p , thì do 2 21 a chia hết cho p nên (,) ap = 1 do đó tồn tại 1a sao cho 1 1(mod) ≡ aap , khi đó ta có:
pp p rrrsss pp p
+++++++≡+++
+−+−++−≡−
1 ......12... 2 11 (11)(21)...1(mod) 22 (2)
−≡ ≡ ≡
2 2 2 1
210(mod) 21(mod) 2(mod)
ap ap ap
Do đó 2 là thặng dư bình phương theo mod p , với p là số nguyên tố dạng 83 + k vô lí. Kết luận được chứng minh.
2. Các bài toán về tồn hay không tồn tại. Biểu diễn số nguyên
Sau đây chúng ta sẽ thấy vai trò của thặng dư bình phương về chứng minh tồn tại hay không tồn tại trong các bài toán số học thi Olympic cũng như sự tồn tại biểu diễn của một số nguyên theo dạng nào đó.
Bài 23. Chứng minh rằng với mỗi số nguyên tố p , thì tồn tại các số nguyên , ab sao cho
22 1++ab là bội số của p
Giải. Ta thấy p | 22 1++ab khi và chỉ khi 22 1(mod)≡−− abp .
Nếu 2 p = thì chỉ cần chọn a chẵn b lẻ ta có điều phải chứng minh.
Xét trường hợp p lẻ.
Xét hai tập hợp 2 2 11 {|(mod),1,},{|1(mod),1,}, 22iijj ArripiBssjpjpp =≡==≡−−=
trong đó các số r đôi một không đồng dư với nhau theo mod p , cũng như các số s đôi
một không đồng dư với nhau theo mod p và { } ,1,2,...,1.ij rsp∈−
Ta có 1,1.ABpABp ==−∪≤−
Nếu 1, ABp∪<− thì AB∩≠∅ nên tồn tại
22221(mod)10(mod). ij rsijpijp = ≡−− ++≡
Nếu 1, ABp∪=− thì AB∩=∅ nên các số , ijrs đôi một phân biệt, suy ra
121121 22 ......12...10(mod) pp rrrssspp +++++++=+++−≡ (1)
Vậy các số nguyên , ab sao cho 22 1++ab là bội số của p
Giải. Giả sử tồn tại số nguyên dương N sao cho với mọi > xN thì cả hai số 2 21 + x và
2 61 + x đều là số nguyên tố. Ta thấy, với >> mnN
222 6131(mod21) +≡++ mmn
Nếu 3 không phải là thặng dư bình phương 2 mod21 + n , thì với 1 2 = n m thì
222 61310(mod21) +≡+≡+ mmn . Vô lí.
2 3 1 21 = + n Mà 2
2
321
2
Giải. Tất cả các đồng dư sau đều theo mod101
Bổ đề: Với các số nguyên , xy , ta có ()()(mod101)(mod101) fxfyxy ≡⇔≡
Thật vậy, hiển nhiên (mod101)()()(mod101).xyfxfy ≡ ≡
Nếu ()()(mod101),fxfy ≡ thì 22 22
Chọn p là ước số số nguyên tố của 32 n + sao cho 2(mod3) ≡ p và số mũ của p trong
phan tích ra thừa số nguyên tố của 32 + n là số lẻ, tức là 21 i p | 32 + n và 2 |32, i pn + /
với * ∈ ℕ i . Thì i p | ++ abc do đó p | ++ abbcca ( 1) . Thế (mod) ≡−− cabp vào (1)
4(()())4(14142)
fxfy xxyyxy xy xyy
=++++− ≡+++−
(214)3(29)(mod101)
Mà 3 1, 101 =− nên 214290(mod101)29(mod101). xyyxy ++≡−≡ ≡≡
Trở lại bài toán của chúng ta.
Từ Bổ Đề ta thấy các số (0),(1),...,(100) fff là một hoán vị của 0,1,..,100 theo mod101.
ta thấy
p | 22 ++ aabb 22(2)3 ++ abb
Suy ra 3 1 = p , vô lí do 2(mod3) ≡ p
Bài 28. (Việt Nam TST 2004) Chứng minh rằng số 21 + n không có ước số nguyên tố nào có dạng 81 k .
Do đó với mỗi {0,1,..,100}, xA ∈= xét 102 số ( ) ( ) ( ) ( ) ( ) ( )
,,...,......fxffxfffx
Theo nguyên lý Dirichlet sẽ tồn tại hai số nguyên 1102, ab ≤<≤ sao cho
Giải. Giả sử 21 + n có ước số nguyên tố p dạng 81 k
Nếu n là số chẵn thì /22 1(2)(mod), n p −≡ do đó 1 là số chính phương modulo p
() ( ) ( ) () ( ) ( )() ............mod101 ab
fffxfffx ≡ () ( ) ( ) () ( )() ......mod101, aba a
fffxffx ≡ theo
bổ đề ta có ( ) ( ) ( ) ( ) ......mod101. ba
Suy ra 1 2 1 1(1)1. p p ==−=−
fffxx ≡
Như vậy với mỗi ,xA ∈ thì tồn tại số tự nhiên dương x n sao cho ()(mod101) xn fxx ≡
Gọi 1 n > là một bội chung của 01100 ,,...,.nnn ta sẽ chứng minh n thỏa mãn bài toán. Thật
vậy: - Nếu ,xA ∈ thì ta có 2 ()(mod101)()()(mod101). x xx nnn fxxfxfxx ≡ ≡≡
Bằng quy nạp ta được
* ()()(mod101),. xx knn fxfxxk ≡≡∀∈ ℕ
Do đó nếu ()(mod101)()101.xnn nnfxxfxx ≡
- Nếu ,xA ∉ thì tồn tại yA ∈ sao cho (mod101).xy ≡
Do đó ()()(mod101)()101.nn n fxfyyxfxx ≡≡≡ ⋮ Bài toán được chứng minh.
Nhận xét. Từ chứng minh trên ta suy ra tồn tại vô hạn số tự nhiên dương n sao cho ()101 n fxx⋮ , với mọi x nguyên.
Bài 27. Chứng minh rằng không tồn tại các số nguyên dương ,, abc sao
cho 222 3() ++ ++ abc abbcca là một số nguyên.
Giải. Giả sử tồn tại các số nguyên dương ,,, abcn sao cho .
222 3()++=++ abcnabbcca
Suy ra 2 ()(32)() ++=+++ abcnabbcca .
Vô lí.
Tương tự nếu là n số lẻ thì (1)/22 2(2)(mod), n p + −≡ do đó 2 là số chính phương modulo p. (1)
Ta lại có () 2 1 1 2 8 212 (1).11 p p ppp
Bài toán được chứng minh.
, mâu thẫn với (1)
Bài 29. (Tạp chí Pi số tháng 8, năm 2017) Chứng minh rằng không tồn tại số nguyên 5, n > sao cho 35nn + chia hết cho 2 25. n
Giải. Giả sử ngược lại, tồn tại số nguyên 5, n > sao cho 35nn + chia hết cho 2 25. n
Nếu n chẵn, tức là 2,nk = thì từ 5, n > suy ra 2 251. n −> Do đó 2 25 n có ước nguyên
k nnkk k p pp p
tố 3(mod4). p ≡ Hơn nữa ()()22 3 35351. 5
+=+ = ⋮ ⋮ ⋮ Vô lí.
Vậy n lẻ. Bây giờ ta xét p là một ước nguyên tố bất kì của 2 25, n thì ta có 35. nn p + ⋮ Do đó 35 , nn pp =
320
nnnn pppp
vì n lẻ.
Suy ra theo luật tương hỗ Gauss, ta có 15135 1.... 35 pp pppp
Suy ra p đồng dư với 1,2,4, hoặc 8 theo (mod15). Như vậy mọi ước nguyên tố của 2 25 n đều có dạng 15, kr + với { } 1,2,4,8. r ∈ Do đó mọi ước nguyên tố của 5 n và 5 n + phải thuộc dạng vừa nêu. Mà tích của hai số dạng 15, kr + với { }1,2,4,8 r ∈ là số coa dạng như vậy, nên suy ra 5 n và 5 n + đồng dư với 1,2,4,8 theo (mod15). Tuy nhiên điều nàu không thể vì hiệu của hai số tùy ý thuộc tập { }1,2,4,8 không chia hết cho
5. Mâu thuẫn nhận được, từ đó suy ra bài toán được chứng minh.
Bài 30. (Indonesia TST, 2009) Chứng minh rằng tồn tại vô hạn số nguyên dương n sao
cho 2 1 n + không là ước của !.n Giải. Ta có bổ đề quên thuộc sau: Tồn tại vô hạn số nguyên tố dạng 41, k + với k là số nguyên dương. (Bổ đề đã được chứng minh trong bài 4).
Trở lại bài toán. Ta xét p là một số nguyên tó có dạng 41.pk=+
Theo tiêu chuẩn Euler, ta có (1)/2 1 (1)1. p p =−=
Hay 1 là số chính phương (mod). op
Từ đó tồn tại { }1,2,...,1mp∈− sao cho 2 10(mod).mp +≡
Ví ,mp < nên !m không chia hết cho p nên 2 1 m + không là ước của !.m
Theo bổ đề tồn tại vô hạn số nguyên tố 41,pk=+ nên tồn tại vô hạn số nguyên dương n
sao cho 2 1 n + không là ước của !.n
Bình luận. Bài toán trên có lẻ xuất phát ý tưởng của từ bài 3 trong kỳ thi Olympic Toán
Quốc tế lần 49 năm 2008 (tác giả là Kestuis Cesnavicius, người Litva), đây là bài toán khó nhất của ngày thi thứ nhất. Từ bài toán này những năm sau nhiều nước dựa trên ý tưởng đó để phát triển thành đề thi Olympic, đề chọn đội tuyển của nước mình.
Bài 31. (IMO shortlist, 2008) Chứng minh rằng tồn tại vô số số nguyên dương n sao cho 2 1 n + có ước nguyên tố lớn hơn 2n10n. +
Giải. Xét số nguyên tố p dạng * 81,.pkk=+∈ ℕ Theo tiêu chuản Euler, ta có 1 4 2 11 (1)(1)1(mod)1, p k p pp
≡−≡−≡ =
vì {} 1 1,1. p ∈−
Nghĩa là 1 là số chính phương modulo p, nên phương trình 2 1(mod),xp ≡− có hai nghiêm thuộc { }1,2,...,1 p và hai nghiệm đó có tổng bằng p (vì 22 ()(mod) pmmp −≡ ).
Gọi n là nghiệm nhỏ hơn trong hai nghiệm đó, ra tồn tại 1 1,2,..., 2 p n ∈
cho 2 10(mod).np +≡
Chúng ta sẽ chứng minh rằng p2n10n. >+ Thật vậy, đặt 1 , 2 p nl =− với 0. l ≥ Thì 2 2 1 10(mod)(21)40(mod). 2 p lplp−+≡⇔++≡
Do đó 2* (21)4,. lrpr ++=∈ ℕ (1)
Ta có 2 (21)1(mod8), lp +≡≡ nên từ (1) suy ra 5(mod8)5.rr ≡ ≥
Do đó 2 541 (21)45. 2 lplp ++≥ ≥ Đặt 54,up=− thì ta được 1 2 lu ≥ Do đó
1 22 ppunl=−≤ Kết hợp với 2 4 5, u p + = ta được bất phương trình
2 51040,uun−−+≥ mà 0, u ≥ nên suy ra 5409 2, n u ++ ≥ kết hợp với , 2 pu n ≤ suy ra 5409 22210. 2 n pnunpnn ++ ≥+≥+ >+
Vì có vô hạn số nguyên tố dạng 81, k + nên có vô hạn số nguyên dương n thỏa mãn bài toán.
Bài 32. (Gabriel Dospinescu) Chứng minh rằng với mỗi số nguyên dương n, số 3 21 + n có ít nhất n ước số nguyên tố dạng 83 + k
Giải. Theo bài trước thì số 21 + k không có ước số nguyên tố nào có dạng 87 + k . Tương tự bài trên ta có số với lẻ k cũng không có ước số nguyên tố dạng 85 + k . Trở lại bài toán, nếu 2 = n thì bài toán là tầm thường. Giả sử 2 > n , khi đó
11322.332.33 21(21)(221)(221)...(221) +=+−+−+−+ n nn
Bổ đề. 2.332.33 gcd(221,221)3 −+−+= iijj với mọi 11 ≤<≤− ijn . Thật vậy giả sử số
2.332.33 gcd(221,221), iijj −+−+ suy ra 3 |21. i p + Nhưng ta lại thấy 1113333
nguyên tố p chia hết số
Mà
Suy ra
+−−−− ≡≡−≡−
≡−+≡
jijiji jj p p
2.33 2(2)(1)1(mod) 02213(mod)
2.332.33 3(gcd(221,221))1. iijj v −+−+=
2.332.33 gcd(221,221)3 −+−+= iijj
Bây giờ ta sẽ chứng minh mỗi số 2.33 221 −+ ii với 11 ≤≤− in , có ít nhất một ước số nguyên tố dạng 83 + k . Thật vậy, mỗi ước số nguyên tố của 2.33 221 −+ ii cũng là ước số
nguyên tố của 3 21 + i nên có dạng 81 + k hoặc 83 + k . Nhưng 2.33 3(221)1, ii v −+= do đó tất cả các ước số nguyên tố khác 3 của 2.33 221 −+ ii đều lớn hơn 3, và nếu tất cả chúng đều có dạng 81 + k thì 2.33 2213(mod8) −+≡ ii , vô lí. Vậy số có ít nhất một ước số
nguyên tố dạng 83 + k
Từ trên suy ra 3 21 + n có ít nhất 1 n ước số nguyên tố dạng 83 + k và lớn hơn 3. Lại
thấy 3 chia hết 3 21 + n . Nên bài toán được chứng minh.
Bài 33. (Turkey TST 2006) Với mỗi số nguyên dương n chúng ta xác định dãy () nx như
sau
222 112 ...,nn xxxx + =+++
với 1x là một số nguyên dương. Tìm 1x nhỏ nhất sao cho 2006 | 2006. x
Giải. Chúng ta thấy rằng 2 1 ,1 + =+≥nnn xxxn , do đó nx là số chẵn với mọi 2. n ≥
Lại chú ý
2 11 0(mod1003)0(mod1003) ≡⇔+≡ nnn xxx
1 0(mod1003) ⇔≡ nx hoặc 1 1(mod1003)≡− nx
Bây giờ ta sẽ chứng minh đồng dư 1(mod1003)≡− nx không thể xảy ra với mọi 2 ≥ n , giả sử tồn tại 2 ≥ n sao cho 1(mod1003)≡− nx thì
xx x
2 11 2 1
1(mod1003) (21)3(mod1003). nn n
+≡− ⇔+≡−
Vậy 3 là thặng dư bình phương mod1003. Mâu thuẫn với Định lí 8. Do đó 2 0(mod1003) ≡ x 1 0(mod1003) ⇔≡ x hoặc 1 1(mod1003). x ≡−
Vậy giá trị nhỏ nhất của 1x là 1002.
Bài 34. Chứng minh rằng tất cả các ước số lẻ của số 2 51 + x có chữ số tận cùng là số lẻ Giải. Nếu số nguyên tố 2 |51 + px , suy ra 5 1 =
Do đó 1,3,7,9(mod20). ≡ p
p . Từ luật tương hỗ ta có 1 2 515 (1).. 5
p p
Bài 35. Cho ba số nguyên dương ,,1abc > với ,a b nguyên tố cùng nhau và 222 aabbc −+= Chứng minh rằng mọi ước nguyên tố của c đều có dạng 61. k + Giải. Từ (,)1, ab = dễ ràng chứng minh dược ,, abc nguyên tố cùng nhau từng đôi một. Gọi p là một ước nguyên tố bất kì của c Do (,)1, ab = nên p lẻ, suy ra c lẻ. Do đó 3. p ≥ Ta có 2220(mod)()(mod), aabcpababp −+≡ −≡− mà (,)1. abp −=− Suy ra 11 1. ababab ppppp
== =
Lại có 2222 3 ()3()3(mod)1. ab aabbababababp p −+=+− +≡ =
Nếu 3, p = thì 222 3 ()33()393 3. a ababababcab b + + +−=
3(3,)11.
Theo luật tương hỗ Gauss, ta có (31)(1)11 42233 3(1)(1)(1). 3 ppp p p pp
Suy ra
2 1 1(1)1. 33 p pp p
Vậy 31 2 1(3)(mod3)31. 3 p pmpdppl
=≡≡ =+ Suy ra 61,pk=+ hoặc 64pk=+ loại (vì p lẻ).
Bài 36. (KHTN Hà Nội, 2018) Cho dãy số nguyen dương () n a thỏa mãn 3 1 4 nnn aaa + =+ với mọi 1. n ≥ Tìm giá trị nhỏ nhất của 1a để 2018 2018 a + chia hết cho 57. Giải. Vì 573.19, = nên để 2018 2018 a + chia hết cho 57 thì hai điều kiện sau đông thời thỏa mãn:
a) 2018 2(mod3). a ≡
b) 2018 4(mod19). a ≡−
Trước hết, xét theo modulo 3 cho toàn bộ các số hạng của dãy (), n a ta được. 1 (mod3). nnnn aaaa + ≡+≡− Như vậy 213120181,,...,(mod3)aaaaaa ≡−≡≡− . Do đó điều kiện a) tương đương với 111(mod3)2(mod3).aa −≡⇔≡
Mặt khác ta lại có ( ) 3 2 1 4444(4)7(4)5(mod19).nnnnnn aaaaaa + +=++≡++++−
Ta sẽ chứng minh rằng 2 75xx+− không chia hết cho 19 với mọi số nguyên .x Thật vậy, xét phương trình đồng dư: 2 2 2 750(mod19)428200(mod19)(27)12(mod19). xxxxx +−≡⇔+−≡⇔+≡
Phương trình này vô nghiệm, vì 12433191 .1. 1919191933
Như vậy 2 (4)7(4)50(mod19), nn aa+++−≡ do đó
1 40(mod19)40(mod19).nn aa + +≡ +≡ Suy ra điều kiện b) tương đương với
1 4(mod19). a ≡− Tóm lại để 2018 2018 a + chia hết cho 57, thì 1a phải thỏa mãn hệ điều
kiện:
2(mod3) 53(mod57). 4(mod19) a a a ≡ ≡ ≡− Vì 1 0, a > nên 1 53 a = là số nhỏ nhất cần tìm.
1 1 1
Chúng ta bây giờ sẽ khảo sát tiếp một vài ứng dụng của thặng dư chính phương trong việc giải các phương trình Diophant, đây là ứng dụng mang tính kĩ thuật của công cụ này.
Một số bài toán được giới thiệu sau là những bài toán hay và khó, được giải quyết mang tính sắc bén và sức mạnh của khái niệm thặng dư bình phương.
3. Các bài toán về phương trình nghiệm nguyên
Bài 37. (Putnam, 1954) Chứng minh rằng không có số nguyên , xy nào sao cho
2232122.xxyy−−=
Giải. Giả sử tồn tại các số nguyên , xy sao cho 2232122−−=xxyy , khi đó ta có: 22 (23)17488 +−=xyy
Vì 48812(mod17) ≡ , Suy ra 12 là thặng dư bình phương mod17 , lại có
12343172
1. 1717171733
Bài 38. Chứng minh rằng phương trình 23 5 xy +≡ không có nghiệm nguyên.
Giải. Giả sử phương trình 23 5 xy +≡ có nghiệm nguyên (,). xy
Nếu y chẵn thì 2250(mod8)3(mod8),xx +≡ ≡ điều này vô lí theo định lí 7. Do đó y phải lẻ. Ta có 23232 538(2)(24). xyxyyyy +=⇔−=−=−++
Từ y lẻ, nên 2 24yy++ lẻ, gọi p là một ước nguyên tố lẻ của 2 24,yy++ có dạng 3(mod4)(1) p ≡
++≡ ++≡
Từ 22 3 240(mod)(1)30(mod)1. yypyp p
Mặt khác 22 3 3(2)(24)0(mod)1.xyyyp p
Nên ta có 3311 11(mod4)(2) p pppp
Vậy (1) và (2) mâu thuẫn với nhau. Vậy phương trình 23 5 xy +≡ vô nghiệm.
Bài 39. (Serbia MO, 2008) Tìm tất cả các nghiệm nguyên không âm của phương trình 4 122008. xz y +=
Giải. Nếu 0, z > thì 0. y >
Nếu x là số chẵn dương thì vế trái của phương trình có dạng 22*,,(1)abab+∈ ℕ
Nếu x là số là số lẻ thì vế trái có dạng 22* 3,,(2) abab+∈ ℕ
Mà 2008251.8 = và số 251 p = là số nguyên tố lẻ và từ phương trình suy ra p không
chia hết ,.ab Nên từ (1) và phương trình suy ra 22 1 (mod)1.bap p ≡− =
Từ (2) và phương trình suy ra 22 3 3(mod)1.bap p ≡− = (1)
326
khác ta có
3313251133 .(1).1
2512512512251251 p
===−=−
312511
22 2512513.822 (1) 1(2) 3333 + =−−====−
Ta có (1) và 92) mâu thuẫn. Vậy 00.zxy = ==
Do đó phương trình có nghiệm duy nhất
Bài 40. (Tukey TST, 2013) Tìm nghiệm nguyên dương của phương trình
61 1. n mnn + =+−
Giải. Xét 1. n = Ta được 1. m =
Xét 1. n > Nếu 13, n + ⋮ thì từ phương trình ta có:
11221
61122613333 11331 nnnn nn n mnnnmmnmnnn ++++ ++ + =+−> > ≥+ ≥+++ 221221 11 3333 1331332. nnnn nn nnnnnnnn ++++ ++ +−≥+++ ≥++ Vô lí.
Do đó 1 n + không chia hết cho 3.
Nếu 6 11(mod3)|31(mod3).nnm +≡ ≡− Vô lí.
Do đó 11(mod3). n +≡− Suy ra tồn tại ước nguyên tố p của 1 n + mà 1(mod3). p ≡−
Nếu 12, n + ⋮ thì lập luận hư trên, ta cũng suy ra mâu thuẫn.
Vậy 1 n + phải lẻ, từ phương trình ta được ( ) ( ) 613111. n mnnnp + +=++++⋮⋮
Suy ra
Bài 41. a) Cho m, n là các số nguyên dương sao cho 1 n + chia hết cho 4. m Chứng minh rằng phương trình đồng dư 2 (mod)xmn ≡ có nghiệm.
b) (Euler) Chứng minh rằng phương trình 2 4xyzxyt =++ không có nghiệm nguyên dương.
Giải.
a) Giải sử 2, a mb = với lẻ. Ta có 2a mb nnn
i) Trước hết ta chứng minh 2 1. a n =
Nếu a chẵn thì hiển nhiên.
Nếu a lẻ thì 2,ml = suy ra 2 (1)/8 22 81(1)11. a n nl
=
Vây, 2 1. a n =
ii) Ta chứng minh 1. b n =
Theo luật tương hỗ suy ra 11 22 (1). nb bn
≡− = =
6 313 3()11.mmodp ppp
inhlí
Theo luật tương hỗ Gaus, ta có
Từ hai kết quả trên, ta suy ra 1 133 1.(1)11. p
Từ (1) và (2) suy ra mâu thuẫn. Vậy phương trình có nghiệm nguyên dương duy nhất là (,)(1;1). mn =
Lại có (1)/2 1(mod)(1). b n nb bb
Vậy 1 (1)1, b b n =−= do 1 b chẵn.
Từ i), và ii) ta có 1. m n = Do đó phương trình 2 (mod)xmn ≡ có nghiệm.
b) Giả sử tồn tại các số nguyên dương ,,, xyzt sao cho
2222 41644416444 xyzxytxyzxytxyzxzyzzt =++⇔=++⇔=++ 22 41(41)(41)(2)(mod), ztxzyzztzn⇔+=−− =− ở đó 41.nyz=−
Vậy z là số chính phương (mod), n do đó 1. z n = Mặt khác 1 n + chia hết cho 4, z
nên theo câu a) 1. z n = Vậy 1 1. n =
Nhưng (1)/2 1 (1)1, n n =−=− do (1)/221 nyz−=− là số lẻ. Ta có mâu thuẫn.
Bài 42. Cho 41pk=− là một số nguyên tố, * . k ∈ ℕ Chứng minh rằng nếu a là một số
nguyên sao cho đồng dư 2 (mod) ≡ xap có nghiệm, thì các nghiệm của nó là =± k xa
Giải: Giả sử đồng dư 2 (mod) ≡ xap có nghiệm, theo tiêu chuẩn Euler ta có 1 21 2 1(mod)1(mod) p k apap ≡⇔≡ . Với =± k xa thì 22 (mod) ≡≡ k xaap Đpcm.
Bài 43. (Korea MO 2000) Chứng minh rằng với mỗi số nguyên tố p tồn tại các số
nguyên ,,, xyzw sao cho 222 0 ++−=xyzwp và 0 <<wp
Giải. Nếu 2 = p chọn 0,1.====xyzw
Giả sử 2 > p . Xét trường hợp 1 là thặng dư bình phương mod p . Thì tồn tại số nguyên
(01) <<−aap sao cho 2 1(mod)≡− ap . Đặt (,,)(0,1,) = xyza . Bởi vì
2222 1++=+xyza chia hết cho p và nó lớn nhất là 22
1(1)+−<pp , nên tồn tại
(0) << wwp sao cho 222 0 ++−=xyzwp . Bây giờ xét 1 không phải là thặng dư bình
phương mod p . Ta chọn k sao cho , kpk đều là thặng dư bình phương mod p . Nếu
1 2 p là thặng dư bình phương mod p thì chọn 1 2 = p k , ngược lại thì
thặng dư bình phương mod p nằm trong các cặp (1,2),(2,3) pp
4. Các bài toán khác. Bài 44. Chứng minh rằng với mỗi ∈ a Z , số các nghiệm nguyên (,,) xyz của đồng dư 222 2(mod) ++≡ xyzaxyzp là '2((1)) +− p p
Giải. Đồng dư đã cho tương đương với 22222 ()(1)(mod). −≡−− zaxyaxyxp (1)
Với mỗi ,{0,1,...,1} ∈−xyp cố định, thì số nghiệm z của phương trình (1) là 2222(1) 1.
Do đó tổng số tất cả số nghiệm của (1) là 11 2222 2
Theo Bài Toán 28, thì
1
2 p số không
31 ,...,(,) 22 −+pp , mỗi
cặp có chứa một số. Từ Định lí Dirichlet, thì tồn tại k sao cho k và pk đều là thặng dư
bình phương mod p . Do đó có thể chọn ,{0,1,..., ∈ xy
2 p sao cho
1 }
2 (mod) ≡ xkp và
2 {0,}∈ p , nên ta có thể chọn wnhư trường hợp trước.
2 (mod) ≡− ypkp . Chọn 1 = z thì 222 ++ xyz chia hết cho p và
Như mọi công cụ hữu ích khác, thặng dư bình phương cũng có những ứng dụng đặc sắc mà chúng ta không thể xếp nó vào đâu được, đó chỉ là phần rời rạc nhưng lại thú vị nhất của công cụ. Phần sau là các bài tập chọn lọc để minh chứng cho nhận xét trên. 330
là kí hiệu phần nguyên của số thực x) Giải. Chú ý rằng 2003 là số nguyên tố. Từ luật tương hỗ ta có
Do đó
20012001 2003|2(21)22. + +=+ iii
mà 2001 2,2 +ii đều không là bội số của 2003. Suy ra 200120012222 1. 200320032003
Cho i chạy từ 0 tới 1000, rồi cộng các đẳng thức lại với nhau ta được 220012002 122...221 10011001. 20032003 ++++− =−=− S
Bài 47. (Korea TST 2000) Cho p là một số nguyên tố dạng 41 + k . Tính :
Giải. Với mỗi số
Khi 1 {} 2 < x thì 2{}{2} = xx . Khi 1 {} 2 > x thì 2{}{2}1
Do đó tổng cần tính bằng số các số k trong [1,1] p sao cho
≥
k p , hay bằng số các số k không thặng dư bình phương mod p sao cho 2 k đồng dư mod p với số nào đó trong 1 ,1. 2 p p +
Vì 1(mod4) ≡ p nên 2 1(mod)−≡ dp , với ∈ ℤ d . Phân chia tập hợp các số không thặng dư bình phương mod p thành 1 2 p cặp có dạng {,} ada , sao cho 22()(mod)≡− aadp trong mỗi cặp. Do đó có chính xác một số trong mỗi cặp có thặng dư bình phương với một số nào đó trong 1 [,1] 2 + p p , có tất cả là 1 2 p thặng dư đó. Suy ra tổng cần tính bằng 1 2 p
Bài 48. (Iberoamerican Olympiad 2003)Các dãy số 00(),()≥≥nnnnab được xác định như sau
001,4==ab và với mọi 0 ≥ n , 20012001 11,.++=+=+ nnnnnn aabbba
Chứng minh rằng 2003 không chia hết bất cứ số nào của cả hai dãy.
Giảỉ. Chúng ta sử dụng quy nạp. Với 0,1 = n kết luận hiển nhiên đúng.
Giả sử kết luận đã đúng với mọi < kn ( 2 ≥ n ). Giả sử 2003| na , thì theo định lí Fermat ta có:
2001 11
=+≡
+≡
0(mod2003) 0(mod2003)
1()()0(mod2003)
+++≡
2002 111 20012001 2222
2002200220012001 222222
++++≡ ++≡
1..0(mod2003)
20012001 2222
Do đó tồn tại số nguyên sao cho 2 31.2003++= xxk . Phương trình bậc hai
2 31.20030 ++−=xxk
có nghiệm nguyên dương, nên 94(1.2003) k phải là số chính phương.
Do đó 5 là số chính phương mod2003, mâu thuẫn với Định lí 8.
Tương tự cũng có 2003 không chia hết bất cứ số hạng nào của dãy (). nb
Bài 49. (JBMO 2007, Bulgaria, problem 4) Chứng minh rằng nếu p là một số nguyên
tố, thì số 734=+− p Ap không phải là số chính phương.
Giải. Giả sử A là số chính phương.
Nếu 2 = p , bài toán là tầm thường.
Giả sử 2 > p , theo Định lí Fermat bé thì 7341(mod)=+−≡− p App
Do đó 1 là thặng dư bình phương mod p , suy ra 41=+pk .
Do đó 7342(mod4)=+−≡− p Ap , nhưng 2 không là thặng dư bình phương mod4 . Vô lí.
Bài 50. Xác định tất cả các số ∈ ℕ n sao cho tập hợp {,1,...,1997}=++Annn có thể phân chia thành ít nhất 2 tập hợp con có cùng tích số các phần tử. Giải. Giả sử có thể phân chia A làm k tập con 12,,..., kAAA có cùng tích các phần tử là m. Do có ít nhất là một phần tử và nhiều nhất là hai phần tử của A chia hết cho số nguyên tố 1997, chúng ta có 1997| m , do đó 2 = k . Ngoài ra, có nhiều nhất một phần tử của A chia hết cho số nguyên tố 1999, nên 1999 không chia hết m. Vì vậy không có phần tử nào của chia hết cho 1999, tức là các phần tử của A đồng dư với 1,2,3,...,1998 mod1999 . Vậy 2 1.2.3....,19981(mod1999) ≡≡− m ( theo Định lí Willson), mâu thuẫn vì 1 không thể là thặng dư bình phương mod19994.4993 =+ .
Bài 51. (Gabriel Dospinescu, Moldova TST, 2005) Cho ,: ++ → ℤℤ fg là các hàm thoã mãn các tính chất
i) g là một song ánh.
ii) 222 2()() =+ fnngn với mọi số nguyên dương n
iii) |()| 2004 −≤ fnnn với mọi số nguyên dương n.
Chứng minh rằng f có vô số điểm cố định.
Giải. Gọi np là dãy các số nguyên tố dạng 83 + k . Ta có
2 1
8 2 (1)1. =−=−
np np
Sử dụng điều kiện i) thì tồn tại nx sao cho () = nngxp với mọi n. Suy ra
222() = nnfxx 2 + np . Vì 2 1 =−
np nên từ đồng dư trước suy ra | nnpx và |()nnpfx . Do
đó tồn tại các dãy số nguyên dương , nnab , sao cho ,() == nnnnnn xapfxbp với mọi n.
Từ ii) suy ra 22 21 =+nnba , với mọi n
Cuối cùng từ |()| 2004 −≤ fnnn , suy ra
fxb xa x tức là 2 1 lim2,. + =→+∞ n n
2004()|1||1|.≥−=− nn nn n
a n a
suy ra lim1,=→+∞ n an . Do đó với n đủ lớn 1 == nnab ( chú ý na là dãy số nguyên dương), tức là (). nnfpp =
Bài 52. (Gabriel Dospinescu, Mathlinks Contest) Giả sử 122004 ,,..., aaa là các số nguyên không âm sao cho 12 ... ++nn aa 2004 + n a là số chính phương với mọi số nguyên dương.
Hỏi số các số bằng 0 nhỏ nhất là bao nhiêu.
Giải. Giả sử 12,,..., kaaa là các số nguyên dương sao cho 12 ... +++ nnn kaaa là số chính phương với mọi n. Thì k là số chính phương, thật vậy, xét số nguyên tố p lớn hơn mỗi
ước số nguyên tố của 12 kaaa . Sử dụng Định lí Fermat nhỏ
111 12 ...(mod).+++≡ppp k aaakp
nhưng 111 12+++ ppp kaaa là số chính phương, nên 1 = k p . Theo hệ quả Bài Toán
36, thì k là số chính phương. Bây giờ bài toán là tầm thường, ta thấy số chính phương lớn nhất không vượt quá 2004 là 2 441936 = . Do đó số mà bài toán chúng ta cần tìm là 68.
Bài 53. Chứng minh rằng một số nguyên dương a là thặng dư bình phương với mỗi số nguyên tố khi và chỉ khi a là số chính phương.
Giải: Giả sử a không phải là số chính phương. Đặt 2 1 kabpp = với 1,..., kpp là các số nguyên tố và b là số nguyên dương. Với mỗi số nguyên tố p ta có 2 11 . kk ii ii
== ∏∏ (1)
pp ab pppp ==
Nếu 2 = a ta có phản thí dụ nếu chọn 5 = p . Trong trường hợp khác a có ước số lẻ, gọi là kp . Chọn số nguyên tố p sao cho 1(mod8),1(mod),≡≡ ippp với 1,2,.., = ik và (mod) ≡ kpap , ở đây a là số tuỳ ý không thặng dư bình phương mod kp . Số nguyên tố p tồn tại từ Định lí Dirichlet về số nguyên tố trong một cấp số cộng. Từ (1) thì 11 ,..., kpp là các thặng dư bình phương mod p , nhưng kp thì không. Do đó a không là thặng dư bình phương mod p . Vô lí.
Nhận xét. Từ lời giải bài toán trên suy ra: Nếu a không phải là số chính phương, thì a không là số chính phương (mod), p với vô số số nguyên tố p
Bài 54. Giả sử [] ∈ℤ fx là đa thức bậc hai sao cho với mỗi số nguyên tố p tồn tại ít nhất
một số nguyên n sao cho |()pfn . Chứng minh rằng f có nghiệm hữu tỉ.
Giải. Gọi 2 () =++ fxaxbxc là đa thức trên. Chúng ta chỉ cần chứng minh biệt số
2 4 bac là số chính phương.
Lấy một số nguyên tố p bất kì và số nguyên n sao cho |()pfn . Thì từ
22 4()(2)4, afnanbacb =++− suy ra 224(2)(mod).−≡+ bacanbp
Do đó
2 4 1. = bac p
Giải. Theo hệ quả của Định lí 1, ta có
Cũng có
Lại thấy
ppppp iii
ipii ipipif ppp
(lại do (1)0 = f )
Do đó với 1(mod4) ≡ p thì '(1)0 = f , nên ()fx chia hết cho 2 (1). x
với mỗi số nguyên tố p
Theo Bài toán trên, thì 2 4 bac là số chính phương. Đpcm.
Bài 55. (Călin Popescu, Romanian TST, 2004) Cho là một số nguyên tố lẻ, và đặt
i fxx p
1 1 1 (). =
= p i i
a) Chứng minh rằng f chia hết cho 1 x nhưng không chia hết cho 2(1) x nếu và chỉ nếu 3(mod4) ≡ p .
b) Chứng minh rằng nếu 5(mod8) ≡ p thì f chia hết cho 2(1) x và không chia hết cho 3(1) x .
Chú
===≡
pp ii
Do đó ()fx chia hết cho 1 x , nhưng không chia hết cho 2(1) x .
ppp iii iii fiiii ppp
p i i fi
2 2 1
i i p
≡−
2 (2)4(mod8). =
Bây giò ta cần tính 11 22 2 11
−≡
Mà 11 22 11
c: 1.
Nếu 1(mod4) ≡ p
pp ii iif pp
221 (1)0. ==
thì 1 , 1 5 (,) 1 , 1 4 = = =− pa p Nap pa p
Do đó
ppp iii
222 111
Lại có: =+
p i
=
21 0 =
Bài 57. Cho p là một số nguyên tố lẻ và 1, > mh là các số tự nhiên sao cho
pp ii
ii pp
+− =+=
2121 1. ==
i p .Bài toán được giải hoàn toàn.
Bài 56. Với mỗi số nguyên tố lẻ p và a thoã mãn 0 <<ap . Đặt (, )Nap là kí hiệu số các số dư r thoã mãn 1 + ==+ rra pp . Tìm một công thức với (, )Nap .
+ =++
2,2.1 <=+ mm hnh .Nếu 1 2 1(mod)≡− p np chứng minh rằng n là một số nguyên tố khi
Do 1 =− n p nên 1 =− p n Đpcm.
Ngược lại, giả sử q là một ước số nguyên tố của n. Ta có 1 1(mod) ≡ n pq và 1 1(mod) ≡ q pq . Suy ra (1,1) 1(mod). ≡ qn pq
Nếu 2m không chia hết 1 q thì 1 (1,) 2 n q nên 1 2 1(mod) ≡ n pq , mâu thuẫn. Vậy
2|1 m q , suy ra .21=+ m qt . Nếu n có ước số nguyên tố ' q khác q , thì
.'(21)2(21).2.1.
≥>+>+>+= mmmm nqqtthhn
Vô lí. Vậy n chỉ có duy nhất một ước số nguyên tố q , tức n là số nguyên tố
p r Srra pp Sử dụng hệ quả của Định lí 1, ta có 1 1 0 0 11() 1.1 44 = = ++ =++=+ p p r r rrarraSp ppp 338 1 1 0 11)1 44 = +− =+= p r rap p p Suy ra 11 (,)(1)113. 44 =−−+−+=−−− aaaa Nappp pppp Từ
Bài 58. (Iurie Boreico, MR) Cho a là một số nguyên dương sao cho với mỗi số nguyên dương n thì số 2 + an có thể viết dưới dạng tổng của hai số chính phương. Chứng minh rằng a là số chính phương.
Giải. Gọi 12,,..., kppp là các số nguyên tố chia hết a với số mũ lẻ trong biểu diễn nguyên tố của a . Nếu 1,2==kp ta có phản ví dụ là 8 = na . Do đó có thể giả sử một trong các số ip là khác 2. Xét các số it không chia hết cho ip . Thì hệ các đồng dư (mod ≡ ixt ),3(mod4) ≡ i px có nghiệm theo Định lí Trung Hoa về phần dư. Ngoài ra theo Định lí
Dirichlet về số nguyên tố trong một cấp số cộng, thì có thể tìm thấy số nguyên tố p không chia hết a và thoã mãn hệ đồng dư trên. Ta có:
Giải. Do | / pa nên các số ,0,1,..,1 +=−axbxp lập thành một hệ thặng dư đầy đủ mod p . Có chính xác 1 2 p số trong chúng là thặng dư bình phương mod p , và chính xác
1 2 p số trong chúng là không thặng dư bình phương mod p . Do đó
==− ∏∏ kk ii ii
pp a pppp
1 . ==
11
Bây giờ sử dụng luật tương hỗ Gauss, thì a p chỉ phụ thuộc i i
t p . Do đó chúng ta có thể chọn các it để i i
t p có dấu nào đó sao cho 1 = a p . Điều này có nghĩa tồn tại n sao cho 2 |. + pan Vì vậy 2 + an không thể viết dưới dạng tổng hai số chính phương, thật vậy giả sử 222 1 1 +=+ = anxy p (sử dụng số nghịch đảo), mâu thuẫn với Định lí 3.
Do đó tập các số 12,,..., kppp là rỗng. Vậy a là số chính phương.
Như đã thấy ở phần lí thuyết thì việc tìm công thức tính tổng của các kí hiệu Lagrange đôi khi lại vô cùng có ích trong giải toán, dưới đây chúng ta sẽ cùng nhau khảo sát một vài bài tập cổ điển.
Bài 59. Với các số nguyên , ab tuỳ ý, và một số nguyên tố p không chia hết a , chứng minh rằng:
1
axb p
IV. Bài tập đề nghị.
axb p
1 1 0. =
Cuối cùng chúng tôi giới thiệu các bài tập để bạn đọc luyện tập, đây là các bài toán hay, tổng kết lại phương pháp, hy vọng sẽ được trao đổi thêm với bạn đọc về nội dung của bài viết này.
Bài 1. (IMO 1996) Cho các số nguyên dương , ab sao cho 1516,1615 +− abab đều là các số chính phương. Hỏi giá trị nhỏ nhất trong hai số chính phương đó?
Bài 2. (CRUX, Problem 2344, Murali Vajapeyam) Tìm tất cả các số nguyên dương n là thặng dư bình phương modulo tất cả các số nguyên tố lớn hơn n .
Bài 3. (Valentin Vornicu, Mathlinks Contest) Cho 1 7 = x và 2 1 2–1 + = nnxx . Chứng minh rằng 2003 không chia hết số hạng nào của dãy.
Bài 4. Laurentiu Panaitopol, Gazeta Matematica) Chứng minh rằng số 32 + n không có ước số nguyên tố nào có dạng 2413 + k
Bài 5. (Varshamov) Tìm nghiệm nguyên của phương trình
23 1=+xyz
trong đó x không có ước số nguyên tố nào có dạng 61 + k
để:
1. Phương trình 22 2 += xyp có nghiệm nguyên dương là 1,3(mod8) ≡ p
1. Phương trình 22 3 += xyp có nghiệm nguyên dương là 1(mod6) ≡ p .
2. Phương trình 22 5 += xyp có nghiệm nguyên dương là 1,9(mod20) ≡ p
+ = p x 340
Bài 7. (Komal, January 2002, problem A.283) Cho n là một số nguyên dương. Chứng minh rằng nếu phương trình 22++= xxyyn có nghiệm hữu tỉ thì nó cũng có nghiệm nguyên.
Bài 8. Kí hiệu nF là số Fibonacci thứ n, gọi p là một số nguyên tố. Chứng minh rằng p
chia hết 5
Bài 9. (Barbeau) Chứng minh rằng một số chính phương n là hiệu của lập phương hai
số tự nhiên liên tiếp, thì n là tổng bình phương của hai số tự nhiên liên tiếp.
Bài 61. Chứng minh rằng nếu p là số nguyên tố chia hết cho hai số có dạng 2 + n
2 1,2 + m , thì nó cũng chia hết cho một số nào đó có dạng 4 1+ k .
Bài 10: Tìm số tự nhiên n nhỏ nhất sao cho 2 41 ++nn không phải là một số nguyên tố
Bài 11: Gọi A là tập hợp các ước số nguyên tố của 23 n , với ∈ ℕ n . Gọi P là tập hợp tất cả các số nguyên tố. Chứng minh rằng các tập hợp , AP \ A đều là các tập vô hạn.
Bài 12. (AMM, Barry Powel)
Cho 43=+pk là một số nguyên tố, và giả sử ,,, xyzt là các số nguyên sao cho
2222 ++= ppppxyzt . Chứng minh rằng p chia hết tích xyzt
Bài 13: Chứng minh rằng tồn tại vô số số tự nhiên dạng 187914 +−− mmmm chia hết cho
59, ở đây m là số tự nhiên lẻ.
Bài 14. (Gabriel Dospinescu) Cho p là một số nguyên tố dạng 41 + k sao cho
2 |22 p p . Chứng minh rằng ước số nguyên tố q lớn nhất của 21 p thoã mãn
2(6) > qp p
Bài 15. (Gabriel Dospinescu) Cho ,, abc là các số nguyên dương sao cho 2 4 bac
không phải là một số chính phương. Chứng minh rằng với mỗi 1 > n tồn tại n số nguyên dương liên tiếp sao cho không có số nào biểu diễn được dưới dạng 22 () ++ z axbxycy , với các số nguyên , xy và số nguyên dương z
Bài 16: Giải phương trình đồng dư 2 0(mod) ++≡ rxsxtm với ,,, ∈ ℤ rstm
Bài 17. Chứng minh rằng với mọi ∈ ℕ n và số nguyên tố p sao cho 1(mod4) ≡ p thì tồn tại ∈ ℤ x để: 2 |1 + n px .
Bài 18. Chứng minh rằng đồng dư 4 2(mod) ≡ xp có nghiệm với 1(mod4) ≡ p nếu và chỉ
Bài 19. (Hungarian practice problem sheet, 1999) Cho , xy là các số nguyên và p là số
nguyên tố dạng 43 + k , sao cho 22 1 | 4 + −+ p pxxyy . Chứng minh rằng tồn tại các số nguyên , uv sao cho 2 −+xxy 22211 () 44 ++ =−+ pp ypuuvv
Bài 20. Chúng ta đã biết rằng các số Fibonacci mod n là tuần hoàn với mỗi n. Tìm độ
dài chu kì của số Fibonacci mod524287 . (chú ý 19 52428721 =− là số nguyên tố).
Bài 21. Chứng minh rằng có vô số số nguyên x sao cho 21 , 521 +
x x x các số đều là các số tự nhiên.
Bài 22. Chứng minh rằng với mỗi số nguyên tố 1(mod4) ≡ p , thì số 1 1 mp m p p là hợp số với mỗi ∈ ℕ m .
Bài 23. (Corvaja và Zannier) Chứng minh rằng nếu 1 1
n n a b là số nguyên với vô số ∈ ℕ n . Chứng minh rằng = kab với ∈ ℕ k .
Bài 24. Hỏi có bao nhiêu số nguyên tố p sao cho tồn tại số nguyên m thoã mãn các đồng dư
3 320(mod)+−≡ mmp
2 450(mod)++≡ mmp
Bài 25. Chứng minh rằng với mỗi + ∈ ℕ k và ∈ℤ c sao cho || c 2 ≥ thì tồn tại vô hạn các số nguyên tố dương 1(mod2) ≡ k p sao cho 1
=−
c p .
Bài 26. Chứng minh rằng có vô số số nguyên tố Gauss.
Bài 27. Chứng minh rằng với mỗi ∈ ℕ n , thì tồn tại các số nguyên nguyên tố cùng nhau , ab ( không cần dương) sao cho
33
13 + = + n ab ab .
Bài 28. Cho , pq là các số nguyên dương lẻ với 21,21 =+=+paqb . Giả sử gcd(,)1 = pq . Gọi k là số các điểm nguyên (,) xy sao cho 0,0, <<xy 2() + xpyq .pq < Chứng minh rằng abk là số chẵn.
PHẦN C. KẾT LUẬN VÀ KIẾN NGHỊ
nếu p có dạng 22 64 + AB với , AB là các số nguyên dương. 342
Bài viết này trình bày các vấn đề cơ bản về thặng dư bình phương và những áp dụng của nó vào giải một số bài toán hay và khó trong các bài thi Olympic. Tác giả hy
vọng rằng bài viết này sẽ là một tài liệu tham khảo bổ ích cho các thầy, cô và các em học sinh chuyên toán. Tác giả rất mong nhận được những góp ý quý báu từ các thầy, cô để chuyên đề được hoàn thiện sâu sắc hơn nữa. Xin trân thành cám ơn!
TÀI LIỆU THAM KHẢO
[1] http: Mathlinks.ro
[2] Quadratic Congruences, Dusan Djukic.
[3] Exercises in Number Theory, D.R Parent.
[4] Elementary theory of numbers, Sierpinski.
[5] An Introduccion to the Theory of Numbers, Hardy, Wright.
[6] Reciprocity Laws From Euler to Eisenstein, Franz Lemmermeyer.
[7] Problems and Theorem in Analysis II, G. Polya, G.Szego.
[8] Problem in algebraic number theory, Ram Murty.
[9] Wining Solutions, E. Lozansky, C. Rousseau.
[10] Đề thi Olympiad các nước.
[11] Số học, Hà Huy Khoái, NXB Giáo dục – 2004.
[12] Legendre symbol, https://lovetoan.wordpress.com/de-thi-hsg/
[13] Các bài giảng về số học, Nguyễn Vũ Lương, Nguyễn Lưu Sơn, Nguyễn Ngọc Thắng, Phạm Văn Hùng, NXB Đại học Quốc Gia Hà Nội - 2006.
[14] Một số chuyên đề toán học chọn lọc bồi dưỡng học sinh giỏi, Nguyễn Văn Mậu, Bùi Công Huấn, Đặng Hùng Thắng, Trần Nam Dũng, Đặng Huy Ruận, Tài liệu bồi dưỡng
HSGQG năm – 2004.
SỬ DỤNG PHI HÀM EULER
ĐỂ GIẢI TOÁN SÔ HỌC QUA CÁC KỲ THI OLYMPIC
BẢNG CÁC KÝ HIỆU
ℕ : Tập hợp các số tự nhiên : { }0;1;2;3;...
*
N : Tập hợp các số tự nhiên khác 0 : { }1;2;3;...
Z: Tập hợp các số nguyên : { }...;3;2;1;0;1;2;3;...
℘: Tập hợp các số nguyên tố
Q : Tập hợp các số hữu tỉ
C: Tập hợp các số phức
R : Tập hợp các số thực
x
∈Z : x thuộc Z; x là số nguyên
ab ⋮ : a chia hết cho b , a là bội của b
I. LÝ DO CHỌN ĐỀ TÀI
Ngạn ngữ Pháp có câu: "Le Mathématique est le Roi des Sciences mais
/
ab
⋮ : a không chia hết cho b
| ba : b là ước của a , b chia hết a
|
ba / : b không là ước của a
(
) mod abm ≡ : a đồng dư với b theo môđun m , ab chia hết cho m
( ) , ab : ƯCLN của a và b
[ ] , ab : BCNN của a và b
( ) ; ab : Cặp số,nghiệm của phương trình hai ẩn số
: Suy ra
⇔ : Tương đương với,khi và chỉ khi
(đpcm), ∆ : Điều phải chứng minh, kết thúc bài toán hay một phép chứng minh
∃, ,,
∀∨∩ : Tồn tại,mọi,hoặc, giao
( ) n ϕ : Phi-hàm Euler của số nguyên dương n
L’Arithmétique est la Reine", dịch nghĩa:"Toán học là vua của các khoa học nhưng Số học là Nữ hoàng". Điều này nói lên tầm quan trọng của Số học trong đời sống và khoa học. Số học giúp con người ta có cái nhìn tổng quát, sâu rộng hơn, suy luận chặt chẽ và tư duy sáng tạo. Phi- hàm Euler là một trong các hàm số học, cùng với các định lý, tính chất gắn liền với nó, có thể được xem là một trong những công cụ mạnh nhất để giải quyết gọn đẹp rất nhiều bài toán Số học. Những bài toán Phi- hàm Euler xuất hiện thường xuyên trong các kì thi học sinh giỏi tỉnh, quốc gia và quốc tế và ngày càng trở nên quen thuộc đối với những người yêu toán. Đặc biệt việc sử dụng Phi -hàm Euler để chứng minh một số bài toán cơ bản của số học. Ngoài việc sử dụng Phi -hàm Euler, ta còn sử dụng thêm một số hàm số học khác như, Hàm số các ước nguyên dương, Hàm tổng các ước nguyên dương. Sử dụng tính chất chia hết, tính chất của số nguyên tố, ƯSCLN, BSCNN của các số nguyên, số chính phương, thặng dư bình phương, số mũ lớn nhất của số nguyên tố của một số… Chiều ngược lại để giải một số bài toán số học như : Chia hết, chứng minh một số có một tính chất số học nào đó, giải phương trình nghiệm nguyên, chứng minh bất đẳng thúc số học… Ta cần sử dụng thuần thạo, định nghĩa và các tính chất của Phi- hàm Euler… Bài viết này sẽ giúp bạn giải quyết tốt một vài vấn đề nhỏ các bài toán Số học Xuất phát từ những ý nghĩ đó tôi đã sưu tầm và hệ thống lại một số bài toán về Phi hàm Euler để viết lên chuyên đề " Sử dụng Phi -hàm Euler để giải toán Số học qua các kỳ thi Olympic ”. Chuyên đề gồm các phần :
- Phần A : Đặt vấn đề
- Phần B : Nội dung
- B1: Kiến thức cơ bản.
-B2: Sử dụng Phi-hàm Euler để giải toán số học qua các kỳ thi Olympic
- Phần C- Kết luận và kiến nghị
- Phần D: Tài liệu tham khảo
Mục tiêu ở đây là một số bài mẫu, một số bài khác biệt căn bản đã nói lên được phần chính yếu của chuyên đề. Tuy vậy, những thiếu sót nhầm lẫn cũng không thể tránh khỏi được tất cả, về phương diện chuyên môn cũng như phương diện sư phạm. Lối trình bày bài giải của tôi không phải là một lối duy nhất. Tôi đã cố gắng áp dụng cách giải cho phù hợp với chuyên đề, học sinh có thể theo mà không lạc hướng. Ngoài ra lúc viết tôi luôn luôn chú ý đến các bạn vì nhiều lí do phải tự học, vì vậy giản dị và đầy đủ là phương châm của tôi khi viết chuyên đề này.
-
Tôi xin trân thành cảm ơn các thầy cô giáo, các em học sinh góp ý thêm cho những chỗ thô lâu và phê bình chân thành để có dịp tôi sửa chữa chuyên đề này hoàn thiện hơn.
II. MỤC ĐÍCH VÀ NHIỆM VỤ NGHIÊN CỨU
- Nghiên cứu vai trò của “Phi hàm Euler ” trong giải các bài toán số học, đại số sơ cấp.
- Vận dụng “Sử dụng Phi- hàm Euler để giải toán số học” trong các tình huống cụ thể nhằm phát huy khả năng tư duy toán học cho học sinh.
Đề xuất một số biện pháp nhằm rèn luyện, phát huy năng lực tư duy và giải các bài toán trong phần “Phi- hàm Euler ” của chương trình chuyên toán THPT.
III. PHƯƠNG PHÁP NGHIÊN CỨU
Trong bản sáng kiến kinh nghiệm sử dụng các phương pháp nghiên cứu chủ yếu sau:
- Phương pháp nghiên cứu lí luận: Nghiên cứu các tài liệu chuyên về “Phi- hàm Euler, hàm số học hàm trên tập số nguyên” đặc biệt là các tài liệu liên quan đến “ Số học và Phi
- Hàm Euler” trong các tạp chí trong và ngoài nước; tài liệu từ Internet...
- Phương pháp trao đổi, tọa đàm (với giáo viên, học sinh các lớp chuyên toán).
- Phương pháp tổng kết kinh nghiệm.
IV. GIẢ THUYÊT KHOA HỌC
Nếu học sinh được học chuyên sâu theo chuyên đề như trên sẽ phát triển năng lực tư duy Toán học, đặc biệt là có phương pháp để giải quyết các bài toán về Số học và phương trình trên tập số nguyên. Đây là phần khó với học sinh các lớp chuyên toán.
V. BỐ CỤC
Bản sáng kiến kinh nghiệm gồm ba phần chính:
Phần A- ĐẶT VẤN ĐỀ.
Phần B- NỘI DUNG
B1 : Kiến thức cơ bản.
B2 : Sử dụng Phi- hàm Euler để giải toán số học
I. Sử dụng: Phi- hàm Euler giải các bài toán chứng minh trong số học.
II. Sử dụng: Phi- hàm Euler giải các bài toán chứng minh sự tồn tại trong số học .
III. Sử dụng: Phi- hàm Euler giải bài toán phương trình nghiệm nguyên.
IV: Bài tập tương tự.
V. Kết quả sau khi áp dụng đề tài
Phần C- Kết luận và kiến nghị
PHẦN B : NỘI DUNG
B1: KIẾN THỨC CƠ BẢN
Định nghĩa 1. Giả sử n là một số nguyên dương. Phi hàm Euler được định nghĩa là các số nguyên dương không vượt quá n và nguyên tố cùng nhau với n
Ký hiệu Phi hàm Euler qua ( )n ϕ
Ví dụ : ( ) ( ) ( ) ( ) ( ) 11,21,32,42,54 ϕϕϕϕϕ===== .
Định nghĩa 2. Một hệ thặng dư thu gọn modulo n là một tập hợp gồm ( )n ϕ số nguyên sao cho mỗi phần tử của tập hợp đều nguyên tố cùng nhau với n
Ví dụ : Tập hợp { }1,3,5,7 là một hệ thặng dư thu gọn modulo 8 . Tập hợp { }3,1,1,3
cũng là một hệ thặng dư thu gọn modulo 8
Định lý 1. Giả sử () { } 12,,..., n rrr ϕ là hệ thặng dư thu gọn modulo n , a là số nguyên dương và ( ) ,1an = . Khi đó tập hợp () { } 12,,..., n ararar ϕ cũng là hệ thặng dư thu gọn modulo n .
Chứng minh. Trước tiên ta chứng tỏ rằng, với mỗi số nguyên jar là nguyên tố cùng nhau với n . Giả sử ngược lại ( ) ,1 j arn > với j nào đó. Khi đó tồn tại ước nguyên tố p của ( ) , j arn . Do đó, hoặc | pa , hoặc | jpr , tức là hoặc | pa và | pn , hoặc | pr và | pn Tuy nhiên, không thể có | jpr và | pn , vì jr và n nguyên tố cùng nhau. Tương tự, không thể có | pa và | pn . Vậy, ar và n nguyên tố cùng nhau với mọi ( ) 1,2,..., jn ϕ = .
Còn phải chứng tỏ không có hai số ( ) , jk ararjk ≠ nào đồng dư với nhau modulo n Giả sử ( ) mod, jk ararnjk ≡≠ và ( ) ( ) 1;1jnkn ϕϕ≤≤≤≤ . Vì ( ) ,1an = nên ta suy ra ( ) mod jk rrn ≡ Điều này mâu thuẫn vì , jkrr cùng thuộc hệ thặng dư thu gọn ban đầu modulo n .
Ví dụ : Tập hợp { }1,3,5,7 là một hệ thặng dư thu gọn modulo 8 . Do ( ) 3,81 = nên tập hợp { }3,9,15,21 cũng là một hệ thặng dư thu gọn modulo 8 .
Định lý 2. (Định lý Euler). Giả sử m là số nguyên dương và a là số nguyên với ( ) ,1am = . Khi đó ( ) ( ) 1mod m am ϕ ≡
Chứng minh. Giả sử () { } 12,,..., m rrr ϕ là một hệ thặng dư thu gọn gồm các số nguyên dương không vượt quá m và nguyên tố cùng nhau với m . Do Định lý 1 và do ( ) ,1am = , tập hợp
() { } 12,,..., m ararar ϕ cũng là một hệ thặng dư thu gọn modulo m .. Như vậy các thặng dư dương bé nhất của () { } 12,,..., m ararar ϕ sẽ là tập () { } 12,,..., m rrr ϕ xếp theo thứ tự nào đó. Vì
thế, nếu ta nhân các hạng tử trong hệ thặng dư thu gọn trên đây, ta được :
()() ( ) 1212 .......mod mm arararrrrm ϕϕ ≡ .
Do đó
Vì () ( ) 12...,1 m rrrm ϕ = nên
( ) ()() ( ) 1212 .......mod m mm arrrrrrm ϕ ϕϕ ≡
( ) ( ) 1mod m am ϕ ≡ □
Ta có thể tìm nghịch đảo modulo m bằng cách sử dụng định lý Euler. Giả sử , am là các số nguyên tố cùng nhau, khi đó ( ) ( ) ( ) 1 .1mod mm aaam ϕϕ=≡
Vậy ( ) 1 m aϕ là một nghịch đảo của a modulo .m
Ví dụ : ( ) ( ) 91 615 222325mod9 ϕ ===≡ là một nghịch đảo của a modulo 9
Cũng có thể giải các đồng dư tuyến tính một ẩn theo nhận xét trên. Giả sử cần phải đồng dư ( ) ( ) mod,,1axbmam≡= . Ta nhân hai vế với ( ) 1 m aϕ thu được ( ) ( ) ( ) 11 mod. mm aaxabm ϕϕ ≡
Như vậy, nghiệm là các số nguyên x sao cho ( ) ( ) 1 mod m xabm ϕ ≡ .
Ví dụ : Giải đồng dư ( ) ( ) ( ) 101 3 37mod10:37379mod10 xx ϕ ≡≡⋅≡⋅≡ Định lý 3. Với số nguyên tố p ta có : ( ) 1 pp ϕ =− . Ngược lại, nếu p là số nguyên dương sao cho ( ) 1 pp ϕ =− , thì p là số nguyên tố
Chứng minh. Nếu p là số nguyên tố thì mọi số nguyên dương nhỏ hơn p đều nguyên tố
cùng nhau với p . Do có 1 p số nguyên dương như vậy nên ( ) 1 pp ϕ =− .
Ngược lại, nếu p là hợp số thì p có ước ,1 ddp << . Tất nhiên p và d không nguyên tố cùng nhau. Như vậy, trong các số 1,2,...,1 p phải có những số không nguyên tố cùng nhau với p , nên ( ) 2 pp ϕ ≤− . Theo giả thiết ( ) 1 pp ϕ =− , vậy p là số nguyên tố Định lý 4. Giả sử p là số nguyên tố và a là số nguyên dương. Khi đó ( ) 1 aaa ppp ϕ =−
Chứng minh. Các số nguyên dương nhỏ hơn a p không nguyên tố cùng nhau với p l ;à các số không vượt quá 1 a p và chia hết cho p . Có đúng 1 a p như vậy. Do đó tồn tại 1 aa pp số nguyên nhỏ hơn a p và nguyên tố cùng nhau với a p . Vậy ( ) 1 aaa ppp ϕ =−
Ví dụ : ( ) ( ) ( ) 332100109 125555100;222512 ϕϕϕ==−==−= .
Định lý 5. (Định lý về tính nhân tính của Phi- hàm Euler). Nếu , mn là các số nguyên dương nguyên tố cùng nhau, thì ( ) ( ) ( ) .. mnmn
Chứng minh. Ta viết các số nguyên dương không vượt quá mn thành bảng
Bây giờ giả sử r là một số nguyên không vượt quá m . Giả sử ( ) ,1.mrd=> khi đó, không có số nào trong dòng thứ r nguyên tố cùng nhau với mn , vì mỗi phần tử của dòng đó đều có dạng kmr + , trong đó ( ) 11,|kndkmr ≤≤−+ vì |,| dmdr .
Vậy để tìm các số trong bảng mà nguyên tố cùng nhau với mn , ta chỉ cần xem các dòng thứ r với ( ) ,1.mr = Ta xét một dòng như vậy, nó chứa các số ( ),,...,1 rmrnmr +−+ .
Vì ( ) ,1mr = nên mỗi số nguyên trong dòng đều nguyên tố cùng nhau với n . Như vậy, n số nguyên trong dòng lập thành hệ thặng dư đầy đủ modulo n . Do đó có đúng ( )n ϕ số
trong hàng đó nguyên tố cùng nhau với n . Do các số đó cũng nguyên tố cùng nhau với
m nên chúng nguyên tố cùng nhau với mn .
Vì có ( )m ϕ dòng, mỗi dòng chứa ( )n ϕ ssos nguyên tố cùng nhau với mn
Nên ta suy ra ( ) ( ) ( )mnmnϕϕϕ =
□
Kết hợp hai định lý trên ta được
Định lý 6 ( Công thức Phi-hàm Euler). Giả sử 12 12 ... ka aa knppp = là phân tích n ra thừa số
nguyên tố. Khi đó
⋯
Chứng minh. Vì ϕ là hàm có tính chất nhân nên nếu n có phân tích như trên, ta được :
===
ϕϕ
B2 : Sử dụng Phi- hàm Euler để giải toán số học qua các kỳ thi Olympic I. Sử dụng: Phi- hàm Euler giải các bài toán chứng minh trong số học Bài I.1.
Lời giải. Rõ ràng f có tính chất nhân nên với mọi số nguyên dương k và với mọi số nguyên tố p thì ta có () ( ) ( ) () 1 11 1 k kk k kk p p ppp fp fp ppppp ϕ ϕ ===−===
Vậy từ đó ta suy ra điều phải chứng minh.
Bài I.2.
Cho n là số nguyên dương. Dãy số { } 1 in ∞ = được xác điịnh bởi ( ) 1 nn ϕ= và ( ) 1 ,1. kk nnk ϕ + =∀≥ Chứng minh rằng tồn tại số nguyên dương r sao cho 1 r n =
Lời giải.
Trước hết ta có ( ) 11 kk ϕ ≤≤− khi 1 k > . Do đó nếu 2 n ≥ thì ( ) 1 nnn ϕ>= , từ đây
k k
111 11...1 111 ...111 111 111
a aa k k a aa k k k ϕ =−−− =−−− =−−− ⋯ □
nppp ppp ppp ppp
n ppp
12 12 12 12 12
Chứng minh. Thật vậy, bằng cách đặt ( ) { } * |,, d Cmmnmnd =∈≤= ℕ .
suy ra được 12 1 nnn>>>≥ ⋯ với ( ) 1 nn ϕ= và ( ) 1 ii nn ϕ= với 1 i > .
Suy ra dãy { } 1 in ∞ là một dãy số nguyên dương giảm dần, vậy sẽ tồn tại một số nguyên
dương r sao cho 1, r n = ta có điều phải chứng minh.
Bài I. 3.
Chứng minh rằng nếu n là một số nguyên dương có k ước số nguyên tố lẻ khác nhau thì ( )n ϕ chia hết cho 2. k
Lời giải.
Hiển nhiên nếu p là số nguyên tố lẻ thì 2|1, p vậy từ đó ta suy ra () () |||
nC = . Chú ý rằng
Thì tập { } 1,2,...,n được phân hoạch thành các tập dC đôi một rời nhau. Do đó d dn
* * |,,1|,,1 d mnnn Cmmnmmm dddd =∈≤==∈≤=
với |
11 11 pnpnpn pn =−==− ∏∏∏ ∏
n p ∈ ∏ Z Từ đây ta suy ra ( ) ( ) ( ) | 12, k pn pnn ϕϕ ∏ trong đó k là số ước
. Pn
nguyên tố lẻ phân biệt của n .
Bài I.4.( Diễn đàn toán học VMF)
351
Cho n là một số nguyên dương lẻ thỏa mãn 5 n ≥ và có các ướ nguyên tố là
{ } 12,,..., kppp . Chứng minh rằng ( ) 21 n ϕ có các ước nguyên tố không thuộc tập hợp { } 12,,..., kppp .
Lời giải.
Ta xét hai trường hợp sau
Trường hợp 1. Nếu n là số nguyên tố thì ( ) 1 nn ϕ =−
Giả sử ) 1 2121 n n ϕ −=− không có ước nguyên dương nào ngoài n từ đây ta suy ra
1 21nk n −= ( )* với * k ∈ ℕ
• Nếu ( ) * 2 khh=∈ ℕ thì từ ( )* ta suy ra 1 2 11 22 1 2
n h nn hh h n h
−= −+=⇔ = += vô lý
21 2210 21
n nnn n
• Nếu ( ) * 21, nhh=+∈ ℕ thì từ ( )* ta suy ra ( )( ) 121221 2111 nhhh nnnnn −+=+=+−+−+ ( ) * 12, r nrrn +=∈< ℕ kết hợp với ( )* ta được
( ) ( ) ( ) ( )
1 22 1111 k mmmm ϕϕϕϕ =−+++
1 2 2 1 k mmnm ϕϕϕ<=+=
( ) ( ) ( )
( ) ( ) 2 12mnϕϕ <+=+
Với vô hạn các cặp số nguyên dương ( ) , mk , trong đó m là số chẵn. Vậy từ đây ta suy ra
tồn tại vô hạn số nguyên dương n thỏa mãn
( ) ( ) ( )12nnnϕϕϕ<+<+
Ta có điều phải chứng minh.
Bài I.6 (Serbia Mathematical Olympyad 2011).
Cho n là một số nguyên dương lẻ thỏa mãn ( )n ϕ và ( )1 n ϕ + đều là lũy thừa của 2 .
Chứng minh rằng 1 n + là lũy thừa của 2 hoặc là 5. n =
Lời giải.
Nếu () () * 1 ,,,21, k a iiii npppa = =∈℘=∈ ∏ ℕ ( hay n là hợp số lẻ ).
Ta có ()() 1 1 1. k a ii i npp ϕ =
−+ −= −= vô lý ( trái với định lý Zsigmondy)
n n
nh r
12121 21
Vậy với n là số nguyên tố thì ( ) 1 2121 n n ϕ −=− không có ước nguyên dương nào
khác n Trường hợp 2. Nếu n là hợp số () * 1 ,, k iii i npp α α = =∈℘∈ ∏ ℕ thì ta thấy rằng
( ) { } 121,1,...,1 knppp ϕ >−−− . Nhưng theo định lý Zsigmondy thì ( ) 21 n ϕ có một
ước nguyên tố mà ( ) 1 21,1,2,..., p ik−= không có. Theo định lý Fermat thì 1 |21 p p do p là số nguyên tố lẻ. Vậy suy ra ( ) 21 n ϕ có ước nguyên tố khác p . Từ đây ta có
( 21 n ϕ có các ước nguyên tố khác { } 12,,..., kppp .
Bài I.5. Chứng minh rằng tồn tại vô hạn số nguyên dương n thỏa mãn ( ) ( ) ( )12nnnϕϕϕ<+<+
Lời giải.
Ta chọn ( ) 2*1,,,2 k nmmkm =−∈ ℕ⋮ . Khi đó ta có () ( ) ()()( ) ( ) 1 2 22 1111...1 k k nmmmmmϕϕϕ =−=−+++
=− ∏ Vì ( )n ϕ là lũy thừa của 21 a = và 12b ip −= ( với * ib ∈ ℕ phân biệt với 1,2,.., ik = ), từ đó suy ra ( ) * 21,1,2 b c ii pikbc =+∈℘∀= =∈ ℕ . Như vậy 2 21, c ip =+ với mỗi * ic ∈ℕ phân biệt. Giả sử rằng 1 n + không là lũy thừa của 2 . Thì từ ( )1 n ϕ + là lũy thừa của 2 , ta thấy rằng tất cả các ước nguyên tố lẻ của 1 n + đều có dạng ( ) 2* 21,,1,2,...,. d djl+∈= ℕ
Do đó ( ) ( ) 2 2 1 1 21,1221 d k t i j nn = = =++=+
Ở đây các phần tử của mỗi bộ ( ) 12,,..., kccc và ( ) 12,,..., lddd là phân biệt.
Không mất tính tổng quát ta giả sử rằng 12 kccc <<< ⋯ và 12 ddd <<< ⋯
Với mỗi * ,, mMmM ∈≤ ℕ , ta chứng minh bằng quy nạp được rằng
1
22222 22222
mimMm mimMm
++− <=⋅<⋅
21212212 2221221
+ + = ∏
M im
Điều này cho chúng ta 1 1 11
212 22 221
22 22
c cc cc n + ⋅≤<⋅ và
22 22
1 1 11
212 22 221
d d dd dd n + ⋅≤<⋅
Với 1
k i i cc = và 1 j dtd =+ , từ đó ta được cd = .
11
Nếu 11 , dc > ta có
Do đó 11dc < , suy ra
dc dc nn + < +< (vô lý).
11
22 22
221 1 212
dc dc cc nn + +≥⋅>⋅> , nên ta có :
11 11
212 122 221
22 22
Tuy nhiên, ta biết rằng () x ϕ là số chẵn với mọi 3 x ≥ nên dãy () n u kể từ số hạng thứ hai trở đi sẽ chứa toàn số lẻ, do đó 1 33 nn uu + =⇔= hay tất cả các số hạng trước đó cũng thế, vô lý. Vì thế nên nếu n u là số nguyên tố thì nó phải lớn hơn 3.Cuối cùng, giả sử rằng 20182 u không phải là số nguyên tố, khi đó tất cả các số trước đó cũng thế Để ý rằng nếu x không phải là số nguyên tố thì ()2 xx ϕ ≤− nên 1 ()11.nnn uuu ϕ + =+≤− Điều này có nghĩa là mỗi lần, giá trị của các số hạng giảm đi ít nhất 1 đơn vị.
Ta có 20192018018
n n ++−
12121 22
>⋅ . Và bởi vì
Bài I.7.
11 11
22 22
dc dc
1 22 211 c na≥+≥+ với ( )( )
2 (22)1(21)121 u ϕϕ 2 =−+=−+≤− nên 20182018 2018 2 221 1(22)1,uuu≤−≤≤−−≤ vô lý.
1 2 2 2 33
11 11 2,1. d aa naa a naa +− +−−
Từ các điều trên ta nhận được 3 2 2 1. 1 a an aa +≤< Bất đẳng thức này xẩy ra chỉ khi
2 a = và 5. n =
Cho 2019 22 a =− . Xét dãy số () n u với 1 ua = và 1 ()1,nn uu ϕ + =+ với 1. n ≥
a) Chứng minh rằng 264 a chia hết cho a và số a có ít nhất 4 ước nguyên tố phân biệt.
b) Chứng minh rằng 20182 u là số nguyên tố lớn hơn 3.
Lời giải.
Vì thế nên 20182 u là một số nguyên tố lớn hơn 3.
Bài I.8.
a) Xét số Fermat 2 21 n n F =+ ,chứngminhrằng 0
F F > ∏ vớimọi n
1 1 2 n i i
b)Xétcácsố nguyên , ab thỏamãn ()2. nn an ϕ +=
i.Chứngminhrằng 2 a ≥ vàkhi 2 a = thì 1. n =
ii.Chứngminhrằngkhi 3 a ≥ thì 3. n ≤ Từ đótìmtấtcả cácbộ số thỏa điềukiệntrên. Lời giải.
a)Tacó 011 22221 01 (1)(1)(1)22 nn n FFF + +++− −−−== ⋯ và 01 1 2222 12 (21)(21)(21)21 nn n FFF + =+++=− .
a) Ta có
201810091009 2(21)2(21)(21) a =−=−+ . Ta thấy 10091009 21,21 −+ là hai số lẻ liên tiếp nên chúng nguyên tố cùng nhau, nghĩa là tập ước nguyên tố của hai số này là rời nhau.
Khi đó, ta chỉ cần chứng minh rằng một trong hai số này có ít nhất 2 ước nguyên tố là
được. Ta thấy 10091009 21(1)10(mod3) +≡−+≡
nên số 1009 21 + chia hết cho 3. Ngoài ra, ta có 1009 33 (21)(21)1vv+=+= nên số này chỉ chia hết cho 3 và không chia hết cho 9, chứng tỏ nó còn một ước nguyên tố khác 3. Suy ra a có ít nhất 4 ước nguyên tố Tiếp theo, ta thấy 6 26464(21) aa −=− . Ta sẽ chứng minh rằng 6 a chia hết cho 2018. Do 201821009 =⋅ và 1009 là số nguyên tố nên theo định lý Fermat nhỏ thì 1008 21(mod1009) ≡ . Suy ra 2019310082 6282(2)80(mod1009) a −=−=⋅−≡ nên suy ra 6 a chia hết cho 2018 Đặt 62018 ak −= thì 62018 2121 ak −=− chia hết cho 2018 21 nên 264 a chia hết cho a b) Ta thấy rằng nếu n u là số nguyên tố thì 1 ()1 nnn uuu ϕ + =+= nên dãy hằng kể từ đó.
Ngoài ra, dễ thấy rằng nếu 1 3 n u + = thì
()13()2{3,4,6} nnn uuuϕϕ+=⇔=⇔∈ .
Cầncó 1 11 1
21 22 2 21 221 2 21
+ ++ + >⇔>− , đúng.
n nn n
b) Với 1, a = ta có ()(1)2 nn annnϕϕ+=+≤< với mọi n + ∈ ℤ nên phương trình đã cho không có nghiệm.
Xét 2, a = nếu 2n mn =+ là số nguyên tố thì 2()121 nn mmn ϕ ==−=+− nên 1. n = Ngược lại, nếu m là hợp số thì gọi p là ước nguyên tố nhỏ nhất của nó. Suy ra
2()22. n nn m mmmmnn p ϕ =≤−≤−=+−+
Do đó 2 2n nn+≤ , cũng không thỏa.
Xét 3, a ≥ đặt 2 21 n n F =+ với số nguyên 0. n ≥
Vì (2) n n ϕ + là lũy thừa của 2 nên 12 22 nk snppp +=⋅… (vì nếu 2 ||() pxpx ϕ → nên các lũy thừa của số nguyên tố lẻ đều phải là 1).
Ngoài ra, 1 12 (2)2(1)(1)(1) nk snppp ϕ +=⋅−−− ⋯ nên với mỗi 1 is≤≤ thì 1 ip là lũy
thừa của 2, kéo theo 12 12
Khi 3 l = thì ( ) 11 333 21.4.(1)2.3.0,2, aa ppap++ −+=⇒== vô lý.
V
ppp ppp ⋅> theo câu a).
1111 2 s s
Suy ra nếu 0 k = thì 21 2 n n an > + , còn nếu 1 k ≥ thì 21 4 n n an > + . Do đó, ta luôn có 2 2. nn an + >+
nn a n >≥ ≤
ậy phương trình ban đầu có tất cả 4 nghiệm phân biệt là (2,1),(3,3),(3,1),(5,1).
Bài I.9.
Cho n là số nguyên dương thỏa mãn () n ϕ (hàm Euler) là lũy thừa của 2.
a) Chứng minh rằng mọi ước nguyên tố lẻ (nếu có) của n đều có dạng 2 21 k + với k ∈ ℕ
b) Tìm n biết rằng n là số hoàn hảo.
(số hoàn hảo là số bằng với tổng các ước nguyên dương nhỏ hơn nó)
Lời giải.
a) Giả sử ()2m n ϕ = với . m + ∈ ℤ Theo công thức tính hàm phi Euler, ta có
1 | ()(1)2 tm pn npp ϕ =−= ∏ với (). p vnt =
Dễ thấy rằng khi đó, ta phải có 1 t = với mọi ước nguyên tố lẻ p của n vì nếu không thì
|2, m p vô lý.
Ngoài ra, 1 p cũng phải là lũy thừa của 2.
Đặt 21 s p =+ với s + ∈ ℤ . Nếu 1 s > và s có ước nguyên tố lẻ là q thì đặt squ = với
1 u ≥ thì 21(2)1 suq+=+ chia hết cho 21 u + , vô lý.
Suy ra s phải là lũy thừa của 2 và đặt 2k s = thì dẫn đến mọi ước nguyên tố lẻ (nếu có)
của n thì đều phải có dạng 2 21. k +
b) Nếu 2a n = thì 1 ()212, a nn σ + =−< không thỏa mãn.
Đặt 23 2..., a l npppa=∈ ℕ với ,2,3,...,(2)i pill=≥ là các ước nguyên tố lẻ tăng ngặt của n thì 1 1 22 ()(21)(1)...(1)2... a a ll npppp σ + + =−++= .
Nếu 2 5 p ≥ thì ( ) 221 1222213,2, kk ii i pi +=+=+≥ ⋮ vô lý.
Nếu 2 3 p = thì 3. l ≤
356
Vậy 6. n = Bài I.10 (USA TSTST-2016).
Cho , nk là các số nguyên dương sao cho : ( ) 1(()) ktimes
Chứng minh rằng 3. k n ≤ ( ký hiệu ( ) 11 ϕ = )
Lời giải.
n ϕϕϕ= ⋯⋯ .
Xét hàm số { } * :\1 f → ℕℕ thỏa mãn ( ) ( ) { } * 2,\1. fn nn ϕ =∀∈ ℕ Ta có ( ) 220nf = = .
Bổ đề 1.
• n chẵn, ( ) 12 * 12 2,,,,1,2,..., k tii npppkpit α αα α =∈∈℘∀= …ℕ kí hiệu ℘ là tập các
ước nguyên tố,ta có : ( ) ( ) ( ) 11 1 tt fnkfpfp αα=−+++
• n lẻ ta có ( ) ( ) ( ) ( ) 1122 tt fnfpfpfp ααα =+++ ⋯
Suy ra : Nếu a hoặc b lẻ thì ( ) ( ) ( )fabfafb =+
Nếu a , b đều chẵn thì ( ) ( ) ( ) 1 fabfafb=+−
Chứng minh bổ đề 1.
( ) () ( )
()()()()
1 22 11 22222121 kk ffkkkkkfffk ϕϕ== =− =−
Ta chứng minh quy nạp theo n. ( ) ( ) ( ) ( ) ( ) ( ) ( ) ( ) ( ) 1 2 1, fnfnnnfnfnnnϕϕϕϕϕ== =−< ( ) ( ) ( ) ( ) 111fpfpfp ϕ =+=−+ () ( ) ( )() () ( ) ( ) () () ( ) 1 1 1 1 1 1 1111 ...1...1...1...1 t tfnfppppfppfpp α α α α α α ϕ =−−=+−−
( ) ( ) ( ) ( ) ( ) ( ) ( ) 11 1 111111 ttt fnfpfpfpfpt αα =−++−+−++−+−+ ⋯⋯ ( ) ( ) ( ) 1122 tt fpfpfp ααα =+++
Bổ đề 2. ( ) 3 fn n ≤ nếu n lẻ và ( ) 2.3 fn n ≤ nếu n chẵn
Ta chứng minh bằng quy nạp theo n
• Nếu ()() ( ) 2 * 11 2212.32.32.22 f tttttntft =∈ =− =>= ℕ
• Nếu n là số nguyên tố ( ) ( ) 11 fnfn =−+
357
()() () () 1133 33.3231 22 fnfnfn nn==⋅⋅≥⋅−≥
• Nếu ( ) ( ) ( ) 1 111 t ttnppfnff α α αααα = =++
13133 logloglog tt ppnαα ≥++=
từ đó suy ra ( ) 3. fn n ≤
• ( ) ( ) ( ) 1 1 11 2...1 k t ttnppfnkff α α αααα = =−+++
() 1 11 1313313 1logloglog3...log3 2 kk ttt k n kppppα α αα ≥−+++==⋅
() () 11 1 33 323 22 kk fnfn kknnn ≥⋅ ⋅≥⋅≥
Từ hai bổ đề trên bài toán được chứng minh. Bài I.11 (USA TST 2015).
Cho ( )n ϕ là số các số nguyên dương nhỏ hơn n và nguyên tố cùng nhau với n. Chứng minh rằng tồn tại số nguyên dương m sao cho phương trình ( ) nm ϕ = có ít nhất 2015 nghiệm nguyên dương của n.
Lời giải.
Đặc biệt hóa. Lấy { }11,13,17,19,29,31,37,41,43,61,71
S = , ta thấy S có tính chất mọi
ước nguyên tố pS ∈ thì các ước nguyên tố 1 p thuộc { }2,3,5,7
Cho , xy là các số nguyên dương. Chứng minh rằng nếu với mọi số nguyên dương n thỏa mãn ( ) ( ) 2 121 n n xy−+ ⋮ thì 1 x = .
Lời giải.
Gọi p là số nguyên tố sao cho
()()( )( ) ( ) 21222 211111...1 n n pypxxxxx + −++++
Nếu chọn được ( )3mod4 p ≡ thì 2 |1,1 t pxt+∀≥ / . Từ đó suy ra ( ) 2 1 px
+
Ta cần phải chứng minh có vô hạn số nguyên tố p sao cho () 21 3mod4
≡
Bổ đề : { }* ,21, n Apppyn =∈℘+∈ ℕ thì A =+∞
Thậtvậygiả sử A chỉ cóhữuhạn ướcnguyêntố 12,,...,.ppp
Giả sử y lẻ,nếu y chẵnthìtathay n bởi ( ) 2 ny υ
.
NN MppNM pp ϕϕϕ ∈∈ ∈∈
=⋅=⋅−==
có các ước nguyên tố thuộc A và chọn TS
()
Vớimọi * u ∈ ℕ chọn ( ) 12...1 t npppu ϕ =+ theo địnhlýEuler’s: ( ) ( ) 12 12 21mod... pppu t pppu ϕ ≡ ( ) ( ) 12 1 * 12 2121mod... pppu t yypppuk ϕ + +≡+ ∃∈ ℕ saocho ( ) ( ) 12 1 12 21...21 pppu t ypppuky ϕ + +≡++
Lấy 21uy=+ ta được ( ) ( )( ) 12 1 12 2121...1 t pppu yypppk ϕ + +≡++
Chọn ( ) 12 ...1,,,1,1,2,...,. ti ppppkpppit +∈℘=∀= ( )1mod4 p ≡−
Thậtvậydo y lẻ ( ) ( )( ) ( ) 12 213mod421...11mod4 t yypppk +≡ ++≡
( ) 12...11mod4pppk +≡− từ đây có ( ) 12 1mod4...1 t pppppk ≡− + mâu thuẫn.
Vậy A =+∞
Bài I.13. ( Shotlits IMO 2006 ).
Chứng minh rằng với mọi số nguyên dương n. Tồn tại số nguyên dương m sao cho 2m m + chiahếtcho n .
Lời giải.
Tachứngminhquynạptheo n.
• 1 n = thỏamãn
• Giả sử bàitoán đúngvớimọisố nguyêndươngnhỏ hơn n.Tacầnchứngminhbài
toán đúngvới n.
Chọn ( ) ( ) * 2,,2221 x mxmxmtntxxmmtn =−∈< +=+− ℕ . Vì vậy ta chỉ cần
chọn x để saocho ( )mxn ϕ⋮ và ( ) 2 xn υ ≥
Trường hợp 1. Nếu ( ) * 2q nq=∈ ℕ ,chọn * 2,2 qm mqqmn =>∀∈ + ℕ⋮
Trường hợp 2. Nếu n có ướcnguyêntố lẻ suyra ( ) ( ) 2 2 n n υ ϕ ⋮
Ta có ( ) ( ) 22 xx mxtnxtnxn ϕ −=−−=−+ ⋮ . Để có điều này ta chỉ cần chọn x
saocho ( ) ( ) 2gcd, x xnn ϕ + ⋮ làxong.
Do ( ) ( ) gcd,nnn ϕ < nên theo giả thiết quy nạp tồn tại * , xxn ∈< ℕ để ( ) ( ) ( ) ( ) * 12122gcd,,, x xnnntnttt ϕϕ +=−∈ ⋮ℕ
* t ∃∈ ℕ để ( ) ( ) 2x xtnn ϕ −++ ⋮
2. Chứng minh rằng 122019 ... uuu chia hết cho 2018! 1009! ⋅
Lời giải.
1) Nhận xét : Nếu ( ) 2 xp ϕ = với 3 p > là số nguyên tố thì 21xp=+ hoặc ( )221xp=+ . Ta chứng minh nhận xét này qua các trường hợp sau.
• Nếu 2k xq = với * q ∈ ℕ lẻ và * ,2kk∈≥ ℕ thì: ( ) ( ) ( ) ( ) 1 22 kk xqq ϕϕϕϕ == , mà ( )q ϕ chẵn với mọi 3 q ≥ , mà ( ) 2 21 p υ = nên ta buộc phải có 2,1kq== tức là 4 x = , loại.
• Nếu 2 xq = với q lẻ thì ( ) ( ) ( ) ( ) 2 xqqϕϕϕϕ == nên ta có thể đưa về xét trường hợp xq = . Nếu q là hợp số ta có các khả năng sau:
Nếu q không là lũy thừa của số nguyên tố thì có thể viết 12qqq = với ( ) 12 gcd,1 qq = và 121,1qq>> là các số lẻ ( ) ( ) ( ) 12qqqϕϕϕ = chia hết cho
4 , không thỏa mãn.
Nếu k qr = với r là số nguyên tố lẻ, *,1kk∈> ℕ , thì ta có
( ) ( ) ( ) 1 1 kk qrrrϕϕ==− . So sánh ( ) 1 12 k rrp −= dễ thấy rằng 12 r −= và
⋮
Ta có ( ) ( ) ( ) ( ) ( ) ( ) ( ) ( ) 2 2gcd,2gcd,2,gcd, k xxxnnxnnknn υ ϕϕϕ +
) ( ) ( ) 2 1 22 2221 k xx xxkxk υ υυ + >+≥ +> ≥ Tiếp theo: ( )2221 mxmx mtn +=+−
2 k = kéo theo 3 p = , không thỏa mãn.
Nếu q là số nguyên tố thì ( ) 12 qqp ϕ =−= , suy ra 21xqp==+
Tóm lại : xq = hoặc 2 xq = , trong đó 21qp=+ là số nguyên tố. Từ đó suy ra:
() () () () ( ) 2
Chú ý: Nếu k có ước lẻ thì ( ) ( ) 2 2. k k υ ϕ ⋮
Bài I.14. (MOCK TEST VMO 2019).
..2
221221221 k k n u nu xmxxx
Với các số nguyên , ab nguyên tố cùng nhau và 1 ab>> , xét dãy số sau ( ) 2121 nn n uab ϕ =+ với 1,2,3,... n =
Trong đó ( )x ϕ là số các số nguyên dương không vượt quá x và nguyên tố cùng nhau với
x ( gọi là hàm Euler của x)
1. Chứng minh rằng nếu 3 p > là số nguyên tố lẻ và có số hạng nào đó của dãy số trên bằng 2 p thì 21abp+=+ hoặc ( )221abp+=+
360
() () 21212121 21 2 221 nnnn p abpab p ϕ + += += +
với 21 p + là số nguyên tố.
Ta biết rằng nếu 1 n > thì 2121 nnab + là hợp số vì nó chia hết cho ab + nên ( ) 2121 221 nn abp+=+
Suy ra 2121 nnab + chỉ có 4 ước ( )1,2,21,221 pp++ , trong khi số này lại có ước là
ab + với 2121 2 2 nnab ab + <+< , vô lý.
Do đó ta phải có 1 n = và 21abp+=+ hoặc ( )221abp+=+
2) Ta sẽ chứng minh bổ đề sau
Bổ đề. Với các số nguyên dương , ab nguyên tố cùng nhau và * n ∈ ℕ thì ( ) nnab ϕ + chia hết cho 2n .
Chứng minh. Thật vậy, số 2 cn = là giá trị nhỏ nhất của c để ccab chia hết cho
nnab + Vì nếu không, giả sử có 0 kn<< để nknkab++ chia hết cho nnab + thì ( ) ( ) knnnkk aabbab +−+ chia hết cho nnab + . Suy ra ( ) nkk bab + chia hết cho
nnab + . Mà ( ) gcd,1 ab = , ( ) gcd,1 nnn bab+= , kéo theo kkab + chia hết cho nnab + ,vô
lý.
Bằng cách tương tự, ta thấy rằng với mọi * c ∈ ℕ mà ccab chia hết cho nnab thì
2|nc . Mặt khác theo định lý Euler thì ( ) ( ) ( ) ( ) ( ) ( ) 1mod,1mod nn nn nnnn abab abab nnnn nnaabbababab ϕ ϕ ϕϕ + + ++ ≡+≡+ −+ ⋮
Điều này cho ta thấy ( ) nnab ϕ + chia hết cho 2n . Bổ đề được chứng minh.
Trở lại bài toán, ta có 122019uuu chia hết cho ( ) 2019 1009 21009!1320172018! 2132017 1009!1009! ⋅⋅== ⋯ ( điều phải chứng minh)
Bài I.15.( Balkan MO 2010)
Với mỗi số nguyên 2 n ≥ , ký hiệu ( )fn là tổng các số nguyên dương không vượt quá n, không nguyên tố cùng nhau với n. Chứng minh rằng ( ) ( )fnpfn +≠ với mỗi n và với mỗi số nguyên tố p .
Lời giải.
Ta có ( ) 1 nn ϕ+− là các số nguyên không âm, không nguyên tố cùng nhau với n, và với bất kỳ số r nằm trong số đó thì nr cũng vậy. Từ đây cho ta công thức
() () () 1 1 2 fnnnn ϕ=+− . Giả sử rằng ( ) ( )fnpfn += . Trước tiên ta quan sát thấy rằng
n và np + chia hết ( ) ( ) 2, fnnnp <+ do đó n và np + không phải là nguyên tố, tức là
ta có nkp = với * k ∈ ℕ . Từ đẳng thức ( ) ( )fnpfn+=⇔
( ) ( ) ( ) ( ) ( ) 111 kkpkpkkppkpp ϕϕ +−=+++−+ , do đó
( ) ( )11 kpkpkx ϕ +−=+ và ( ) ( )11kppkppkx ϕ ++−+= , với * ,. xxp ∈< ℕ Từ
( ) ( ) ( ) 1 xkppkpp ϕϕ =+−− . Bởi vì ( )kp ϕ và ( )kpp ϕ + đều chia hết cho 1 p ( từ
công thức của ( )n ϕ ), nên ta thu được ( )1mod1xp≡−− .
+ Nếu 2 p = thì 1 x = và ( ) 223 kk ϕ +=+ , vô lý bởi vì ( ) 221 kk ϕ +≤+
+ Nếu 3 p = thì 1 x = và ( ) 3324 kk ϕ +=+ , vô lý bởi vì ( ) ( )3321 kk ϕ +≤+
+Do đó 5 p ≥ , nhưng ( )1mod1xp≡−− nên ta chọn 2 xp=− , thay giá trị này vào ( )1 dẫn đến : ( ) 23 kpkp ϕ =+− và ( ) 21 kppkp ϕ +=++
Nếu ( ) kpkpp ϕ ⋮⋮ cho nên |23|3pkp + vô lý, vậy | pk /
Tương tự |1pk + / .
Cho nên ( ) ( ) ( ) 1 kppkϕϕ =− và ( ) ( ) ( )11kpppkϕϕ+=−+ kết hợp với ( )1 được
() 22 1 1 k k p ϕ + =− và () 22 11 1 k k p ϕ + +=+
Từ đây ta thấy ( ) t ϕ không chia hết cho 4 với tk = hoặc 1 tk=+ itq⇔= hoặc 2 tq =
với q là số nguyên tố lẻ và * i ∈ ℕ hoặc { }1,2,4 t ∈ . Dễ dàng loại trừ 1,2,4 t = . Do đó k hoặc 1 k + có dạng q hoặc 2 i q Với ( ) ( ) 1 ,111 iii tkqqqq ϕ ==+=−+ chia hết 22 q + vô lý bởi vì
() () 1 2 11122 3 iii qqqq +>−+>+
Ba trường hợp còn lại được chứng minh tương tự được điều không thể xẩy ra.
Vậy điều giả sử là sai nên ( ) ( )fnpfn +≠ với mỗi n và với mỗi số nguyên tố p ( đpcm)
Bài II.16.
Chứng minh rằng với mọi 1 n ≥ ta có ( ) ( ) ( ) 12 12 2 212121 n
n ϕϕϕ+++< ⋯
Lời giải. Ta thấy rằng ( ) ( ) () 1
11 1 2122 1 2 kkjk j k
=⋅=
kk k ϕϕ ϕ ≥
kk k ϕϕ ϕ ==≥≥=≤
==
1 2122 nn kjkd kkjdjkdkn
Bây giờ với mọi 1 d ≥ ta có ( kết hợp định lý Gauss’) ( ) ( ) ,| jkdknkd kkdϕϕ =≤ ≤=
Do đó ( ) 11212 n kd kd
Từ
k d ϕ =≥ ≤⋅
2 2 1 2 1
( ) () 21
nn n nxnxx xxnx x
++−++ +++= ⋯
Xét ()() 1000000000 2.3.5.7, NMN ϕ== . Mỗi tập TS ⊂ ta có
Ta suy ra bằng cách chọn 12 x = và cho n →∞ thì
d ∞ =
=
1 2 2d d
và ta có kết quả sau đó, từ đây ta có điều phải chứng minh
II.Sử dụng: Phi- hàm Euler giải các bài toán tồn tại trong số học
Bài II.1
Chứng minh rằng tồn tại vô hạn số nguyên dương n thỏa mãn ( ) ( ) ( )12nnnϕϕϕ<+<+
Lời giải.
Ta chọn ( ) 2*1,,,2 k nmmkm =−∈ ℕ⋮ . Khi đó ta có () ( ) ()()( ) ( ) 1 2 22 1111...1 k k nmmmmmϕϕϕ =−=−+++
( ) ( ) ( ) ( ) 1 22 1111 k mmmm ϕϕϕϕ =−+++ ⋯ ( ) ( ) ( ) 1 2 2 1 k mmnm ϕϕϕ<=+=
( ) ( ) 2 12mnϕϕ <+=+
Với vô hạn các cặp số nguyên dương ( ) , mk , trong đó m là số chẵn. Vậy từ đây ta suy ra
tồn tại vô hạn số nguyên dương n thỏa mãn ( ) ( ) ( )12nnnϕϕϕ<+<+
Ta có điều phải chứng minh.
Bài II.2 (USA TST 2015).
Cho ( )n ϕ là số các số nguyên dương nhỏ hơn n và nguyên tố cùng nhau với n . Chứng
minh rằng tồn tại số nguyên dương m sao cho phương trình ( ) nm ϕ = có ít nhất 2015
nghiệm nguyên dương của n
Lời giải.
Đặc biệt hóa. Lấy { }11,13,17,19,29,31,37,41,43,61,71
S = , ta thấy S có tính chất mọi
ước nguyên tố pS ∈ thì các ước nguyên tố 1 p thuộc { }2,3,5,7
364
() () ()() 1 11pTpT pTpT
pppppp ppppp ppp
⋅==−=
αα α αα α α α ϕϕϕϕϕ ∈ ∈∈
∈
a mãn ( ) (
ϕϕ==
ọ
Lời giải.
i số nguyên dương n. Tồn tại số nguyên dương m sao cho
Ta chứng minh quy nạp theo n
• 1 n = thỏa mãn
• Giả sử bài toán đúng với mọi số nguyên dương nhỏ hơn n. Ta cần chứng minh bài toán đúng với n.
Chọn ( ) ( ) * 2,,2221 x mxmxmtntxxmmtn =−∈< +=+− ℕ . Vì vậy ta chỉ cần
chọn x để sao cho ( )mxn ϕ⋮ và ( ) 2 xn υ ≥
Trường hợp 1. Nếu ( ) * 2q nq=∈ ℕ , chọn * 2,2 qm mqqmn =>∀∈ + ℕ⋮
Trường hợp 2. Nếu n có ước nguyên tố lẻ suy ra ( ) ( ) 2 2 n n υ ϕ ⋮
Ta có ( ) ( ) 22 xx mxtnxtnxn ϕ −=−−=−+ ⋮ . Để có điều này ta chỉ cần chọn x sao cho ( ) ( ) 2gcd, x xnn ϕ + ⋮ là xong.
Do ( ) ( ) gcd,nnn ϕ < nên theo giả thiết quy nạp tồn tại * , xxn ∈< ℕ để
( ) ( ) ( ) ( ) * 12122gcd,,, x xnnntnttt ϕϕ +=−∈ ⋮ℕ
* t ∃∈ ℕ để ( ) ( ) 2x xtnn ϕ −++ ⋮
Ta có ( ) ( ) ( ) ( ) ( ) ( ) ( ) ( ) 2 2gcd,2gcd,2,gcd, k xxxnnxnnknn υ ϕϕϕ + +≥≥= ⋮ ( ) ( ) ( ) 2 1 22 2221 k xx xxkxk υ υυ + >+≥ +> ≥
Trở lại bài toán. Giả sử với bất kỳ các số nguyên dương a và b tồn tại vô hạn n sao cho a chia hết n bn
Ta chứng minh bài toán bằng quy nạp theo a .
+ 1 a = luôn đúng
2 2212. 2 k n xx k n n υ
ϕ
() () 2 2
Chú ý: Nếu k có ước lẻ thì ( ) ( ) 2 2. k k υ ϕ ⋮
Bài II.4
Xác định xem có hay không tồn tại các số nguyên dương a và b sao cho a không chia
hết n bn với mọi số nguyên dương n
Lời giải.
Câu trả lời là không
Bổ đề: Cho các số nguyên dương a và b với số nguyên dương n đủ lớn ta có ( ) ( ) mod na n bba ϕ+ ≡
Thật vậy. Đặt 12 12 ... k kappp α αα = , trong đó 12,,..., kppp là các số nguyên tố phân biệt,
* ,1, i ik α ∈∀= ℕ
Ta thấy hàm Phi Euler ϕ có tính chất nhân nghĩa là
() ()() () () () 1211 1 1 1211 1 αααααααϕϕϕϕ ==−−=−− …⋯
Cụ thể ( ) ( ),1 i paik α ϕϕ ∀≤≤ và ( ) aa ϕ < .
Ta xét với mỗi ip
+ Nếu i pb thì ( ) 0mod n ibpα ≡ vói mọi 1 i n α ≥+ .
Do đó ( ) ( ) ( ) 0mod naann ibbbbp ϕϕ α + ≡≡≡ vói mọi 1 n α ≥+
+ Nếu ip không chia hết ( ) gcd,1. i bpb α = Theo định lý Euler’s, ta có
366
+ Giả sử khẳng định đúng đến mọi số nguyên dương nhỏ hơn 0aa < . Vì ( ) aa ϕ < theo giả thiết quy nạp và theo bổ đề thì với mọi n ta có
( ) n abn ϕ và ( ) ( ) mod na n bba ϕ+ ≡ . Với mỗi n như vậy ta đặt () n bn t a ϕ = và
( ) 1 n nbnta ϕ ==+ , từ đó ta có. ( ) ( ) ( ) ( ) ( ) 1 1 0mod nta n n bnbntabntaa ϕ ϕϕ + −≡−+≡−−≡ . Từ đó 1n thỏa mãn tính chất mong muốn. Theo giả thiết quy nạp rõ ràng có vô hạn 1 nnb = thỏa mãn các điều kiện đúng với a Vậy bài toán đúng với mọi số nguyên dương a Bài II.5 (Polish MO 2015 ). Cho số nguyên dương a . Chứng minh rằng tồn tại số nguyên dương ba > sao cho 123bb++ chia hết cho 123aa++ .
Lời giải.
Với 1 a = tồn tại 31 ba =>= sao cho () ( ) 33 612312336 =++++= kết luận đúng. Giả
sử 2 a ≥ , đặt 12323 aars m ++= , trong đó ,, rsm là các số tự nhiên và 1 m ≥ nguyên tố cùng nhau với 6 . Đặt ( )km ϕ= . Khi đó ( ) ( ) 21mod,31mod kk mm≡≡ ( theo định lý Euler’s). Vì thế với mọi số nguyên dương n ta có 21 kn m⋮ và 31 kn m⋮ .
Nhận xét: với mọi số tự nhiên n, ta có ( ) () 2 2 31312mod4 nn+=+≡ và ( )( ) ( ) 21221 31313331421 nnn M +− +=+−+−+=+ , với M là số nguyên dương lẻ. Từ đó ta có : ( ) () 212 313.314mod8 n n+ +=+≡ . Suy ra 1 r = nếu a là số chẵn và 2 r = nếu a là số lẻ. Mặt khác ta có 31232.3 aaaa <++< (vì 123,1 aa a +<∀> ).
Nên suy ra 3|123 aaa ++ / dẫn đến sa < . Vì thế 12a + chia hết cho 3s .
Bây giờ ta chọn 2 bakaa =+> khi đó ta có :
367
( ) ( ) 1231232233 bbaababa ++=+++−+−
( ) ( ) 22 123221331 aaakaaka =+++−+−
Mà 2121;3131 kakkak mm⋮⋮⋮⋮ nên 123bb++ chia hết cho m
Mặt khác, 3b chia hết cho 3s ( do )bas >> và 22 2121213. kaaas −−+⋮⋮⋮
Cuối cùng do 2a và 2 31 ka chia hết cho 4 nên ( )123123mod4 bbaa++≡++ , chứng tỏ
rằng 123bb++ chia hết cho 2r . Từ đây suy ra 123bb++ chia hết cho
123aa++ . Bài toán được giải quyết. Bài II.6 (TST Quảng Ninh 2021).
a) Cho các số nguyên dương , ak với 2 a ≥ . Chứng minh rằng ( )|1 kka ϕ , trong đó
ϕ là hàm Euler.
b) Cho số nguyên m và p là một số nguyên tố không là ước của m nhưng p là ước
của ( )m ϕ . Chứng minh rằng tồn tại số nguyên tố q sao cho ( ) 1mod qp ≡
c) Cho p là một số nguyên tố. Chứng minh rằng có vô hạn sô nguyên tố có dạng 1 pk + .
Lời giải.
a) Đặt 1 k ma=− thì ( ) 1mod k am ≡ . Gọi ( ) ord m ha = thì | hk và ( ) | hm ϕ . Do | hk nên
hk ≤ . Mặt khác ( ) 1mod h am ≡ nên 1 hk amahk ≥+= ≥ . Từ đó ta có hk = , suy ra ( ) ( ) |1. kkma ϕϕ=−
b) Giả sử m có phân tích tiêu chuẩn 1
ik i i mp =
= ∏ thì () () 1 1. t k ii i mpp ϕ =
=− ∏ Do p là ước
của ( )m ϕ nhưng không là ước của m nên () 1 |1 i i pp = ∏ . Từ đây suy ra tồn tại
{ } 1;2;...; it ∈ sao cho |1 ipp hay ( ) 1mod, i pp ≡ và iqp =
c) Giả sử chỉ có hữu hạn số nguyên tố dạng 1 pk + là 12,,..., qqq .
Đặt 12..;1 p t apqqqma ==− .
Theo phần a) thì ( )|1 p pa ϕ hay ( ) |.pm ϕ Do p là ước của ( )m ϕ và không là ước của
m nên theo phần b) tồn tại số nguyên tố 1 q + sao cho ( ) 1 1mod t qp + ≡ , trong đó 1 q + là
một ước nguyên tố của 1. p ma=−
Rõ ràng 1 , ti qqit + ≠∀≤ , vì nếu ngược lại thì 1 | t qa + hay 1 |1, tq + vô lý.
Vậy điều giả sử là sai, tức là có vô số nguyên tố có dạng 1 pk + .
Bài II.7( TST Iranian 2017).
Sắp xếp tất cả các số nguyên tố theo thứ tự tăng dần : 1232.ppp=<<<
Cũng giả sử rằng 123 nnn<<< ⋯là một dãy số nguyên dương tăng dần, biết rằng với mọi 1,2,3,... i = thì phương trình ( ) 2mod n xp ≡ , có một nghiệm cho x . Hỏi liệu luôn luôn có một số x thỏa mãn tất cả các phương trình hay không ?
Lời giải. Xét một dãy số 123 nnn<<< ⋯ các số nguyên dương sao cho
( ) 3 3:21,7 iiii inpppn ϕ ∀≠=+=−=
Với mọi 3 i ≠ , 2 ix = và với 3,3 iix== là nghiệm của phương trình
( ) 2mod in ii xp ≡
Giả sử rằng tồn tại một số x thỏa mãn tất cả các phương trình.
Chú ý rằng ( ) 3 3 22mod n p ≡ / , do đó 2 x ≠ . Gọi kp là ước nguyên tố 1 x ta có
( ) 1mod, kn kxp ≡ mặt khác ( ) 2mod kn kxp ≡ nên suy ra ( ) 12mod kp ≡ , vô lý. Vậy không tồn tại một số nguyên dương x thỏa mãn tất cả các phương trình
Bài II.8.
Tìm số nguyên tố ( )5 pp ≥ nhỏ nhất sao cho tồn tại hai đa thức [ ] , fgx ∈ Z , thỏa mãn ( ) ( ) * |23, fngn pn+∀∈ / ℕ
Lời giải. Nhận xét : Nếu 00,|23nm pp∈℘+ với 00 , nm ∈ ℕ . T
suy ra ()()22 0011 * |23,. pnnpnm pn −+−+
Vậy ( ) ( ) * |23, fngn pn+∀∈ ℕ , trong đó : ( ) ( ) [ ]
00|23nm p ⇔+ với 00 , mn ∈ ℕ
Ta thấy: 202131224041 5|23,7|23,11|23,13|23,17|23,19|23 ++++++ .
Xét số nguyên tố 23. Ta có 231 1;1 232323
Giả sử () 1 23|23,,32mod231 23 abba ab +∀∈ ≡− =
Vậy số nguyên tố nhỏ nhất thỏa mãn yêu cầu bài toán là 23.
Bài II.9 ( Iranian MO 2013).
Chứng minh rằng với mỗi số nguyên dương n thì tồn tại các số nguyên dương 12 n aaa <<< sao cho ( ) ( ) ( ) 12 n aaa ϕϕϕ >>> ⋯ , ký hiệu ( ) ia ϕ là hàm Euler của ia .
Lời giải.
Ta giải bài toán bằng phương pháp quy nạp theo n
+ Với 1 n = thì khẳng định đúng + Giả sử bài toán đúng đến mọi số nguyên dương nhỏ hơn n. Tức là tồn tại các số nguyên dương 121 n bbb <<< ⋯ thỏa mãn tính chất ( ) ( ) ( ) 121 n bbb ϕϕϕ >>>
Đặt 1232,3,5,ppp=== ta có dãy vô hạn các số nguyên tố. Rõ ràng thấy
Theo định lý thặng dư Trung Hoa số nghiệm của phương trình !|1 n na là tích của số nghiệm của mỗi phương trình |1 i n i pa α .
Vì vậy theo yêu cầu bài toán trở thành ( ) 1mod n ap α ≡ có nghiệm duy nhất với mỗi i
Nếu 2 ip ≠ thì ta có căn nguyên thủy modulo ip α số nghiệm của ( ) 1mod n ap α ≡ đúng
bằng ( ) ( ) , np α ϕ . Vì vậy ta phải có ( ) ( ) ,1 inp α ϕ =
∞ =
1 10 k kp
1
−=
Nhưng nếu n là hợp số thì ta lấy p là một trong các số nguyên tố chia hết n . Trong đó lũy thừa của p trong !n lớn hơn 1 , khi đó ( ) | k pp ϕ trong đó k là lũy thừa của p trong !n . Cho nên ( ) ( ) ,1 k np ϕ ≠ , vì vậy n không thỏa mãn ( nếu n là lũy thừa của 2 ta thấy rằng ( ) 2| k p ϕ với p lẻ vậy do đó ta phải có 2 n ≤ hay 2 n = ).
Do đó tồn tại các số nguyên tố phân biệt 12 mqqq <<< ⋯ sao cho 1
11 1 4 m k kq =
−= ∏ và ( ) 12112 gcd...,...1 nm bbbqqq = . Do đó
Vì vậy, cho n là số nguyên tố lẻ, thì ( ) ( ) ,1 i pn α ϕ = bởi vì n lớn hơn ( ) ip α ϕ . Do đó ta cần kiểm tra có nghiệm duy nhất modulo 2
Đặt:
11 ......1... 4 m mm m k k bqqqbqqqbqqq q ϕ =
()112112112 1
≤−< ∏
Theo bổ đề Bertrand’s thì tồn tại số nguyên tố p sao cho () () 112112 112 1 ......1... 4 mm mbqqqbqqqpppbqqq ϕϕ<<−=<<
Ta chọn ( ) 1 nfxx=− , khi đó ( ) ( ) 0mod2 fx ≡ có nghiệm duy nhất và đối với nghiệm này ( )1 ta có ( ) ( ) 1 10mod2.ntfxnx ′ =≡≠
Vì vậy ta có một nghiệm duy nhất modulo 2t với mỗi t nguyên dương.
Kết luận n là số nguyên tố lẻ. Bài
= = =
Bài II.9 ( Iranian TST 2008).
... ... ...
m m nnm
1 2112 3212 112
ap abqqq abqqq abqqq
Thì 12 n aaa <<< ⋯ đồng thời ( ) ( ) ( ) ( ) ( ) ( ) ( ) 12112 112 1...... mnnm apabqqqabqqq ϕϕϕϕϕϕϕ =−>=>>=
Vậy khẳng định bài toán đúng với .n Bài toán được chứng minh.
Tìm tất cả các số nguyên dương n để tồn tại duy nhất 0! an≤< sao cho
!|1 n na
Lời giải.
Gọi 12 12 ! k knppp α αα = … , với ( ) 1, i pik = là số nguyên tố và ( ) * 1, i ik α ∈∀= ℕ .
370
Bài II.11( Swiss Final Round 2020).
G
Lời giải.
Ta giải quyết bài toán bằng phương pháp quy nạp + 1 n = bài toán đúng + Giả sử bài toán đúng cho * 1(,2)nnn−∈≥ ℕ tức là () ( ) 1 1 2 2|3221 n nn ϕ , ta cần phải chứng minh bài toán đúng với n () ( ) ( ) ( ) 11 1 222 2|3221322121 nnn nn ϕϕϕ + ⇔⋅−=⋅−⋅+ ( ) () 1 22 2|21* n n ϕ ⇔+
Nhận xét 1. Tất cả các ước nguyên tố của 1 2 21 n + đều có dạng ( ) * 41kk+∈ ℕ , ngược lại có ước nguyên tố () 2 1 43,|111mod4pkpap p =++ =⇔≡ vô lý
Nhận xét 2. 1 2 21 n + có ít nhất n ước số nguyên tố.
Thật vậy : ( )( ) ( )( ) 12311 22222 2121212121 nnn −=+++− nó có nghĩa là trên 1 22 n có ít nhất n ước nguyên tố khác nhau,tất cả các ước nguyên tố này đều có dạng
Bài II.13. Cho k là số nguyên dương. Chứng minh rằng tồn tại một số nguyên dương n sao cho ( ) ( ) nnkϕϕ=+
Lời giải. Gọi p là số nguyên tố nhỏ nhất không là ước của k và chọn ( )1 npk =− . Khi đó ( ) ( ) ( ) ( ) 1 nkpkpk ϕϕϕ +==− . Nhưng mặt khác ta có
1 111 qpk npkpk q ϕϕ =−=−−
Và vì tất cả các thừa số nguyên tố q của 1 p đều là ước nguyên tố của k ( theo cách chọn nhỏ nhất của p ), nên chúng ta suy ra rằng:
=−−=−=+
()() ()()() | ∏
Bài II.14
với mọi 11. kn ≤≤−
+==− ∏∏ suy ra ()() () ( ) 1 2 1211...1|21 n nppp ϕ −−−+ ta có điều phải chứng minh
( ) * 41kk+∈ ℕ . Mặt khác () () 1 1 2 11 211 n ii knkn iii ii ppp αα ϕϕ ≥≥ ==
Bài II,12 (Mathematical Reflections 5-2010).
Cho , ab là các sô nguyên dương sao cho a không chia hết b và b không chia hết a .
Chứng minh rằng tồn tại một số nguyên x sao cho 1 xa<≤ và cả hai số a và b chia hết ( ) 1 , b xx ϕ + với ϕ là phi hàm Euler’s.
Lời giải.
Ta có () ( )1 b xxϕ chia hết cho x và với ( ) gcd,1 bx = , theo định lý Euler’s thì nó cũng chia hết cho b .
Ta chọn ()gcd, a x ab = thì ta có () gcd,1 gcd, a b ab = , khi đó () () () () () ()10mod0mod, gcd,gcd,gcd,
b) Có vô hạn 1 n > sao cho ( ) ( ) ( ) nknkϕϕϕ≤+− với mọi 11 kn ≤≤− ?
Lời giải.
a) Ta khẳng định rằng bất kỳ số nguyên tố lẻ p nào cũng có tính chất này. Thật vậy ta có: ( ) ( ) ( ) 1121 kpkkpkppp ϕϕϕ+−≤−+−−=−<=−
b) Câu trả lời là có. Gọi 12,,...pp là dãy các số nguyên tố tăng dần và ký hiệu 12 dd nppp = . Ta chứng minh rằng có một nghiệm của bài toán cho 2 d ≥ . Chọn
bất kỳ { }1,2,...,1 dkn∈− và gọi 12 lqqq <<< là các ước nguyên tố của k . Lưu
ý rằng 1122,,...qpqp ≥≥ và kể từ đó 121212 ......... ddll pppnkqqqppp =>≥≥
chúng ta phải có ld < . Ta suy ra ( ) ( ) 11
11 11 ld d iiiid ∏∏
ϕ −≡⋅≡
b aaa blcmab ababab
Do đó yêu cầu tồn tại số ()gcd, a x ab = và nó cũng thỏa mãn 1. xa<≤
372
áp dụng cho bất đẳng thức tương tự khi thay k bởi d nk , ta có kết quả () () ( ) () ( ) (), dd ddd dd
nn knkknkn nn ϕϕ ϕϕϕ +−≥⋅+−⋅=
Chứng tỏ rằng dn là một nghiệm của bài toán.
Bài II.15 (Shotlits IMO 2020.N6)
Với mỗi số nguyên dương n
• Gọi ( )dn là số các ước nguyên dương của n, và
• Gọi ( )n ϕ là số các số nguyên dương nhỏ hơn n , nguyên tố cùng nhau với n .
Có tồn tại hay không một hằng số C sao cho
( ) ( ) () () dn C dn ϕ ϕ ≤
Với mọi 1? n ≥ ( ( )n ϕ còn gọi là hàm phi Euler)
Lời giải.
Câu trả lời là không.
Gọi 12,,..., mppp là m số nguyên tố đầu tiên và gọi 1,..., rqq là các số nguyên tố với tính
chất 2 mim pqp << .Theo định lý số nguyên tố, ta có thể cho r đủ lớn.
Xét n có dạng
1 2121 11 s s m mr nppqq =⋅
Ta có ( ) 1 2 m ssr dn +++ = và ( ) ( )
III.Sử dụng: Phi- hàm Euler giải bài toán phương trình nghiệm nguyên
Bài III.1
Tìm tất cả các cặp số nguyên dương ( ) , mn sao cho ( ) |21 n m ϕ + và ( ) |21 m n ϕ +
Lời giải.
Với 1 m = ta có |3 n từ đó 3,1nn== . Ta có ba cặp ( ) ( ) ( )1,1,1,3,3,1 thỏa mãn ycbt.
k M k k mp α =
1 1 2 m ssr dn ϕ +++− = Hơn thế nữa ( ) ( ) ( )
1 2121 11 11. m mr nppqq ϕ =⋅−−
Từ 1 2 i m q p ≤ , các thừa số của 1 iq nằm trong số { } 12,,, mppp … , vì vậy chúng ta có thể viết ( )
1 1 22 22 1 , m mf f mnpp ϕ −+ −+ = ⋯
=−+ ∏
Tiếp theo () () () 1 21 m s i dnf ϕ =
Vì thế ( ) ( ) () ()
∪∪∪⋯
Với ,:mn ,3mn ≥ . Giả sử 1
= ∏ và 1
k N k k nqβ =
= ∏ . Giả sử 1,1iMjN≤≤≤≤ sao cho ( ) ( ) 22 1 1min1ik kM vpvp ≤≤ −=− và ( ) ( ) 22 1 1min1jk kN vqvq ≤≤ −=−
Trong đó ( ) 2 2| vnn α α =⇔ và 1 2| n α+ / và ( ) 2 2| vmm β β =⇔ và 1 2| n α+ /
Ta thấy ( ) ( ) ( ) ( ) 2 21mod21mod nn ii pp ϕϕαα ≡− ≡ . Gọi ord2 i p d α = , qua đó ( ) | i dn ϕ / và ( ) |2 i dn ϕ . Từ đó ta thu được ( ) ( ) ( ) 22 1 vdvn ϕ =+
Cuối cùng ( ) ( ) 1 |1 ii iiii dppp αα ϕ =− ( định lý Euler) ( ) ( ) 22 1 ii vdvp ≤− .
Như vậy ( ) ( ) ( ) ( ) 2221111 i j vpvnvq ϕ −≥+≥+− ( )1
Lập luận tương tự như trên với j q β ta được: ( ) ( ) 22111ji vqvp−≥+− ( )2
Lấy ( ) ( )12 + theo vế ( ) ( ) ( ) ( ) 222211112 ijij vpvqvpvq −+−≥−+−+ vô lý. Như
vậy không tồn tại nghiệm với ,3mn ≥
Bài III.2 (TST Saudi Arbian 2021)
Tìm tất cả các số nguyên dương n , sao cho n là một số hoàn hảo và ( )n ϕ là một lũy thừa của 2 . ( Chú ý : một số nguyên dương n được gọi là số hoàn hảo nếu tổng các ước dương của nó bằng 2n . Ký hiệu: ( )n ϕ hàm phi Euler của n )
Lời giải.
Gi
dn dnf ϕ ϕ = ∏
là
2 2 21 i i
s m r s i i
≥⋅ −+
Khi các is ngày càng lớn, điều này dẫn đến 1 2r có thể lớn tùy ý.
ả sử rằng ( ) 2m n ϕ = với * m ∈ ℕ . Dựa trên công thức của ( )n ϕ ta có ( ) ( ) 1 * 12,(,)tm pn npppt ϕ =−=∈℘∈ ∏ ℕ
Với ( ) p vnt = , vậy 1 t = với tất cả các ước nguyên tố lẻ của n , ngược lại 1 t > thì |2m p vô lý. Như vậy 1 p là lũy thừa của 2 .
Với p là ước nguyên tố lẻ của n , đặt 21 s p =+ với * s ∈ ℕ
+) Nếu 1 s = thì 3 p = . +) Nếu 1 s > thì s phải chẵn vì ( ) ( ) ( )110mod3,1 s ps ≡−+≡> với s là số lẻ (vô lý
).Trong trường hợp s chẵn có một ước nguyên tố lẻ q thì đặt ( ),1sqtt=> thì
( ) 212121 q stt p =+=++ ⋮ ( vô lý)
Từ hai điều trên vậy ( ) 2, k sk=∈ ℕ khi đó ta có 2 21 s p =+
Bây giờ đặt 12 2... a lnppp = với 0,0al≥≥ và 12,,..., lppp là các số nguyên tố lẻ theo
thứ tự tăng dần, trong đó 2 1 21 k p =+ . Vì thế
( ) ( )( )( ) ( ) 1 12 221111 a lnnppp σ + ==−+++ … ( )* , là tổng các ước của n
Nếu 0 l = thì ta có ( ) 1 212 a nn σ + =−= vô lý, từ đó 1 l ≥
Nếu 1 3 p > thì 3|2n / và
( ) ()( ) () 21 221 1 1222212110mod3 k kk p +=+=+≡−+≡ ( )**
Nghĩa là 1 3|13|2 pn + vô lý. Như vậy 1 3 p = và ( ) 3 21vn = dẫn đến 2 l ≤ , do đó có hai trường hợp.
+) Nếu 2 l = từ ( )* suy ra ( ) ( ) 11 22 232141 aa pp ++⋅⋅=−⋅⋅+ , chú ý rằng theo ( )** thì ( ) 22 11 vp += , từ đây ta có ( ) ( ) ( ) ( ) 1 1 2222 2141323132 a a vpvpaa + + −⋅⋅+= ⋅⋅=+= =
Như vậy ta có: ( ) 22 671 pp=+ vô lý.
+) Nếu 1 l = thì từ ( )* ta được ( ) 11 232141 aa a ++ ⋅=−⋅ = khi đó 236 n =⋅= là số hoàn hảo.
Do đó 6 n = là nghiệm của bài toán.
Bài III.3
Tìm tất cả các số nguyên dương n thỏa mãn phương trình: ( ) ( ) 22 212. nn ϕϕ −=
Lời giải. Gọi 2 21 k kF =+ là số Fermat thứ k . Bởi vì tất cả các số Fermat nguyên tố cùng nhau đôi một và
2 011 21 n n FFF−=⋅ ⋯
Chúng ta có thể viết lại phương trình đã cho dưới dạng ()
Ngược lại nếu 5 n ≤ , thì iF là số nguyên tố với 1 in≤− , do đó
()() 1 11 12221 00 122 nn nn ii ii FF ϕ +++− ==
=−==∏∏
Như vậy các số nguyên dương 1,2,3,4,5 n = thỏa mãn bài toán.
Nhận xét: 7 641251 =⋅+ , suy ra ( ) ( ) 7284 251mod641251mod641 ⋅≡− ⋅≡ . Bởi vì 4464125 =+ nên ta có ( ) 44 52mod641 ≡− . Từ đó ( ) 28432 252mod641 ⋅≡− tức là
()() 5 32 232 5 21mod64121210mod641 F −≡⇔=+=+≡
Bài III.4 (Mathematical Reflections 2-2012)
Giải phương trình ( ) ( ) 34nnϕτ = , với ( )n ϕ là hàm Euler và ( )n τ là số các ước dương của n Lời giải.
Ta xét hàm số: () ( ) () () 1 1 1 p p pp f p
α α α ϕ α α τ == +
Với p là số nguyên tố và α là số nguyên dương. Dễ dàng ta xác minh được ( ) ( )1 ppffαα<+ và ( ) ( ) pq ffαα < với pq <
Vì vậy: ()() 1 11 2 pp p ff α ≥=≥ khi 2 p > hoặc 2 p = và 2 α > ( bởi vì () () 22 12 1,2 23ff== ). Bây giờ ta đưa ra các khẳng định sau đây
1) Nếu 7 p ≥ là một số nguyên tố chia hết n thì ( ) () ()()()() 227 34 1111 23 p
n ffff n ϕ τ ≥≥=>
2) Nếu 25 chia hết n thì ( ) () ()()25 104 12 33 n ff n ϕ τ ≥=>
3) Nếu 33 chia hết n thì
= ∏
1 21 0 2, n n iF ϕ =
Nhờ tính chất nhân của hàm Phi Euler’s.
Nếu 6 n ≥ ta suy ra rằng ( ) 5F ϕ là một lũy thừa của 2 . Bởi vì 5 641| F , nên 640 chia hết ( ) 5F ϕ và ( ) 5F ϕ không là lũy thừa của 2 . Do đó bất kỳ nghiệm nào thỏa mãn bài toán thì
( ) () ()()23 94 13 43 n ff n ϕ τ ≥=>
4) Nếu 42 chia hết n thì ( ) () () 2 84 4 53 n f n ϕ τ ≥=>
5. n ≤ 377
376
Do đó nghiệm phương trình đã cho nếu có phải có dạng 235abc với 0,1,2,3, a = 0,1,2 b =
và 0,1 c = . Kiểm tra trong toàn bộ tập hữu hạn này ta sẽ thu được các nghiệm
20,36nn== và 60. n =
Bài III.5(Mathematical Reflections 1-2010)
Tìm tất cả các cặp số nguyên dương ( ) , mn sao cho ( ) ( ) 1 m n ϕϕ = , với ϕ là Phi hàm
Euler’s.
Lời giải. Chúng ta sẽ chỉ ra rằng các cặp ( ),1m với mọi số nguyên dương m, ( )3,2 và
( )2,4 thỏa mãn yêu cầu bài toán.
Đầu tiên chúng ta lưu ý rằng:
1) Vì ( )x ϕ là số lẻ nếu 1 x = hoặc 2 x = thì khi đó ( ) ( ) m nn ϕϕ = với n lẻ, nếu
1 n = và với tất cả các số nguyên dương m
2) Vì ( ) nn ϕ < với 1 n > , vậy nếu 1 m = thì 1 n =
Vì vậy ta có thể giả sử rằng 2k nj = , với 1,1kj≥≥ lẻ và 2 m ≥
Trường hợp 1. Nếu 1, j ≠ thì theo 1) ta có ( ) 2 ms jh ϕ = với 1,1sh≥≥ lẻ và ( ) ( ) ( ) ( ) ( ) ( )
12 22222 mkmmkmskmsk njhhnjϕϕϕϕϕϕ −+− ==⋅===
Do đó 2,kmsk +−≤ tức là ( ) 121kms−≤−≤ , có nghĩa là 1,2km== và 1 s = . Khi đó ( ) hj ϕ = và theo 1) ta có 1 hj== điiều này vô lý.
Trường hợp 2. Nếu 1, j = thì 2k n = và ( ) ( ) ( ) 12 2222 kmkmkmkϕϕϕ===
Do đó ta có ( ) 2121,3kmkkmkm −=⇔−= == hoặc 2,2km== tương ứng với
( ) ( ),3,2mn = hoặc ( ) ( ),2,4mn =
Bài III.6.
Tìm tất cả các số nguyên dương n sao cho ( ) 22. n n ϕ ≤
Lời giải.
Bổ đề Với mọi số nguyên dương n thì () 1 2 nnn ϕ ≤≤
Chứng minh +) () 1 2 nn ϕ ≥ . Thật vậy 1 n = thì bất đẳng thức đúng. Giả sử 1 n > ta
viết 0 12 12 2a aaa rnqqq = ⋯ , với 12 2 rqqq<<<< là các số nguyên tố và ( ) 0, i air = là các
=−
Vì ( ) ( )2 3110pppp −+=−−> với 3 p ≥ , nên ta có 1 pp −> . Đồng thời cũng có
với mỗi số nguyên dương ia thì 11 22 ii aa −≥ ( 1, ir∀= ). Vì thế
raaaa
rr nqqqqqq
c ch
ng minh. Tr
đề ta có () 1 2 nn ϕ ≥ hay () () 2 22* n n
lại bài toán. theo b
≥ . Vì v
i m
i 9,5 x ≥ thì hàm số ( ) 2 28 x fxx =− đồng biến, theo đó ứng với 361 n ≥ , thì 2 2 282 2
Do đó để bất đẳng thức đề bài đúng với điều kiện 1361 n ≤≤ . Kiểm tra trực tiếp chúng ta nhận được các giá trị : 1,2,3,4,6,8,10 n = và 12.
Bài III.7.
Chứng minh rằng tất cả các nghiệm nguyên dương của phương trình ( ) 2 3 n n ϕ = đều có dạng 3,1,2,3,... k nk==
Lời giải.
Ta xây dựng bài toán sau đây: ( ) 2 3,1,2,3,... 3 k n nk n ϕ =⇔==
( ) ( ) ()() 2 32* 3 n nn n ϕ ϕ = = do ( ) 2,31 = , nên ta có 3| n . Từ đó suy ra tồn tại hai số nguyên dương k và r với ( ) ,31r = sao cho 3k nr = . Khi đó
( ) ( ) ( ) ( ) *33.2.3.3.3.2.3. kkkk rrrrϕϕϕ ⇔=⇔=
( ) 1 3.3.2.3. kk rr ϕ ⇔=
số nguyên dương. Ta có 379
( ) 1 rrr ϕ ⇔= = . Tức là 3,1,2,3,... k nk==
( )⇐ () ( ) 1 3 3.22 333
k k kk n n ϕ ϕ ===⋅ Bài toán được chứng minh.
Bài III.8.
Chứng minh rằng số nguyên dương n là nghiệm của phương trình ( ) 4 3.7 7 n n n αβ ϕ =⇔= , với 1,2,...;1,2,...αβ==
Lời giải.
( ) ( ) () 4 7.47| 7 n nnn n ϕ ϕ = = do đó tồn tại hai số nguyên dương , r β với ( ) ,71r = sao cho 7 nr β =
Từ giả thiết ( ) ( ) 7.47.74.7 kk nnrϕϕ=⇔= ( ) ( ) 7.7.4.7. kk rr ϕϕ ⇔= ( ) ( ) 1 7.7.71.4.7. rrββ ϕ ⇔−=
( ) 2 3,1,2,... 3 r r r α ϕ α ⇔= ==
Vậy 3.7,1,2,...;1,2,... n αβ αβ ===
Cho p là số nguyên tố sao cho21 p + là hợp số. Chứng minh rằng phương trình ( ) 2 xp ϕ = không có nghiệm nguyên dương x
Lời giải.
Gọi 12 12 k kxqqq α αα = với iq ( ) 1, ik = : các số nguyên tố phân biệt, ( ) * 1, in α ∈= ℕ
Thì ta có () () 1 1 12 k k ii i xqqp ϕ =
=−= ∏
Do đó 1|2 i qp . Tuy nhiên, vì ước của 2 p là 1,2, p và 2 p . Mặt khác 21 p + là hợp số, nên các khả năng xẩy ra chỉ là 11 iq −= và 12 iq −= . Khi đó x thỏa mãn phương trình phải có dạng 23(,) ab xab=∈ ℕ . Nhưng ( ) 11 2232 ab xpp ϕ =⇔= Từ đó ta được 2 p = hoặc 3 p = , và vì vậy 215 p += hoặc 217 p += là hợp số, vô lý. Điều giả sử là sai hay phương trình ( ) 2 xp ϕ = không có nghiệm nguyên dương x thỏa mãn 21 p + là hợp số ( với p là số nguyên tố).
Bài III.11 (TYCM,vol.26,1995)
Cho , an là các số nguyên dương và p là số nguyên tố. Chứng minh rằng ( ) ( )261 a pn ϕ =+ khi và chỉ khi ( )6,11mod12pp>≡ và a chẵn.
() () ( ) ( ) ( ) ()()11 3.737 3.31.7.71 4 3.73.73.77
αβαβ αβ αβαβαβ ϕϕϕ ϕ ⇐==== ( đpcm)
n n
Bài III.9.
Tìm tất cả các nghiệm nguyên dương x của phương trình ( ) 24 x ϕ = Lời giải.
Đặt 1 , i r i i xq α =
= ∏ với iq là số nguyên tố phân biệt, * iα ∈ ℕ với mọi 1,2,..., ir =
==−=
Vì vậy ước nguyên tố lớn nhất của x nhỏ hơn hoặc bằng 23. Cho nên các ước nguyên tố của x có thể là 2,3,5,7,11,13,17,19,23.Vì 10|24,16|24,18|24 /// và 22|24 / , nên ta thấy rằng các số nguyên tố 11,17,19và 23 không là ước của x. Do đó các ước nguyên tố của x có thể là 2,3,5,7 p = và 13 .
Đặt: ( ) 235713,,,,, abcdf xabcdf =⋅⋅⋅⋅∈ ℕ .
Vì ( ) 24 x ϕ = suy ra 05,02 ab ≤≤≤≤ và 0,,1 cdf ≤≤
Kiểm tra ta có các nghiệm của ( ) 24 x ϕ = là 35,39,45,52,56,70,72,78,84 và 90
380
Lời giải. ( )⇐ Nếu ( ) 1211, pkk=+∈ ℕ và a là số chẵn, thì ( ) ()()() 1 1 112111210 a aa pppkk ϕ =−=++
=++=+=++∈
ℕ
( ) ( ) ( ) ( ) 1 261.1 aa nppp ϕ +==− , vậy p là số nguyên tố lẻ.
Ta thấy: ( ) ( ) ( ) ( ) 222611113mod4 nppυυ += −= ≡ ( )1
Và ( ) ( ) ( ) ( ) ( ) 1 12612mod32mod32 a ppnp −=+≡ ≡
Từ ( )1 và ( )2 ta suy ra: 2,3,56pp ≠ > , ( )11mod12 p ≡ , tiếp theo ( ) 1 2mod3 a pa ≡ chẵn ( đpcm)
Bài III.12 ( Iranian TST 2006).
Giả sử rằng p là một số nguyên tố. Tìm tất cả các số tự nhiên n sao cho ( ) | pn ϕ và với mọi a sao cho ( ) ,1an = ta có : ( ) |1 n p na ϕ .
Lời giải.
381
Với mỗi số nguyên dương n, ta nói k là số tốt đối với n khi và chỉ khi ta có: ( ) ,1|1 k anna = .
Gọi ( ) |1 h n hordana = . Vậy điều kiện cần và đủ để k là tốt đối với n là ( ) { } |,1| n lcmhordaank ==
Vơi mỗi số n gọi số tốt nhỏ nhất là ( )fn . Theo định lý Thặng dư Trung Hoa ta có ( ) ( ) ( ) ( ) , fmnlcmfmfn = nếu ( ) * ,,,1mnmn∈= ℕ . Với một lũy thừa của số nguyên tố k p sao cho p là nguyên tố lẻ. ( ) ( ) kk fpp ϕ= bởi vì ta có căn nguyên thủy modulo k p .
Với lũy thừa của 2 ta có ( ) 21 f = và ( ) 42 f = và từ đó chúng ta có ( ) 2 2|2kk f với
3. k ≥ Bởi vì chúng ta không có căn nguyên thủy modulo2k với 3 k ≥ ( thật vậy bài toán quen thuộc mọi số nguyên dương lẻ 1 a > thì 2 2 12,3) k k ak−∀≥ ⋮ . Vậy rõ ràng ( ) 2 2|2kk f
Trở lại bài toán, nếu
1 1 1 1 2...2,,..., k k tt k kfnfpplcmfpp α α α α ϕϕ ==
1 1 2... k t knpp α α = ( ip : các số nguyên tố lẻ phân biệt, ,)t α ∈ ℕ ( ) ( ) ( ) ( ) ( ) ( )
Nhưng ( ) ( ) ( ) ( ) 1 1 2... k knpp α α ϕϕϕϕ = + Với 2 p ≠ là số nguyên tố thì cả hai số ( ) 2t ϕ và ( ) 2 f đều không chia hết cho p .
Nên điều kiện cần và đủ trong trường hợp này hai trong số ( ) ip α ϕ chia hết cho p
+ Với 2 p = , nếu ta có hai số nguyên tố lẻ thì giá trị của f của chúng chia hết cho 2 và n
có cùng tính chất.
Nếu ta chỉ có một lũy thừa của số nguyên tố lẻ thì 2tk np = , nếu 1 t > thì cả hai ( ) 2t f và
( ) kfp đều chia hết cho 2 ta có điều chứng minh.( Cũng lưu ý rằng ( ) ( ) 2|2 tt f ϕ ).
Với 1 t = ta có một căn nguyên thủy và n không có tính chất.
Cũng với 2t n = chúng ta có tính chất nếu và chỉ nếu 3 t ≥ như ta đã giải quyết về ( ) 2t f .
Bài III.13( Mathematical Reflections 5-2006)
Cho số nguyên 2 n ≥ thỏa mãn điều kiện ( ) ( ) ( ) 23nnn n ϕϕϕ +++ ⋯ chia hết cho n, trong đó
( )n ϕ là số các số nguyên dương không vượt quá n và nguyên tố cùng nhau với n
a) Chứng minh rằng n không có ước số lớn hơn 1 nào là số chính phương.
b) Biết rằng n có không quá 3 ước nguyên tố, tìm tất cả các số n thỏa mãn điều kiện trên.
Lời giải.
382
a) Giả sử tồn tại số nguyên dương n có ước chính phương lớn hơn 1 thỏa mãn đề bài.
Khi đó, n có ước dạng 2 p , với p là số nguyên tố. Từ giả thiết: ( ) ( ) ( ) 23nnn nnϕϕϕ +++⋯⋮ , suy ra ( ) ( ) ( ) 2 23. nnn npϕϕϕ +++⋯⋮ Theo định lý Euler, với số nguyên dương a mà ( ) ,1an = thì ( ) ( ) 1mod, n an ϕ ≡ suy ra ( ) ( ) 2 1mod. n ap ϕ ≡
Chú ý rằng trong tập { } 2,3,..., n có đúng n p số chia hết cho p và 1 n n p số
không chia hết cho p . Do ( ) ( ) 12 npp ϕ ≥−≥ nên với mỗi số nguyên b chia hết cho p , thì ( ) 2 n bp ϕ ⋮ . Vì thế ta có
t
≡≡+≡−− ,
n aaanp p ϕϕϕ ==
()() () () () 2 2,1| 01mod n nnn aappa
suy ra : () 2 10mod n np p −−≡ . Điều này vô lý, vì 2 np ⋮ . Vậy điều giả sử là sai,
ức là n không có ước chính phương lớn hơn 1 .
b) Gọi p là một ước nguyên tố của n, thì từ giả thiết ta có :
( ) ( ) ( ) 23nnn npϕϕϕ +++⋯⋮ hay () ()10mod10mod nn npp pp −−≡⇔+≡ . Xảy ra
ba trường hợp sau.
• Nếu np = là số nguyên tố thì ()0112mod np p pp ≡+≡+≡ , suy ra 2 p = hay
2 n =
• Nếu npq = với pq < là các số nguyên tố từ giả thiết ta có:
+≡ ⋮ ⋮
n p qp p npq q q ⇔⇔ + +≡
+
Do 1 pq +≤ và 1 pq + ⋮ nên 1 pq += .Mà , pq làcácsố nguyêntố nên 2,3pq== hay 2.36 n ==
i.Nếu npqr = ,với ;; pqr làcácsố nguyêntố từ giả thiếttacó:
+
⇔+≡⇔+⇔+++
()()() +
() () ()
+≡
10mod 1 10mod1111 1 10mod
1 pqqrrppqr⇔+++ ⋮ tứclà * 11111 pqqrrp pqrpqrpqr +++ =+++∈ ℕ .
Khôngmấttínhtổngquátgiả sử 1;2.pqrqprp << ≥+≥+
Khi đó: ()() 11111111 1212pqrpqrpppppp +++<+++ ++++
1111112 2 12121 ppppppp <+++−=+< +++++
Vậysuyra: () 1111 1* pqrpqr +++=
Tathấy 2 p = ngượclại * * 111172 1 3573.5.7105 VTVP <+++=<=
Khi () 1111 2;* 2..2 p qrqr =⇔++= lậpluậntươngtự suyra 3 q =
Cuốicùngtathu được 7 r = .Vậytrườngnàyta được 2.3.742 n ==
Vậycácsố n cầntìmlà 2;6;42.nnn===
Bài III.14 ( MOCK TEST VMO- 2019).
Gọi α lànghiệmdươngcủaphươngtrình 1 48 x x +=
a) Chứngminhrằngvớimọisố nguyêndương n tacóbất đẳngthức ( ) ( ) ( ) ( ) 222220202019 nnnn ϕααϕαα +>+
b) Chứngminhrằng 3030 33 S αα=+ cóítnhất 355.2 ướcnguyêndương.
Lời giải.
Mộtsố kếtquả quenthuộc
• ( )n ϕ và ( )n τ đềulàhàmnhântính( ( )n τ làhàmsố ướcdươngcủa n)
• ( ) 1 kkk ppp ϕ =− với p làsố nguyêntố, k làsố nguyêndương.
• ( ) 1 k pk τ =+ với p làsố nguyêntố, k làsố nguyêndương.
a) Đặt 22 nn n u αα=+ ,khi đó 2 1 2 nn uu + =− và 0 48 u = .Với nnguyêndương,xét p là
số nguyên tố lẻ bất kỳ của n u , khi đó ( ) 2 1 2mod n up ≡ hay 2 1 p =
( )17mod8 p ≡∨ .Suyra 222 1 35101
384
hay
Nên ( ) ( ) ( ) ,3,5,1011 nnn uuu=== vớ
() ()()() 20192.3.673.236731344 222 n aaa u ϕϕϕϕϕϕϕ
() () ()() 33 20202.5.101.251011600 222 n aaa u ϕϕϕϕϕϕϕ === ( ) ( ) 20202019 nn uuϕϕ >
• Nếu 0 k > ,khi đó:
() ()() () 1 2019236732.672.673 22 kk n aa u ϕϕϕϕϕϕ + ==
() () ()() () 3 1 2020251016731600.672.673 22 kk n aa u ϕϕϕϕϕϕϕ
( ) ( ) 20202019 nn uuϕϕ >
Nên ( ) ( ) 20202019 nn uuϕϕ > vớimọisố nguyêndương n b) Đặt 33 v nn n αα=+ ,khi đó 3 1 vv3v nnn + =− vớimọi n nguyêndương. Dễ thấy v n chia hết cho 6 với mọi n tự nhiên. Ta chứng minh các số 2 v 1 3 đôi một nguyêntố cùngnhau.
Với ij > thì vi chia hết cho ( ) 2 1 vvv3 jjj + =− hay vi chia hết cho 2 v3 j nên 2 v 3 i
−−=
là bội của 2 v3 3 hay 2 2 v v 1,11 33 i
. Do v n chẵn nên 2 v 1 3 n lẻ, gọi np là
ước nguyên tố lẻ bất kỳ của 2 v 1 3 n , suy ra các số ip đôi một phân biệt và khác 3 (
do 2 v 1 3 không chia hết cho 3).
+ ==
=−=−
do ( ) ( ) ττnd ≥ vơi d là ước
nguyên dương của n nên ( ) ( ) ( ) 41 01 ττ 2.3....522, nn n nppn + ≥=+ suy ra ( ) ( ) 3035 30 ττ v5.32.25.2 S =≥=
Vậy 3030 33 S αα=+ có ít nhất 355.2 ước nguyên dương.
Bài III.15.
Tìm số nguyên tố ( )5 pp ≥ nhỏ nhất sao cho tồn tại hai đa thức [ ] , fgx ∈ Z , thỏa mãn ( ) ( ) * |23, fngn pn+∀∈ / ℕ .
Lời giải.
Nhận xét : Nếu 00,|23nm pp∈℘+ với 00 , nm ∈ ℕ . Từ đó suy ra ()()22 0011 * |23,. pnnpnm pn −+−++∀∈ ℕ
Khả năng 1. Nếu n lẻ suy ra 1 k = ( ngược lại vế trái ( )2 chia hết cho 8 vô lý)
=−+∈ =−+∈ Z Z 00|23nm p ⇔+ với 00 , mn ∈ ℕ
1 1 fnpnnx gnpnmx
Vậy ( ) ( ) * |23, fngn pn+∀∈ ℕ , trong đó : ( ) ( ) [ ] ()() [] 2 0 2 0
Ta thấy: 202131224041 5|23,7|23,11|23,13|23,17|23,19|23 ++++++
===−
Xét số nguyên tố 23. Ta có 231 1;1 232323
Giả sử () 1 23|23,,32mod231 23 abba ab +∀∈ ≡− = ℕ vô lý. Từ đó suy ra 23|23,, ab ab +∀∈ / ℕ
Vậy số nguyên tố nhỏ nhất thỏa mãn yêu cầu bài toán là 23.
Bài III.16 (Taiwanese TST 1999)
Tìm tất cả các số nguyên dương , mn sao cho ( ) ,1mn = và ( ) 5151 mn ϕ −=− .
Lời giải.
Gọi : ( ) 1 ** 1 512,2,,,1,..., k m kiii ppppik α α α αα −=⋅⋅⋅∈℘>∈∈= ℕℕ
Từ điều kiện bài toán ( ) ( ) ( ) 1 1 1 1 11 5151211 k nm kk pppp α α α ϕ −=−=⋅⋅⋅−− ⋯
Do ( ) ( ) 51,5151514 mn mn −−=−=−= . Từ đó suy ra 11,2,...,ik α =∀=
Khi đó: 12 512 m kppp α −= và ( )( ) ( ) 1 12 512111 n kppp α −=−−−
+Nếu ( ) 3851,514 mn s > −−= (vô lý )
+Nếu 30sk = = ( ngược lại () ( ) 1 112851,514 mn kp ≥ −−= ⋮ ( vô lí))
Khi đó ta có 518 514
−= −= (vô lí)
m n
p p −= −=−+⇔=+ ≡ ⋮⋮
1 5 55mod111mod51,2,..., 5 m i i
p ppik p +
()
(vô
lí)).
Khi đó xét modulo 5 của (1) và (2) : ( ) ( ) ( ) 12 1054...14.11mod5 k m k pppk ⇔≡=+≡−+⇔ chẵn ( ) ( )( ) ( ) ( ) ( ) ( ) 12 205211...112.21mod53mod4 k n k pppk ⇔≡=−−−+≡−+⇔≡ hai điều trên mâu thuẫn với nhau
Vậy không tồn tại các số nguyên dương , mn sao cho ( ) ,1mn = , ( ) 5151 mn ϕ −=− .
Tìm tất cả các số nguyên dương n sao cho ( )n ϕ chia hết cho 4.
Lời giải.
Bổ đề : Nếu n là một số nguyên dương có k ước số nguyên tố lẻ khác nhau thì ( )n ϕ chia hết cho 2. k
Thật vậy. Hiển nhiên nếu p là số nguyên tố lẻ thì 2|1, p vậy từ đó ta suy ra
() () |||
11 11 pnpnpn pn pn nnnp ppp ϕ =−==− ∏∏∏ ∏
n p ∈ ∏ Z Từ đây ta suy ra ( ) ( ) ( ) 12, k pn pnn ϕϕ ∏ trong đó k là số ước nguyên tố lẻ phân biệt của.
Trường hợp2. ( ) ( ) * 2,2,1,, ks npppks =∈℘=∈ ℕ , khi đó
• Với 3 k ≥ ta có ( ) ( ) ( ) ( ) ( ) ( ) ( ) 1 422,22 kk ksks nnpp ϕϕϕϕϕϕ ===
+Nếu 2 s = cho ta ( ) ()() () () 12 12
5141 5121112
ppp ppp
m k n k
386
• Với 2 k = ta có
−= −=−−− … 387
() () () () () 2 1 11 2441121 2 sss s nppppp p ϕϕϕϕ ===−−=− , do ( ) 2|14|pn ϕ với mọi * . s ∈
Kết hợp hai trường hợp ta thu được giá trị của n thỏa mãn yêu cầu bài toán khi và chỉ khi n có dạng ( ) * 23,, ks nks=∈ℕ . Ngoài ra nếu 0 s > thì cần có 0. k >
=
= = =
Vậy để ( ) ( ) 1 4|4|14|1 s nppp ϕ ⇔−⇔− hay ( )1mod4 p ≡
Tổng hợp các trường hợp ta thấy rằng để ( ) 4| n ϕ khi và chỉ khi n là số nguyên
dương có ít nhất hai ước nguyên tố lẻ khác nhau hoặc n chia hết cho 8 hoặc n chia hết cho ( ) ( )4,2,1 ppp∈℘= hoặc n chia hết cho số nguyên p mà ( ) 1mod4. p ≡
Bài III.18
Tìm tất cả các số nguyên dương n sao cho n chia hết cho ( )n ϕ
Lời giải.
IV.BÀI TÂP TƯƠNG TỰ
Trường hợp 1. Nếu 2, k nk=∈ ℕ thì () 1 2|2khi0 1|khi0
Vậy nên n thỏa mãn yêu cầu bài toán.
kk nk n nnk ϕ => = ==
Trường hợp 2. Nếu ( ) ( )2,,,,21,1 k ntkttt =∈=> ℕ , ta thấy t có ít nhất một ước
nguyên tố lẻ nên theo kết quả Bài III 18 ta suy ra : ( ) ( ) ( ) ( ) ( ) 2|2|22 kk tttn ϕϕϕϕϕ ==
vậy trong trường hợp này, nếu n thỏa mãn yêu cầu bài toán thì ta phải có
2|0 nk >
và do đó ( ) ( ) ( ) ( ) ( ) 1 222 kkk nttt ϕϕϕϕϕ ===
Từ đó suy ra: ( ) ( ) ||2 nnttϕϕ ⇔ ( )1
Theo kết quả bài 5 thì từ ( )1 ta thấy t chỉ có một ước nguyên tố lẻ, tức là ( ) ( ) * ,,,2,1 s tpspp =∈∈℘= ℕ , nhưng khi đó ( ) ( ) 1 1 s tpp ϕ =− nên từ đây ( )1 tương
đương với
Bài 1. Chứng minh rằng: () ( ) 1
nn n d d ϕ =
+ =
Bài 2. Với số nguyên dương 1 n > , gọi ( )Sn là tập hợp các số nguyên dương k mà phần thập phân của n k ít nhất là 1 2 . Chứng minh rằng ( ) ()
2 kSn kn ϕ ∈
= .
Bài 3. Chứng minh rằng với mọi 1 n > ta có
a) ()() 2 2 2 n nnn σϕ >⋅>
b) () 4log n n n ϕ >⋅
Bài 4. Cho 2 n ≥ . Chứng minh rằng n là số nguyên tố nếu và chỉ nếu ( ) |1nn ϕ và ( ) 1| nn σ + ( với ( )n σ là tổng các ước dương của n ).
Bài 5.(AMM 11544). Chứng minh rằng với mỗi số nguyên 1 m > ta có
−==
() 112 1|21|2,,113 123 pp pppppp pp
+ += +
() 1 2
−−= ⇔ = −== ( ) * 223,, ksks npks ==∈ ℕ 389
0 21 21 m k
mk km k ϕ =
Bài 6.(Turkey TST 2013). Tìm tất cả các cặp số nguyên dương ( ) , mn sao cho
( ) ( ) 21!1. nm nnn ϕ +−−=+
Bài 7. Tìm tất cả các cặp số nguyên dương ( ) , mn sao cho ( ) ( ) ( )mnmnϕϕϕ =+
Bài 8. Chứng minh rằng không tồn tại số nguyên dương n sao cho ( ) ( )271 m nm ϕ =⋅≥
Bài 9. Chứng minh rằng () 2 5 n n ϕ = khi và chỉ khi * 25,, rs nrs=∈ℕ
Bài 10. Cho số nguyên dương n
a) Chứng minh rằng () 1 . 2 nnn ϕ ≤≤
b) Chứng minh rằng phương trình ( ) xn ϕ = có hữu hạn nghiện nguyên x
Bài 11. Chứng minh rằng với mỗi số nguyên 2 n > ta có bất đẳng thức ( ) ()( ) 2 22 12 nnnϕϕ++<
Bài 12. Chứng minh rằng nếu n là số nguyên chẵn, thì ( ) ( ) | 10 nd dn d ϕ −=
Nếu n là số nguyên lẻ thì điều gì xảy ra?
Bài 13. Chứng minh rằng ( )n ϕ là lũy thừa của 2 khi và chỉ khi 1 2... r nFF α = , với 0 α ≥
và 2 21,1,2,..., i Fir =+= là các số Fermat.
Bài 14. Cho một số nguyên lẻ 3 n > , chứng minh rằng tồn tại một số nguyên tố p chia
hết () ( )21 n ϕ nhưng không chia hết n
Bài 15. Giả sử n là sô nguyên dương. Lập dãy 123,,,...nnn bằng cách đặt
( ) ( ) 11,,1,2,3,... kk nnnnk ϕϕ + ===
Chứng minh rằng tồn tại r sao cho 1 r n = .
Bài 16. Chứng minh rằng số nguyên dương n là hợp số nếu và chỉ nếu
( ) nnn ϕ ≤− .
Bài 17. Cho , nk là các số nguyên dương và p là số nguyên tố lẻ. Chứng minh rằng tồn
tại số nguyên dương a sao cho k p là ước của tất cả các số ( ) ( ) ( ) ,1,..., aaanϕϕϕ++
Bài 18.Tìm các số nguyên dương n sao cho ( ) ( ) 33nnϕϕ = .
Bài 19. Tìm tất cả các cặp số nguyên dương ( ) , an sao cho ( ) 2 nn an ϕ += .
Bài 20. Cho số nguyên dương :1100nn≤≤ Tìm giá trị lớn nhất của ( ) ( ) 22 2 nnnϕϕ +− .
Bài 21.(HMMT-2015) Gọi S là tập các số nguyên dương n sao cho bất đẳng thức ()() 3 3 n nnϕτ⋅≥
ở đây ( )n ϕ là Phi-hàm Euler của n và ( )n τ là số ước dương của n . Chứng minh rằng S có hữu hạn phần tử.
Bài 22.(HMMT-2016) Số nguyên dương n được gọi là tốt nếu ( ) ( ) 4 nnnϕτ+= ,( ví
dụ số 44 là tốt vì ( ) ( ) 4444444ϕτ+= ). Tìm tổng tất cả các số tốt n .
Bài 23.(( VMO 2021, Day 1 P4). Với số nguyên 2 n ≥ , gọi ( )sn là tổng các số nguyên dương không vượt quá n và không nguyên tố cùng nhau với n
a) Chứng minh rằng () () () 1 2 n snnn ϕ=+− , trong đó ( )n ϕ là Phi-hàm Euler của n
b) Chứng minh rằng không tồn tại số nguyên 2 n ≥ thỏa mãn ( ) ( )2021 snsn=+
Bài 24.
Cặp số nguyên dương ( ) , mn được gọi là cặp số Bạn bè khi ( ) ( ) mnmnσσ==+ ( ví dụ cặp ( )220,384 có ( ) ( ) 220284504σσ== , ( )m σ là hàm tổng các ước dương của m ). Chứng minh rằng cặp ( ) ( ) , nn ϕ không là cặp Bạn bè
Bài 25(China TST 2021, Test 2, Day 2 P4). Tìm tất cả các hàm : f ++ → ℤℤ sao cho với tất cả các số nguyên dương , mn với mn ≥ thì ( ) ( ) ( ) ( ) 33 fmnfmn ϕϕ ⋅=⋅ ( ở đây ( )a ϕ là Phi- hàm Euler của số nguyên dương a ).
V. KẾT QUẢ SAU KHI ÁP DỤNG ĐỀ TÀI
Trong 03 năm học vừa qua tôi đã thực hiện đề tài này với các nhóm học sinh có học lực khá và giỏi (lớp 10A1,11A1của trường THPT Chuyên Vĩnh Phúc niên khóa 20172020). Để đánh giá hiệu quả của đề tài, tôi đã thực hiện hai bài kiểm tra trước và sau khi áp dụng, cụ thể như sau:
Đề 1 (Trước khi thực hiện chuyên đề)
Đề 2 (Sau khi thực hiện chuyên đề)
Hai đề có mức độ khó tương đương
390
391
Kết quả cho thấy điểm số trung bình ở các lớp 10A1, 11A1 tăng 68,74%,.Như vậy, việc áp dụng đề tài này rất có hiệu quả đối với lớp học sinh khá và giỏi.
Chuyên đề đã được giảng dạy cho học sinh giỏi toán trong trại hè Hùng Vương tháng 08 năm 2015 với kết quả tốt, giảng dạy cho học sinh giỏi toàn quốc tại Viện toán cao cấp nghiên cứu về toán Việt Nam tháng 8 năm 2014, với kết quả tốt.
1. Mathematical Olympiad Challenges-2001- Titu Andreescu.
2. Mathematical Olympiad Treasures-2004 Birkhauser Boston,USA- Titu Andreescu.
3. Gazeta Matematică-A bridge - Vasile Berinde.
4. Vô địch các quốc gia và vùng lãnh thổ từ 1978-2021.
5. Tuyển tập dự tuyển OLYMPIC toán học Quốc tế 1991-2020.
6. Tạp chí Mathematical Reflections.
7. Tạp chí Crux Mathemticarum : 1995-2021.
8. Đề thi học sinh giỏi lớp 12 các Tỉnh,Thành phố.
9. Chuyên đề bồi dưỡng học sinh giỏi toán THPT Số học-2004 - Hà Huy Khoái.
PHẦN C: KẾT LUẬN VÀ KIẾN NGH
1. Một số hướng phát triển Đề tài
- Khai thác thêm các kỹ năng đánh giá khác
Ị
- Kỹ năng tư duy của học sinh với các kiến thức số học liên quan đến chuyên đề
- Ứng dụng trong giải các lớp bài tập Số học liên quan đến Phi – hàm Euler.
2. Kiến nghị, đề xuất về việc triển khai áp dụng đề tài
Đề tài này thực sự cần thiết phải giảng dạy học sinh giỏi và học sinh chuẩn bị thi học sinh giỏi các cấp tỉnh, quốc gia, khu vực và quốc tế. Có thể áp dụng rộng rãi cho học sinh các lớp chuyên toán 10, 11, 12 trong toàn quốc, và các bạn yêu thích môn toán sơ cấp.
Vấn đề mới/cải tiến SKKN đặt ra và giải quyết so với các SKKN trước đây (ở trong nhà trường hoặc trong Tỉnh):
Vấn đề Phi- hàm Euler không phải là vấn đề mới của Số học sơ cấp, nhưng việc sử dụng kiến thức của nó để giải một số các bài toán Số học không phải là dễ.Trong SKKN này đã giúp thầy và trò có một cách tiếp cận bài toán một cách dễ gần hơn, cách giải các lớp bài toán trong SKKN cũng linh hoạt hơn, trong sáng hơn, lời dẫn dơn giản bạn đọc có thể tự học được và học tốt môn Số học và yêu quý môn Số học nói riêng cũng như môn Toán sơ cấp nói chung.
TÀI LIỆU THAM KHẢO.
392
393